You are on page 1of 419

Paper 4 – Set A with Solutions

Regn No: __________________

Name : __________________
(To be written by the candidate)

18th NATIONAL CERTIFICATION EXAMINATION


FOR
ENERGY MANAGERS & ENERGY AUDITORS – September, 2017

PAPER – 4:Energy Performance Assessment for Equipment and Utility Systems

Date:24.09.2017Timings: 14:00-16:00 HRS Duration: 2 HRS Max. Marks: 100

General instructions:
o Please check that this question paper contains 7 printed pages
o Please check that this question paper contains 16 questions
o The question paper is divided into three sections
o All questions in all three sections are compulsory
o All parts of a question should be answered at one place

Section - I: BRIEF QUESTIONS Marks: 10 x 1 = 10

(i) Answer all Ten questions


(ii) Each question carries One mark

S-1 A rise in conductivity of boiler feed water indicates a rise in ____ level of feed water.

Ans TDS

S-2 In a parallel flow heat exchanger the hot fluid inlet temperature is 150 °C . The cold
fluid inlet and outlet temperatures are 45 °C and 60 °C. Calculate the effectiveness.

Ans = 15/105 =0.14

S-3 Integrated Part Load Value (IPLV) in a vapour compression refrigeration refers to
average of ____with partial loads

Ans kW/TR

S-4 A pure resistive load in an alternating current (AC) circuit draws only reactive power
– True or False

Ans False (active power)


Paper 4 – Set A with Solutions

S-5 In a reciprocating air compressor, if the speed is reduced to 80%, the power will
reduce by about 50% -True or False

Ans False

S-6 If slip of an induction motor increases, the shaft speed also increases – True or False

Ans False

S-7 The advantage of evaporative cooling is that it is possible to obtain water


temperatures below the wet bulb economically. True or false

Ans False

S-8 In a step down transformer for a given load the current in the primary will be more
than the current in the secondary. True or false

Ans False

S-9 For two pumps to be operated in parallel their ______heads should be the same

Ans Shut off (or ’closed discharge valve’ heads)

S-10 A fluid coupling changes the speed of the driven equipment without changing the
speed of the motor. True or false

Ans True

…………. End of Section - I ………….

Section - II: SHORT NUMERICAL QUESTIONS Marks: 2 x 5 = 10

(i) Answer all Twoquestions


(ii) Each question carries Five marks

L-1 In a Process Industry the L.P and H.P boilers have the same efficiency of
83%. The operating parameters and data are given below:

Boiler L.P. (Low Pressure) H.P. (High Pressure)


Efficiency on G.C.V. 83% 83%
Fuel Furnace Oil Furnace Oil
G.C.V. 10,000 Kcal/Kg. 10,000 Kcal/Kg.
Steam enthalpy 666 Kcal/Kg. 737 Kcal/Kg.
Feed water temperature 95oC 105oC
Paper 4 – Set A with Solutions

The cost of steam fromL.Pboiler is Rs. 3000 per tonne. Find out the cost of
steam from H.P boiler.

Ans % Boiler Efficiency = (TPH of Stm) x 1000 x (Enth of Stm – Enth of FW) x 100
(Mass of Fuel x GCV Fuel)

Evaporation ratio of LP Boiler; ER LP =0.83 X 10000 = 14.53


(666 – 95)
………………..1.5 marks
Evaporation ratio of HP Boiler; ER HP = 0.83 X 10000= 13.13
(737 – 105)
………………..1.5 marks
ER HP is less than ER LP ;
Thus, the specific fuel consumption (kg fuel / kg steam) is more in the case
of the HP boiler than in the case of the LP boiler.
Therefore, the cost of steam from HP boiler is higher than the cost of steam
from LP boiler.
HP Steam Cost = 14.54x 3000 = Rs.3322 per tonne
13.13 ………2 marks

OR
1 T of FO – 14.54 T of LP steam
Cost of LP steam – Rs.3000/T
cost of 1 T of FO= Rs.3000 x 14.54 = Rs.43620/-
………………..1 mark
1 T of FO – 13.13 T of HP steam
cost of 1T of HP steam = Rs.43620/13.13 = Rs.3322/T

………………..1 mark
L-2 A shell-and-tube heat exchanger with 2-shell passes and 8-tube passes is
used to heat ethyl alcohol (cp= 2670 J/kgoC) in the tubes from 25 oC to 70oC
at a rate of 2.1 kg/s.
The heating is to be done by water (cp= 4190 J/kgoC) that enters the shell
side at 95oC and leaves at 45oC.
The LMTD correction factor for this heat exchanger is 0.82
If the overall heat transfer coefficient is 950 W/m2oC, determine the flow
rate of water in kg/s and surface area of the heat exchanger in m2.
Ans Heat duty
Cold fluid (ethyl alcohol)
Qcold= 2.1 x 2670 x (70-25) J/s
= 252315 Watts
Paper 4 – Set A with Solutions

= 252.315 kW
………………..1 mark
Hot fluid (water)
Qhot= mw x 4190 x (95 -45)
= mw x 209500 J/s
= (209500 mw) Watts
= (209.5 mw) kW
………………..1 mark
Qcold= Qhot
252.315 kW = (209.5 mw) kW
mw=1.204 kg/s
LMTD = [(95-70) – (45-25)] / [ln (95-70) / (45-25)]
= 22.42oC
Corrected LMTD = 0.82 x 22.42
= 18.38oC
………………..2 marks
Q = U*A*LMTD
A = 252315 / (950x 18.38)
= 14.5m2
………………..1 mark

…………. End of Section - II …………

Section - III: LONG NUMERICAL QUESTIONS Marks: 4 x 20 = 80

(i) Answer all Four questions


(ii) Each question carries Twentymarks

N-1 A Process industry is operating a natural gas fired boiler of 10 tonnes/hr to cater
to a steam load of 8 tonnes/hr at 10.5 kg/cm2(g). The O2 in the flue gas is 4%
and the exit flue gas temperature is180oC. Due to increased cost of natural gas,
the management has decided to revert to operating the furnace oil fired boiler,
having an efficiency of 84% on G.C.V. for meeting the above load.

In keeping with its sustainability policy the management proposes to offset the
additional CO2 emissions due to the use of furnace oil by sourcinga part of its
total electrical energy consumption from green power (wind source).
Paper 4 – Set A with Solutions

The following is the additional data.


COMPOSITION OF FUELS (% BY WEIGHT)

Constituents Natural gas Furnace oil

Carbon 73 84
Hydrogen 23 11
Nitrogen 3 0.5
Oxygen 1 0.5
Sulphur - 4

 G.C.V. of natural gas -13000 kcal/kg


 Enthalpy of steam at 10.5 kg/cm2(g) -665 kcal/kg.
o
 Inlet feed water temperature -90 C
 Heat loss due to Radiation and moisture in air -1.2%
o
 Specific heat of flue gases -0.29 kcal/kg C
o
 Specific heat of super heated water vapour -0.45 kcal/kg C
 G.C.V. of furnace oil - 10,000 kcal/kg
o
 Ambient temperature -30 C
,

Substitution by 1 kwh of green electrical energy in place of grid electricity,


reduces 0.80 kg. of CO2
Determine the monthly amount of green electrical energy from wind, (for 720
hours operation) required to be purchased to maintain the existing level of CO 2
emissions.

 Theoretical air required = 11.6 C + [34.8 (H2 – O2/8)] + 4.35 S


Ans = 11.6x0.73 + [34.8 (0.23 – 0.01/8)]
= 16.43 kg. air / kg. gas
 Excess Air % = % O2 / (21 - % O2) x100
= [(4 ) / (21 – 4)] x100
= 23.5 %
 Actual Air Supplied (AAS) = (1 + 0.235) x 16.43
= 20.29 kg.air / kg.gas

………………..3 marks
 Mass of dry flue gas mdfg = mass of combustion gases due
to PresenceofC,N,S + mass of
N2 in the fuel + mass of nitrogen
in air supplied + mass of excess
O2in flue gas
Paper 4 – Set A with Solutions

=(0.73 x 44/12) + 0.03 + (20.29 x 0.77)


+ (20.29–16.43) x 0.23

= 19.22 kg. dry flue gas / kg. gas


………………..2. marks
 (Mair+Mfuel) ie (20.29+1) = 21.29 may also be considered.
 L1 = % heat loss due to dry flue gases
= MdfgxCpx (Tq – Ta)x 100
GCV of fuel(NG)
= 19.22 X 0.29 X (180 – 30)x 100
13000
= 6.43 %
………………..2 marks
 L2 = % Loss due to water vapour from hydrogen

= 9 H [584 + Cps (Tq – Ta)] x100


13000
= [9x0.23x[584+0.45x(180-30)]x100
13000
= 10.37 %

………………..2 marks
 Heat loss due to Radiation and
moisture in air= 1.2% (given)

 Efficiency of natural gas boiler


on GCV = 100 – [6.43 + 10.37 + 1.2]
= 82%

 Steam Load = 8 tonnes /hr.

 Amount of Gas required= 8000 (665 – 90)


0.82 X 13000 ,,

= 431.52 kg / hr
………………..2 marks
 Amount of CO2 emission with
natural gas = (431.52 X 0.73 X 3.67)
= 1156.1 Kg/hr.
 Amount of furnace oil required for
the same steam load =8000 (665 – 90)
0.84 X 10000
Paper 4 – Set A with Solutions

= 547.62 kg / hr
………………..2 marks
 Amount of CO2 emission with F.O = (547.62 X 0.84 X 3.67)
= 1688.2kg CO2/hr

………………..2. marks
(Note: 1 Kg. Carbon Combustion emits 3.67 Kg. CO 2)

 Increase in CO2 emission due


to switchingfrom natural gas
to furnace oil= (1688.2 – 1156.1)
= 532.1 kg. CO2/hr.
………………..2.5 marks
[Substituting 1 kWh grid (Thermal) electrical energy by green electrical energy
reduces 0.80 Kg.of CO2)]
 Green energy to be purchased to
offset higher CO2 emissions per
month= [(532.1x 720)/ 0.8] =4,78,890 Kwh

………………..2.5 marks
N-2 The monthly energy consumption for 30 days operation in a 25 TPD (Tonneper day)
ice plant, producing block ice, is 37,950 kWh. The daily output of the ice plant is 15
Tonnes of block ice by freezing 16.5 m3 of water at 30oC. The higher water
consumption is due to loss of ice, while removing the block ice from ice cans, for
customer delivery. The following data has been given:
o
 Temperature of ice block = (-) 8 C
 Latent heat of freezing of ice = 80 kcal/kg.
o
 Specific heat of water = 1 kcal/kg C
o
 Specific heat of ice = 0.5 kcal/kg C
 Energy consumption in the
ice plant chiller compressor = 85% of the total energy consumption
 Efficiency of compressor motor = 88%

Estimate the,
a) Energy consumption per tonne of ice ‘output’,
b) Total daily cooling load in kcals for freezing water into ice blocks,
c) Refrigeration load on the chiller in TR (Tonne refrigeration) and
d) E.E.R. of ice plant chiller compressor.
The Management intends to pre-cool the inlet water from 30oCto 12oC using a
separate water chiller, drawing0.8 kW/TR.

e) Find out the reduction in energy consumption per tonne of ice block output
f) % reduction in the condenser heat load of the plant chiller due to the use of pre-
cooled water.
Paper 4 – Set A with Solutions

Assume overall auxiliary energy consumption of the plant remains same and only
consider water chiller compressor energy consumption for estimating the savings.

Ans a) Monthly energy consumption = 37950 Kwh


Daily energy consumption = 37950 / 30 = 1265 kWh
 Energy consumption per tonne of ice delivered = 1265/15
= 84.33 kWh/tone
………………..3 marks
b)
3
Quantity of water input for the production 16.5 m = 16500 kg.
(sp.wt of water = 1000 Kg./m3)

Total cooling load per day


Q = Q1 + Q 2 + Q 3
o o
Q1 = Heat removed from lowering temperature from inlet 30 C to 0 C in kcals
o
Q2 = Latent heat removed in freezing water to ice at 0 C in kcals
o o
Q3= Heat removed for sub-cooling of ice from 0 C to -8 C in kcals

Q = (16,500 X 1 X (30-0)) + (16,500 X 80) + {16,500 X 0.5 X [0 – (-8)]}


= 4,95,000 + 13,20,000 + 66,000

Total Daily Cooling Load = 18,81,000 kCals


………………..3 marks
c)
18,81,000
Refrigeration load on the Chiller = ------------ =25.92 TR
24 X 3024
………………..3 marks

d)E.E.R. ice plant chiller

Ice plant chiller consumption per day = 0.85 X 1265


1075.25 kWh
Ice plant auxiliary consumption per day = 1265 – 1075.25
189.75 kWh
Power consumption of the chiller = 1075.25 / 24
44.80 KW
Input KW/TR Ice Plant chiller = 44.80 / 25.92
1.728
Motor Efficiency = 88%
Input power to the ice plant compressor = 0.88 X 1.728
Paper 4 – Set A with Solutions

1.52 KW / TR
E.E.R. ice plant chiller (3024)kcal/hr/(1.52X860)
kcal/hr
2.313
………………..4 marks

e) Reduction in energy consumption per tonne of ice block output

Condenser heat rejection load in the existing = QE + Q C


case Q1
(25.92 X 3024) + (25.92 X
1.52 X 860)
1,12,264 kcals/hr
o 16500 X 1 X (30 – 12)/ (24 X
Refrigeration load for pre-cooling from 30 C
o 3024)
to 12 C in a separate water chiller
4.09 TR
Energy consumption in water chiller = 0.8 X 4.09 X 24 = 78.53 kWh
Reduced ice plant chiller load = 25.92 – 4.09 = 21.83 TR
Energy consumption for the plant chiller = 21.83 X 1.728 X 24 =
905.33 kWh
Total energy consumption per day by = Energy consumption in ice
resorting to pre-cooling of inlet water in a plant chiller+ Auxiliaries in ice
separate water chiller is plant
(no change) + Energy
consumption in water chiller
for pre-cooling
= 905.33 + 189.75 + 78.53
1173.61 kWh/day
Reduction in energy consumption = (1265 – 1173.61) /15
kWh/tone for ice delivered
6.092
………………..4 marks

f)
Heat rejection load in the ice plant = (21.83 X 3024) + (21.83 X
condenser 1.52 X 860)
94550 kcal/hr
% reduction in ice plant condenser heat = (1,12,264–94,550)x100
load /(1,12,264)
15.8 %

………………..3 marks
Paper 4 – Set A with Solutions

N3 In a Petrochemical Industry a gas turbine cogeneration system comprising of 20


MW gas turbine generator along with a waste heat boiler (WHB) of 70 Tonne
per hour capacity at 10 kg/cm2 (g) are operated to meet the power and steam
requirements. The existing operating data is given below:

Power supplied by the Cogenerator = 16000 kW


Power drawn from the grid = 1500 kW
Grid power cost = Rs 5 /kWh
2
Steam at 10 kgf/cm g supplied by WHB = 48 Tonne/hr
(without supplementary fuel firing)
Efficiency of gas turbine on G.C.V. = 28%
Efficiency of generaror= 95%
G.C.V. of fuel (Natural Gas) = 13000 Kcal/Kg
3
Density of natural gas = 0.7 Kg./m
3
Cost of natural gas = Rs.25/m
o
Temperature of gas turbine exhaust gas entering WHB = 515 C
o
Specific heat of exhaust gas =0.3 kcal/kg C
o
Ambient temperature = 30 C
Air to natural gas ratio for gas turbine combustion = 60:1
Enthalpy of steam at 10 kgf/sq.cm.g = 665 Kcal/Kg
Enthalpy of feed water = 105 Kcal/Kg

a) Find out the heat rate of the gas turbine generator and

b) Estimate the efficiency of the waste heat boiler.

The plant personnel claim and believe that by resorting to supplementary fuel
firing to increase steam generation in the WHB. is likely to improve its efficiency
by 1.5% points.

c) Determine if it is economical to generate additional steam requirement of 10


Tonne per hour by supplementary fuel firing in WHB. as against in a separate
natural gas fired smoke tube boiler of 82% efficiency on G.C.V.

The plant operations are steady and continuous with 8760 yearly hours of operation.

Ans a)

Efficiency of gas turbine generator= 28%


Heat Rate = 860 / 0.28
= 3071.43 kcal/kWh
………………..4 marks
b)
Paper 4 – Set A with Solutions

Gas Rate = 3071.43 / 13000


= 0.236 kg.Natural gas/kWh
………………..2 marks
Power generated by Gas turbine = 16000 KW
Steam supplied by WHB = 48000 Kg./hr
 power to Steam ratio = 3 KW / Kg. steam
Air to fuel ratio of gas turbine combustion = 60 : 1
Exhaust gas per Kg. of natural gas fired = 60 + 1 = 61 Kg. per Kg
of natural gas

48000 x (665 – 105)


Efficiency of waste heat boiler = ---------------------------------------
(without supplementary fuel firing) 6000 X 0.236 X 61 X 0.3 X 515

= 75.5%
………………..4 marks
c)

Efficiency of WHB with supplementary firing (as per claim)= 75.5+1.5


= 77%

Additional gas consumption for meeting 10 Tonne/hr steam through


supplementary firing in WHB =

10000 (665 – 105)


= ---------------------- = 559.44 Kg./hr.
0.77 X 13000

10000 (665 – 105)


Gas consumption in separate gas fired boiler with 82% on GCV = -----------------------
0.82 X 13000

= 525.33 Kg/hr
………………..5 marks

Operating separate gas fired boiler is economical.


Saving in gas consumption by meeting additional steam through
gas fired boiler =
= 559.44 – 525.33
= 34.1 Kg/hr
= 34.1 / 0.7
3
= 48.714 m /hr
Paper 4 – Set A with Solutions

Yearly monetary savings = 48.714 X 25 X 8760


= Rs.1,06,68,366
= Say Rs.10.67 million
………………..5 marks
N-4 Answer any one of the following
A) The heat balance of a stenter in a textile industry is given below:

Heat used for Drying = 48%


Heat loss in exhaust air = 42%
Heat loss through insulation = 6%
Heat loss due to air infiltration = 4%

The above stenter is drying 75 meters per min. of cloth to final moisture of 7%
o
with inlet moisture of 50%. Temperature of cloth at inlet and outlet is 25 C and
o
75 C respectively.

The hot air for drying in the stenter is heated by thermic fluid. The thermic fluid
heater is fired by furnace oil, having an efficiency of 84%. The following data
has been given:

Density of furnace oil = 0.95 Kg/litre


GCV = 10000 kcal/kg
Cost of furnace oil = Rs.24 per litre

Weight of 10 mts of outgoing dried cloth= 1 Kg

a) Find out the existing furnace oil consumption for stenter drying.

b) What will be the annual furnace oil savings and annual monetary saving if
the overall thermal efficiency of the stenter is improved by reducing the
combined thermal insulation loss and the loss due to air infiltration, by half,
for operations at 22 hours per day and 330 days per year.

Ans Stenter speed = 75 meters / min


Dried cloth output = 75 x 60 /10
= 450 kg/hr
Weight of bone dry cloth per hr. = 450 x 0.93
i.e. W = 418.5 kg./hr
………………..2.5 marks
Weight of outlet moisture per kg. of bone dry cloth
mo = (450 – 418.5) / 450
= 0.0753 kg/kg
Paper 4 – Set A with Solutions

………………..2.5 marks
Inlet moisture = 50%
Inlet wet cloth flow rate = 418.5/ 0.5 = 837kg/hr
mi inlet moisture per Kg. of bone dry cloth= (837 – 418.5) / 418.5
mi = 1 kg/kg bone dry cloth
Heat load on the dryer = Wx(mi – mo)x[(Tout – Tin)
+ 540] Kcal/hr
Tout= Outlet cloth temperature
o
= 75 C
Tin= Inlet cloth temperature
o
= 25 C

Heat load on the dryer=418.5 kg/hrx


(1 – 0.0753)kg/kg dry.clthx
[(75 – 25) + 540]
= 2,28,322.3 kcal/hr
………………..2.5 marks

Based on heat balance, dryer efficiency is 48%.


Heat input to the dryer = 228322.3 / 0.48
= 4,75,671.46 kcal/hr

Furnace oil consumption in =


Thermic fluid heater = 4,75,671.46/(0.84x10000)
= 56.63 kg./hr.
………………..2.5 marks

After reducing insulation and air infiltration loss by half, the heat energy
input will reduce by 100% – 0.5 (6 + 4)% = 95%

Dryer efficiency will increase to = (48/0.95) x 100


= 50.52%

Furnace oil consumption with = 2,28,322.3/(0.5052x0.84x


10000)
improved dryer efficiency
= 53.80 kg/hr
………………..4 marks
Saving in Furnave oil
consumption due to
improved stenter efficiency = 56.63 – 53.80
= 2.83 kg/hr
AnnualFurnace oil savings = 2.83x22x330
Paper 4 – Set A with Solutions

= 20545.8 kgs/year
………………..3 marks

Annual monitory savings = 20545.8x(1/0.95)x24


= Rs.5,19,051.8
………………..3 marks
Note:
If candidates had done the calculation with temperature of cloth at inlet at 75 oC and outlet
at 25oC. the marks can be awarded according the steps.
B) In a secondary steel manufacturing unit, steel scrap is melted in an arc furnace.
The molten metal is then taken for ladle refining followed by vacuum degassing,
before being cast into ingots.

After the ingots are cooled down to ambient temperature, the entire lot is loaded
in a batch forging furnace and heated to 1150 oC. The heated ingots are forged
into desired shapes. The monthly number of batches are 160.

The management has decided to improve energy efficiency of the system by


incorporating a holding furnace ( electric resistance furnace) in between the
electric arc furnace and the fuel fired forging furnace, in order that the hot ingots (
after casting) could directly fed into the intermediate holding furnace to maintain
temperature and be fed at high temperature to the forging furnace, instead of at
atmospheric temperature.

Following are the data obtained in the energy audit study of the unit.

1. Scrap material fed into the arc furnace = 10 tons per heat
2. Yield of ingot casting from scrap = 95%
3. Temperature of casting after removal of mould = 600 oC
4. Ambient temperature = 30 oC
5. Specific heat of steel = 0.682 kJ/ kg oC
6. Efficiency of forging furnace = 25 %
7. Calorific value of Furnace oil fuel = 10500 kcal/ kg
8. Specific gravity of F.O = 0.9
9. Yield of forged steel in forging furnace = 97 %
10. Melting point of steel = 1650 0C
11. Latent heat of melting of steel = 272 kJ/kg
12. Electrical energy consumption measured per ton of steel melted = 800 kWh
13. Electrical energy consumption for holding ingots at 600oC in electric furnace
= 75kWh per batch
14. Cost of electricity = Rs.6 /kWh
15. Cost of Furnace oil = Rs. 30,000 / ton
Paper 4 – Set A with Solutions

Calculate
a. Efficiency of electric arc furnace ignoring heat loss due to slag
b. Specific oil consumption in litres per ton of finished forged product.
c. Annual net savings in energy cost by holding the hot forged casting in an
intermediate electric furnace at 600 oC before feeding into forging furnace.

Ans a) Efficiency of the arc furnace.

Theoretical heat required for melting one ton of steel


*
1,000 x 0.682 x 1650 − 30 + 272
=
3600
{kJ per ton of molten metal/(4.18kj / kcal x 860kcals/kwh)}

= 382.45 kWh per ton of molten steel


………………..3 marks

Efficiency = 382.45 x 100 /800 = 47.8 %


………………..2 marks

b) Specific oil consumption in liters per ton of finished forged product


from the forging furnace

Amount of material heated in forging furnace


= 10,000 kg x (0.95) = 9500 kg steel / batch
Oil consumption = 9500 x (0.682 /4.18) x (1150-30) / (10500 x 0.25)
= 661.3 kg FO
………………..3 marks
Amount of material forged = 9500 kg x (0.97) = 9215 kg steel / batch
Specific oil consumption = 661.3 kg FO / 9.215 tons steel = 71.76 kg FO/ton
= 71.76 / 0.9 = 79.73 Lts FO / ton of forged steel
………………..3 marks
Paper 4 – Set A with Solutions

c) Net Savings in energy cost by holding the hot forged casting in an


intermediate electric furnace at 600oC before feeding into forging
furnace
Oil consumption = 9500 x (0.682 /4.18) x (1150-600) / (10500 x 0.25)
= 324.76 kg FO per batch
………………..2.5 marks
Additional electrical energy consumption for holding ingots at 600 oC
= 75kWh per batch
Reduction in FO consumption by hot charging the forge furnace
= 661.3 - 324.76 = 336.54 kg FO per batch
………………..2.5 marks
Net savings in energy cost = (336.54 x 30) – (75 x 6) = Rs. 9646.2 per batch
Annual Net savings in energy cost = 9646.2 x 12 x 160 = Rs. 185,20,704 /yr
………………..4 marks

C) A steam power plant consisting of high pressure Turbine(HP Turbine) and low pressure
Turbine(LP Turbine) is operating on Reheat cycle(schematic of power plant is
represented below).

Steam from Boiler at a pressure of 150 bar(a) and a temperature of 550 0C expands
through the HP Turbine. The exhaust steam from HP Turbine is reheated in a reheater at
a constant pressure of 40 bar(a) to 550 0C and then expanded through LP Turbine. The
exhaust steam from LP Turbine is condensed in a condenser at a pressure of 0.1 bar (a).

The isentropic efficiencies of HP Turbine and LP Turbine are same and is 90%. The
generator efficiency is 96%

The other data of the power plant is given below:

Main steam flow rate : 228 TPH


Enthalpy of main steam: 3450 kJ/kg
Enthalpy of feed water : 990.3kJ/kg
Isentropic Enthalpy of cold reheat steam : 3050 kJ/kg
Enthalpy of hot reheat steam : 3560 kJ/kg
Condenser pressure and temperature: 0.1 bar(a)
and 45.80C
Isentropic enthalpy of LP Turbine exhaust steam : 2300 kJ/kg
Enthalpy of dry saturated steam at 0.1 bar(a) and 45.8 0C : 2584.9kJ/kg
Enthalpy of water at 0.1 bar(a) and 45.8 0C:191.9 kJ/kg

Based on the above data calculate the following parameters


(a) Power developed by the Generator
(b) Turbine heat rate
(c) Turbine cycle efficiency
(d) Specific steam consumption of turbine cycle.
Paper 4 – Set A with Solutions

Ans (a) Power developed by the Generator: Turbine output x Generator efficiency------------ (1)
Turbine output = Q1 (H1 – h2) + Q2(H3 – h4)/860 MW ---------------------------------------(2)
Where, Q1=main steam flow rate =228 TPH
H1=main steam enthalpy=3450 KJ/Kg
h2=actual enthalpy at HP Turbine outlet= ?(cold reheat enthalpy)
Q2=steam flow through reheater=228TPH
H3=enthalpy of hot reheat steam=3560 KJ/kg
h4= actual enthalpy of LP turbine exhaust steam=?

HP Turbine isentropic efficiency= Actual enthalpy drop/isentropic enthalpy drop


0.9= (H1- h2)/(H1-h2is) , h2is=isentropic enthalpy of cold
reheat
Steam=3050KJ/kg
0.9= (3450 –h2)/(3450—3050)
h2= 3090KJ/kg

LP Turbine isentropic efficiency= (H3—h4)/(H3—h4is), h4is=isentropic enthalpy of LP


Turbine
Exhaust steam=2300KJ/kg
0.9=( 3560-h4)/(3560—2300)
h4= 2426 KJ/kg

Substituting the values in equation-2,we get

Turbine output = 228(3450—3090) + 228(3560—2426)/3600 = 94.62MW


Generator output= 94.62 x 0.96= 90.83 MW--------------------ANSWER (9 MARKS)

(b) Turbine heat rate=Q1 (H1—hfw) +Q2(H3—h2)/Generator output =KJ/kwhr-------------(3)


Paper 4 – Set A with Solutions

hfw=enthalpy of feed water=990.3KJ/kg


Substituting the values in the above equation-3, we get

Turbine heat rate=228 (3450—990.3) + 228(3560—3090)/90.83


=7354.08 KJ/kWhr------------------------------ANSWER (5 MARKS)

(C) Turbine cycle efficiency= 860/Turbine heat rate


=860/(7354.08/4.18) =48.95%------------------ANSWER (3MARKS)

(d) Specific steam consumption of cycle=Steam flow/generator output


=228/90.83
=2.51 tons/MWhr----ANSWER(3MARKS)

D) In a cement kiln producing 4500 TPD of clinker output, the grate cooler hot exhaust air
temperature is vented to atmosphere at 275 oC.

It is proposed to generate hot water from this waste exhaust for operating a Vapour
Absorption Machine(VAM)chiller. This will replace the existing Vapour Compression
Chiller (VCR) of 50 TR capacity used for air-conditioning of control rooms and office
buildings.

The following are the data:


 Diameter of the cooler vent : 2 m
 Velocity of cooler exhaust air : 18.6 m/s
 Density of cooler exhaust air at 275 oC : 0.64 kg / m3
 Existing VCR Chiller Specific power consumption : 0.9 kW/TR
 Existing VCR condenser water pump power
consumption : 2.8 kW
 Investment towards 50TR VAM & its associated system :Rs 30 lakhs
 CoP of VAM system : 0.75
 Power consumption of VAM auxillaries: 2.83 kW
 Temperature of circulating hot water of VAM generator: Inlet - 90oC;
outlet - 80 oC
 Specific heat of exhaust cooler air : 0.24 kcal/ kgoC
 The efficiency of all pumps and their drive motors are 75% & 90%
respectively.
 The cost of electricity :Rs.6/kWh
 No of hours of operation : 8000 hrs/ yr

Calculate
a) Cooler Exhaust air temperature after heat recovery
b) Payback period by replacement of VCR by VAM

Ans a) Cooler Exhaust air temperature after heat recovery


Paper 4 – Set A with Solutions

 Area of the duct= 𝜋r 2 = 3.14 x (2/2)^2 = 3.14 m2


 Volume of cooler exhaust air2750C= 3.14 x 18.6 = 58.4 m3/s = 2,10,240 m3/h
 Mass flow rate of cooler exhaust air275oCmcxa= 210240 x 0.64 = 134553 kg/ hr
Capacity of existing chiller= 50 TR
 Cooling load = 50 x 3024
 = 151200 kcal/ hr
CoP of VAM= 0.75
= (Cooling Load / Heat Input)
 Heat Input to VAM generator = 151200 / 0.75
 = 201600 kcal/hr
201600 kcal/hr= mhwxCp-hw x (90oC -80oC)
 Hot water flow rate mhw= 201600 / (1 x 10) = 20160 kg/hr
 Heat input to VAM generator = Heat recovered from Cooler Exhaust Air (m cxaxCp-cxax
(275-To)
 Cooler Exhaust air temperature after heat recovery
To = 275- [201600 / (134553 x 0.24)]
= 268.76oC
………………..5 marks
b) Payback period by replacement of VCR by VAM

Hot water circulation pump capacity


 motor input power Pm= mhw x head developed x 9.81 / (1000 x Pump η x motor η m)
Pm= [(20160 /3600) x 20 x 9.81/ (1000 x 0.75x 0.9)] = 1.63 kW
Heat load in the cooling tower= heat load from chilled water + heat load from
generator hot water
= 151200 + 201600 = 352800 kcal/ hr
 Condenser water circulation rate = 352800 / 5 = 70560 kg / hr

………………..3 marks

Condenser water circulation pump capacity


 motor input power Pm = mhw x head developed x 9.81 / (1000 x Pump η x motor η m)
Pm= [(70560 /3600) x 20 x 9.81/ (1000 x 0.75x 0.9)] = 5.69 kW
………………..4 marks
Savings
 Existing VCR Chiller Specific power consumption = 0.9 kW/TR
 Existing VCR Chiller total power consumption = 50 x 0.9
= 45 kW
 Existing VCR condenser water pump power consumption = 2.8 kW
 Total Energy Saving = Existing VCR Chiller total power consumption – (Proposed
VAM chiller power consumption)
=(45+2.8) – (1.63+2.83+5.69)
= 37.65 kW

………………..5 marks
Paper 4 – Set A with Solutions

 Annual Energy savings = 37.65 x 8000 = 301200 kWh/yr


 Annual Monetary savings = 301200 x 6 = Rs. 18.07 Lakhs /y
 Investment towards 50TR VAM & its associated system = Rs 30 lakhs
 Simple payback period = 30 / 18.07 = 1.7 yrs or 19.9 months

………………..3 marks

-------- End of Section - III ---------


Paper 4 – Set B with Solutions

Regn No: __________________

Name : __________________
(To be written by the candidate)

18th NATIONAL CERTIFICATION EXAMINATION


FOR
ENERGY MANAGERS & ENERGY AUDITORS – September, 2017

PAPER – 4:Energy Performance Assessment for Equipment and Utility Systems

Date:24.09.2017 Timings: 14:00-16:00 HRS Duration: 2 HRS Max. Marks: 100

General instructions:
o Please check that this question paper contains 7 printed pages
o Please check that this question paper contains 16 questions
o The question paper is divided into three sections
o All questions in all three sections are compulsory
o All parts of a question should be answered at one place

Section - I: BRIEF QUESTIONS Marks: 10 x 1 = 10

(i) Answer all Ten questions


(ii) Each question carries One mark

S-1 In a parallel flow heat exchanger the hot fluid inlet temperature is 150 °C . The cold
fluid inlet and outlet temperatures are 50 °C and 70 °C. Calculate the effectiveness.

Ans = 20/100 =0.2

S-2 A pure resistive load in an alternating current (AC) circuit draws only active power –
True or False

Ans True (active power)

S-3 Integrated Part Load Value (IPLV) in a vapour compression refrigeration refers to
average of ____with partial loads

Ans kW/TR

S-4 If slip of an induction motor increases, the shaft speed decreases – True or False

Ans True
Paper 4 – Set B with Solutions

S-5 In a reciprocating air compressor, if the speed is reduced to 80%, the power will
reduce by about 50% -True or False

Ans False

S-6 The advantage of evaporative cooling is that it is possible to obtain water


temperatures below the wet bulb economically. True or false

Ans False

S-7 A fluid coupling changes the speed of the driven equipment without changing the
speed of the motor. True or false

Ans True

S-8 In a step down transformer for a given load the current in the primary will be less than
the current in the secondary. True or false

Ans True

S-9 A rise in conductivity of boiler feed water indicates a rise in ____ level of feed water.

Ans TDS

S-10 For two pumps to be operated in parallel their ______heads should be the same

Ans Shut off (or ’closed discharge valve’ heads)


…………. End of Section - I ………….

Section - II: SHORT NUMERICAL QUESTIONS Marks: 2 x 5 = 10

(i) Answer all Two questions


(ii) Each question carries Five marks

L-1 A shell-and-tube heat exchanger with 2-shell passes and 8-tube passes is
used to heat ethyl alcohol (cp = 2670 J/kgoC) in the tubes from 25 oC to 70oC
at a rate of 2.1 kg/s.
The heating is to be done by water (cp = 4190 J/kgoC) that enters the shell
side at 95oC and leaves at 45oC.
The LMTD correction factor for this heat exchanger is 0.82
If the overall heat transfer coefficient is 930 W/m2oC, determine the flow rate
of water in kg/s and surface area of the heat exchanger in m2.
Paper 4 – Set B with Solutions

Ans Heat duty


Cold fluid = Qcold = 2.1 x 2670 x (70-25)
= 252315 Watts
= 252.315 kW ……………….1 mark
Hot fluid (water) Qhot = mw x 4190 x (95 -45)
= mw x 209500 kJ/s
= (209500 mw ) Watts
= (209.5 mw) kW ……………….1 mark
Qcold = Qhot
252.315 kW = (209.5 mw) kW
mw = 1.204 kg/s
LMTD = (95-70) –(45-25) / {ln (95-70)/ (45-25)}
= 22.42 oC
Corrected LMTD = 0.82 x 22.42
= 18.38 oC ……………….2 marks
Q = UA LMTD
A = 252315 / (930 x 18.38)
= 14.76m2 ……………….1 mark
L-2 In a Process Industry the L.P and H.P boilers have the same efficiency of
83%. The operating parameters and data are given below:

Boiler L.P. (Low Pressure) H.P. (High Pressure)


Efficiency on G.C.V. 83% 83%
Fuel Furnace Oil Furnace Oil
G.C.V. 10,000 kcal/kg. 10,000 kcal/kg.
Steam enthalpy 666 kcal/kg. 737 kcal/kg.
Feed water temperature 85oC 95oC

The cost of steam from L.P boiler is Rs. 3000 per tonne. Find out the cost of
steam from H.P boiler.

Ans % Boiler Efficiency = (TPH of Stm) x 1000 x (Enth of Stm – Enth of FW) x 100
(Mass of Fuel x GCV Fuel)

Evaporation ratio of LP Boiler; ER LP = 0.83 X 10000 = 14.29


(666 – 85)
Paper 4 – Set B with Solutions

……………….1.5 marks
Evaporation ratio of HP Boiler; ER HP = 0.83 X 10000 = 12.93
(737 – 95)
……………….1.5 marks
ER HP is less than ER LP ;
Thus, the specific fuel consumption (kg fuel / kg steam) is more in the case of
the HP boiler than in the case of the LP boiler.
Therefore, the cost of steam from HP boiler is higher than the cost of steam
from LP boiler.
HP Steam Cost = 14.29x 3000 = Rs.3315.5 per tonne
12.93
……………….2 marks

Or
1 T of FO – 14.29 T of LP steam
Cost of LP steam – Rs.3000/T
cost of 1 T of FO= Rs.3000 x 14.29 = Rs.42870/-
……………….1 mark
1 T of FO – 12.93 T of HP steam
cost of 1T of HP steam = Rs.42870/12.93 = Rs.3315.5/T
……………….1 mark

…………. End of Section - II …………


Section - III: LONG NUMERICAL QUESTIONS Marks: 4 x 20 = 80

(i) Answer all Four questions


(ii) Each question carries Twenty marks

N-1 The monthly energy consumption for 30 days operation in a 25 TPD (Tonne Per Day) ice plant
producing block ice is 36,950 kWh. The ice plant produces 15 Tonnes of block ice daily by
freezing 16.5 m3 of water at 30oC. The higher water consumption is due to loss of ice while
removing the block ice from ice cans for customer delivery. The following data have been
given:

o
Temperature of ice block = (-) 8 C
Latent heat of freezing of ice = 80 kcal/kg.
o
Specific heat of water = 1 kcal/kg C
o
Specific heat of ice = 0.5 kcal/kg C
Energy consumption in the = 85% of the total energy consumption
ice plant chiller compressor

Efficiency of compressor motor = 88%


Paper 4 – Set B with Solutions

Estimate

a) Energy consumption per tonne of ice ‘output’,


b) Total daily cooling load in kcals for freezing water into ice blocks
c) Refrigeration load on the chiller in TR (Tonne refrigeration) and
d) E.E.R. of ice plant chiller compressor.

The Management intends to pre-cool the inlet water from 30 oC to 12oC using a separate
water chiller drawing 0.8 kW/TR.

e) Find out the reduction in energy consumption per tonne of ice block output
f) % reduction in the condenser heat load of the plant chiller due to the use of pre-cooled
water.

Assume absolute auxiliary energy consumption of the plant remains same and only
consider water chiller compressor energy consumption for estimating the savings.
a) Monthly energy consumption = 36950 kWh
Ans Daily energy consumption = 36950 / 30 = 1231.67 kWh
 Energy consumption per tonne of ice delivered = 1231.67/15
= 82.11 kWh/tone
……………….3 marks
b)
3
Quantity of water input for the production 16.5 m = 16500 kg.
(sp.wt of water = 1000 Kg./m3)

Total cooling load per day


Q = Q1 + Q 2 + Q 3
o o
Q1 = Heat removed from lowering temperature from inlet 30 C to 0 C in kcals
o
Q2 = Latent heat removed in freezing water to ice at 0 C in kcals
o o
Q3 = Heat removed for sub-cooling of ice from 0 C to -8 C in kcals

Q = (16,500 X 1 X (30-0)) + (16,500 X 80) + {16,500 X 0.5 X [0 – (-8)]}


= 4,95,000 + 13,20,000 + 66,000

Total Daily Cooling Load = 18,81,000 kcals ……………….3 marks

c) 18,81,000
Refrigeration load on the Chiller = ------------ = 25.92 TR
24 X 3024
……………….3 marks
d)

Ice plant chiller consumption per day = 0.85 X 1231.67


1046.92 kWh
Paper 4 – Set B with Solutions

Ice plant auxiliary consumption per day = 1231.67 – 1046.92


184.75 kWh
Power consumption of the chiller = 1046.92 / 24
43.62 KW
Input KW/TR Ice Plant chiller = 43.62 / 25.92
1.683
Motor Efficiency = 88%
Input power to the ice plant compressor = 0.88 X 1.683
1.48 kW / TR
E.E.R. ice plant chiller (3024)kcal/hr/(1.48X860) kcal/hr
2.376
……………….4 marks
e) reduction in energy consumption per tonne of ice delivered

Condenser heat rejection load in the existing = QE + QC


case Q1
(25.92 X 3024) + (25.92 X 1.48 X 860)
111373.1 kcals/hr
o 16500 X 1 X (30 – 12)/ 24 X 3024
Refrigeration load for pre-cooling from 30 C
o
to 12 C in a separate water chiller
4.09 TR
Energy consumption in water chiller = 0.8 X 4.09 X 24 = 78.53 kWh
Reduced ice plant chiller load = 25.92 – 4.09 = 21.83 TR
Energy consumption for the plant chiller = 21.83 X 1.683 X 24 = 881.76 kWh
Total energy consumption per day by Energy consumption in ice plant chiller+
resorting to pre-cooling of inlet water in a Auxiliaries in ice plant
separate water chiller is (no change) + Energy consumption in
water chiller for pre-cooling
= 881.76 + 189.75 + 78.53
1150 kWh/day
Reduction in energy consumption = (1231.67 – 1150) /15
Kwh/tone
for ice delivered
5.44
……………….4 marks
f)
Heat rejection load in the ice plant = (21.83 X 3024) + (21.83 X 1.48 X 860)
condenser
93799.14 kcal/hr
% reduction in ice plant condenser heat = (1,11,373.1 –93799.14)x100 /(1,11,373.1)
load
15.78 %
……………….3 marks
Paper 4 – Set B with Solutions

N-2 In a Petrochemical Industry a gas turbine cogeneration system comprising of 20 MW


gas turbine generator along with a waste heat boiler (WHB) of 70 Tonne per hour
capacity at 10 kg/cm2 (g) are operated to meet the power and steam requirements.
The existing operating data is given below:

Power supplied by the Cogenerator = 16000 KW


Power drawn from the grid = 1500 KW
Grid power cost = Rs 5 /kwh
2
Steam at 10 Kgf/cm g supplied by WHB = 48 Tonne/hr
(without supplementary fuel firing)
Efficiency of gas turbine on G.C.V. = 28%
Efficiency of generaror = 95%
G.C.V. of fuel (Natural Gas) = 13000 Kcal/Kg
3
Density of natural gas = 0.7 Kg./m
3
Cost of natural gas = Rs.25/m
Temperature of gas turbine exhaust gas
o
entering WHB = 515 C
o
Specific heat of exhaust gas 0.3 Kcal/Kg C
o
Ambient temperature = 30 C
Air to natural gas ratio for gas turbine combustion = 60:1
Enthalpy of steam at 10 Kgf/sq.cm.g = 665 Kcal/Kg
Enthalpy of feed water = 105 Kcal/Kg

a) Find out the heat rate of the gas turbine generator and

b) Estimate the efficiency of the waste heat boiler.

The plant personnel claim and believe that by resorting to supplementary fuel firing to
increase steam generation in the WHB. is likely to improve its efficiency by 1.5 % points.

c) Determine if it is economical to generate additional steam requirement of 10 Tonne


per hour by supplementary fuel firing in WHB. as against in a separate natural gas fired
smoke tube boiler of 80% efficiency on G.C.V.

The plant operations are steady and continuous with 8600 yearly hours of operation.

Ans a)

Efficiency of gas turbine generator = 28%


 Heat Rate = 860 / 0.28
= 3071.43 kcal/kWh
……………….4 marks
Paper 4 – Set B with Solutions

b)
Gas Rate = 3071.43 / 13000
= 0.236 kg.Natural gas/kWh
………… …….2 marks
Power generated by Gas turbine = 16000 kW
Steam supplied by WHB = 48000 kg./hr
 power to Steam ratio = 3 kW / kg. steam
Air to fuel ratio of gas turbine combustion= 60 : 1
Exhaust gas per Kg. of natural gas fired = 60 + 1 = 61 kg. per kg of natural
gas

48000 (665 – 105)


Efficiency of waste heat boiler = --------------------------------------- =75.5%
(without supplementary fuel firing) 16000 X 0.236 X 61 X 0.3 X 515

……………….4 marks
c)

Efficiency of WHB with supplementary firing (as per claim) = 75.5 + 1.5
= 77%

Additional gas consumption for meeting 10 Tonne/hr steam through


supplementary firing in WHB =

10000 (665 – 105)


= ---------------------- = 559.44 kg./hr.
0.77 X 13000

10000 (665 – 105)


Gas consumption in separate gas fired boiler with 85% on GCV = -----------------------
0.85 X 13000

= 506.79 kg/hr
……………….5 marks
Operating separate gas fired boiler is economical.
Saving in gas consumption by meeting additional steam through gas
fired boiler =
= 559.44 – 506.79
= 52.65 kg/hr
= 52.65 / 0.7
3
= 75.21 m /hr
Paper 4 – Set B with Solutions

Yearly monetary savings = 75.21 X 25 X 8600


= Rs.1,61,70,150
= Say Rs.16.17 million
……………….5 marks
N3 A Process Industry is operating a natural gas fired boiler of 10 tonnes/hr to cater to a
steam load of 8 tonnes/hr at 10.5 kg/cm2(g). The O2 in the flue gas is 4 % and the exit
flue gas temperature is 180oC. Due to increased cost of natural gas the management
has decided to revert to operating the furnace oil fired boiler having an efficiency of 82%
on G.C.V. for meeting the above load.

In keeping with its sustainability policy the management proposes to offset the additional
CO2 emissions due to the use of furnace oil by sourcing a part of its total electrical
energy consumption from green power (wind source).

The following is the additional data.

Composition of fuels (% by weight)

Constituents Natural gas Furnace oil

Carbon 73 84
Hydrogen 23 11
Nitrogen 3 0.5
Oxygen 1 0.5
Sulphur - 4

G.C.V. of natural gas - 13000 kcal/kg


Enthalpy of steam at 10.5 kg/cm2(g) - 665 kcal/kg.
o
Inlet feed water temperature - 90 C
Heat loss due to Radiation and moisture in air - 1.2%
o
Specific heat of flue gases - 0.29 kcal/kg C
o
Specific heat of super heated water vapour - 0.45 kcal/kg C
G.C.V. of furnace oil - 10,000 kcal/kg
o
Ambient temperature -. 30 C

Substitution of 1 kwh of green electrical energy in place of grid electricity reduces 0.80
kg. of CO2
Determine the monthly amount of green electrical energy from wind, (for 700 hours
operation) required to be purchased to maintain the existing level of CO 2 emissions.

Ans Theoretical air required = 11.6 C + [34.8 (H2 – O2/8)] + 4.35 S


= 11.6 x 0.73 + [34.8 (0.23 – 0.01/8)]
Kg.air/Kg.gas
= 16.43 kg. air / kg. gas
Paper 4 – Set B with Solutions

Excess Air % =% O2 / (21 - % O2) x100


=(4 / 21 – 4)
=23.5 %
Actual Air Supplied (AAS) =(1 + 0.235) X 16.43
=20.29 kg.air / kg.gas
……………….3 marks
Mass of dry flue gas mdfg = mass of combustion gases due to presence
C,N,S + mass of N2 in the fuel + mass nitrogen
In air supplied + mass of excess O2 in flue gas

= (0.73 X 44/12) + 0.03 + (20.29 X 0.77) + (20.29–


16.43) X 0.23
= 19.217
= Say 19.22 kg. dry flue gas / kg. gas
……………….2 marks
*(Mair+Mfuel) ie (20.29+1) = 21.29 may also be considered.

L1 = % heat loss due to dry flue gases


MdfgX Cp X (Tq – Ta)
= ------------------------- X 100
G.C.V. of fuel

19.22 X 0.29 X (180 – 30)


= ------------------------------ X 100 = 6.43%
13000
……………….2 marks
L2 = Loss due to presence of hydrogen forming water vapour

9 H [584 + Cps (Tq – Ta)]


= ------------------------------- X 100 = 10.37%
13000
……………….2 marks
Heat loss due to Radiation and moisture in air = 1.2% ...............Given

Efficiency of natural gas boiler on G.C.V. = 100 – [6.43 + 10.37 + 1.2]

= 82%

Steam Load = 8 tonne/hr.

8000 (665 – 90)


Amount of Gas required = -------------------- = 431.52 kg./hr.
0.82 X 13000

……………….2marks
Paper 4 – Set B with Solutions

Amount of CO2 emission with natural gas = 431.52 X 0.73 X 3.67 = 1156.1 kg/hr.

8000 (665 – 90)


Amount of furnace oil required for the= -------------------- = 560.97 kg./hr
same steam load 0.82 X 10000
……………….2 marks
Amount of CO2 emission with F.O = 560.96 X 0.82 X 3.67 = 1688.15 kg. CO2/hr
……………….2 marks
(Note: 1 Kg. Carbon Combustion emits 3.67 Kg. CO2)

Increase in CO2 emission due to switching =1729.39 – 1156.1 = 532.05 kg. CO2/hr.
from natural gas to furnace oil
……………….2.5 marks
[Substituting 1 kWh grid (Thermal) electrical energy by green electrical energy
reduces 0.80 Kg. of CO2)

Green energy to be purchased to offset higher CO 2 emissions per month


= 532.05 X (1/0.8) X 700 =465543.75 kWh
…………….2.5 marks
N-4 Answer any one of the following
A) In a secondary steel manufacturing unit, steel scrap is melted in an arc furnace. The
molten metal is then taken for ladle refining followed by vacuum degassing, before being
cast into ingots.

After the ingots are cooled down to ambient temperature, the entire lot is loaded in a
batch forging furnace and heated to 1150 oC. The heated ingots are forged into desired
shapes. The monthly numbers of batches are 160.
The management has decided to improve energy efficiency of the system by
incorporating a holding furnace ( electric resistance furnace) in between the electric arc
furnace and the fuel fired forging furnace, in order that the hot ingots ( after casting)
could directly fed into the intermediate holding furnace to maintain temperature and be
fed at high temperature to the forging furnace, instead of at atmospheric temperature.

Following are the data obtained in the energy audit study of the unit.

1. Scrap material fed into the arc furnace = 10 tons per heat

2. Yield of ingot casting from scrap = 95%

3. Temperature of casting after removal of mould = 600 oC

4. Ambient temperature = 30 oC

5. Specific heat of steel = 0.682 kJ/ kgoC


Paper 4 – Set B with Solutions

6. Efficiency of forging furnace = 25 %

7. Calorific value of Furnace oil fuel = 10500 kcal/ kg

8. Specific gravity of F.O = 0.9

9. Yield of forged steel in forging furnace = 97 %

10. Melting point of steel = 1650 0C

11. Latent heat of melting of steel = 272 kJ/kg

12. Electrical energy consumption measured per ton of steel melted = 850 kWh

13. Electrical energy consumption for holding ingots at 600 oC in electric furnace
= 85 kWh per batch

14. Cost of electricity = Rs.6 /kWh

15. Cost of Furnace oil = Rs. 30,000 / ton

Calculate

a. Efficiency of electric arc furnace ignoring heat loss due to slag

b. Specific oil consumption in litres per ton of finished forged product.

c. Annual net savings in energy cost by holding the hot forged casting in an intermediate
electric furnace at 600oC before feeding into forging furnace.

Ans a) Efficiency of the arc furnace.

Theoretical heat required for melting one ton of steel

= 382.45 kWh per ton of molten steel


……………….3 marks
Paper 4 – Set B with Solutions

Efficiency = 382.45 x 100 /850 = 45 %


……………….2 marks
b) Specific oil consumption in liters per ton of finished forged product from the
forging furnace

Amount of material heated in forging furnace


= 10,000 kg x (0.95) = 9500 kg steel / batch
Oil consumption = 9500 x (0.682 /4.18) x (1150-30) / (10500 x 0.25)
= 661.3 kg FO ……………….3 marks
Amount of material forged = 9500 kg x (0.97) = 9215 kg steel / batch
Specific oil consumption = 661.3 kg FO / 9.215 tons steel = 71.76 kg FO/ton
= 71.76 / 0.9 = 79.73 Lts FO / ton of forged steel
……………….3 marks
c) Net Savings in energy cost by holding the hot forged casting in an intermediate
electric furnace at 600oC before feeding into forging furnace
Oil consumption = 9500 x (0.682 /4.18) x (1150-600) / (10500 x 0.25)
= 324.76 kg FO per batch ……………….2.5 marks
o
Additional electrical energy consumption for holding ingots at 600 C
= 85 kWh per batch
Reduction in FO consumption by hot charging the forge furnace
= 661.3 - 324.76 = 336.54 kg FO per batch
……………….2.5 marks
Net savings in energy cost = (336.54 x 30) – (85 x 6) = Rs. 9586.2 per batch
Annual net savings in energy cost = 9586.2 x 160 x 12 = Rs.184,05,504 /yr
……………….4 marks
B) The heat balance of a stenter in a textile industry is given below:

Heat used for Drying = 48%


Heat loss in exhaust air = 42%
Heat loss through insulation = 6%
Heat loss due to air infiltration = 4%

The above stenter is drying 75 meters per min. of cloth to final moisture of 7 % with inlet
o o
moisture of 50%. Temperature of cloth at inlet and outlet is 25 C and 75 C respectively.

The hot air for drying in the stenter is heated by thermic fluid. The thermic fluid heater
is fired by furnace oil , having an efficiency of 82%. The following data has been given:

Density of furnace oil = 0.95 Kg/litre


GCV = 10000 kcal/kg
Cost of furnace oil = Rs.24 per litre
Paper 4 – Set B with Solutions

Weight of 10 mts of outgoing dried cloth = 1 Kg

a) Find out the existing furnace oil consumption for stenter drying.
b) What will be the annual furnace oil savings and annual monetary saving if the
overall thermal efficiency of the stenter is improved by reducing the combined
thermal insulation loss and the loss due to air infiltration, by half, for operations at
20 hours per day and 330 days per year.

Ans Stenter speed = 75 meters / min


Dried cloth output = 75 x 60 /10
= 450 kg/hr
Weight of bone dry cloth per hr. = 450 x 0.93
i.e. W = 418.5 kg./hr
……………….2.5 marks
Weight of outlet moisture per kg. of bone dry cloth
mo = (450 – 418.5) / 450
= 0.0753 kg/kg
……………….2.5 marks
Inlet moisture = 50%
Inlet wet cloth flow rate = 418.5/ 0.5 = 837 kg/hr
mi inlet moisture per kg.
of bone dry cloth = (837 - 418.5) / 418.5
mi = 1 kg/kg bone dry cloth
Heat load on the dryer = W x (mi – mo) x [(Tout – Tin) + 540]
kcal/hr
Tout = Outlet cloth temperature
o
= 75 C
Tin = Inlet cloth temperature
o
= 25 C

Heat load on the dryer = 418.5 x (1 – 0.0753) x [(75 – 25) + 540]


= 2,28,322.3 kcal/hr

……………….2.5 marks
Based on heat balance, dryer efficiency is 48%.
Heat input to the dryer = 228322.3 / 0.48
= 4,75,671.46 kcal/hr

Furnace oil consumption in = 4,75,671.46 / (0.82 x 10000)


Thermic fluid heater
= 58.01 kg./hr
……………….2.5 marks
Paper 4 – Set B with Solutions

After reducing insulation and air infiltration loss by half, the heat energy input
will reduce by 100% – 0.5 (6 + 4)% = 95%

Dryer efficiency will increase to = (48/0.95) x 100


= 50.52%

Furnace oil consumption with = 2,28,322.3 / (0.5052 x 0.82 x 10000)


improved dryer efficiency
= 55.12 kg/hr
……………….4 marks
Saving fuel consumption due to = 58.01 – 55.12
improved dryer efficiency
= 2.89 kg/hr
Annual Furnace oil savings = 2.89 x 20 x 330
= 19074 kgs/year

……………….3 marks
Annual monitory savings = 19074 x (1/0.95) x 24
= Rs.4,81,870/year
……………….3 marks
Note:
If candidates had done the calculation with temperature of cloth at inlet at 75 oC and outlet at
25oC. the marks can be awarded according the steps.
C) In a cement kiln producing 4500 TPD of clinker output, the grate cooler hot exhaust air
temperature is vented to atmosphere at 275oC.

It is proposed to generate hot water from this waste exhaust for operating a Vapour Absorption
Machine (VAM) chiller. This will replace the existing Vapour Compression Chiller (VCR) of 50
TR capacity used for air-conditioning of control rooms and office buildings.

The following are the data:


 Diameter of the cooler vent :2m

 Velocity of cooler exhaust air : 18.6 m/s

 Density of cooler exhaust air at 275 oC : 0.64 kg / m3

 Existing VCR Chiller Specific power consumption : 0.88 kW/TR

 Existing VCR condenser water pump power consumption : 3.1 kW

 Investment towards 50TR VAM & its associated system : Rs 30 lakhs

 CoP of VAM system : 0.75


Paper 4 – Set B with Solutions

 Power consumption of VAM auxillaries : 2.83 kW

 Temperature of circulating hot water of VAM generator : Inlet - 90 oC;


Outlet - 80 oC
 Specific heat of exhaust cooler air : 0.24 kcal/ kg oC

 The efficiency of all pumps and their drive motors are 75% & 90% respectively.

 The cost of electricity : Rs.6/kWh

 No of hours of operation : 8200 hrs/ yr

Calculate
a) Cooler Exhaust air temperature after heat recovery
b) Payback period by replacement of VCR by VAM

Ans a) Cooler Exhaust air temperature after heat recovery

 Area of the duct = = 3.14 x (2/2)^2 = 3.14 m2

 Volume of cooler exhaust air2750C = 3.14 x 18.6 = 58.4 m3/s = 2,10,240 m3/h

 Mass flow rate of cooler exhaust air2750C mcxa= 210240 x 0.64 = 134553 kg/ hr

Capacity of existing chiller = 50 TR

 Cooling load = 50 x 3024 = 151200 kcal/ hr

CoP of VAM = 0.75

= (Cooling Load / Heat Input)

 Heat Input to VAM generator = 151200 / 0.75 = 201600 kcal/hr

201600 kcal/hr = mhw x Cp-hw x (90 oC -80 oC)

 Hot water flow rate mhw = 201600 / (1 x 10) = 20160 kg/hr

 Heat input to VAM generator = Heat recovered from Cooler Exhaust Air (m cxaxCp-cxax (275-To)

 Cooler Exhaust air temperature after heat recovery


To = 275- [201600 / (134553 x 0.24)]
= 268.76 oC
………………5 marks
b) Payback period by replacement of VCR by VAM

Hot water circulation pump capacity


Paper 4 – Set B with Solutions

 motor input power Pm = mhw x head developed x 9.81 / (1000 x Pump η x motor ηm)
Pm= [(20160 /3600) x 20 x 9.81/ (1000 x 0.75x 0.9)] = 1.63 kW
Heat load in the cooling tower = heat load from chilled water + heat load from
generator hot water
= 151200 + 201600 = 352800 kcal/ hr
 Condenser water circulation rate = 352800 / 5 = 70560 kg / hr

……………….3 marks

Condenser water circulation pump capacity


 motor input power Pm = mhw x head developed x 9.81 / (1000 x Pump η x motor ηm)
Pm= [(70560 /3600) x 20 x 9.81/ (1000 x 0.75x 0.9)] = 5.69 kW
……………….4 marks
Savings
 Existing VCR Chiller Specific power consumption = 0.88 kW/TR

 Existing VCR Chiller total power consumption = 50 x 0.88 = 44 kW

 Existing VCR condenser water pump power consumption = 3.1 kW

 Total Energy Saving = Existing VCR Chiller total power consumption – (Proposed
VAM chiller power consumption)

= (44+3.1) – (1.63+2.83+5.69) = 36.95 kW

……………….5 marks

 Annual Energy savings = 36.95 x 8200 = 302990 kWh/yr

 Annual Monetary savings = 302990x 6 = Rs. 18.18 Lakhs /y

 Investment towards 50TR VAM & its associated system = Rs 30 lakhs

 Simple payback period = 30 / 18.18 = 1.65 yrs or 19.8 months

……………….3 marks

D) A steam power plant consisting of high pressure Turbine (HP Turbine) and low pressure Turbine
(LP Turbine) is operating on Reheat cycle (schematic of power plant is represented below).

Steam from Boiler at a pressure of 150 bar(a) and a temperature of 550 0C expands through the
HP Turbine. The exhaust steam from HP Turbine is reheated in a reheater at a constant
pressure of 40 bar (a) to 550 0C and then expanded through LP Turbine. The exhaust steam from
LP Turbine is condensed in a condenser at a pressure of 0.1 bar (a).

The isentropic efficiencies of HP Turbine and LP Turbine are same and is 90%. The generator
efficiency is 94%

The other data of the power plant is given below:


Paper 4 – Set B with Solutions

Main steam flow rate : 228 TPH


Enthalpy of main steam : 3450 KJ/kg
Enthalpy of feed water : 990.3KJ/kg
Isentropic Enthalpy of cold reheat steam : 3050 KJ/kg
Enthalpy of hot reheat steam : 3560 KJ/kg
Condenser pressure and temperature : 0.1 bar(a) and 45.80C
Isentropic enthalpy of LP Turbine exhaust steam : 2300 KJ/kg
Enthalpy of dry saturated steam at 0.1 bar(a) and 45.8 0C : 2584.9KJ/kg
Enthalpy of water at 0.1 bar(a) and 45.8 0C :191.9 KJ/kg

Based on the above data calculate the following -:


(a) Power developed by the Generator
(b) Turbine heat rate
(c) Turbine cycle efficiency
(d) Specific steam consumption of turbine cycle.

Ans (a) Power developed by the Generator: Turbine output x Generator efficiency------------ (1)
Turbine output = Q1 (H1 – h2) + Q2(H3 – h4)/860 MW ---------------------------------------(2)
Where, Q1=main steam flow rate =228 TPH
H1=main steam enthalpy=3450 KJ/Kg
h2=actual enthalpy at HP Turbine outlet= ?(cold reheat enthalpy)
Q2=steam flow through reheater=228TPH
H3=enthalpy of hot reheat steam=3560 KJ/kg
h4= actual enthalpy of LP turbine exhaust steam=?

HP Turbine isentropic efficiency= Actual enthalpy drop/isentropic enthalpy drop


0.9= (H1- h2)/(H1-h2is) , h2is=isentropic enthalpy of cold reheat
Steam=3050KJ/kg
0.9= (3450 –h2)/(3450—3050)
h2= 3090KJ/kg
Paper 4 – Set B with Solutions

LP Turbine isentropic efficiency= (H3—h4)/(H3—h4is), h4is=isentropic enthalpy of LP Turbine


Exhaust steam=2300KJ/kg
0.9=( 3560-h4)/(3560—2300)
h4= 2426 KJ/kg

Substituting the values in equation-2,we get

Turbine output = 228(3450—3090) + 228(3560—2426)/3600 =94.62MW


Generator output= 94.62 x 0.94= 88.94 MW--------------------ANSWER (9 MARKS)

(b) Turbine heat rate=Q1 (H1—hfw) +Q2(H3—h2)/Generator output =KJ/kwhr-------------(3)


hfw=enthalpy of feed water=990.3KJ/kg
Substituting the values in the above equation-3, we get

Turbine heat rate=228 (3450—990.3) + 228(3560—3090)/88.94


=7510.4 KJ/kwhr------------------------------ANSWER (5 MARKS)

(C) Turbine cycle efficiency= 860 x4.18/Turbine heat rate


=860 x4.18 /7510.4=47.8%------------------ANSWER (3MARKS)

(d) Specific steam consumption of cycle=Steam flow/generator output


=228/88.94=2.56 tons/MW hr-------ANSWER (3MARKS)

-------- End of Section - III ---------


SUPPLEMENTARY Paper 4 – Set A

Regn No: __________________

Name : __________________
(To be written by the candidate)

16th NATIONAL CERTIFICATION EXAMINATION


FOR
ENERGY MANAGERS & ENERGY AUDITORS – September, 2016

PAPER – 4:Energy Performance Assessment for Equipment and Utility Systems

Date: 25.09.2016 Timings: 14:00-16:00 HRS Duration: 2 HRS

General instructions:
o Please check that this question paper contains 6 printed pages
o Please check that this question paper contains 16 questions
o The question paper is divided into three sections
o All questions in all three sections are compulsory
o All parts of a question should be answered at one place

Section - I: BRIEF QUESTIONS

S-1 A pump handling water is now handling brine at same flow and head. Will the power
consumption increase or decrease or remain the same
Ans Increase
S-2 If the steam generation in a boiler is reduced to 50%, the radiation loss from the
surface of boiler will reduce by the same ratio. True or false
Ans False
S-3 The speed of an A.C. induction motor is inversely proportional to number of poles.
True or False
Ans True
S-4 If the speed of a centrifugal fan is reduced with a VFD, the power drawn will reduce as
cube of speed. True or False
Ans True
S-5 A fluid coupling varies the speed of the driven equipment by varying the speed of the
motor. True or False
Ans False
S-6 With increase of steam pressure, the enthalpy of evaporation and specific volume
increases. True or False
Ans False
S-7 A pump is retrofitted with a VFD and operated at full speed. Will the power
consumption increase or decrease?
Ans Increase
S-8 Which parameter in the proximate analysis of coal is an index of ease of ignition.
Ans Volatile matter
S-9 The major source of heat loss in a coal fired thermal power plant is through flue gas
SUPPLEMENTARY Paper 4 – Set A

losses in the boiler. True or false


Ans False
S-10 With evaporative cooling, it is possible to attain water temperatures below the
atmospheric wet bulb temperature. True or False
Ans False

…………. End of Section - I ………….

Section - II: SHORT NUMERICAL QUESTIONS

L-1 In a petrochemical industry the LP & HP boilers have the same evaporation ratio of
14 using the same fuel oil. The operating details of LP & HP boiler are given below:
Particulars LP Boiler HP Boiler
Pressure 10 Kg./cm2a 32 Kg./cm2a
Temperature Saturated Steam 400oC
Enthalpy of steam 665 Kcal/kg 732 Kcal/kg
o
Enthalpy of feed water 80 C 105oC
Evaporation Ratio 14 14

Find out the efficiency of HP boiler if the LP boiler efficiency is 80%.


Ans Effy ᶯ = ER. (hg – hf) / GCV
EffyL.P ᶯ1 = 0.8 = 14 x(665 – 80) / GCV
EffyH.P ᶯ2 = 14 x(732 – 105) / GCV
EffyH.P ᶯ2 / EffyL.P ᶯ1 = (732 – 105)0.8 / (665 – 80) = 0. 8574 =85.74%

Or

EffyL.P ᶯ1= 0.8 = 14 x(665 – 80) / GCV


GCV= 14x(665-80) / 0.8 = 10237.5kcal/kg
EffyH.P ᶯ2 = 14 x(732 – 105) / GCV
= 14 x(732 – 105) / 10237.5 = 0.8574 = 85.74%

L-2 The following sketch shows the details of an installed pumping system. The rated
parameters of the pump are:

Flow (Q) : 30 lps


Head (H) : 20 m
Power (P) : 10 kW
Efficiency (ηP) : 65%
SUPPLEMENTARY Paper 4 – Set A

Dia: 3”

6m
NRV

Pump

2m

3m
1m

Under normal operating conditions,


1. What will be the total head delivered by the pump if pressure drop across
the NRV is 0.1 kg/cm2
2. What will be the impact on flow rate and power consumption of this pump
due to above operation condition?
Ans
1. Operating head = Discharge head (6 m) - Suction head (- 4 m) + Head loss
in NRV (1m)
= 11 m.
2. Actual flow rate from the pump will be higher than the rated flow rate due to lower
operating head.
3. Actual power consumption will increase due to higher flow rate.
4. Pump operating efficiency will be less than the design efficiency under actual
conditions.

…………. End of Section - II ………….

Section - III: LONG NUMERICAL QUESTIONS

N-1 In a plant, a single cell 3000 millionCal/hr, cooling tower with one CW pump is operated for
cooling water system. The operating parameters are tabulated as below.

S.No Parameter Before After refurbishment


refurbishment
1 CW inlet temp to CT 35oC 35oC
SUPPLEMENTARY Paper 4 – Set A

2 Atmospheric air conditions WbT -25 oC, DbT - 38 WbT -25 oC, DbT -
o
C 38 oC
3 COC 3.5 5
4 Suction head of CW pump -1m -1m
5 Discharge pressure of CW 4kg/cm2 4kg/cm2
pump
6 Efficiency
CW Pump 54% 53%
CW Pump motor 89% 89%
CT fan 55% 54%
CT fan motor 90% 90%
7 Pressure developed by CT 20mmwc 20mmwc
fan
8 Approach 4oC 3oC
9 L/G ratio 1.5 1.5
10 Density of air 1.29kg/m3 1.29kg/m3

The cooling tower is refurbished as a result of which the effectiveness has increased to 70 %.
Also with improved water treatment the COC is increased to 5.
Find out
1. Reduction in power consumption of pump and fan due to improvements in cooling tower.
2. Reduction in make up water consumption ignoring drift losses in KL/day

Ans
Paramet Equation / formulae Before After
er refurbishment refurbishment
approach = TCW0-WbT =4+25 =29oC =3+25 =28oC
Twco = approach+ Wbt
CW flow = heat load/( TCWi-TCW0) =(3000x106/103) / ( =(3000x106/103)/(35-
rate Q 35- 29) 28)
= 500000 kg/h = 428571 kg/h
= 500 m3/h = 429 m3/hr
Evaporati =1.8*.00085*CW flow x 1.8x0.00085x500x 1.8x.00085x429x(35
on loss Range (35-29) -28) = 4.59 m3/h
= 4.59 m3/h
Blow = Evaporation = 4.59/(3.5-1) 4.59/(5-1)
down loss Loss/(COC-1) = 1.84 m3/h = 1.15 m3/h
Total = Eva loss+ Blow down = 4.59+1.84 =4.59+1.15
water loss =6.43 m3/h =5.74 m3/h
loss
Make-up = Total water loss x = 6.43 x 24 = 5.74 x 24
water 24hrs = 154.2 m3/day = 137.76m3/day
=154.2KL/day =137.76 KL/day
Total = discharge head- = 40-(-1) = 41 mWC = 40-(-1) = 41 mWC
head H suction head
Pump = = =
LKW ((Q*1000/3600)*(H*9.8 (500*1000/3600)*(41* (429*1000/3600)*(4
1))/1000 9.81)/1000 1*9.81)/1000
= 55.86KW = 47.9 kW
SUPPLEMENTARY Paper 4 – Set A

Pump = Pump LKW/Eff.Pump =55.86/0.54 = 47.9/0.53


input =103.4 kW =90.4 kW
Motor = Pump input/motor = 103.4/0.89 =90.4/0.89
input eff =116.2 kW = 101.6kW
Air flow in =[( CW flow)x1000]/ = =
CT fan [((L/G)]*1.29) (500x1000/(1.5x1.29) (429x1000/(1.5x1.29
Qf = 258398 m3/h )
= 221705m3/h
Hf Pressure developed by = 20mmWC = 20mmWC
fan Hf
Air KW = [(Qf in m3/h)*(Hf in =(258398*20)/(3600*1 =(221705*20)/(3600
mmWC)]/(3600*102) 02) *102)
=14.07 kW = 12.08 kW
Fan =Air KW/(FanEffi =14.07/(0.55*0.9) =12.058/(0.54*0.9)
motor xMotor Eff) = 28.43kW = 24.9 kW
input
(1) Reduction in power of pump and motor = (116.2+28.43) - (101.6+24.9) = 18.13 kW

(2) Reduction in makeup water = 154.2-137.76 = 16.44 or 16.5 KL/day

N-2 An old stoker fired boiler of 24 Tonne/hr delivering 20 tonne/hr of steam on a continuous basis
was converted to atmospheric fluidised boiler (AFBC) to improve the efficiency. The existing
stoker fired boiler was operating with the following data and parameters:

The ultimate analysis of coal (fuel):


Carbon Hydrogen Sulphur Oxygen Nitrogen Moisture Ash G.C.V
40% 4% 0.6% 7% 1% 4.4% 43% 4000kCal
/kg

Operating Parameters (in both the cases):


Parameter Unit value
Steam Pressure Kg./cm2a 20
Enthalpy of Steam kCal/Kg 690
o
Steam temp C 250
Feed water enthalpy kCal/kg 95
Heat loss due to presence of H2 & Moisture % 6.5
o
Flue gas temperature C 165
o
Ambient temperature C 30
Specific heat of flue gases kCal/kgoC 0.27
Radiation and other unaccounted losses % 1.87
CO2 actual in dry flue gas % 11

Operating parameters (that have changed):


Parameter CO in flue GCV of GCV of fly Ratio of
gas Bottom Ash Ash bottom ash
SUPPLEMENTARY Paper 4 – Set A

to fly ash
units PPM kCal/kg kCal/kg
Before 700 950 500 90:10
conversion
After nil 800 400 20:80
conversion

Find out the daily reduction in coal consumption after converting to AFBC boiler.
Ans Carbon = 40%; H2 = 4%; S = 0.6%; Ash = 43%; O2 = 7%; N2 = 1%; Moisture = 4.4%

Theoretical air reqd. = [11.6 C + 34.8 (H – O/8) + 4.35 S]


Where C, H, O, S are percentages by weight per Kg of coal.
= [11.6x 40 + {34.8 (4 – 7/8)} + 4.35 x 0.6] x 1/100
= 5.754 Kg. air / Kg. coal

Mole C
% CO2 (th) = ------------------------------------
Mole N2 + Mole C + Mole S

Wt. of N2 in theoretical air Wt. N2 in Fuel


Mole N2 = ------------------------------------ + ----------------------
Mol Wt. of N2 Mol Wt. of N2

5.754 X 0.77 0.01


= ----------------- + -------- = 0.1586
28 28
Mole C = 0.4 / 12 = 0.033

Mole S = 0.006 / 32 = 0.0001875

. 0.033
. . CO2 (th) = ------------------------------------
0.033 + 0.1586 + 0.0001875

= 0.033 / 0.1916 = 0.1722 or 17.2 %

Actual CO2 measured = 11%

7900 X [(CO2 %)t – (CO2 %)a]


% EA = --------------------------------------
(CO2 %)a X (100 – (CO2 % )th)

7900 X (17.2 – 11) 49.138


= --------------------------- = -------------- = 53.8 = say 54%
SUPPLEMENTARY Paper 4 – Set A

11 X (100 – 17.20) 11 X 82.78

Actual Air Supplied (AAS) = 5.754 x (1.54) = 8.86 Kg. air / Kg. coal

Mass of Dry flue gas = Mass of combustion gases due to C, H, O & S + Mass of N2 in fuel +
Mass of N2 in AAS + Mass of oxygen in flue gas due to excess air supplied

= 0.4 X 44/12 + 0.01 + 8.86 X 0.77 + (8.86 – 5.754) 0.23 + .006 X 64/32

= 1.46 + 0.01 + 6.822 + 0.7144 + 0.012 = 9.02 Kg / Kg. coal

Tfg = 165oC

9.02 X 0.27 X (165 – 30)


*Heat loss due to dry flue gas = ------------------------------- x 100 = 8.2 %
4000

*Heat loss due to H2 & moisture in fuel = 6.5 % given

Ratio of bottom to fly ash = 90 : 10


GCV fly ash = 450 Kcal
Fly ash in Coal = 0.1 X .43 = 0.043 Kg/Kg. Coal
Heat loss due to fly ash = 0.043 X (500 / 4000) X 100
*Heat loss due to fly ash = 0.54%

Bottom ash qty. = 0.9 X 0.43 = 0.387 Kg. / Kg. Cal


*Heat loss in bottom ash = 0.387 X( 950 / 4000)x100 = 9.19 %

% CO X C 5654
CO loss = ------------------- X ----------------- X 100
% CO X % CO2a GCV of fuel

CO = 700 p.p.m
= 0.07 %

0.07 X 0.4 5654


= ------------ X --------x100
0.07 + 11 4000
*Heat loss due to partial combustion of carbon to CO = 0.357 % = 0.36%
SUPPLEMENTARY Paper 4 – Set A

*Heat loss in dfg = 8.2%


* Heat loss due to presence of H2 & M = 6.5%
* Heat loss in fly ash = 0.54%
* Heat loss in bottom ash = 9.19%
*Heat loss due to CO = 0.36%
* Radiation & other unaccounted losses = 1.87 %

Effy ᶯ = 100 – % total losses


ᶯ = 100 – [8.2 + 6.5 + 0.36 + 0.54 + 9.19 + 1.87] = 73.34%

In case of FBC boiler bottom to fly ash is 20 : 80


GCV of fly ash = 400 Kcal/Kg; GCV of bottom ash = 800 Kcal/Kg.
*Heat loss in fly ash = 0.8 X 0.43 = 0.344 Kg. fly ash / Kcal.
*Heat loss due to fly ash = 0.344 X( 400 / 4000)X100
= 3.44%

Bottom ash = 0.2 X 0.43 = .086 Kg/Kg. Cal


*Heat loss in bottom ash = 0.086 X( 800 / 4000)X100 = 1.72%

Heat loss =
*Heat loss due to dfg = 8.2 %
*Heat loss due to H2 & M = 6.5 %
*Heat loss due to fly ash = 3.44 %
*Heat loss due to bottom ash = 1.72 %
*Radiation other losses = 1.87 %
Total Loss = 21.73 %

Effy ᶯ = 100 – 21.73


Efficiency of boiler after conclusion to FBC = 78.27 %

ms (hg – hf)
Effy ᶯ = --------------
Mf X GCV

20000 (690 – 95)


mf = ----------------------
0.7334 X 4000

Coal consumption with stoker fired system = 4056 Kgs/hr = 4.06 Tonnes/hr.

20000 (690 – 95)


With AFBC conversion, mf = ----------------------
SUPPLEMENTARY Paper 4 – Set A

0.7827 X 4000

Coal consumption = 3800.9 Kg./hr = say 3.8 Tonnes / hr.

Saving in daily coal consumption = (4.06-3.8) X 24 = 6.24 Tonnes / day

N-
3
SUPPLEMENTARY Paper 4 – Set A

b. cooling water flow rate circulation in the condenser if the range is 7oC
Condenser load = 17,000 x (2633.4 – 191.8)
= 415072000kJ/hr
= 415072000 /4.18 = 9929952.15kCal/hr

At a range of 7oC cooling water flow rate = 9929952.15 / 7


= 1418.56m3/hr

N-4 Answer ANY ONE OF THE FOLLOWING among A, B, C and D

A) A utility type captive thermal power plant of 65 MW is generating an output of 60 MW at the


generator . Steam generated in the boiler at 105 kg/cm 2a and 510oC is expanded in the steam
turbine exhausting to condenser maintained at 0.1 kg/cm2a and 45.5oC. The cooling water flow
rate through the condenser is 166m3 per min. The other operating data and particulars are,
SUPPLEMENTARY Paper 4 – Set A

Enthalpy of steam at 105 kg/cm2a & 510oC = 805 Kcal/kg.


Enthalpy of steam at turbine outlet = 565 Kcal/Kg.
Enthalpy of water at condenser pressure 0.1 kg/cm2a &at 45.5oC = 45.5 Kcal/Kg.
Inlet/outlet temperature of cooling water at the condenser =26oC/38oC
The efficiency of the generator = 97%
Enthalpy of saturated steam at 10kg/cm2a = 665 Kcal/Kg.

Based on the above, find out,


a. Heat load on the condenser in million kcal/hr
b. Output of the steam turbine in KW
c. Loss in the gear box in KW
d. Condense effectiveness
e. DM water at 135oC to be sprayed for desuperheating of boiler steam after pressure reduction
to 10 kg/cm2a required for auxiliary service in kgs/tonne steam
Ans
a. Heat load on the condenser = 166 x 60 x 1000(38-26)/1000000
= 119.52 million kcal/hr

b. Inlet enthalpy of steam to condenser = outlet enthalpy of steam from turbine


Steam flowrate through the turbine = 119.52x 1000000/(565-45.5)
= 230000 kg/hr
= 230.0 tonne/hr

Inlet enthalpy to turbine = 805 kcal/kg


Steam turbine out put =230000 x (805-565)
= 64186.00 KW

c. Generator out put = 60000.0 KW


Generator efficiency =97%
Generator input =60000.00 / 0.97 = 6I,856.00 KW

Loss in gear box = 64186 .00 – 61856.00


= 2330.00 kW

d. Cooling water inlet temperature = 26oC


Cooling water outlet temperature = 38oC
Inlet steam temperature to condenser = 45.5oC
.
. . Condenser effectiveness i.e. € =
= (Cooling water temp. rise) / (Inlet steam temp. to condenser –
inlet cooling water temp.)
= (38 – 26) / (45.5 – 26)
= 0.615
SUPPLEMENTARY Paper 4 – Set A

e. Quantity of DM water to be sprayed for desuperheating


ms= mass ofhigh pressure superheated steam
mw= mass of DM water to be sprayed
ms x 805 + mw x 135 = (ms + mw) x 665

Enthalpy of 105 kg/cm2a, 510oC superheated steam 805 kcal/kg


Enthalpy of saturated steam at 10 kg/cm2a 665 kcal/kg
Enthalpy of DM water at 135oC = 135 kcal/kg assumed

ms (805 – 665) = mw (665 – 135)


mw / ms = (805 – 665) /( 665 – 135)
= 0.264 kg water / kg steam
= 264 kg water/ tonne steam

or
B) The preheater exhaust gas from a cement kiln has the following composition on dry
basis : CO2 – 23.9%, O2 – 5.9%, CO – 0.2%, remaining is N2. The static
pressure and temperature measured in the duct are -730 mmWC and 3500C
respectively. The velocity pressure measured with a pitot tube is 19 mmWC in a duct of
2800 mm diameter ( Pitot tube constant = 0.89 ). The atmospheric pressure at the site is
10350 mmWC universal gas constant is 847.84 mmWCm3/kg mol k. The specific heat
capacity of preheater exhaust gas is 0.25 kcals/kg0C.

The static pressure developed by PH exhaust fan is 630mmWC and power drawn is
1582 kW. Calculate the efficiency of fan given that the motor efficiency is 92%.

The plant has decided to install a waste heat recovery power plant with the heat rate of
5595 kcals/kWh. The temperature drop across the waste heat boiler of the power plant
is 1000C. Calculate the maximum possible power generation from this system?
Ans
Molecular weight exhaust gas (dry basis) M
= %CO2xMCO2 + %O2xMO2 + %COxMCO + %N2 x MN2
= {(23.9 x 44) + (5.9 x 32) + (0.2 x 28) + (70 x 28)}/100
= 32.06 kg/kg mole

Exhaust Gas density at operating temperature = γ = [ PM / RT ]


= [ (10350 – 730) x 32.06 ) / { 847.84 x (273+350) }
= 0.584 kg/m3

Duct Area = 3.14 x( 2.8/2)2 = 6.15 m2

Volume flow rate


= A Cp (2 x g x ∆P / γ)1/2 = 6.15 x 0.89 (2 x 9.81x 19/0.0.584)1/2
= 138.4 m3/s
SUPPLEMENTARY Paper 4 – Set A

Volume flow rate = 498194 m3/ h

Fan efficiency = volumetric flow rate x pressure developed


(102 x power drawn x motor eff)

= 138.4 x 630 x 100 = 59%


(102x1582x0.92)
Mass flow rate of preheater exhaust gas= Volume flow rate x density
= 498194*0.584 = 2,90,945 kg/hr

Heat given up to power plant by exhaust gas = 290945 x0.25 x100


= 7273625 kcals/hr

Maximum possible power generation from the WHR power plant


=7273625/5595
= 1300kW

or
C) In a textile process house the production from the stenter machine is 72000 mtrs per day. The
effective operation of stenter is 20 hours per day. The percentage moisture in the dried cloth
(output) is 6% and its temperature is 75oC and wet cloth inlet is at 25oC . The stenter is heated
by steam at 8 Kg./cm2a and the daily steam consumption for the stenter is 16.5 tonnes. The
efficiency of the stenter dryer is 47%. Calculate the
(i) Linear speed of the stenter machine
(ii) Inlet moisture
(iii) Feed rate of the stenter.

The following data have been provided


Weight of 10 meter of dried cloth = 1 kg.
Enthalpy of the steam to the stenter = 665 kcal/kg.
Enthalpy of condensate at the exit of stenter = 130 kcal/kg.
Ignore losses in start-up and stoppage.

Ans Production per day = 72000 meters


Actual hours of operation = 20 hours/ day
Linear speed of the stenter = 72000 / (20x60) = 60 meters per min

Dried cloth output = 72000 / (20x10) = 360 kg/hr.

Moisture in dry cloth = 6%


Bone dry cloth = 360 x 0.94 = 338.4 kg/hr

Moisture in outlet cloth mo = (360 – 338.4) / 338.4


SUPPLEMENTARY Paper 4 – Set A

= 0.0638 Kg./Kg. bone dry cloth

Steam consumption per day = 16.5 tonnes


= 16500 / 20 = 825 Kg./hr.

Heat load on the dryer = Energy input in steam x Dryer Efficiency


= Steam flow rate x (Enthalpy steam – Enthalpy condensate) x Efficiency Dryer
= 825 x (665 – 130) x 0.47
= 207446.3 Kcal/hr.

Further Heat load on the dryer = w x (mi – mo) X [(Tout – Tin) + 540] Kcal/hr.
w = weight of bone dry cloth rate kg/hr
mi = weight of cloth inlet moisture Kg./Kg. bone dry cloth
Tout = dried cloth outlet temperature = 75oC
Tin = wet cloth inlet temperature = 25oC

338.4 x (mi – 0.0638) X [(75 – 25) + 540] = 207446.3 Kcal/hr


mi = 1.1028 Kg./Kg. bone dry cloth(1.1028) / (1.1028+1)x100
% inlet moisture in wet cloth = 52.44 %

total moisture in inlet cloth = 1.1028x338.4= 373.2 kg/hr

feed rate(inlet cloth rate), = total inlet moisture/hr +bone dry cloth/hr
= 373.2+338.4
= 711.6 Kg./hr.

or
D) The following data are given for a commercial building.

Outdoor conditions : DBT = 37C, WBT = 26.5C, Humidity = 17.5 g of water / kg of dry air
Desired indoor conditions : DBT = 24C, RH = 55 %, Humidity = 10.2 g of water / kg of dry air

Total area of wall = 320 m2, out of which 50% is window area.
U – Factor ( Wall ) = 0.33 W/m2K
U – Factor ( Roof ) = 0.323 W/m2K
U – factor [ fixed windows with aluminum frames and a thermal break ] = 3.56 W/m2K

Other data:

 15 m x 25 m roof constructed of 100 mm concrete with 90 mm insulation & steel decking.


 CLTD at 17:00 hr : Details : Wall = 12C; Roof = 44C; Glass Window = 7C
 SCL at 17 : 00 hr : Details : Glass Window = 605 W/ m2
 Shading coefficient of Window = 0.74
 Space is occupied from 8:00 to 17:00 hr by 25 people doing moderately active work.
 Sensible heat gain / person = 75 W ; Latent heat gain / person = 55 W ; CLF for people
SUPPLEMENTARY Paper 4 – Set A

= 0.9
 Fluorescent light in space = 21.5 W/m2 ; CLF for lighting = 0.9
 Ballast factor details = 1.2 for fluorescent lights & 1.0 for incandescent lights
 Computers and office equipment in space produces 5.4 W/m2 of sensible heat
 One coffee maker produces 1050 W of sensible heat and 450 W of latent heat.
 Air changes/hr of infiltration = 0.3
 Height of building = 4 m
 Supply air dry bulb temperature is 140C

(i) Determine the building cooling load in TR


(ii) Calculate the supply air quantity to the cooling space m3/s

Ans (i) Cooling Load Determination:

I. External Heat Gain

(i) Conduction heat gain through the wall = U – factor x net area of wall x CLTD
= 0.33 x (160) x 12 ] = 633.6 W

(ii) Conduction heat gain through the roof= U – factor x net area of roof x CLTD
= 0.323 x ( 15 x 25 ) x 44
= 5329.5 W

(iii) Conduction heat gain through the windows = U – factor x net area of windows x
CLTD
= (3.56 x 160 x 7) = 3987.2 W

(iv) Solar radiation through glass


= Surface area x Shading coefficient x SCL
= (160 x 0.74 x 605) = 71632 W

II. Internal Heat Gain

(i) Heat gain from people =Sensible heat gain + Latent heat gain

Sensible heat gain = (No. of people x Sensible heat gain / person x


CLF)
=(25 x 75 x 0.9) = 1687.5 W

Latent heat gain = No. of people x Latent heat gain / person


= (25 x 55 ) = 1375 W
Therefore, Heat gain from people = (1687.5 + 1375 ) = 3062.5 W

(ii) Heat gain from lighting = (Energy input x Ballast factor x CLF)
Energy input = (Amount of lighting in space / unit area)x Floor area
= 21.5 x (15 x 25) = 8062.5 W
SUPPLEMENTARY Paper 4 – Set A

Therefore, heat gain from lighting = (8062.5 x 1.2 x 0.9) =8707.5 W

(iii) Heat generated by equipment :

Sensible heat generated by coffee maker =1050 W


Latent heat generated by coffee maker = 450 W
Sensible heat gain by computers and office equipment= 5.4 x 375 = 2025 W
Therefore, Heat generated by equipment = 3525 W

(iv)Heat gain through air infiltration = (Sensible heat gain + Latent heat gain)

Sensible heat gain =(1210 x airflow x ∆T )


Airflow = (Volume of space x air change rate ) / 3600
= { (15 x 25 x 4 ) x 0.3 } / 3600
= 0.125 m3 / s

Therefore, sensible heat gain =1210 x 0.125 x ( 37 – 24 ) =1966.25 W

Latent heat gain = 3010 x 0.125 x ( 17.5 – 10.2 ) =2746.6 W

No. Space Load Components Sensible Heat Load (W) Latent Heat Load (W)
1. Conduction through exterior wall 633.6 ---
2. Conduction through roof 5329.5 ---
3. Conduction through windows 3987 ---
4. Solar radiation through windows 71632 ---
5. Heat gained from people 1687.5 1375
6. Heat gained from lighting 8707.5 ---
7. Heat gained from equipment 3075 450
8. Heat gained by air infiltration 1966.25 2746.6
Total space cooling load 97018.35 4571.6
Total Cooling Load = 101589.4 W/ 3516 = 29TR

(ii) Supply Air Quantity Calculation:

Supply air flow = Sensible heat gain / {1210 * (Room dry bulb temperature – Supply dry bulb
temperature)}
= 97018.35 W / {1210 J/m3K*(24 – 14) 0C}
= 8.02 m3/s

-------- End of Section - III ---------


SUPPLEMENTARY Paper 4 – Set B

Regn No: __________________

Name : __________________
(To be written by the candidate)

16th NATIONAL CERTIFICATION EXAMINATION


FOR
ENERGY MANAGERS & ENERGY AUDITORS – September, 2016

PAPER – 4:Energy Performance Assessment for Equipment and Utility Systems

Date: 25.09.2016 Timings: 14:00-16:00 HRS Duration: 2 HRS

General instructions:
o Please check that this question paper contains 6 printed pages
o Please check that this question paper contains 16 questions
o The question paper is divided into three sections
o All questions in all three sections are compulsory
o All parts of a question should be answered at one place

Section - I: BRIEF QUESTIONS

S-1 If the speed of a reciprocating compressor is reduced with a VFD, the power drawn
will reduce as cube of speed. True or False
Ans False
S-2 A fluid coupling varies the speed of the driven equipment by varying the speed of the
motor. True or False
Ans False
S-3 With increase of steam pressure, the enthalpy of evaporation and specific volume
increases. True or False
Ans False
S-4 A pump handling water is now handling brine at same flow and head. Will the power
consumption increase or decrease or remain the same
Ans Increase
S-5 If the steam generation in a boiler is reduced to 45 %, the radiation loss from the
surface of boiler will reduce by the same ratio. True or false
Ans False
S-6 The speed of an A.C. induction motor is proportional to number of poles. True or
False
Ans False
S-7 A pump is retrofitted with a VFD and operated at full speed. Will the power
consumption increase or decrease?
Ans Increase
S-8 Which parameter in the proximate analysis of coal is an index of ease of ignition ?.
SUPPLEMENTARY Paper 4 – Set B

Ans Volatile matter


S-9 With evaporative cooling, it is possible to attain water temperatures below the
atmospheric wet bulb temperature. True or False
Ans False
S-10 The major source of heat loss in a coal fired thermal power plant is through flue gas
losses in the boiler. True or false
Ans False

…………. End of Section - I ………….

Section - II: SHORT NUMERICAL QUESTIONS

L-1 The following sketch shows the details of an installed pumping system. The rated
parameters of the pump are:

Flow (Q) : 30 lps


Head (H) : 20 m
Power (P) : 10 kW
Efficiency (ηP) : 65%

Dia: 3”

6m
NRV

Pump

2m

3m
1m

Under normal operating conditions,


1. What will be the total head delivered by the pump if
pressure drop across the NRV is 0.2 kg/cm2
2. What will be the impact on flow rate and power
consumption of this pump due to above operation condition?
Ans
1. Operating head = Discharge head (6 m) - Suction head (- 4 m) + Head loss
in NRV (2m)
= 12 m.
SUPPLEMENTARY Paper 4 – Set B

1. Actual flow rate from the pump will be higher than the rated flow rate due
to lower operating head.
2. Actual power consumption will increase due to higher flow rate
3. Pump operating efficiency will be less than the design efficiency under
actual conditions.

L-2 In a petrochemical industry the LP & HP boilers have the same evaporation ratio of
14 using the same fuel oil. The operating details of LP & HP boiler are given below:
Particulars LP Boiler HP Boiler
Pressure 10 Kg./cm2a 32 Kg./cm2a
Temperature Saturated Steam 400oC
Enthalpy of steam 665 Kcal/kg 732 Kcal/kg
Enthalpy of feed water 80oC 100oC
Evaporation Ratio 14 14

Find out the efficiency of HP boiler if the LP boiler efficiency is 80%.


Ans Effy ᶯ = ER. (hg – hf) / GCV
EffyL.P ᶯ1 = 0.8 = 14 x(665 – 80) / GCV
EffyH.P ᶯ2 = 14 x(732 – 100) / GCV
EffyH.P ᶯ2 / EffyL.P ᶯ1 = (732 – 100)0.8 / (665 – 80) = 0. 8643 =86.43%

Or
EffyL.P ᶯ1= 0.8 = 14 x(665 – 80) / GCV
GCV = 14x(665-80) / 0.8 = 10237.5kcal/kg
EffyH.P ᶯ2= 14 x(732 – 100) / GCV
= 14 x(732 – 100) / 10237.5 = 0.8643 = 86.43%

…………. End of Section - II ………….

Section - III: LONG NUMERICAL QUESTIONS

N- An old stoker fired boiler of 24 Tonne/hr delivering 20 tonne/hr of steam on a continuous basis was
1 converted to atmospheric fluidised boiler (AFBC) to improve the efficiency. The existing stoker fired
boiler was operating with the following data and parameters:

The ultimate analysis of coal (fuel):


Carbon Hydrogen Sulphur Oxygen Nitrogen Moisture Ash G.C.V
40% 4% 0.6% 7% 1% 4.4% 43% 4000kCal
/kg

Operating Parameters (in both the cases):


Parameter Unit value
Steam Pressure Kg./cm2a 20
Enthalpy of Steam kCal/Kg 690
SUPPLEMENTARY Paper 4 – Set B

o
Steam temp C 250
Feed water enthalpy kCal/kg 95
Heat loss due to presence of H2 & Moisture % 6.5
o
Flue gas temperature C 165
o
Ambient temperature C 30
Specific heat of fluegases kCal/kgoC 0.27
Radiation and other unaccounted losses % 2.0
CO2 actual in dry flue gas % 11

Operating parameters (that have changed):


Parameter CO in flue GCV of GCV of fly Ratio of
gas Bottom Ash Ash bottom ash
to fly ash
units PPM kCal/kg kCal/kg
Before 700 950 500 90:10
conversion
After nil 800 400 20:80
conversion

Find out the daily reduction in coal consumption after converting to AFBC boiler.
Carbon = 40%; H2 = 4%; S = 0.6%; Ash = 43%; O2 = 7%; N2 = 1%; Moisture = 4.4%
Theoretical air reqd. = [11.6 C + 34.8 (H – O/8) + 4.35 S]
Where C, H, O, S are percentages by weight per Kg of coal.
= [11.6x 40 + {34.8 (4 – 7/8)} + 4.35 x 0.6] x 1/100
= 5.754 Kg. air / Kg. coal

Mole C
% CO2 (th) = ------------------------------------
Mole N2 + Mole C + Mole S

Wt. of N2 in theoretical air Wt. N2 in Fuel


Mole N2 = ------------------------------------ + ----------------------
Mol Wt. of N2 Mol Wt. of N2

5.754 X 0.77 0.01


= ----------------- + -------- = 0.1586
28 28

Mole C = 0.4 / 12 = 0.033

Mole S = 0.006 / 32 = 0.0001875

. 0.033
. . CO2 (th) = ------------------------------------
SUPPLEMENTARY Paper 4 – Set B

0.033 + 0.1586 + 0.0001875

= 0.033 / 0.1916 = 17.2 %

Actual CO2 measured = 11%

7900 X [(CO2 %)t – (CO2 %)a]


% EA = --------------------------------------
(CO2 %)a X (100 – (CO2 % )th)

7900 X (17.2 – 11) 49.138


= --------------------------- = -------------- = 53.8 = say 54%
11 X (100 – 17.20) 11 X 82.78

.
Actual Air Supplied (AAS) = 5.754 x (1.54) = 8.86 Kg. air / Kg. coal

Mass of Dry flue gas = Mass of combustion gases due to C, H, O & S + Mass of N2 in fuel +
Mass of N2 in AAS + Mass of oxygen in flue gas due to excess air supplied

= 0.4 X 44/12 + 0.01 + 8.86 X 0.77 + (8.86 – 5.754) 0.23 + .006 X 64/32

= 1.46 + 0.01 + 6.822 + 0.7144 + 0.012 = 9.02 Kg / Kg. coal

Tfg = 165oC
9.02 X 0.27 X (165 – 30)
*Heat loss due to dry flue gas = ------------------------------- x 100 = 8.2 %
4000

*Heat loss due to H2 & moisture in fuel = 6.5 % given


Ratio of bottom to fly ash = 90 : 10
GCV fly ash = 450 Kcal
Fly ash in Coal = 0.1 X .43 = 0.043 Kg/Kg. Coal
Heat loss due to fly ash = 0.043 X (500 / 4000) X 100
*Heat loss due to fly ash = 0.54%

Bottom ash qty. = 0.9 X 0.43 = 0.387 Kg. / Kg. Cal


*Heat loss in bottom ash = 0.387 X( 950 / 4000)x100 = 9.19 %

% CO X C 5654
CO loss = ------------------- X ----------------- X 100
% CO X % CO2a GCV of fuel
SUPPLEMENTARY Paper 4 – Set B

CO = 700 p.p.m
= 0.07 %

0.07 X 0.4 5654


= ------------ X --------x100
0.07 + 11 4000
*Heat loss due to partial combustion of carbon to CO = 0.357 % = 0.36%

*Heat loss in dfg = 8.2%


* Heat loss due to presence of H2 & M = 6.5%
* Heat loss in fly ash = 0.54%
* Heat loss in bottom ash = 9.19%
*Heat loss due to CO = 0.36%
* Radiation & other unaccounted losses = 2.00 %

Effy ᶯ = 100 – % total losses


ᶯ = 100 – [8.2 + 6.5 + 0.36 + 0.54 + 9.19 + 2.00] = 73.21%

In case of FBC boiler bottom to fly ash is 20 : 80


GCV of fly ash = 400 Kcal/Kg; GCV of bottom ash = 800 Kcal/Kg.
*Heat loss in fly ash = 0.8 X 0.43 = 0.344 Kg. fly ash / Kcal.
*Heat loss due to fly ash = 0.344 X( 400 / 4000)X100
= 3.44%

Bottom ash = 0.2 X 0.43 = .086 Kg/Kg. Cal


*Heat loss in bottom ash = 0.086 X( 800 / 4000)X100 = 1.72%

Heat loss =
*Heat loss due to dfg = 8.2 %
*Heat loss due to H2 & M = 6.5 %
*Heat loss due to fly ash = 3.44 %
*Heat loss due to bottom ash = 1.72 %
*Radiation other losses = 2.0 %
Total Loss = 21.86 %

Effy ᶯ = 100 – 21.73


Efficiency of boiler after conclusion to FBC = 78.14 %

ms (hg – hf)
Effy ᶯ = --------------
Mf X GCV
SUPPLEMENTARY Paper 4 – Set B

20000 (690 – 95)


mf = ----------------------
0.7321 X 4000

Coal consumption with stoker fired system = 4063.6 Kgs/hr = 4.06 Tonnes/hr.

20000 (690 – 95)


With AFBC conversion, mf = ----------------------
0.7814 X 4000

Coal consumption = 3807.3 Kg./hr = say 3.8 Tonnes / hr.

Saving in daily coal consumption = (4.06-3.8) X 24 = 6.24 Tonnes / day

N-
2
SUPPLEMENTARY Paper 4 – Set B

b. cooling water flow rate circulation in the condenser if the range is 7oC
Condenser load = 17,000 x (2633.4 – 191.8)
= 415072000kJ/hr
= 415072000 /4.18 = 9929952.15kCal/hr

At a range of 7oC cooling water flow rate = 9929952.15 / 7


= 1418.56m3/hr
SUPPLEMENTARY Paper 4 – Set B

N-3 In a plant, a single cell 3000 million cal/hr, cooling tower with one CW pump is operated for cooling
water system. The operating parameters are tabulated as below.

S.No Parameter Before After refurbishment


refurbishment
1 CW inlet temp to CT 35oC 35oC
2 Atmospheric air conditions WbT -25 oC, DbT - 38 WbT -25 oC, DbT -
o
C 38 oC
3 COC 3.5 5
4 Suction head of CW pump -1m -1m
5 Discharge pressure of CW 4kg/cm2 4kg/cm2
pump
6 Efficiency
CW Pump 54% 53%
CW Pump motor 89% 89%
CT fan 60% 64%
CT fan motor 90% 90%
7 Pressure developed by CT 20mmwc 20mmwc
fan
8 Approach 4oC 3oC
9 L/G ratio 1.5 1.5
10 Density of air 1.29kg/m3 1.29kg/m3

The cooling tower is refurbished as a result of which the effectiveness has increased to 70 %.
Also with improved water treatment the COC is increased to 5.
Find out
1. Reduction in power consumption of pump and fan due to improvements in cooling tower.

2. Reduction in make up water consumption ignoring drift losses in KL/day

Parameter Equation / formulae Before refurbishment After


refurbishment
approach = TCW0-WbT =4+25 =29oC =3+25 =28oC
Twco = approach+ Wbt
CW flow = heat load/( TCWi-TCW0) =(3000x106/103) / ( 35- =(3000x106/103)/(35-
rate Q 29) 28)
= 500000 kg/h = 428571 kg/h
= 500 m3/h = 429 m3/hr
Evaporation =1.8*.00085*CW flow x 1.8x0.00085x500x (35- 1.8x.00085x429x(35
loss Range 29) -28) = 4.59 m3/h
= 4.59 m3/h
Blow down = Evaporation = 4.59/(3.5-1) 4.59/(5-1)
loss Loss/(COC-1) = 1.84 m3/h = 1.15 m3/h
Total water = Eva loss+ Blow down = 4.59+1.84 =4.59+1.15
loss loss =6.43 m3/h =5.74 m3/h
Make-up = Total water loss x = 6.43 x 24 = 5.74 x 24
water 24hrs = 154.2 m3/day = 137.76m3/day
=154.2KL/day =137.76 KL/day
SUPPLEMENTARY Paper 4 – Set B

Total head = discharge head- = 40-(-1) = 41 mWC = 40-(-1) = 41 mWC


H suction head
Pump LKW = = =
((Q*1000/3600)*(H*9.81 (500*1000/3600)*(41*9. (429*1000/3600)*(4
))/1000 81)/1000 1*9.81)/1000
= 55.86KW = 47.9 kW
Pump input = Pump LKW/Eff.Pump =55.86/0.54 = 47.9/0.53
=103.4 kW =90.4 kW
Motor = Pump input/motor = 103.4/0.9 =90.4/0.9
input eff =116.2 kW = 101.6kW
Air flow in =[( CW flow)x1000]/ = (500x1000/(1.5x1.29) =
CT fan Qf [((L/G)]*1.29) = 258398 m3/h (429x1000/(1.5x1.29
)
= 221705m3/h
Hf Pressure developed by = 20mmWC = 20mmWC
fan Hf
Air KW = [(Qf in m3/h)*(Hf in =(258398*20)/(3600*10 =(221705*20)/(3600
mmWC)]/(3600*102) 2) *102)
=14.07 kW = 12.08 kW
Fan motor =Air KW/(FanEffi xMotor =14.07/(0.60*0.9) =12.058/(0.64*0.9)
input Eff) = 26.05kW = 20.93 kW

(1) Reduction in power of pump and motor = (116.2+26.05) - (101.6+20.93)= 19.72 kW

(2) Reduction in makeup water = 154.2-137.76 = 16.44 or 16.5 KL/day

N-4 Answer ANY ONE OF THE FOLLOWING among A, B, C and D

A) A utility type captive thermal power plant of 65 MW is generating an output of 60 MW at the


generator . Steam generated in the boiler at 105 kg/cm 2a and 510oC is expanded in the steam
turbine exhausting to condenser maintained at 0.1 kg/cm2a and 45.5oC. The cooling water flow
rate through the condenser is 166m3 per min. The other operating data and particulars are,

Enthalpy of steam at 105 kg/cm2a & 510oC = 805 Kcal/kg.


Enthalpy of steam at turbine outlet = 565 Kcal/Kg.
Enthalpy of water at condenser pressure 0.1 kg/cm2a &at 45.5oC = 45.5 Kcal/Kg.
Inlet/outlet temperature of cooling water at the condenser =26oC/38oC
The efficiency of the generator = 95%
Enthalpy of saturated steam at 10kg/cm2a = 665 Kcal/Kg.

Based on the above, find out,


a. Heat load on the condenser in million kcal/hr
b. Output of the steam turbine in KW
c. Loss in the gear box in KW
d. Condenser effectiveness
e. DM water at 135oC to be sprayed for desuperheating of boiler steam after pressure reduction to
10 kg/cm2a required for auxiliary service in kgs/tonne steam
Ans
SUPPLEMENTARY Paper 4 – Set B

a. Heat load on the condenser = 166 x 60 x 1000(38-26)/1000000


= 119.52 million kcal/hr

b. Inlet enthalpy of steam to condenser = outlet enthalpy of steam from turbine


Steam flowrate through the turbine = 119.52x 1000000/(565-45.5)
= 230000 kg/hr
= 230.0 tonne/hr

Inlet enthalpy to turbine = 805 kcal/kg


Steam turbine out put =230000 x (805-565)
= 64186.00 KW

c. Generator out put = 60000.0 KW


Generator efficiency =95%
Generator input =60000.00 / 0.95 = 63157.9 KW

Loss in gear box = 64186 .00 – 63157.9


= 1028.1 kw

d. Cooling water inlet temperature = 26oC


Cooling water outlet temperature = 38oC
Inlet steam temperature to condenser = 45.5oC
.
. . Condenser effectiveness i.e. € =
= (Cooling water temp. rise) / (Inlet steam
temp. to condenser – inlet cooling water temp.)
= (38 – 26) / (45.5 – 26)
= 0.615

e. Quantity of DM water to be sprayed for desuperheating


ms= mass ofhigh pressure superheated steam
mw= mass of DM water to be sprayed
ms x 805 + mw x 135 = (ms + mw) x 665

Enthalpy of 105 kg/cm2a, 510oC superheated steam 805 kcal/kg


Enthalpy of saturated steam at 10 kg/cm2a 665 kcal/kg
Enthalpy of DM water at 135oC = 135 kcal/kg assumed

ms (805 – 665) = mw (665 – 135)


mw / ms = (805 – 665) /( 665 – 135)
= 0.264 kg water / kg steam
SUPPLEMENTARY Paper 4 – Set B

= 264 kg water/ tonne steam

or
B) The preheater exhaust gas from a cement kiln has the following composition on dry basis :
CO2 – 23.9%, O2 – 5.9%, CO – 0.2%, remaining is N2. The static pressure and temperature
measured in the duct are -730 mmWC and 3500C respectively. The velocity pressure
measured with a pitot tube is 19 mmWC in a duct of 2800 mm diameter ( Pitot tube
constant = 0.89 ). The atmospheric pressure at the site is 10350 mmWC universal gas
constant is 847.84 mmWCm3/kg mol k. The specific heat capacity of preheater exhaust
gas is 0.25 kcals/kg0C.

The static pressure developed by PH exhaust fan is 630mmWC and power drawn is 1582
kW. Calculate the efficiency of fan given that the motor efficiency is 92%.

The plant has decided to install a waste heat recovery power plant with the heat rate of
5200 kcals/kWh. The temperature drop across the waste heat boiler of the power plant is
1000C. calculate the maximum possible power generation from this system?
Ans Molecular weight exhaust gas (dry basis) M
= %CO2xMCO2 + %O2xMO2 + %COxMCO + %N2 x MN2
= {(23.9 x 44) + (5.9 x 32) + (0.2 x 28) + (70 x 28)}/100
= 32.06 kg/kg mole

Exhaust Gas density at operating temperature= γ = [ PM / RT ]


= [ (10350 – 730) x 32.06 ) / { 847.84 x (273+350) }
= 0.584 kg/m3

Duct Area = 3.14 x( 2.8/2) 2= 6.15 m2

Volume flow rate


=A Cp (2 x g x ∆P / γ)1/2 = 6.15 x 0.89 (2 x 9.81x 19/0.0.584)1/2
= 138.4 m3/s
Volume flow rate = 498194 m3/ h

Fan efficiency = volumetric flow rate x pressure developed


(102 x power drawn x motor eff)

= 138.4 x 630 x100 = 59%


(102x1582x0.92)

Mass flow rate of preheater exhaust gas = Volume flow rate x density
= 498194*0.584 = 2,90,945 kg/hr

Heat given up to power plant by exhaust gas = 290945 x0.25 x100


SUPPLEMENTARY Paper 4 – Set B

= 7273625 kcals/hr

Maximum possible power generation from the WHR power plant =7273625/5200
= 1399kW

or
C) In a textile process house the production from the stenter machine is 72000 mtrs per day. The
effective operation of stenter is 20 hours per day. The percentage moisture in the dried cloth
(output) is 6% and its temperature is 75oC and wet cloth inlet is at 25oC . The stenter is heated by
steam at 8 Kg./cm2a and the daily steam consumption for the stenter is 16.5 tonnes. The efficiency
of the stenter dryer is 53%. Calculate the
(i) Linear speed of the stenter machine
(ii) Inlet moisture
(iii) Feed rate of the stenter.

The following data have been provided


Weight of 10 meter of dried cloth = 1 kg.
Enthalpy of the steam to the stenter = 665 kcal/kg.
Enthalpy of condensate at the exit of stenter = 130 kcal/kg.
Ignore losses in start-up and stoppage.

Ans Production per day = 72000 meters


Actual hours of operation = 20 hours/ day
Linear speed of the stenter = 72000 / (20x60) = 60 meters per min

Dried cloth output = 72000 / (20x10) = 360 kg/hr.

Moisture in dry cloth = 6%


Bone dry cloth = 360 x 0.94 = 338.4 kg/hr

Moisture in outlet cloth mo = (360 – 338.4) / 338.4


= 0.0638 Kg./Kg. bone dry cloth

Steam consumption per day = 16.5 tonnes


= 16500 / 20 = 825 Kg./hr.

Heat load on the dryer =Energy input in steam x Dryer Efficiency


= Steam flow rate x (Enthalpy steam – Enthalpy condensate) x Efficiency Dryer
= 825 x (665 – 130) x 0.53
= 233928.75 Kcal/hr.

Further Heat load on the dryer = w x (mi – mo) X [(Tout – Tin) + 540] Kcal/hr.
w =weight of bone dry cloth rate kg/hr
mi = weight of cloth inlet moisture Kg./Kg. bone dry cloth
Tout = dried cloth outlet temperature = 75oC
SUPPLEMENTARY Paper 4 – Set B

Tin = wet cloth inlet temperature = 25oC

338.4 x (mi – 0.0638) X [(75 – 25) + 540] = 233928.75 Kcal/hr


mi = 1.235 Kg./Kg. bone dry cloth(1.235) / (1.235+1)x100
% inlet moisture in wet cloth = 55.25 %

total moisture in inlet cloth = 1.235x338.4= 417.924 kg/hr

feed rate(inlet cloth rate), = total inlet moisture/hr +bone dry cloth/hr
= 417.92+338.4
= 756.32 Kg./hr.

or
D) The following data are given for a commercial building.

Outdoor conditions : DBT = 37C, WBT = 26.5C, Humidity = 17.5 g of water / kg of dry air
Desired indoor conditions : DBT = 24C, RH = 55 %, Humidity = 10.2 g of water / kg of dry air

Total area of wall = 320 m2, out of which 50% is window area.

U – Factor ( Wall ) = 0.33 W/m2K

U – Factor ( Roof ) = 0.323 W/m2K

U – factor [ fixed windows with aluminum frames and a thermal break ] = 3.56 W/m2K

Other data:

 15 m x 25 m roof constructed of 100 mm concrete with 90 mm insulation & steel decking.


 CLTD at 17:00 hr : Details : Wall = 12C; Roof = 44C; Glass Window = 7C
 SCL at 17 : 00 hr : Details : Glass Window = 605 W/ m2
 Shading coefficient of Window = 0.74
 Space is occupied from 8:00 to 17:00 hr by 25 people doing moderately active work.
 Sensible heat gain / person = 75 W ; Latent heat gain / person = 55 W ; CLF for people =
0.9
 Fluorescent light in space = 21.5 W/m2 ; CLF for lighting = 0.9
 Ballast factor details = 1.2 for fluorescent lights & 1.0 for incandescent lights
 Computers and office equipment in space produces 5.4 W/m2 of sensible heat
 One coffee maker produces 1050 W of sensible heat and 450 W of latent heat.
 Air changes/hr of infiltration = 0.3
SUPPLEMENTARY Paper 4 – Set B

 Height of building = 4 m
 Supply air dry bulb temperature is 170C

(i) Determine the building cooling load in TR

(ii) Calculate the supply air quantity to the cooling space m3/s

Ans (i) Cooling Load Determination:

I. External Heat Gain

(i) Conduction heat gain through the wall = U – factor x net area of wall x CLTD
= 0.33 x (160) x 12 ] = 633.6 W

(ii) Conduction heat gain through the roof= U – factor x net area of roof x CLTD
= 0.323 x ( 15 x 25 ) x 44
= 5329.5 W

(iii) Conduction heat gain through the windows = U – factor x net area of windows x CLTD
= (3.56 x 160 x 7) = 3987.2 W

(iv) Solar radiation through glass


= Surface area x Shading coefficient x SCL
= (160 x 0.74 x 605) = 71632 W

II. Internal Heat Gain

(i) Heat gain from people =Sensible heat gain + Latent heat gain

Sensible heat gain = (No. of people x Sensible heat gain / person x CLF)
=(25 x 75 x 0.9) = 1687.5 W

Latent heat gain = No. of people x Latent heat gain / person


= (25 x 55 ) = 1375 W
Therefore, Heat gain from people = (1687.5 + 1375 ) = 3062.5 W

(ii) Heat gain from lighting = (Energy input x Ballast factor x CLF)
Energy input = (Amount of lighting in space / unit area)x Floor area
= 21.5 x (15 x 25) = 8062.5 W

Therefore, heat gain from lighting = (8062.5 x 1.2 x 0.9) =8707.5 W

(iii) Heat generated by equipment :

Sensible heat generated by coffee maker =1050 W


SUPPLEMENTARY Paper 4 – Set B

Latent heat generated by coffee maker = 450 W


Sensible heat gain by computers and office equipment= 5.4 x 375 = 2025 W
Therefore, Heat generated by equipment = 3525 W

(iv)Heat gain through air infiltration = (Sensible heat gain + Latent heat gain)

Sensible heat gain =(1210 x airflow x ∆T )


Airflow = (Volume of space x air change rate ) / 3600
= { (15 x 25 x 4 ) x 0.3 } / 3600
= 0.125 m3 / s

Therefore, sensible heat gain =1210 x 0.125 x ( 37 – 24 ) =1966.25 W

Latent heat gain =3010 x 0.125 x ( 17.5 – 10.2 ) =2746.6 W

No. Space Load Components Sensible Heat Load (W) Latent Heat Load
(W)
1. Conduction through exterior wall 633.6 ---
2. Conduction through roof 5329.5 ---
3. Conduction through windows 3987 ---
4. Solar radiation through windows 71632 ---
5. Heat gained from people 1687.5 1375
6. Heat gained from lighting 8707.5 ---
7. Heat gained from equipment 3075 450
8. Heat gained by air infiltration 1966.25 2746.6
Total space cooling load 97018.35 4571.6
Total Cooling Load = 101589.4 W/ 3516 = 29TR

(ii) Supply Air Quantity Calculation:

Supply air flow = Sensible heat gain / {1210 * (Room dry bulb temperature – Supply dry bulb temperature)}
= 97018.35 W / {1210 J/m3K*(24 – 17) 0C}
= 11.45 m3/s

-------- End of Section - III ---------


Regular set A

Regn No: __________________


Name : __________________
(To be written by the candidate)

17th NATIONAL CERTIFICATION EXAMINATION


FOR
ENERGY MANAGERS & ENERGY AUDITORS – September, 2016

PAPER – 4:Energy Performance Assessment for Equipment and Utility Systems

Date: 25.09.2016 Timings: 14:00-16:00 HRS Duration: 2 HRS

General instructions:
o Please check that this question paper contains 6 printed pages
o Please check that this question paper contains 16 questions
o The question paper is divided into three sections
o All questions in all three sections are compulsory
o All parts of a question should be answered at one place

Section - I: BRIEF QUESTIONS

S-1 An air washer cools the water and a cooling tower cools the air. True or False.
Ans False.
S-2 A 11 kW induction motor has an efficiency of 90% what will be its maximum delivered
output?
Ans 11 kW.
S-3 The COP of a vapour absorption refrigeration system is lower than the COP of a
vapour compression refrigeration system-True /false.
Ans True.
S-4 An industrial electrical system is operating at unity power factor. Addition of further
capacitors will reduce the maximum demand (kVA). True or False.
Ans False.
S-5 Which parameter in the proximate analysis of coal is an index of ease of ignition?
Ans Volatile matter.
S-6 The major source of heat loss in a coal fired thermal power plant is through flue gas
losses in the boiler. True or false.
Ans False.
S-7 With evaporative cooling, it is possible to attain water temperatures below the
atmospheric wet bulb temperature. True or False
Ans False
S-8 A pump is retrofitted with a VFD and operated at full speed. Will the power
consumption increase or decrease or remain the same?
Ans Increase
S-9 De-aeration in boiler refers to removal of dissolved gases. True or false

Ans True
Regular set A

S-10 In a compressed air system, the function of the after cooler is to reduce the work of
compression. True or False

Ans False
…………. End of Section - I ………….
Section - II: SHORT NUMERICAL QUESTIONS

L-1 In a petrochemical industry the LP & HP boilers have the same evaporation ratio
of 14 using the same fuel oil. The operating details of LP & HP boiler are given
below:
Particulars LP Boiler HP Boiler
Pressure 10 Kg./cm2a 32 Kg./cm2a
Temperature Saturated Steam 400oC
Enthalpy of steam 665 Kcal/kg 732 Kcal/kg
Enthalpy of feed water 80oC 105oC
Evaporation Ratio 14 14

Find out the efficiency of HP boiler if the LP boiler efficiency is 80%.


Ans Effy ᶯ = ER. (hg – hf) / GCV
EffyL.P ᶯ1 = 0.8 = 14 x(665 – 80) / GCV
EffyH.P ᶯ2 = 14 x(732 – 105) / GCV
EffyH.P ᶯ2 / EffyL.P ᶯ1 = (732 – 105)0.8 / (665 – 80) = 0. 8574 =85.74%

Or

EffyL.P ᶯ1= 0.8 = 14 x(665 – 80) / GCV


GCV= 14x(665-80) / 0.8 = 10237.5kcal/kg
EffyH.P ᶯ2 = 14 x(732 – 105) / GCV
= 14 x(732 – 105) / 10237.5 = 0.8574 = 85.74%

L-2 While carrying out an energy audit of a pumping system, the treated water flow
(in open channel) was measured by the tracer method. 20% salt solution was
used as the tracer which was dosed @ 2 lts/min. The water analysis about 500
mtrs away revealed salt concentration of 0.5%. Assuming complete mixing and
no losses, calculate the water flow rate.
Ans 20% salt solution = 200 gms of salt in 1 Litre of water
0.5% salt solution = 5 gms of salt in 1 litre of water
Dosing rate = 2 lts/min
Salt added in water = 2 x 200 = 400 gms/min
Total flow = 400/5 = 80 lts/min
Water flow rate = 80 – 2 = 78 lts/min
Or

C1V1 = C2V2
V2 = C1V1/C2 = 0.2x2/0.005 =80 lts/min
Actual flow = total flow – dosage flow = 80-2 =78 lts/min

…………. End of Section - II ………….


Regular set A

Section - III: LONG NUMERICAL QUESTIONS

N-1 In a chemical plant, a 3000 Million Cal/hr cooling tower with one CW pump caters to the
cooling water requirements. The management had decided to refurbish the cooling tower
as its performance is felt to be low. The operating parameters of the CW system before
and after refurbishment are presented below.

S.No Parameter Before After


refurbishment refurbishment
1 CW inlet temp to CT 35oC 35oC
2 Atmospheric air WbT -25 oC, WbT -25 oC,
conditions DbT - 38 oC DbT - 38 oC
3 COC 3.5 5
4 Suction head of CW -1m -1m
pump
5 Discharge pressure of 4kg/cm2(g) 4kg/cm2(g)
CW pump
6 Efficiency
CW Pump 54% 53%
CW Pump motor 89% 89%
CT fan 55% 54%
CT fan motor 90% 90%

7 Pressure developed by 20mmwc 20mmwc


CT fan
8 Effectiveness of CT 60 % 70%
9 L/G ratio 1.5 1.5
10 Density of air 1.29kg/m3 1.29kg/m3

As a result of cooling tower refurbishment the effectiveness has increased from 60% to 70
%. Also with improved water treatment the COC has increased to 5.

Find out
1. Reduction in power consumption of pump and fan due to improvements in cooling
tower.
2. Reduction in make up water consumption (ignoring drift losses) in KL/day
Regular set A

Ans

Paramet Equation / Before After refurbishment


er formulae refurbishment
Effectiven = (TCWi-TCW0)/(TCWi- 0.6=(35- TCWO)/(35- 0.7=(35- TCWO)/(35-
ess WbT) 25) 25)
TCWO = 29 oC TCWO = 28 oC
CW flow = heat load/( TCWi- =(3000x106/103) / ( =(3000x106/103)/(35-
rate Q TCW0) 35- 29) 28)
= 500000 kg/h = 428571 kg/h
= 500 m3/h = 429 m3/hr
Evaporati =1.8*.00085*CW 1.8x0.00085x500x 1.8x.00085x429x(35-
on loss flow x Range (35-29) 28) = 4.59 m3/h
= 4.59 m3/h
Blow = Evaporation = 4.59/(3.5-1) 4.59/(5-1)
down loss Loss/(COC-1) = 1.84 m3/h = 1.15 m3/h
Total = Eva loss+ Blow = 4.59+1.84 =4.59+1.15
water down loss =6.43 m3/h =5.74 m3/h
loss
Make-up = Total water loss = 6.43 x 24 = 5.74 x 24
water x 24hrs = 154.2 m3/day = 137.76m3/day
=154.2KL/day =137.76 KL/day
Total = discharge head- = 40-(-1) = 41 mWC = 40-(-1) = 41 mWC
head H suction head
Pump = = =
LKW ((Q*1000/3600)*(H* (500*1000/3600)*(41 (429*1000/3600)*(41
9.81))/1000 *9.81)/1000 *9.81)/1000
= 55.86KW = 47.9 kW
Pump = Pump =55.86/0.54 = 47.9/0.53
input LKW/Eff.Pump =103.4 kW =90.4 kW
Motor = Pump = 103.4/0.89 =90.4/0.89
input input/motor eff =116.2 kW = 101.6kW
Air flow in =[( CW flow)x1000]/ = =
CT fan [((L/G)]*1.29) (500x1000/(1.5x1.29) (429x1000/(1.5x1.29
Qf = 258398 m3/h )
= 221705m3/h
Hf Pressure developed = 20mmWC = 20mmWC
by fan Hf
Air KW = [(Qf in m3/h)*(Hf =(258398*20)/(3600* =(221705*20)/(3600*
in 102) 102)
mmWC)]/(3600*102 =14.07 kW = 12.08 kW
)
Fan =Air KW/(FanEffi x =14.07/(0.55*0.9) =12.058/(0.54*0.9)
motor Motor Eff) = 28.43kW = 24.9 kW
input

(1) Reduction in power of pump and motor = (116.2+28.43) - (101.6+24.9) = 18.13 kW

(2) Reduction in makeup water = 154.2-137.76 = 16.44 or 16.5 KL/day


Regular set A

N-2 In a beverages industry the product stream (liquid) flowing at a rate of 5000 kgs/hr at 90 oC
is first cooled in counter type cooling water (CW) heat exchanger to 55 oC and then by a
chilled water (ChW) heat exchanger, to reduce temperature of the product to 11oC. The
specific heat of the product is 0.9 kCal/kgoC. The other operating data and parameters are:

Cooling Water heat exchanger Chilled Water heat exchanger


Inlet temp Outlet temp Inlet temp Outlet temp
Product 90oC 55 oC Product 55oC 11 oC

Cooling 25 oC 32 oC Chilled 7 oC 12 oC
Water water

The chilled water is supplied by a reciprocating chiller, whose motor is drawing 60 KW with
a motor efficiency of 87%. The management decides to upgrade cooling water heat
exchanger by providing additional heat exchanger area to further enhance heat recovery
i.e. to reduce the temperature of product at its outlet to 40oC.
A. Depict the heat exchanger in existing and upgraded (improved) heat recovery case
in a simple block diagram
B. Calculate

i. The additional heat exchanger area (as a % of the existing area) for cooling water
heat exchanger, assuming there is no change in cooling water circulation rate and
the overall heat transfer coefficient.
ii. The COP of the chiller.
iii. Reduction in refrigeration /chiller load and yearly energy savings at 600 hours per
month operation, assuming energy consumption is proportional to load delivered.
Regular set A

Ans (A) Existing


mh = 5000 Kg/hr, 90oC product 55oC 11oC
Cooling water heat Chilled Water heat exchanger
Δ T1 exchanger area (A1) Δ T2

cw.out 32oC
ch.w out 12oC ch.w in 7oC

cw.in 25oC To chiller From chiller

Heat load on CW heat exchanger = heat gained by CW


Heat rejected by product stream
Q1 = mc X 1 X (Δ Tc) = 5000 X 0.9 X (90 – 55)
Q1 = 1,57,500 Kcal/hr
mc = Cooling Water flow rate = (5000X0.9 X 35)/(32 – 25)
= 22,500 Kg/hr.
Heat exchanger duty with increased heat = 5000 X 0.9 X (90 – 40)
recovery Q2 = 2,25,000 Kcal/hr
Cooling water temp. Rise with increased heat = 5000 X 0.9 X (90 – 40)
exchanger duty for cooling product stream to ------------------------------ = 10oC
40oC 22,500

Cooling water outlet temp with the above case = 25 + 10 = 35oC

(A) Upgraded
mh = 5000 Kg/hr, 90oC product 40oC 11oC
Upgraded Cooling water heat Chilled Water heat exchanger
Δ T1 exchanger area (A2) Δ T2

cw.out 35oC
ch.w out 12oC ch.w in 7oC

cw.in 25oC To chiller From chiller


…..2.5 marks
Δ T1 - Δ T2
LMTD1 with existing case = ----------------------
Ln Δ T1 / Δ T2

(90 – 32) –(55 – 25)


= -------------------------------- = 42.49oC
Ln (90 – 32) / (55 – 25)
Regular set A

(90 – 35) – (40 – 25)


LMTD2 with additional heat recovery = -------------------------------- = 30.77oC
Ln (90 – 35) / (40 – 25)

Q2 = U X A2 X LMTD2 Q1 = U X A1 X LMTD1
Q2 = 2,25,000 = U X A2 X 30.77 Q1 = 1,57,500 = U X A1 X 42.49

A2 / A1 = (2,25,000 /1,57,500) X (42.49 / 30.77) = 1.973

Additional area required = 97.3% of existing heat exchanger area of CW


heat exchanger
Refrigeration load in existing = 5000 X 0.9 X (55 – 11)
case
= 1,98,000 Kcal/hr
= 1,98,000 /3024
= 65.476 TR
Motor input power = 60 KW
Motor eff. = 87%
C.O.P. of refrigeration chiller = 198000 /( 60 X 0.87 X 860)
= 4.41
Input KW / TR = 60 / 65.476 = 0.916
Reduction in refrigeration load = 5000 X 0.9 X (55 – 40) / 3024
due to lower input temperature = 22.32 TR
of the product to chilled water
heat exchanger
= 22.32 X 0.916 X 600 X 12
Yearly energy savings at 600 1,47,204.86 Kwh
hrs. operation per month =

N-3 In a continuous process industry Steam and Power are supplied through a cogeneration
plant interconnected with grid. The design and actual operating parameters of the
cogeneration plant as represented in the schematic are given in the table below.

Double Extraction – Condensing Steam Turbine Cogeneration System


Regular set A

Design actual
B- Boiler 68.75tph,
75tph,64kg/cm2(a), 450oC @82%
64kg/cm2(a), 450oC
efficiency
@81% efficiency
T- Steam Turbine Double Extraction – Condensing type
G- Generator 10MW 7.2MW

Stream Steam flow location Steam Flow Steam Steam Steam


Ref (tph) Pressure Temp (oC) enthal
(kg/cm2) py
(kCal/k
g)
1 Steam input to 68.75 64 450 745
turbine
2 First extraction 18.75 17 270 697
3 Second extraction 31.25 9 200 673
Condenser in 18.75 0.1 - 550
4
Condenser out 18.75 - - 46

The industry is installing a 1200 TR double effect absorption chiller to meet the
refrigeration load due to product diversification. Additional steam will be generated by the
boiler, which will go into the turbine and be extracted at 9kg/cm 2(a) to meet the VAM
requirement. The additional power thus generated will reduce the imported grid power.

The following additional data has been provided:


Maximum allowable steam flow the extraction at 9
40 TPH
Kg/cm2a
Minimum allowable steam to condenser 9 TPH
Critical power requirement of the plant 3800 KW
Power import from grid 500 KW
Cost of grid power Rs.4.25 / Kwh
G.C.V. of coal 4000 Kcal/Kg.
Cost of coal Rs. 4000/ton
Feed Water temperature 105oC
Feed Water enthalpy 105 Kcal/Kg.
Combined efficiency of gear box and generator 96%
Steam requirement for double effect absorption chiller 4.5 Kg./TR hr at 9 Kg/cm2a
Annual hours of operation. 8000 hrs/y
Steam rate at 9 Kg/cm2a at 2nd extraction for 1 KW
12 (Kg/hr)/kW
turbine output
Ignore auxiliary power consumption and also return condensate from extracted steam to
process.

Calculate
(i) The Energy Utilization Factor (EUF) for the existing operating case
(ii) The net additional annual operating cost, after installation of VAM.
(iii) The Energy Utilization Factor (EUF) after installation of VAM.
Regular set A

Ans (i) Q thermal + P electrical


Energy Utilization Factor (EUF) = -----------------------------------
(before VAM installation) Fuel Consumption X G.C.V.

Q thermal = m2 h2 + m3h3 + m4h’4

m (h1 – hf)
Q in = -------------
 X G.C.V.

Q thermal = 18750 X 697 + 31250 X 673 + 18750 X 46


= (18.75 X 697 + 31.25 X 673 + 18.75 X 46) X 103 Kcal/hr
= (13068 X 21031 + 862.5) X 103 Kcal/hr
= 34962.5 X 103 Kcal/hr

Pe = 7200 X 860
= 6192 X 103 Kcal/hr

(745 – 105) X 68.750x1000


Fuel Consumption = -------------------------- = 13.58 TPH
0.81X 4000 X 1000

34962.5 X 103 + 6192 X 103


EUF = ---------------------------------- -x 100 = 75.76%
13.58 X 103 X 4000

(ii)
Refrigeration Load = 1200 TR

1TR requires 4.5 Kg./hr steam at 9 Kg./cm2a


Steam consumption in double effect absorption chiller = 1200 X 4.5
= 5400 Kg./hr.
Increase in steam extraction at 9 Kg./ cm2a = 5400Kg/hr

Every 12 Kg./hr extraction at 9 kg/ cm2a gives 1 KW output at turbine ,


efficiency of generator and gear box = 0.96

Additional power recovery due to increase in extraction = (5400 / 12) X 0.96


= 432 KW

Additional coal consumption due to increase in extraction


= (745 – 105)X 5400/(0.81x4000)
= 1066 kg/h
Additional cost of coal = 4000x1.066 = Rs 4266.6 /hr
…..2 marks
Monetary realisation by reducing import cost of purchased electricity = 4.25 Rs./unit
= 432 X 4.25
= 1836 Rs./hr
Regular set A

Net additional annual operating cost after VAM installation = (4266.6-1836)*8000


= Rs 1.94 crore/y

(iii)
Stream Steam flow Steam Steam Steam Steam
Ref location Flow Pressure Temp enthalpy
(tph) (kg/cm2) (oC) (kCal/kg)
1 Steam input to 68.75+5.4 64 450 745
turbine =74.15
2 First extraction 18.75 17 270 697
3 Second extraction 31.25+5.4 9 200 673
=36.65
Condenser in 18.75 0.1 - 550
4
Condenser out 18.75 - - 46

Q thermal + P electrical
Energy Utilization Factor (EUF)= -----------------------------------
(after VAM installation) Fuel Consumption X G.C.V.

Q thermal = m2 h2 + m3h3 + m4h’4

m (h1 – hf)
Q in = -------------
 X G.C.V.

Q thermal = 18750 X 697 + 36650 X 673 + 18750 X 46


=(18.75 X 697 + 36.65 X 673 + 18.75 X 46) X 103 Kcal/hr
= (13068 X 21031 + 862.5) X 103 Kcal/hr
= 38596.7 X 103 Kcal/hr

Pe = (7200+432) X 860
= 6563.5 X 103 Kcal/hr

Fuel Consumption = 13.58 TPH +1.066 TPH = 14.646TPH

38596.7 X 103 + 6563.5 X 103


EUF = ----------------------------------------x100 = 77.08%
14.646 X 103 X 4000

Answer ANY ONE OF THE FOLLOWING among A, B, C and D


N-4
A) The operating parameters observed w.r.t. design in a 110 MW power generation unit are
given below:
Regular set A

Parameters Design Operation

Generator output 110 MW 110 MW


Steam generator outlet super heat 540oC 525oC
temperature

Steam generator outlet pressure 140 Kg/cm2a 130 Kg/cm2a


Feed water inlet temperature 135oC 135oC
Boiler  87.5% 87.5%
GCV of Coal 3800 3800
Turbine exhaust pressure 0.09 0.11
Kg./cm2a Kg./cm2a
Dryness fraction of exhaust steam 0.87 0.89

Turbine heat rate 2362.5 Kcal ------


/Kwh

Efficiency generator 96% 96%


Energy loss in gear box 4420 KW 4420 KW
Enthalpy of steam at 520oC, 130 Kg/cm2a, 810
Kcal/Kg.
Enthalpy of steam at 0.11 Kg./cm2a 550
Kcal/Kg

Calculate the
I. Actual steam flow to the turbine
II. Specific steam consumption of turbine
III. % increase in gross unit heat rate compared to design
IV. Increase in monthly (720 hours/month) coal consumption due to deviation in
operation w.r.t. design at a plant load factor of 80%
Ans
Generator output = 110 MW
 of generator 96%
Generator input = 110 / 0.96
= 114.58 MW

Energy loss in gear box = 4420 KW


= 4.42 MW

Turbine output = Total input at gear box +


energy loss in gear box
Turbine out put = 114.58 + 4.42
= 119 MW

Turbine out put = ms (810 – 550) X (1 / 860)


Regular set A

ms (810 – 550) X (1 / 860) = 119 X 1000


Steam flow rate through the = 3,93,615 Kg /hr
turbine = 393.615 Tonne/hr

Specific steam consumption = (393.615 X 1000) / (110 X


1000)
= 3.58 Kg./kW
Boiler  = Ms (hg – hw)
--------------- X 100
GCV X mf
393615 (810 – 135)
Coal consumption mf -------------------------
0.875 X 3800
79906.8 Kg./hr

= 79906.8 / (110 X 1000)


Specific coal consumption = 0.726 Kg./Kwh

Actual unit heat rate = 0.726 X 3800


= 2758.8 Kcal/Kwh

Design turbine heat rate 2362.5 Kcal / KW


 of steam generator or boiler 87.5%
Design unit heat rate = 2362.5 / 0.875
= 2700 Kcal/Kwh
% increase in heat rate w.r.t. = [(2758.8 – 2700) / 2700] X 100
design = 2.17 %
Specific coal consumption for = 2700/3800
design heat rate = 0.71kg/kwh
= (0.726 – 0.71) X 110 X 1000 X 0.8 X 720
------------------------------------------------
Additional coal consumption per 1000
month with a PLF of 80% 1013.76 tonnes
=
Or
B) In a textile process house the production from the stenter machine is 72000 mtrs per day.
The effective operation of stenter is 20 hours per day. The percentage moisture in the
dried cloth (output) is 6% and its temperature is 75oC and wet cloth inlet is at 25oC . The
stenter is heated by steam at 8 kg/cm2a and the daily steam consumption for the stenter is
16.5 tonnes. The efficiency of the stenter dryer is 47%. Calculate the
(i) Linear speed of the stenter machine
(ii) Inlet moisture
(iii) Feed rate of the stenter.

The following data have been provided


Weight of 10 meter of dried cloth = 1 kg.
Enthalpy of the steam to the stenter = 665 kcal/kg.
Regular set A

Enthalpy of condensate at the exit of stenter = 130 kcal/kg.


Ignore losses in start-up and stoppage.

Ans Production per day = 72000 meters


Actual hours of operation = 20 hours/ day
Linear speed of the stenter = 72000 / (20x60) = 60 meters per min

Dried cloth output = 72000 / (20x10) = 360 kg/hr.

Moisture in dry cloth = 6%


Bone dry cloth = 360 x 0.94 = 338.4 kg/hr

Moisture in outlet cloth mo = (360 – 338.4) / 338.4


= 0.0638 Kg./Kg. bone dry cloth

Steam consumption per day = 16.5 tonnes


= 16500 / 20 = 825 Kg./hr.

Heat load on the dryer =Energy input in steam x Dryer Efficiency


= Steam flow rate x (Enthalpy steam – Enthalpy condensate) x Efficiency Dryer
= 825 x (665 – 130) x 0.47
= 207446.3 Kcal/hr.

Further Heat load on the dryer = w x (mi – mo) X [(Tout – Tin) + 540] Kcal/hr.
w =weight of bone dry cloth rate kg/hr
mi = weight of cloth inlet moisture Kg./Kg. bone dry cloth
Tout = dried cloth outlet temperature= 75oC
Tin = wet cloth inlet temperature = 25oC

338.4 x (mi – 0.0638) X [(75 – 25) + 540] = 207446.3 Kcal/hr


mi = 1.1028 Kg./Kg. bone dry cloth(1.1028) / (1.1028+1)x100
% inlet moisture in wet cloth = 52.44 %

total moisture in inlet cloth = 1.1028x338.4= 373.2 kg/hr

feed rate(inlet cloth rate), = total inlet moisture/hr +bone dry cloth/hr
= 373.2+338.4
= 711.6 Kg./hr.

or
C) The preheater exhaust gas from a cement kiln has the following composition on dry basis
:CO2 – 23.9%, O2 – 5.9%, CO – 0.2%, remaining is N2. The static pressure and
temperature measured in the duct are -730 mmWC and 3500C respectively. The velocity
pressure measured with a pitot tube is 19 mmWC in a duct of 2800 mm diameter ( Pitot
tube constant = 0.89 ). The atmospheric pressure at the site is 10350 mmWC and
universal gas constant is 847.84 mmWCm3/kg mol k. The specific heat capacity of
preheater exhaust gas is 0.25 kcals/kg0C.
Regular set A

The static pressure developed by PH exhaust fan is 630mmWC and power drawn is 1582
kW. Calculate the efficiency of fan given that the motor efficiency is 92%.

The management had decided to install a 1.3 MW power plant with a cycle efficiency of
15% by using this preheater exhaust gas. Calculate the exhaust gas temperature at the
outlet of waste heat recovery boiler of the power plant.
Ans Molecular weight exhaust gas (dry basis) M
= %CO2xMCO2 + %O2xMO2 + %COxMCO + %N2 x MN2
= {(23.9 x 44) + (5.9 x 32) + (0.2 x 28) + (70 x 28)}/100
= 32.06 kg/kg mole

Exhaust Gas density at operating temperature = γ = [ PM / RT ]


= [ (10350 – 730) x 32.06 ) / { 847.84 x (273+350) }
= 0.584 kg/m3

Duct Area = 3.14 x( 2.8/2) 2= 6.15 m2

Volume flow rate


=A Cp (2 x g x ∆P / γ)1/2 = 6.15 x 0.89 (2 x 9.81x 19 /0.584)1/2
= 138.3 m3/s
Volume flow rate= 497880 m3/ h

Fan efficiency = volumetric flow rate x pressure developed


(102 x power drawn x motor eff)

= 138.3 x 630 x100 = 58.69%


(102x1582x0.92)

Mass flow rate of preheater exhaust gas = Volume flow rate x density
= 497880*0.584 = 2,90,762 kg/hr

Heat equivalent of power generated from power plant =1.3MW =1300 x 860
= 1118000 kCals/hr

Heat given up to power plant by exhaust gas = 290762 x0.25 x(350-To)x0.15

To = 350 - (1118000/(290945x0.25x0.15)) = 247.50C

or
D) For a commercial building, using the following data,
(i) Determine the building cooling load in TR
(ii) Calculate the supply air quantity to the cooling space in m3/s

Outdoor conditions : DBT = 40C, WBT = 28C, Humidity = 19 g of water / kg of dry air
Desired indoor conditions : DBT = 25C, RH = 60 %, Humidity = 12 g of water / kg of dry
air
Regular set A

Total area of wall = 324 m2, out of which 50% is window area.

U – Factor ( Wall ) = 0.33 W/m2K


U – Factor ( Roof ) = 0.323 W/m2K
U – factor [ fixed windows with aluminium frames and a thermal break ] = 3.56 W/m2K
Other data:

 20 m x 25 m roof constructed of 100 mm concrete with 90 mm insulation & steel


decking.
 CLTD at 17:00 hr : Details : Wall = 12C; Roof = 44C; Glass Window = 7C
 SCL at 17 : 00 hr : Details : Glass Window = 605 W/ m2
 Shading coefficient of Window = 0.74
 Space is occupied from 8:00 to 17:00 hr by 30 people doing moderately active
work.
 Sensible heat gain / person = 75 W ; Latent heat gain / person = 55 W ; CLF for
people = 0.9
 Fluorescent light in space = 21.5 W/m2 ; CLF for lighting = 0.9
 Ballast factor details = 1.2 for fluorescent lights & 1.0 for incandescent lights
 Computers and office equipment in space produces 5.4 W/m2 of sensible heat
 One coffee maker produces 1050 W of sensible heat and 450 W of latent heat.
 Air changes/hr of infiltration = 0.3
 Height of building = 3.6 m
 Supply air dry bulb temperature is 150C
Ans (i) Cooling Load Determination:

I. External Heat Gain

(i) Conduction heat gain through the wall = U – factor x net area of wall x CLTD
= 0.33 x (324*0.5) x 12 ] = 641.5 W

(ii) Conduction heat gain through the roof = U – factor x net area of roof x CLTD
= 0.323 x ( 20 x 25 ) x 44
= 7106 W

(iii) Conduction heat gain through the windows = U – factor x net area of windows x CLTD
= (3.56 x 162 x 7) = 4037 W

(iv) Solar radiation through glass


= Surface area x Shading coefficient x SCL
= (162 x 0.74 x 605) = 72527 W

II. Internal Heat Gain

(i) Heat gain from people =Sensible heat gain + Latent heat gain
Regular set A

Sensible heat gain = (No. of people x Sensible heat gain / person x CLF)
=(30 x 75 x 0.9) = 2025 W

Latent heat gain = No. of people x Latent heat gain / person


= (30 x 55 ) = 1650 W
Therefore, Heat gain from people = (2025 + 1650 ) = 3675 W

(ii) Heat gain from lighting = (Energy input x Ballast factor x CLF)
Energy input = (Amount of lighting in space / unit area)x Floor area
= 21.5 x (20 x 25) = 10750 W

Therefore, heat gain from lighting = (10750 x 1.2 x 0.9) =11610 W

(iii) Heat generated by equipment :

Sensible heat generated by coffee maker =1050 W


Latent heat generated by coffee maker = 450 W
Sensible heat gain by computers and office equipment = 5.4 x 500 = 2700 W
Therefore, Heat generated by equipment = 4200 W

(iv)Heat gain through air infiltration = (Sensible heat gain + Latent heat gain)

Sensible heat gain =(1210 x airflow x ∆T )


Airflow = (Volume of space x air change rate ) / 3600
= { (20 x 25 x 3.6 ) x 0.3 } / 3600
= 0.15 m3 / s

Therefore, sensible heat gain =1210 x 0.15 x ( 40 – 25 ) =2722.5 W

Latent heat gain =3010 x 0.15 x ( 19 – 12 ) =3160.5 W

No. Space Load Components Sensible Heat Load Latent Heat


(W) Load (W)
1. Conduction through exterior wall 641.5 ---
2. Conduction through roof 7106 ---
3. Conduction through windows 4037.0 ---
4. Solar radiation through windows 72527 ---
5. Heat gained from people 2025 1650
6. Heat gained from lighting 11610 ---
7. Heat gained from equipment 3750 450
8. Heat gained by air infiltration 2722.5 3160.5
Total space cooling load 104419 5260.5
Total Cooling Load = 109679.5W/3516 =31.2 TR

(ii) Supply Air Quantity Calculation:


Supply air flow = Sensible heat gain / {1210 * (Room dry bulb temperature
– Supply dry bulb temperature)}
= 104419 W / {1210 J/m30K*(25 – 15)0C}
= 8.63 m3/s
Regular set A

-------- End of Section - III ---------


Regular Reg set B

Regn No: __________________


Name : __________________
(To be written by the candidate)

17th NATIONAL CERTIFICATION EXAMINATION


FOR
ENERGY MANAGERS & ENERGY AUDITORS – September, 2016

PAPER – 4:Energy Performance Assessment for Equipment and Utility Systems

Date: 24.09.2016 Timings: 14:00-16:00 HRS Duration: 2 HRS

General instructions:
o Please check that this question paper contains 6 printed pages
o Please check that this question paper contains 16 questions
o The question paper is divided into three sections
o All questions in all three sections are compulsory
o All parts of a question should be answered at one place

Section - I: BRIEF QUESTIONS

S-1 With evaporative cooling, it is possible to attain water temperatures below the
atmospheric wet bulb temperature. True or False
Ans False
S-2 Which parameter in the proximate analysis of coal is an index of ease of ignition?
Ans Volatile matter.
S-3 The major source of heat loss in a coal fired thermal power plant is through flue gas
losses in the boiler. True or false.
Ans False.
S-4 The COP of a vapour absorption refrigeration system is higher than the COP of a
vapour compression refrigeration system-True /false.
Ans False
S-5 In a compressed air system, the function of the inter cooler is to reduce the work of
compression. True or False
Ans True
S-6 De-aeration in boiler refers to removal of dissolved gases. True or false
Ans True
S-7 An industrial electrical system is operating at unity power factor. Addition of further
capacitors will reduce the maximum demand (kVA). True or False.
Ans False.
S-8 An air washer cools the air and a cooling tower cools the water. True or False.
Ans True.
S-9 A pump is retrofitted with a VFD and operated at full speed. Will the power
consumption increase or decrease or remain the same?
Ans Increase
Regular Reg set B

S-10 A 11 kW induction motor has an efficiency of 95% what will be its maximum delivered
output?
Ans 11 kW.
…………. End of Section - I ………….
Section - II: SHORT NUMERICAL QUESTIONS
L-1 While carrying out an energy audit of a pumping system, the treated water flow (in
open channel) was measured by the tracer method. 20% salt solution was used as the
tracer which was dosed @ 3 lts/min. The water analysis about 500 mtrs away revealed
salt concentration of 0.5%. Assuming complete mixing and no losses, calculate the
water flow rate.
Ans 20% salt solution = 200 gms of salt in 1 Litre of water
0.5% salt solution = 5 gms of salt in 1 litre of water
Dosing rate = 2 lts/min
Salt added in water = 2 x 200 = 400 gms/min
Total flow = 400/5 = 80 lts/min
Water flow rate = 80 – 3 = 77 lts/min
Or
C1V1 = C2V2
V2 = C1V1/C2 = 0.2x2/0.005 =80 lts/min

Actual flow = total flow – dosage flow = 80-3 =77 lts/min

L-2 In a petrochemical industry the LP & HP boilers have the same evaporation ratio of 14
using the same fuel oil. The operating details of LP & HP boiler are given below:
Particulars LP Boiler HP Boiler
Pressure 10 Kg./cm2a 32 Kg./cm2a
Temperature Saturated Steam 400oC
Enthalpy of steam 665 Kcal/kg 732 Kcal/kg
Enthalpy of feed water 80oC 100oC
Evaporation Ratio 14 14
Find out the efficiency of HP boiler if the LP boiler efficiency is 80%.
Ans Effy ᶯ = ER. (hg – hf) / GCV
EffyL.P ᶯ1 = 0.8 = 14 x(665 – 80) / GCV
EffyH.P ᶯ2 = 14 x(732 – 100) / GCV
EffyH.P ᶯ2 / EffyL.P ᶯ1= (732 – 100)0.8 / (665 – 80) = 0. 8643 =86.43%

Or
EffyL.P ᶯ1= 0.8 = 14 x(665 – 80) / GCV
GCV = 14x(665-80) / 0.8 = 10237.5kcal/kg
EffyH.P ᶯ2= 14 x(732 – 100) / GCV
= 14 x(732 – 100) / 10237.5 = 0.8643 = 86.43%

…………. End of Section - II ………….


Regular Reg set B

Section - III: LONG NUMERICAL QUESTIONS

N-1 In a beverages industry the product stream (liquid) flowing at a rate of 5000 kgs/hr at 90oC
is first cooled in counter type cooling water (CW) heat exchanger to 55 oC and then by a
chilled water (ChW) heat exchanger, to reduce temperature of the product to 11oC. The
specific heat of the product is 0.9 kCal/kgoC. The other operating data and parameters are:

Cooling Water heat exchanger Chilled Water heat exchanger


Inlet temp Outlet temp Inlet temp Outlet temp
o o o
Product 90 C 55 C Product 55 C 11 oC

Cooling 25 oC 32 oC Chilled 7 oC 12 oC
Water water

The chilled water is supplied by a reciprocating chiller, whose motor is drawing 60 KW with
a motor efficiency of 90%. The management decides to upgrade cooling water heat
exchanger by providing additional heat exchanger area to further enhance heat recovery
i.e. to reduce the temperature of product at its outlet to 40oC.
A. Depict the heat exchanger in existing and upgraded (improved) heat recovery
case in a simple block diagram

B. Calculate

i. The additional heat exchanger area (as a % of the existing area) for cooling water
heat exchanger, assuming there is no change in cooling water circulation rate and
the overall heat transfer coefficient.
ii. The COP of the chiller.
iii. Reduction in refrigeration /chiller load and yearly energy savings at 500 hours per
month operation, assuming energy consumption is proportional to load delivered.

Ans (A) Existing


mh = 5000 Kg/hr, 90oC product 55oC 11oC
Cooling water heat Chilled Water heat exchanger
Δ T1 exchanger area (A1) Δ T2

cw.out 32oC
ch.w out 12oC ch.w in 7oC

cw.in 25oC To chiller From chiller


…..2.5 marks

Heat load on CW heat exchanger = heat gained by CW


Heat rejected by product stream
Q1 = mc X 1 X (Δ Tc) = 5000 X 0.9 X (90 – 55)
Regular Reg set B

Q1 = 1,57,500 Kcal/hr
mc = Cooling Water flow rate = (5000X0.9 X 35)/(32 – 25)
= 22,500 Kg/hr.
Heat exchanger duty with increased heat = 5000 X 0.9 X (90 – 40)
recovery Q2 = 2,25,000 Kcal/hr
Cooling water temp. Rise with increased heat = 5000 X 0.9 X (90 – 40)
exchanger duty for cooling product stream to ------------------------------ = 10oC
40oC 22,500

Cooling water outlet temp with the above case = 25 + 10 = 35oC

(A) upgraded
mh = 5000 Kg/hr, 90oC product 40oC 11oC
Upgraded Cooling water heat Chilled Water heat exchanger
Δ T1 exchanger area (A2) Δ T2

cw.out 35oC
ch.w out 12oC ch.w in 7oC

cw.in 25oC To chiller From chiller

Δ T1 - Δ T2
LMTD1 with existing case = ----------------------
Ln Δ T1 / Δ T2

(90 – 32) - (55 – 25)


= -------------------------------- = 42.49oC
Ln (90 – 32) / (55 – 25)

(90 – 35) – ( 40 – 25)


LMTD2 with additional heat recovery = -------------------------------- = 30.77oC
Ln (90 – 35) / (40 – 25)

Q2 = U X A2 X LMTD2 Q1 = U X A1 X LMTD1
Q2 = 2,25,000 = U X A2 X 30.77 Q1 = 1,57,500 = U X A1 X 42.49

A2 / A1 = (2,25,000 /1,57,500) X (42.49 / 30.77) = 1.973

Additional area required = 97.3% of existing heat exchanger area of CW


heat exchanger
Refrigeration load in existing = 5000 X 0.9 X (55 – 11)
case
= 1,98,000 Kcal/hr
= 1,98,000 /3024
Regular Reg set B

= 65.476 TR
Motor input power = 60 KW
Motor eff. = 90%
C.O.P. of refrigeration chiller = 198000 /( 60 X 0.9 X 860)
= 4.26
Input KW / TR = 60 / 65.476 = 0.916
Reduction in refrigeration load = 5000 X 0.9 X (55 – 40) / 3024
due to lower input temperature = 22.32 TR
of the product to chilled water
heat exchanger
= 22.32 X 0.916 X 500 X 12
Yearly energy savings at 500 = 1,22,670.7 Kwh
hrs. operation per month

N-2 In a continuous process industry Steam and Power are supplied through a cogeneration
plant interconnected with grid. The design and actual operating parameters of the
cogeneration plant as represented in the schematic are given in the table below.

Double Extraction – Condensing Steam Turbine Cogeneration System


Design actual
B- Boiler 2 o 68.75tph,
75tph,64kg/cm (a), 450 C @82%
64kg/cm2(a), 450oC
efficiency
@81% efficiency
T- Steam Turbine Double Extraction – Condensing type
G- Generator 10MW 7.2MW

Stream Steam flow location Steam Flow Steam Steam Steam


Ref (tph) Pressure Temp (oC) enthalpy
(kg/cm2) (kCal/kg)
1 Steam input to 68.75 64 450 745
turbine
2 First extraction 18.75 17 270 697
3 Second extraction 31.25 9 200 673
Condenser in 18.75 0.1 - 550
4
Condenser out 18.75 - - 46

The industry is installing a 1200 TR double effect absorption chiller to meet the
Regular Reg set B

refrigeration load due to product diversification. Additional steam will be generated by the
boiler, which will go into the turbine and be extracted at 9kg/cm 2(a) to meet the VAM
requirement. The additional power thus generated will reduce the imported grid power.

The following additional data has been provided:


Maximum allowable steam flow the extraction at 9
40 TPH
Kg/cm2(a)
Minimum allowable steam to condenser 9 TPH
Critical power requirement of the plant 3800 KW
Power import from grid 500 KW
Cost of grid power Rs.7.00 / Kwh
G.C.V. of coal 4000 Kcal/Kg.
Cost of coal Rs. 4000/ton
Feed Water temperature 105oC
Feed Water enthalpy 105 Kcal/Kg.
Combined efficiency of gear box and generator 96%
Steam requirement for double effect absorption chiller 4.5 kg./TR hr at 9 Kg/cm2(a)
Annual hours of operation. 8000 hrs/y
Steam rate at 9 Kg/cm2(a) at 2nd extraction for 1 KW
12 (Kg/hr)/kW
turbine output
Ignore auxiliary power consumption and also return condensate from extracted steam to
process.
Calculate
(i) The Energy Utilization Factor (EUF) for the existing operating case
(ii) The net additional annual operating cost, after installation of VAM.
(iii) The Energy Utilization Factor (EUF) after installation of VAM.

Ans (i) Q thermal + P electrical


Energy Utilization Factor (EUF)= -----------------------------------
(before VAM installation) Fuel Consumption X G.C.V.

Q thermal = m2 h2 + m3h3 + m4h’4

m (h1 – hf)
Q in = -------------
Effy  X G.C.V.

Q thermal = 18750 X 697 + 31250 X 673 + 18750 X 46


= (18.75 X 697 + 31.25 X 673 + 18.75 X 46) X 103 Kcal/hr
= (13068 X 21031 + 862.5) X 103 Kcal/hr
= 34962.5 X 103 Kcal/hr

Pe = 7200 X 860
= 6192 X 103 Kcal/hr
Regular Reg set B

(745 – 105) X 68.750x1000


Fuel Consumption = -------------------------- = 13.58 TPH
0.81X 4000 X 1000

34962.5 X 103 + 6192 X 103


EUF = ---------------------------------- -x 100 = 75.76%
13.58 X 103 X 4000

(ii)
Refrigeration Load = 1200 TR

1TR requires 4.5 Kg./hr steam at 9 Kg./cm2a


Steam consumption in double effect absorption chiller = 1200 X 4.5
= 5400 Kg./hr.

Increase in steam extraction at 9 Kg./ cm2a = 5400Kg/hr

Every 12 Kg./hr extraction at 9 kg/ cm2a gives 1 KW output at turbine ,


efficiency of generator and gear box = 0.96

Additional power recovery due to increase in extraction = (5400 / 12) X 0.96


= 432 KW

Additional coal consumption due to increase in extraction


= (745 – 105)X 5400/(0.81x4000)
= 1066 kg/h
Additional cost of coal = 4000x1.066 = Rs 4266.6 /hr

Monetary realisation by reducing import cost of purchased electricity = 7 Rs./unit


= 432 X 7.00
= 3024 Rs./hr

Net additional annual operating cost after VAM installation = (4266.6-3024)*8000


= Rs. 99.41 lakhs /y

(iii)
Stream Steam flow Steam Steam Steam Steam
Ref location Flow Pressure Temp enthalpy
(tph) (kg/cm2) (oC) (kCal/kg)
1 Steam input to 68.75+5.4 64 450 745
turbine =74.15
2 First extraction 18.75 17 270 697
3 Second extraction 31.25+5.4 9 200 673
=36.65
Condenser in 18.75 0.1 - 550
4
Condenser out 18.75 - - 46
Regular Reg set B

Q thermal + P electrical
Energy Utilization Factor (EUF) = -----------------------------------
(after VAM installation) Fuel Consumption X G.C.V.

Q thermal = m2 h2 + m3h3 + m4h’4

m (h1 – hf)
Q in = -------------
Effy X G.C.V.

Q thermal = 18750 X 697 + 36650 X 673 + 18750 X 46


= (18.75 X 697 + 36.65 X 673 + 18.75 X 46) X 103 Kcal/hr
= (13068 X 21031 + 862.5) X 103 Kcal/hr
= 38596.7 X 103 Kcal/hr

Pe = (7200+432) X 860
= 6563.5 X 103 Kcal/hr

Fuel Consumption = 13.58 TPH +1.066 TPH = 14.646TPH

38596.7 X 103 + 6563.5 X 103


EUF = ----------------------------------------x100 = 77.08%
14.646 X 103 X 4000

N-3 In a chemical plant, a 3000 Million Cal/hr cooling tower with one CW pump caters to the
cooling water requirements. The management had decided to refurbish the cooling tower
as its performance is felt to be low. The operating parameters of the CW system before
and after refurbishment are presented below.
S.No Parameter Before After
refurbishment refurbishment
1 CW inlet temp to CT 35oC 35oC
o
2 Atmospheric air WbT -25 C, DbT - WbT -25 oC, DbT -
o
conditions 38 C 38 oC
3 COC 3.5 5
4 Suction head of CW -1m -1m
pump
5 Discharge pressure of 4kg/cm2(g) 4kg/cm2(g)
CW pump
6 Efficiency
CW Pump 54% 53%
CW Pump motor 89% 89%
CT fan 60% 64%
CT fan motor 90% 90%
7 Pressure developed by 20mmwc 20mmwc
CT fan
8 Effectiveness of CT 60 % 70%
Regular Reg set B

9 L/G ratio 1.5 1.5


10 Density of air 1.29kg/m3 1.29kg/m3

As a result of cooling tower refurbishment the effectiveness has increased from 60% to 70
%. Also with improved water treatment the COC has increased to 5.
Find out
1. Reduction in power consumption of pump and fan due to improvements in cooling
tower.

2. Reduction in make up water consumption (ignoring drift losses) in KL/day

Ans
Paramet Equation / formulae Before After refurbishment
er refurbishment
Effectiven = (TCWi-TCW0)/(TCWi- 0.6=(35- TCWO)/(35- 0.7=(35- TCWO)/(35-
ess WbT) 25) 25)
TCWO = 29 oC TCWO = 28 oC
CW flow = heat load/( TCWi-TCW0) =(3000x106/103) / ( =(3000x106/103)/(35-
rate Q 35- 29) 28)
= 500000 kg/h = 428571 kg/h
= 500 m3/h = 429 m3/hr
Evaporati =1.8*.00085*CW flow x 1.8x0.00085x500x 1.8x.00085x429x(35-
on loss Range (35-29) 28) = 4.59 m3/h
= 4.59 m3/h
Blow = Evaporation = 4.59/(3.5-1) 4.59/(5-1)
down loss Loss/(COC-1) = 1.84 m3/h = 1.15 m3/h
Total = Eva loss+ Blow down = 4.59+1.84 =4.59+1.15
water loss =6.43 m3/h =5.74 m3/h
loss
Make-up = Total water loss x = 6.43 x 24 = 5.74 x 24
water 24hrs = 154.2 m3/day = 137.76m3/day
=154.2KL/day =137.76 KL/day
Total = discharge head- = 40-(-1) = 41 mWC = 40-(-1) = 41 mWC
head H suction head
Pump = = =
LKW ((Q*1000/3600)*(H*9.8 (500*1000/3600)*(41 (429*1000/3600)*(41
1))/1000 *9.81)/1000 *9.81)/1000
= 55.86KW = 47.9 kW
Pump = Pump LKW/Eff.Pump =55.86/0.54 = 47.9/0.53
input =103.4 kW =90.4 kW
Motor = Pump input/motor = 103.4/0.9 =90.4/0.9
input eff =116.2 kW = 101.6kW
Air flow in =[( CW flow)x1000]/ = =
CT fan [((L/G)]*1.29) (500x1000/(1.5x1.29 (429x1000/(1.5x1.29)
Qf ) = 221705m3/h
= 258398 m3/h
Hf Pressure developed by = 20mmWC = 20mmWC
fan Hf
Regular Reg set B

Air KW = [(Qf in m3/h)*(Hf in =(258398*20)/(3600* =(221705*20)/(3600*


mmWC)]/(3600*102) 102) 102)
=14.07 kW = 12.08 kW
Fan =Air KW/(FanEffi =14.07/(0.60*0.9) =12.058/(0.64*0.9)
motor xMotor Eff) = 26.05kW = 20.93 kW
input
(1) Reduction in power of pump and motor = (116.2+26.05) - (101.6+20.93)= 19.72 kW

(2) Reduction in makeup water = 154.2-137.76 = 16.44 or 16.5 KL/day

N-4 Answer ANY ONE OF THE FOLLOWING among A, B, C and D

A) The operating parameters observed w.r.t. design in a 110 MW power generation unit are
given below:
Parameters Design Operation

Generator output 110 MW 110 MW


Steam generator outlet super heat 540oC 525oC
temperature

Steam generator outlet pressure 140 Kg/cm2a 130 Kg/cm2a


Feed water inlet temperature 135oC 135oC
Boiler  87.5% 87.5%
GCV of Coal 3800 3800
Turbine exhaust pressure 0.09 0.11
Kg./cm2a Kg./cm2a
Dryness fraction of exhaust steam 0.87 0.89
Regular Reg set B

Turbine heat rate 2362.5 Kcal ------


/Kwh

Efficiency generator 96% 96%


Energy loss in gear box 4420 KW 4420 KW
Enthalpy of steam at 520oC, 130 Kg/cm2a, 810
Kcal/Kg.
Enthalpy of steam at 0.11 Kg./cm2a 550
Kcal/Kg

Calculate the
I. Actual steam flow to the turbine

II. Specific steam consumption of turbine

III. % increase in gross unit heat rate compared to design

IV. Increase in monthly (720 hours/month) coal consumption due to deviation in


operation w.r.t. design at a plant load factor of 90%

Ans
Generator output = 110 MW
 of generator 96%
Generator input = 110 / 0.96
= 114.58 MW
Energy loss in gear box = 4420 KW
= 4.42 MW
Turbine out put = Total input at gear box +
Energy loss in gear box
Turbine out put = 114.58 + 4.42
= 119 MW

Turbine out put = ms (810 – 550) X (1 / 860)


ms (810 – 550) X (1 / 860) = 119 X 1000
Steam flow rate through the = 3,93,615 Kg /hr
turbine = 393.615 Tonne/hr

Specific steam consumption = (393.615 X 1000) / (110 X


1000)
= 3.58 Kg./kW
Boiler  = Ms (hg – hw)
--------------- X 100
GCV X mf
Regular Reg set B

= 393615 (810 – 135)


Coal consumption mf -------------------------
0.875 X 3800
79906.8 Kg./hr
= 79906.8 / (110 X 1000)
Specific coal consumption = 0.726 Kg./Kwh

Actual unit heat rate = 0.726 X 3800


= 2758.8 Kcal/Kwh

Design turbine heat rate = 2362.5 Kcal / KW


 of steam generator or boiler 87.5%

Design unit heat rate = 2362.5 / 0.875


= 2700 Kcal/Kwh
% increase in heat rate w.r.t. = [(2758.8 – 2700) / 2700] X 100
design = 2.17 %
Specific coal consumption for = 2700/3800
design heat rate = 0.71kg/kwh

= (0.726 – 0.71) X 110 X 1000 X 0.9 X 720


Additional coal consumption per ---------------------------------------------
month with a PLF of 90% --- 1000
= 1140.48 tonnes
Or
B) In a textile process house the production from the stenter machine is 72000 mtrs per day.
The effective operation of stenter is 20 hours per day. The percentage moisture in the
dried cloth (output) is 6% and its temperature is 75oC and wet cloth inlet is at 25oC . The
stenter is heated by steam at 8 Kg./cm2a and the daily steam consumption for the stenter
is 16.5 tonnes. The efficiency of the stenter dryer is 53%. Calculate the
(i) Linear speed of the stenter machine
(ii) Inlet moisture
(iii) Feed rate of the stenter.

The following data have been provided


Weight of 10 meter of dried cloth = 1 kg.
Enthalpy of the steam to the stenter = 665 kcal/kg.
Enthalpy of condensate at the exit of stenter = 130 kcal/kg.
Ignore losses in start-up and stoppage.

Ans Production per day = 72000 meters


Actual hours of operation = 20 hours/ day
Linear speed of the stenter = 72000 / (20x60) = 60 meters per min

Dried cloth output = 72000 / (20x10) = 360 kg/hr.

Moisture in dry cloth = 6%


Regular Reg set B

Bone dry cloth = 360 x 0.94 = 338.4 kg/hr


Moisture in outlet cloth mo = (360 – 338.4) / 338.4
= 0.0638 Kg./Kg. bone dry cloth

Steam consumption per day = 16.5 tonnes


= 16500 / 20 = 825 Kg./hr.

Heat load on the dryer =Energy input in steam x Dryer Efficiency


= Steam flow rate x (Enthalpy steam – Enthalpy condensate) x Efficiency Dryer
= 825 x (665 – 130) x 0.53
= 233928.75 Kcal/hr.

Further Heat load on the dryer = w x (mi – mo) X [(Tout – Tin) + 540] Kcal/hr.
w =weight of bone dry cloth rate kg/hr
mi = weight of cloth inlet moisture Kg./Kg. bone dry cloth
Tout = dried cloth outlet temperature = 75oC
Tin = wet cloth inlet temperature = 25oC

338.4 x (mi – 0.0638) X [(75 – 25) + 540] = 233928.75 Kcal/hr


mi = 1.235 Kg./Kg. bone dry cloth(1.235) / (1.235+1)x100
% inlet moisture in wet cloth = 55.25 %

total moisture in inlet cloth = 1.235x338.4 = 417.924 kg/hr

feed rate(inlet cloth rate), = total inlet moisture/hr +bone dry cloth/hr
= 417.92+338.4
= 756.32 Kg./hr.

or
C) The preheater exhaust gas from a cement kiln has the following composition on dry basis
:CO2 – 23.9%, O2 – 5.9%, CO – 0.2%, remaining is N2. The static pressure and
temperature measured in the duct are -730 mmWC and 3500C respectively. The velocity
pressure measured with a pitot tube is 19 mmWC in a duct of 2800 mm diameter ( Pitot
tube constant = 0.89 ). The atmospheric pressure at the site is 10350 mmWC and
universal gas constant is 847.84 mmWCm3/kg mol k. The specific heat capacity of
preheater exhaust gas is 0.25 kcals/kg0C.

The static pressure developed by PH exhaust fan is 630mmWC and power drawn is 1582
kW. Calculate the efficiency of fan given that the motor efficiency is 92%.

The management had decided to install a 1.3 MW power plant with a cycle efficiency of
12% by using this preheater exhaust gas. Calculate the exhaust gas temperature at the
outlet of waste heat recovery boiler of the power plant.
Ans Molecular weight exhaust gas (dry basis) M
= %CO2xMCO2 + %O2xMO2 + %COxMCO + %N2 x MN2
= {(23.9 x 44) + (5.9 x 32) + (0.2 x 28) + (70 x 28)}/100
= 32.06 kg/kg mole

Exhaust Gas density at operating temperature = γ= [ PM / RT ]


Regular Reg set B

= [ (10350 – 730) x 32.06 ) / { 847.84 x (273+350) }


= 0.584 kg/m3

Duct Area= 3.14 x( 2.8/2) 2 = 6.15 m2

Volume flow rate


=A Cp (2 x g x ∆P / γ)1/2 = 6.15 x 0.89 (2 x 9.81x 19/0.0.584)1/2
= 138.3 m3/s
Volume flow rate = 497880 m3/ h

Fan efficiency = volumetric flow rate x pressure developed


(102 x power drawn x motor eff)

= 138.3 x 630 x100 = 58.69%


(102x1582x0.92)

Mass flow rate of preheater exhaust gas = Volume flow rate x density
= 498194*0.584 = 2,90,945 kg/hr

Heat equivalent of power generated from power plant =1.3MW =1300 x 860
= 1118000 kCals/hr

Heat given up to power plant by exhaust gas = 290762 x0.25 x(350-To)x0.12

To = 350 - (1118000/(290945x0.25x0.12)) = 221.910C

or
D) For a commercial building, using the following data,

(i) Determine the building cooling load in TR

(ii) Calculate the supply air quantity to the cooling space in m3/s

Outdoor conditions : DBT = 40C, WBT = 28C, Humidity = 19 g of water / kg of dry air
Desired indoor conditions : DBT = 25C, RH = 60 %, Humidity = 12 g of water / kg of dry
air

Total area of wall = 324 m2, out of which 50% is window area.

U – Factor ( Wall ) = 0.33 W/m2K

U – Factor ( Roof ) = 0.323 W/m2K

U – factor [ fixed windows with aluminium frames and a thermal break ] = 3.56 W/m2K
Regular Reg set B

Other data:

 20 m x 25 m roof constructed of 100 mm concrete with 90 mm insulation & steel


decking.
 CLTD at 17:00 hr : Details : Wall = 12C; Roof = 44C; Glass Window = 7C
 SCL at 17 : 00 hr : Details : Glass Window = 605 W/ m2
 Shading coefficient of Window = 0.74
 Space is occupied from 8:00 to 17:00 hr by 30 people doing moderately active
work.
 Sensible heat gain / person = 75 W ; Latent heat gain / person = 55 W ; CLF for
people = 0.9
 Fluorescent light in space = 21.5 W/m2 ; CLF for lighting = 0.9
 Ballast factor details = 1.2 for fluorescent lights & 1.0 for incandescent lights
 Computers and office equipment in space produces 5.4 W/m2 of sensible heat
 One coffee maker produces 1050 W of sensible heat and 450 W of latent heat.
 Air changes/hr of infiltration = 0.3
 Height of building = 3.6 m
 Supply air dry bulb temperature is 170C
Ans i) Cooling Load Determination:

I. External Heat Gain

(i) Conduction heat gain through the wall = U – factor x net area of wall x CLTD
= 0.33 x (324*0.5) x 12 ] = 641.5 W

(ii) Conduction heat gain through the roof = U – factor x net area of roof x CLTD
= 0.323 x ( 20 x 25 ) x 44
= 7106 W

(iii) Conduction heat gain through the windows = U – factor x net area of windows x CLTD
= (3.56 x 162 x 7) = 4037 W

(iv) Solar radiation through glass


= Surface area x Shading coefficient x SCL
= (162 x 0.74 x 605) = 72527 W

II. Internal Heat Gain

(i) Heat gain from people =Sensible heat gain + Latent heat gain
Regular Reg set B

Sensible heat gain = (No. of people x Sensible heat gain / person x CLF)
=(30 x 75 x 0.9) = 2025 W

Latent heat gain = No. of people x Latent heat gain / person


= (30 x 55 ) = 1650 W
Therefore, Heat gain from people = (2025 + 1650 ) = 3675 W

(ii) Heat gain from lighting = (Energy input x Ballast factor x CLF)
Energy input = (Amount of lighting in space / unit area)x Floor area
= 21.5 x (20 x 25) = 10750 W

Therefore, heat gain from lighting = (10750 x 1.2 x 0.9) =11610 W

(iii) Heat generated by equipment :

Sensible heat generated by coffee maker =1050 W


Latent heat generated by coffee maker = 450 W
Sensible heat gain by computers and office equipment = 5.4 x 500 = 2700 W
Therefore, Heat generated by equipment = 4200 W

(iv)Heat gain through air infiltration= (Sensible heat gain + Latent heat gain)

Sensible heat gain =(1210 x airflow x ∆T )


Airflow = (Volume of space x air change rate ) / 3600
= { (20 x 25 x 3.6 ) x 0.3 } / 3600
= 0.15 m3 / s
Therefore, sensible heat gain =1210 x 0.15 x ( 40 – 25 ) =2722.5 W

Latent heat gain =3010 x 0.15 x ( 19 – 12 ) =3160.5 W

No. Space Load Components Sensible Heat Load Latent Heat


(W) Load (W)
1. Conduction through exterior wall 641.5 ---
2. Conduction through roof 7106 ---
3. Conduction through windows 4037.0 ---
4. Solar radiation through windows 72527 ---
5. Heat gained from people 2025 1650
6. Heat gained from lighting 11610 ---
7. Heat gained from equipment 3750 450
8. Heat gained by air infiltration 2722.5 3160.5
Total space cooling load 104419 5260.5
Total Cooling Load = 109679.5W/3516 =31.2 TR

(ii) Supply Air Quantity Calculation:

Supply air flow = Sensible heat gain / {1210 * (Room dry bulb temperature – Supply dry
bulb temperature)}
= 104419 W / {1210 J/m30K*(25 – 17)0C}
Regular Reg set B

= 10.78 m3/s

-------- End of Section - III ---------


SUPPLEMENTARY Paper 4 – Set A

16th NATIONAL CERTIFICATION EXAMINATION


FOR
ENERGY MANAGERS & ENERGY AUDITORS – September, 2015

PAPER – 4:Energy Performance Assessment for Equipment and Utility Systems

Date: 20.09.2015 Timings: 14:00-16:00 HRS Duration: 2 HRS Max. Marks: 100

General instructions:
o Please check that this question paper contains 6 printed pages
o Please check that this question paper contains 16 questions
o The question paper is divided into three sections
o All questions in all three sections are compulsory
o All parts of a question should be answered at one place

Section - I: BRIEF QUESTIONS Marks: 10 x 1 = 10

(i) Answer all Ten questions


(ii) Each question carries One mark

S-1 A direct driven centrifugal fan delivers more air after replacing its standard motor drive
with an energy efficient motor. Why?
Ans Since motor slip is reduced, speed increases and hence fan flow increases.
S-2 Between a natural gas fired boiler and oil fired boiler which will have a higher
percentage of hydrogen loss in flue gas? Why?
Ans Gas fired boiler. Because the hydrogen percentage is more in natural gas compared
to oil.
S-3 Due to gradual choking of AHU filter, AHU fan power decreased. Why?

Ans Due to increased resistance, the air flow decreased.

S-4 For a process requiring indirect heating to 200 oC, thermic fluid is preferred to steam
as a heat carrier. Why?
Ans Because for steam to be heated to high temperatures, the pressure required will be
very high.

S-5 If the condenser back pressure is 76 mm Hg, calculate the condenser vacuum.
if the atmospheric pressure is 745 mmHg.

Ans Condenser vacuum, mmHg = (Atmospheric pressure, mmHg - Condenser back


pressure, mmHg)
= (745 - 76) = 669 mmHg.
S-6 If the heat rate of a power plant is 2867 kCal/kWh, what is the efficiency of power
plant?

Ans 860/2867 x 100 = 30%


SUPPLEMENTARY Paper 4 – Set A

S-7 In low load region, current measurements are not a right indicator of motor loading.
Why?
Ans PF will be low.
S-8 The dry bulb and wet bulb temperatures of air entering an air washer are 35 and 28
0
C respectively. If the saturation efficiency is 90 %, calculate the air temperature
leaving the air washer.

Ans 90% = 35 -Tout


35-28

Tout = 28.7oC

S-9 Why can’t a boiler in normal operating conditions deliver its rated capacity?
Ans Because boiler are rated from and at 100 oC.
S-10 Why are water-cooled condensers more efficient than air-cooled condensers for
refrigeration applications?
Ans In water cooled condensers, the cooling water temperature can be bought below dry
bulb temperature.

…………. End of Section - I ………….

Section - II: SHORT NUMERICAL QUESTIONS Marks: 2 x 5 = 10

(i) Answer all Two questions


(ii) Each question carries Five marks

L-1 In a counter current heat exchanger, the hot stream enters at 70°C and leaves at
40°C. On the other hand, the cold stream enters at 20°C and leaves the heat
exchanger at 50°C. Determine the heat transferred in Kcal/hour if the area is 30 m2
and overall heat transfer coefficient is 800 W/m2 K

Ans For counter-current type:

LMTD = (70-50) – (40-20)


ln(70-50/40-20)

= 20 - 20
ln (20/20)

=0
---- (1.5 marks)
In this case LMTD is the same as the temperature difference on each end of the
heat exchanger (terminal temperature difference).

Hence LMTD = 20oC for counter-current flow.


---- (1.5 marks)
SUPPLEMENTARY Paper 4 – Set A

Heat transfer = (800 /1000) x 30 x 20 x 860 = 4,12,800 kcal/hr


---- (2 marks)

L-2 The gross heat rate of a thermal power plant is 2550 kcal/kWh and its net
heat rate is 2833.33 kcal/kWh. The plant is targeting to improve the net heat
rate by 30 kcal/kWh through reduction in auxiliary power consumption. What
will be its % auxiliary power consumption with the above improvement and
incremental reduction in auxiliary power consumption.

Solution:

Existing case:
Gross heat rate = 2550 kcal/kwh , Net heat rate: 2833.33 kcal/kwh

% Auxiliary power consumption in the existing case

= [ I – (2550/2833.33)] x 100

= 10
---- (2 marks)
Improved case:

Net heat rate = 2833.33 – 30.00 = 2803.33 kcal/kwh

% Auxiliary power consumption in the improved case


= [ I – (2550/2803.33)] x 100

=9
---- (2 marks)
Incremental reduction in Auxiliary power consumption

= 10 - 9
=1%

. ----- (1 mark)

…………. End of Section - II ………….

Section - III: LONG NUMERICAL QUESTIONS Marks: 4 x 20 = 80

(i) Answer all Four questions


(ii) Each question carries Twenty marks
SUPPLEMENTARY Paper 4 – Set A

N-1 A gas engine-based trigeneration plant operates in two modes:

• Power and heating mode (10 hours per day) :


Pel= 650 kW of electricity and 975 kg/h of steam with enthalpy addition of 530 kcal/kg of steam
EUFheat = 0.85

• Power and cooling mode (14 hours per day) :


Pel = 650 kW of electricity and chilling load of 213 TR for absorption chillers
EUFcool= 0.73

• Calorific value of natural gas = 8500 kcal/Sm3


• Average operating days/year = 330
• Alternator efficiency = 0.95
• The energy loss in the flue gas and that in the cooling water is same as engine power
output and other losses are negligible

Answer the following:

(1) What is the average plant energy utilization factor


(2) Calculate the useful energy produced daily by the trigeneration plant in MTOE
(3) Determine the daily plant natural gas requirements based on average energy utilization
factor
(4) The plant proposes to install a 60 TR hot water driven Vapour absorption chiller with a
COP of 0.5 using waste heat from jacket cooling water. Check if it is feasible with
supporting calculations.
SUPPLEMENTARY Paper 4 – Set A

Ans 1) Plant average energy utilization factor

Plant average energy utilization factor= (0.85 x 10 + 0.73 x 14)/24


= 0.78
….(3 marks)
2) The useful energy produced daily by the trigeneration plant in Gcal

PEle = 650 KW
QHeat = 975 x 530
= 516750 kcal/h
QCool = 213 x 3024
= 644112 kcal/h
….(2 marks)
Total daily useful energy production of (650 x 860 x 24 +516750 x 10 +
=
the plant 644112 x 14)
= 13416000 + 5137500 + 9017568
The useful energy produced daily = 27601068 kcal/day
The useful energy produced in
= 2.760
MTOE/day
….(4 marks)
3)The daily plant natural gas requirements
Input heat = 27601068 / 0.78
= 35385985 kcal/day
Natural gas requirements = 35385985 / 8500
= 4163 Sm3/day

….(4 marks)

4) Justification for a 60 TR Vapour Absorption chiller from waste heat of the


jacket cooling water

Heat required for operating 60 TR at = 60 x 3024/0.5


COP of 0.5 = 362880 Kcal/hr
….(2 marks)
650 /0.95
Power output of the engine =
684.2 KW
….(2 marks)
= 684.2 x 860
Heat in the jacket cooling water
= 588412 kcal/hr
….(2 marks)
Since the heat requirement (362880 Kcal/hr) is much less than heat available
(588412 kcal/hr) the proposal is feasible.
….(1 mark)
3
N-2 An engineering industry has a compressor of capacity 1700 m /h in operation. Free air delivery
test of the compressor was carried out by filling the receiver. The test and other data are given
SUPPLEMENTARY Paper 4 – Set A

below.

Receiver capacity : 10 m3
Interconnecting pipe : 1 m3
Atmospheric pressure : 1.03 kg/cm2a
Initial pressure in receiver : 1.0 kg/cm2a
Inlet air pressure to compressor : 1.0 kg/cm2a
Final pressure : 3.5 kg/cm2a
Time taken to fill the receiver : 3 minutes (180 seconds)
Inlet air temperature : 30 oC
Air temperature in the receiver : 40 oC

Motor rpm (N1) : 1400 rpm


Motor pulley diameter (D1) : 300mm
Compressor rpm (N2) : 700 rpm
Compressor Pulley diameter (D2) : 600 mm
Average duration of loading : 40 minutes in an hour
Average duration of unloading : 20 minutes in an hour
Power consumption during loading : 150 kW
Power consumption during unloading : 45 kW
Cost of energy : Rs. 5.00 per kWh

A: What is the operating free air delivery of the compressor?

B: Evaluate the cost of energy per day (24hrs operation).

C: The Plant was interested in reducing the unloading time of the compressor by reducing
the pulley diameter of the motor. Evaluate the speed of the compressor required for a
cycle of 10 minutes unloading and 50 minutes loading and accordingly evaluate the
diameter of the pulley of the motor.

D: Estimate the hourly power consumption and energy savings after replacement of the
pulley and payback period. Consider the cost of pulley and belt is Rs 40,000 and
operating hours of the compressor is 8000 in a year. (consider that the power
consumption was 120 kW during loading and 35 kW during unloading)
Ans P2  P1 V
A. Operating free air delivery of the compressor, Q   Nm3 / Minute
P0 T
Applying for temperature correction factor (273 + t1) / (273 + t2), Operating free air delivery
is:
Q1 = 3.5 – 1.0 x 11 x (273 + 30)
1.03 3 (273 + 40)
3
= 8.9 m /hr x 0.968
= 8.6 Nm3/min
= 516 Nm3/hr.
---- (5 marks)
SUPPLEMENTARY Paper 4 – Set A

B. Cost of energy per day


Average power consumption per hour = (150 x 40) + (45 x 20)
(40 + 20)
= 115 kW.

Average energy consumption per day = 115 x 24


= 2760 kWh.

Cost of energy per day = 2760 x 5 = Rs. 13,800 per day.


---- (5 marks)
C. Speed of compressor and Pulley diameter of motor
(for 10 minutes unloading and 50 minutes loading)

Air flow rate Q2 = (516 x 50) + (0 x 10)


60
3
= 430 m /hr.

(Q1 / Q2) compressor = (RPM 1 / RPM 2) compressor

516 / 430 = 700 / RPM2

RPM2 = 583 rpm.

(RPM1 / RPM2)Compressor = (D1/D2 )Motor

700/583=300/ D2

D2 = 250 mm.

(or) (Q1 / Q2) compressor = (D 1 / D 2)Motor

516 / 430 = 300 / D2

D2 = 250 mm.

Reduced motor pulley diameter = 250 mm


Reduced speed of the compressor = 583 rpm
---- (5 marks)
D. Energy Savings and Payback period(after replacement of pulley)

Average power consumption per hour = (120 x 50) + (35 x 10)


(50 + 10)
SUPPLEMENTARY Paper 4 – Set A

= 105.8 kW.
Power Savings = 115 – 105.8 = 9.2 kW.
Annual energy savings = 9.2 x 8000 = 73600 kWh/year.
Annual cost savings = 73600 x 5 = Rs. 3,68,000/year.
Payback Period = 40,000 / 3,68,000 = 1.3 months.
---- (5 marks)

N-3 Chilled water is circulated through the evaporator of a vapor compression chiller and
the outlet chilled water temperature is 7 oC. The evaporator is maintained at 2.5 oC.
Terminal Temperature Difference (TTD) on the chilled water inlet side is 5 oC higher
than chilled water outlet side.

Other given data:

 Overall heat transfer coefficient of the evaporator is 542.42 kcal/hr m2oC


 Area of the evaporator is 250 m2
 Efficiency of the compressor motor is 88%
 Condenser heat load is 20% more than the evaporator cooling load.

Calculate

a. LMTD of the evaporator


b. Refrigeration load or evaporator cooling load in tonne of refrigeration(TR)
c. C.O.P. of the chiller
d. Compressor input kW/TR
e. Indicate the operating parameters and the calculated values in a simple
schematic diagram
f. Find out the energy savings if the chilled water supply to one of the process
heat exchanger with a heat load of 90,000 kcal/hr operating for 6000 hrs in a
year is eliminated by process modification

Solution
a. ∆T2 = To - TE = 7 – 2.5 = 4.5oC

∆T1 = ∆T2+ 5 = 4.5 + 5 = 9.5oC

∆T1 = Ti - TE = 9.5oC

Ti = 9.5 + 2.5 = 12oC

Ti – To = 12 – 7 = 5oC
SUPPLEMENTARY Paper 4 – Set A

∆T1 - ∆T2
LMTD of evaporator = -------------- = 6.69 oC
ln 9.5 / 4.5
---- 4 marks

b. Refrigeration load or evaporator heat load

Heat load = U(kcal/hr m2 oC) x A (m2) x LMTD oC

= 542.42 x 250 x 6.69

= 9,07,197.45 kcal/hr

Refrigeration load = 9,07,197.45/3024

= 300 TR
---- 4 marks

c. Condenser heat load = 300 x 1.2 = 360 TR

d. Compressor input (heat energy) equivalent = 360 – 300 = 60 TR

COP = 300 / 60 = 5

Electrical input power to compressor motor

Input KW for compressor = 60 x 3024 / 860 = 210.98

Compressor chiller KW / TR = 210.98 / 300 = 0.703


---- 4 marks
e.
SUPPLEMENTARY Paper 4 – Set A

Schematic of the Vapor Compression Water Chiller indicating the operating parameters
and the calculated values
---- 4 marks

f. Reduction in chiller load = 90,000.00 kcal/hr

Reduction in refrigeration load = (90000.00) / 3024 = 29.76 TR

Compressor input power is 0.703kW/TR, motor efficiency is 88%

Saving in electrical energy = (29.76 x 0.703) / 0.88 = 23.774 KW

Annual energy savings = 23.774 x 6000 = 1,42,644 kWh


---- 4 marks

N-4 Answer ANY ONE OF THE FOLLOWING among A, B, C and D

A) A steam power plant consisting of high pressure Turbine (HP Turbine) and low
pressure Turbine (LP Turbine) is operating on Reheat cycle(schematic of power plant
is represented below).Steam from Boiler at a pressure of 150 bar(a) and a
temperature of 5500C expands through the HP Turbine. The exhaust steam from HP
SUPPLEMENTARY Paper 4 – Set A

Turbine is reheatedin a Reheater at a constant pressure of 40 bar (a) to 5500C and


then expanded through LP Turbine. The exhaust steam from LP Turbine is condensed
in a condenser at a pressure of 0.1 bar (a).The isentropic efficiency of HP Turbine and
LP Turbine is same and is 0.9. Generator efficiency is 95%

The other data of the power plant is as given below:

Main steam flow rate : 228 TPH


Enthalpy of main steam : 3450 KJ/kg
Enthalpy of feed water : 990.3KJ/kg
Isentropic Enthalpy of cold reheat steam : 3050 KJ/kg
Enthalpy of hot reheat steam : 3560 KJ/kg
Condenser pressure and temperature : 0.1 bar(a) and 45.80C
Isentropic enthalpy of LP Turbine exhaust steam : 2300 KJ/kg
Enthalpy of dry saturated steam at 0.1 bar(a) and 45.80C : 2584.9KJ/kg
Enthalpy of water at 0.1 bar(a) and 45.80C:191.9 KJ/kg

Based on the above data calculate the following parameters


(a) Power developed by the Generator
(b) Turbine heat rate
(c) Turbine cycle efficiency
(d)Dryness fraction of LP Turbine Exhaust steam
(e) Specific steam consumption of turbine cycle.
SUPPLEMENTARY Paper 4 – Set A

Ans SOLUTION:

(a) Power developed by the Generator: Turbine output x Generator efficiency---- (1)
Turbine output = Q1 (H1 – h2) + Q2(H3 – h4)/860 MW --------------- (2)
Where, Q1=main steam flow rate =228 TPH
H1=main steam enthalpy=3450 KJ/Kg
h2=actual enthalpy at HP Turbine outlet= cold reheat enthalpy
Q2=steam flow through reheater =228TPH
H3=enthalpy of hot reheat steam=3560 KJ/kg
h4= actual enthalpy of LP turbine exhaust steam=?
---- (1 mark)

HP Turbine isentropic efficiency= Actual enthalpy drop/isentropic enthalpy drop


0.9= (H1- h2)/(H1-h2is) ,
h2is =isentropic enthalpy of cold reheat Steam=3050KJ/kg
0.9= (3450 –h2)/(3450—3050)
h2= 3090KJ/kg
---- (3 marks)

LP Turbine isentropic efficiency= (H3—h4)/(H3—h4is),


h4is=isentropic enthalpy of LP Turbine Exhaust steam=2300KJ/kg
0.9=( 3560-h4)/(3560—2300)
h4= 2426 KJ/kg
---- (3 marks)

Substituting the values in equation-2,we get

Turbine output = 228(3450—3090) + 228(3560—2426)/860 =75.73MW


Generator output= 75.73 x 0.95= 71.5 MW
---- (3 marks)

(b) Turbine heat rate=Q1 (H1—hfw) +Q2(H3—h2)/Generator output =KJ/kwhr--------(3)


hfw=enthalpy of feed water=990.3KJ/kg
Substituting the values in the above equation-3, we get

Turbine heat rate=228 (3450—990.3) + 228(3560—3090)/71.5


=9342 KJ/kwhr
---- (3 marks)

(C) Turbine cycle efficiency= 860/Turbine heat rate


=860x 4.18/9342=38.5%
---- (2 marks)

(d) Dryness fraction of steam at 0.1 bar(a) and 45.80C

Actual enthalpy of LP Exhaust steam= enthalpy of water + dryness fraction of steam x


SUPPLEMENTARY Paper 4 – Set A

L.H of vaporisation of steam


2426 = 191.9+ dryness fraction of steam x(2584.9—191.9)

Dryness fraction of steam= 93.35%


---- (3 marks)

(e) Specific steam consumption of cycle=Steam flow/generator output


=228/71.5=3.19 tons/MWhr
---- (2 marks)
or
B) Stenter operations in a textile process were significantly improved to reduce inlet
moisture of from 60% to 55% in wet cloth while maintaining the same outlet moisture
of 7% in the dried cloth. The Stenter was operated at 80 meters/min in both the cases.
The dried cloth weighs 0.1 kg /meter. Further steps were taken to improve the
efficiency of the fuel oil fired thermic fluid from 80% to 82%, which was supplying heat
energy from to the dryer. The other data and particulars are,

Latent heat of water evaporated = 540kcal/kg,


Inlet temperature of wet cloth = 28oC ,
Outlet temperature of dried cloth = 80 oC,
Dryer efficiency = 50% ,
G.C.V of fuel oil = 10,300.00 kcal/kg,
Yearly operation of the stenter = 5000 hours

Find out the % reduction in Dryer heat load ,


Estimate the overall yearly fuel savings in tonnes by reducing moisture and efficiency
improvement compared to the initial case. Assume only energy for moisture
evaporation only for dryer heat load
Ans Solution:
Initial case: inlet moisture, 60%, outlet moisture 7%, dryer efficiency
50%,thermic fluid heater efficiency 80%

Output of stenter = 80 mts/min x 0.1 x 60 = 480 Kg/hr.

Moisture in the dried output cloth = 7%

Wt of bone- dry cloth = 480 X (1 – 0.07)


i.e. W = 446.4 Kg/hr.
---- (1.5 mark)

mo = moisture in outlet cloth= (480 – 446.4) /446.4 = 0.0753 Kg/Kg bone-dry cloth

---- (1mark)
Inlet moisture = 60%
Wt of inlet cloth = 446.4 / (1 – 0.60) = 1116.00 Kg./hr.
mi = moisture in inlet cloth
SUPPLEMENTARY Paper 4 – Set A

= (1116-446.4) ÷ 446.4
= 1.5 Kg./Kg. bone- dry cloth
---- (1 mark)
Inlet temperature of cloth Tin = 28oC
Final temperature of clothTout = 80oC

Heat load on the dryer = w x (mi – mo) X [(T out – Tin) + 540] Kcal/hr.

Heat load on the dryer = 446.4 (1.5 – 0.0753) X [(80 – 28) + 540]
= 3,76,503.76 Kcal/hr
---- (3 marks)

Efficiency of the dryer is50%, Efficiency of the thermic fluid heater is 80%

Fuel oil consumption in the thermic fluid heater


= 3,76503.76/( 0.5x 0.8x10300) = 91.40 kg/hr
---- (2.5 marks)

Improve case: inlet moisture, 55%, outlet moisture 7%, dryer efficiency
50%,thermic fluid heater efficiency 82%

Inlet moisture = 55%


Wt of inlet cloth = 446.4 / (1 – 0.55) = 992.00 Kg./hr.

mi = moisture in inlet cloth


= (992-446.4) ÷ 446.4
= 1.22 Kg./Kg. bone-dry cloth
---- (1.5 marks)
Heat load on the dryer = w x (mi – mo) X [(T out – Tin) + 540] Kcal/hr.

Heat load on the dryer = 446.4 (1.22 – 0.0753) X [(80 – 28) + 540]
= 3,02508.00 Kcal/hr
---- (3 marks)
Efficiency of the dryer is50%, Efficiency of the thermic fluid heater is 82%

Fuel oil consumption in the thermic fluid heater in impoved case


= 3,02,508.00/( 0.5x 0.82x10300) = 71.63 kg/hr
---- (2.5 marks)
% reduction in dryer load due to reduction inlet moisture
(3,76,504-3,02,508) x 100
= -----------------------------------------
(3,76,504)
= 19.65%
---- (2 marks)
Saving in fuel oil consumption in improved case
= 91.4- 71.63
= 19.77 kg/hr
SUPPLEMENTARY Paper 4 – Set A

Yearly fuel oil savings =19.77x5000 x1/1000


=98.85 tonnes
---- (2 marks)
or
C) In a steel industry, the composition of blast furnace gas by volume is as follows
CO – 27%, H2 - 2%, CO2 – 11%, N2 - 60%.

i) Calculate the stoichiometric air for combustion


ii) Calculate the gross calorific value of gas in kcal/Nm 3
iii) Calculate the net calorific value of gas in kcal/Nm 3
iv) If 3,00,000 Nm3/hr of gas is available and is to be co-fired in a coal fired boiler. How
much coal it can replace if the GCV of coal is 4000 kcal/kg.

Ans (i) Stoichiometric air for combustion:

C + O2 ------ CO2 + 8,084 kcal/kg Carbon


2C + O2 ------ 2 CO + 2,430 kcal/kg Carbon Available in
H2 + ½O2 -------H2O + 28,922 kcal/kg Hydrogen Book-2
CO + ½ O2 -------CO2 + 5,654 kcal/kg Carbon
---- (2 marks)
1 mole CO + 0.5 mole O2 ------ 1 mole CO2 + 5654 kCal/kg
For 27% CO, O2 required is (0.5/1) x 0.27 = 0.135 O 2
---- (2 marks)\
1 mole H2 + 0.5 mole O2 ------ 1 mole H2O + 28922 Kcal/kg
For 2 % of H2, O2 required is (0.5/1) x 0.02 = 0.01 O2
---- (2 marks)
Total stoichiometric oxygen required = 0.135 + 0.01 = 0.145 O2

Stoichiometric air required = 100 x 0.145 = 0.690 m3 air / m3 blast furnace gas
21
---- (3 marks)
(ii) Gross calorific value of gas:

1 kg mole of any gas at STP occupies 22.4 m3 of volume.


---- (1 mark)
Therefore,

((5654 x 12) / 22.4) x 0.27 = 817,83 kCal/m3 (molecular weight of Carbon = 12)
---- (2 marks)
((28922 x 2) / 22.4) x 0.02 = 51.64 kCal/m3 (molecular weight of Hydrogen = 2)
SUPPLEMENTARY Paper 4 – Set A

---- (2 marks)
Gross Calorific Value = 817.83 + 51.64 = 869.5 kcal/m3
---- (1 mark)

(iii) Replacement of coal by blast furnace gas:

Gross calorific value of coal = 4000 kcal/kg (given)


Blast furnace gas available = 3,00,000 m3/hr (given)

Heat content available from gas = 3,00,000 m3/hr x 869.5 kcal/m3


= 2608.5 x 105 kcal/hr
---- (2.5 marks)
If X is the coal quantity to be replaced, then
4000 kcal/kg x X = 2608.5 x 105 kcal/hr
X = 65212 kg/hr of coal can be replaced by gas of 3,00,000 m3/hr.
---- (2.5 marks)
or
D) As an energy auditor, auditing a cement plant, it is essential to assess the specific coal
consumption for the production of the clinker. With the following data available, calculate the
specific coal consumption (kgCoal/ kg Clinker).

S.No. Parameter Value

1. Reference temperature 20OC

2. Barometric pressure 10329 mmWC

3. Density of the gas at Pre-heater at NTP 1.436 kg/m3

4. Density of Air 1.293 kg/m3

5. Pitot Tube Constant 0.85

6. Clinker production rate 4127 TPD

7. Static Pressure of the Pre-heater gas in the pre-heater duct 640 mmWC

8. Dynamic pressure of the pre-heater gas in the duct 15.8 mmWC

9. Temperature of the Pre-heater gas 320OC

10. Specific heat of the Pre-heater gas 0.247kCal/kg 0C

11. Area of the Pre-heater Duct 8.5 m2


SUPPLEMENTARY Paper 4 – Set A

12. Temperature of the exit clinker 128OC

13. Specific heat of the clinker 0.193 kCal/kg 0C

14. Static Pressure of the Cooler Exhaust gas in the duct 42mmWC

15. Dynamic pressure of the Cooler Exhaust gas in the duct 15.5mmWC

16. Temperature of the Cooler Exhaust gas gas 290

17. Specific heat of the Cooler Exhaust gas 0.247kCal/kg 0C

18. Area of the Cooler exhaust duct 7.1m2

19. Heat of Formation of Clinker 405 kcal/kgClinker

20. All other heat loss except heat loss through Pre-heater gas, exiting 84.3 kcal/kg Clinker
clinker and cooler exhaust gases
21. All heat inputs except heat due to Combustion of fuel (Coal) 29 kcal/kg Clinker

22. GCV of the Coal 5500 kcal/kg

Ans Solution:

Heat Lost in the along with the Exiting pre-heater gases:


QPH Gas = mphgas × Cpphgas × (tephgas-tr)
mphgas = VphgasX  Phgas
Vphgas = vph gas X A

Corrected density of the pre-heater gas:


10329  640 273
 Phgas = 1.436  
10334 273  320
= 0.6198 kg/ m3

Velocity (v) = Pt × √(2g(ΔPdynamic)avg /  Phgas) m/sec

2  9.81  15.8
= 0.85  m/sec
0.6198
= 19.0 m/sec

VPH gas = 19.0m3/s X 8.5 m2


= 161.5 m3/sec
= 5,81,400m3/hr

Mph gas = 581400 m3/hr X 0.6198 kg/m3


= 3,60,351/72 kg/hr
SUPPLEMENTARY Paper 4 – Set A

mphgas = 3,60,351 kg/hr / 1,71,958 kg/hr = 2.095 Kgph gas/ Kg clinker


QPH Gas = 2.095 X 0.247 X (320 -20)
= 155.24 kcal/kgClinker
---- (7 marks)

Heat Lost along with the Exiting Hot Clinker:


QHot clinker = mclinker× Cpclinker× (tclinker - tr)
= 1 x 0.193 x (128 – 20)
= 20.84 kCal/kgClinker
---- (2 marks)
Heat Lost along with the Exiting Cooler Exhaust gases:
QCooler Exhaust Gas = mCooler Exhaust Gas × CpCooler Exhaust Gas × (tCooler Exhaust Gas-tr)
mCooler Exhaust Gas = VCooler Exhaust Gas X  Cooler Exhaust Gas
VCooler Exhaust Gas = vCooler Exhaust Gas X A

Corrected density of the pre-heater gas:


10329  42 273
 Cooler Exhaust gas = 1.293  
10334 273  290
= 0.624 kg/ m3

Velocity (v) =Pt × √(2g(ΔPdynamic)avg /  Cooler Exhausts) m/sec

2  9.81  15.5
= 0.85  m/sec
0.624
= 18.76 m/sec

VcoolerExhaustgas = 18.76m/s X 7.1 m2


= 133.196 m3/sec
= 4,79,505m3/hr

McoolerExhaustgas = 479505 m3/hr X 0.624 kg/m3


=2,99,211 Kg/hr

mcoolerExhaustgas = 2,99,211 kg/hr / 1,71,958 kg/hr =1.74 KgcoolerExhaustgas/ Kg clinker

QcoolerExhaustgas = 1.74 X 0.244 X (290 -20)


= 114.63Kcal/KgClinker

---- (7 marks)

Heat Input = Heat output


Heat Inputcoal + Heat inputothers = HeatClikerfrmtn+ HeatPH gas + HeatCliker+ Heatcooler exhaust gas +
Heatothers
SUPPLEMENTARY Paper 4 – Set A

GCVcoalX m coal + 29 = 405 + 155.24 + 20.84 +114.63 + 84.3


mcoal = 751 / 5500
= 0.137 Kgcoal/Kgclinker

---- (4 marks)

-------- End of Section - III ---------


SUPPLEMENTARY Paper 4 – Set B

Note to Evaluator: Please give marks for the steps & logic. A mistake in value in initial step
would lead to subsequent steps getting wrong values. Consider 75% marks, if step is right.

16th NATIONAL CERTIFICATION EXAMINATION


FOR
ENERGY MANAGERS & ENERGY AUDITORS – September, 2015

PAPER – 4:Energy Performance Assessment for Equipment and Utility Systems

Date: 20.09.2015 Timings: 14:00-16:00 HRS Duration: 2 HRS Max. Marks: 100

General instructions:
o Please check that this question paper contains 7 printed pages
o Please check that this question paper contains 16 questions
o The question paper is divided into three sections
o All questions in all three sections are compulsory
o All parts of a question should be answered at one place

Section - I: BRIEF QUESTIONS Marks: 10 x 1 = 10

(i) Answer all Ten questions


(ii) Each question carries One mark

S-1 In low load region, current measurements are not a right indicator of motor loading.
Why?
Ans PF will be low.

S-2 Due to gradual choking of AHU filter, AHU fan power decreased. Why?

Ans Due to increased resistance, the air flow decreased.

S-3 Why are water-cooled condensers more efficient than air-cooled condensers for
refrigeration applications?
Ans In water cooled condensers, the cooling water temperature can be bought below dry
bulb temperature

S-4 The dry bulb and wet bulb temperatures of air entering an air washer are 35 and 28
0
C respectively. If the saturation efficiency is 90%, calculate the air temperature
leaving the air washer.

Ans 90% = 35 -Tout


35-28

_______________________
Bureau of Energy Efficiency
1
SUPPLEMENTARY Paper 4 – Set B

Tout = 28.7oC

S-5 For a process requiring indirect heating to 200 oC, thermic fluid is preferred to steam
as a heat carrier. Why?
Ans Because for steam to be heated to high temperatures, the pressure required will be
very high.

S-6 If the condenser back pressure is 82 mm Hg, calculate the condenser vacuum.
if the atmospheric pressure is 745 mmHg.

Ans Condenser vacuum, mmHg = (Atmospheric pressure, mmHg - Condenser back


pressure, mmHg)
= (745 - 82) = 663 mmHg.

S-7 A direct driven centrifugal fan delivers more air after replacing its standard motor drive
with an energy efficient motor. Why?
Ans Since motor slip is reduced, speed increases and hence fan flow increases.
S-8 Between a natural gas fired boiler and oil fired boiler which will have a higher
percentage of hydrogen loss in flue gas? Why ?
Ans Gas fired boiler. Because the hydrogen percentage is more in natural gas compared
to oil.
S-9 Why can’t a boiler in normal operating conditions deliver its rated capacity?
Ans Because boiler are rated from and at 100 oC.
S-10 If the heat rate of a power plant is 1967 kCal/kWh, what is the efficiency of power
plant?
Ans 860/1967 x 100 = 43.7%

…………. End of Section - I ………….

Section - II: SHORT NUMERICAL QUESTIONS Marks: 2 x 5 = 10

(i) Answer all Two questions


(ii) Each question carries Five marks

L-1 The gross heat rate of a thermal power plant is 2550 kcal/kWh and its net
heat rate is 2833.33 kcal/kWh. The plant is targeting to improve the net heat
rate by 50 kcal/kWh through reduction in auxiliary power consumption. What
will be its % auxiliary power consumption with the above improvement and
incremental reduction in auxiliary power consumption.

_______________________
Bureau of Energy Efficiency
2
SUPPLEMENTARY Paper 4 – Set B

Solution:

Existing case:
Gross heat rate = 2550 kcal/kWh, Net heat rate: 2833.33 kcal/kwh

% Auxiliary power consumption in the existing case

= [ I – (2550/2833.33)] x 100

= 10
---- 2 marks
Improved case:

Net heat rate = 2833.33 – 50.00 = 2783.33 kcal/kwh

% Auxiliary power consumption in the improved case


= [ I – (2550/2783.33)] x 100

= 8.38
---- 2 marks
Incremental reduction in Auxiliary power consumption

= 10 -8.38
= 1.62%

---- 1 mark
L-2 In a counter current heat exchanger, the hot stream enters at 80°C and leaves at
50°C. On the other hand, the cold stream enters at 20°C and leaves the heat
exchanger at 50°C. Determine the heat transferred in Kcal/hour if the area is 30 m 2
and overall heat transfer coefficient is 800 W/m 2 K.

Ans For counter-current type:

LMTD = (80-50) – (50-20)


ln(80-50/50-20)

= 30 - 30
ln (30/30)

=0
--- 1.5 marks

_______________________
Bureau of Energy Efficiency
3
SUPPLEMENTARY Paper 4 – Set B

In this case LMTD is the same as the temperature difference on each end of the
heat exchanger (terminal temperature difference).

Hence LMTD = 20oC for counter-current flow.


--- 1.5 marks

Heat transfer = (800 /1000) x 30 x 30 x 860 = 6,19,200 kcal/hr


--- 2 marks

…………. End of Section - II ………….

Section - III: LONG NUMERICAL QUESTIONS Marks: 4 x 20 = 80

(i) Answer all Four questions


(ii) Each question carries Twenty marks

N-1 A gas engine-based trigeneration plant operates in two modes:

• Power and heating mode (10 hours per day) :


Pel= 650 kW of electricity and 975 kg/h of steam with enthalpy addition of 530 kcal/kg of
steam
EUFheat = 0.85

• Power and cooling mode (14 hours per day) :


Pel = 650 kW of electricity and chilling load of 256 TR for absorption chillers
EUFcool = 0.73

• Calorific value of natural gas = 8500 kcal/Sm3


• Average operating days/year = 330
• Alternator efficiency = 0.95
• The energy loss in the flue gas and that in the cooling water is same as engine power output
and other losses are negligible

Find out the following:

(1) Plant average energy utilization factor


(2) The useful energy produced daily by the trigeneration plant in MTOE
(3) The daily plant natural gas requirements based on average energy utilization factor
(4) The plant proposes to install a 60 TR hot water driven Vapour absorption chiller with a
COP of 0.5 using waste heat from jacket cooling water. Check if it is feasible with
supporting calculations.

_______________________
Bureau of Energy Efficiency
4
SUPPLEMENTARY Paper 4 – Set B

Ans PEle = 650 KW


QHeat = 975 x 530
= 516750 kcal/h
QCool = 256 x 3024
= 774144 kcal/h
….(2 marks)
Plant average energy utilization = (0.85 x 10 + 0.73 x 14)/24
factor
= 0.78
….(3 marks)
Total daily useful energy production of = (650 x 860 x 24 + 516750 x 10 + 774144 x 14)
the plant
= 13416000 + 5167500 + 10838016
= 29421516 kcal/day
The useful energy produced daily = 29421516/107
by trigeneration plant in MTOE/day
= 2.94
….(4 marks)
Input heat = 29421516 / 0.78
= 37719892 kcal/day
Natural gas requirements = 37719892 / 8500
= 4438 Sm3/day
….(4 marks)
4) Justification for a 60 TR Vapour Absorption chiller from waste heat of the jacket
cooling water
Heat required for operating 60 TR at = 60 x 3024/0.5
COP of 0.5 = 362880 Kcal/hr
….(2 marks)
650 /0.95
Power output of the engine =
684.2 KW
….(2 marks)
= 684.2 x 860
Heat in the jacket cooling water
= 588412 kcal/hr
….(2 marks)
Since the heat requirement (362880 kcal/hr) is much less than heat available (588412
kcal/hr) the proposal is feasible.
….(1 mark)
3
N-2 An engineering industry has a compressor of capacity 2500 m /h in operation. Free air delivery
test of the compressor was carried out by filling the receiver. The test and other data are given
below.

Receiver capacity : 9 m3
Interconnecting pipe : 1 m3
_______________________
Bureau of Energy Efficiency
5
SUPPLEMENTARY Paper 4 – Set B

Atmospheric pressure : 1.03 kg/cm2a


Initial pressure in receiver : 1.0 kg/cm2a
Inlet air pressure to compressor 1.0 kg/cm2a
Final pressure : 5 kg/cm2a
Time taken to fill the receiver : 3 minutes (180 seconds)
Inlet air temperature : 30 oC
Air temperature in the receiver : 40 oC

Motor rpm (D1) : 1440


Motor pulley diameter (N1) : 300mm
Compressor rpm (D2) : 650 rpm
Compressor Pulley diameter (N2) : 600 mm
Average duration of loading : 40 minutes in an hour
Average duration of unloading : 20 minutes in an hour
Power consumption during loading : 150 kW
Power consumption during unloading : 45 kW
Cost of energy : Rs. 5.00 per kWh

A: What is the operating free air delivery of the compressor?

B: Evaluate the cost of energy per day (24hrs operation).

C: The Plant was interested in reducing the unloading time of the compressor by reducing
the pulley diameter of the motor. Evaluate the speed of the compressor required for a
cycle of 10 minutes unloading and 50 minutes loading and accordingly evaluate the
diameter of the pulley of the motor.

D: Estimate the hourly power consumption and energy savings after replacement of the
pulley and payback period. Consider the cost of pulley and belts is Rs 40,000 and
operating hours of the compressor is 8000 in a year. (consider that the power
consumption was 120 kW during loading and 35 kW during unloading)
Ans P2  P1 V
A. Operating free air delivery of the compressor, Q   Nm3 / Minute
P0 T
Applying for temperature correction factor (273 + t1) / (273 + t2), Operating free air delivery
is:
Q1 = 5 – 1.0 x 10 x (273 + 30)
1.03 3 (273 + 40)
= 12.95 m3/hr x 0.968
= 12.5 Nm3/min
= 750 Nm3/hr.

_______________________
Bureau of Energy Efficiency
6
SUPPLEMENTARY Paper 4 – Set B

---- 5 marks
B. Cost of energy per day
Average power consumption per hour = (150 x 40) + (45 x 20)
(40 + 20)
= 115 kW.

Average energy consumption per day = 115 x 24


= 2760 kWh.

Cost of energy per day = 2760 x 5 = Rs. 13,800 per day.


---- 5 marks

C. Speed of compressor and Pulley diameter of motor


(for 10 minutes unloading and 50 minutes loading)

Air flow rate Q2= (750 x 50) + (0 x 10)


60
= 625 m3/hr.

( Q1 / Q2) compressor= (RPM 1 / RPM 2) compressor

750 / 625 = 650 / RPM2

RPM2 = 542 rpm.

(RPM1 / RPM2)Compressor = (D1/D2 )Motor

650/542=300/ D2

D2 = 250 mm.

(or) (Q1 / Q2) compressor = (D 1 / D 2)Motor

750 / 625 = 300 / D2

D2 = 250 mm.

Reduced motor pulley diameter = 250 mm

_______________________
Bureau of Energy Efficiency
7
SUPPLEMENTARY Paper 4 – Set B

Reduced speed of the compressor = 542 rpm


---- 5 marks

D. Energy Savings and Payback period (after replacement of pulley)

Average power consumption per hour = (120 x 50) + (35 x 10)


(50 + 10)
= 105.8 kW.
Power Savings = 115 – 105.8 = 9.2 kW.
Annual energy savings = 9.2 x 8000 = 73600 kWh/year.
Annual cost savings = 73600 x 5 = Rs. 3,68,000/year.
Payback Period = 40,000 / 3,68,000 = 1.3 months.
---- 5 marks
N-3 Chilled water is circulated through the evaporator of a vapor compression chiller and
the outlet chilled water temperature is 7 oC. The evaporator is maintained at 2.5 oC.
Terminal Temperature Difference (TTD) on the chilled water inlet side is 5 oC higher
than chilled water outlet side.

Other given data:

 Overall heat transfer coefficient of the evaporator – 542.42 kcal/hr m2oC


 Area of the evaporator – 250 m2
 Efficiency of the compressor motor is 88%
 Condenser heat load is 20% more than the evaporator cooling load.

Calculate

a. LMTD of the evaporator


b. Refrigeration load or evaporator cooling load in tonne refrigeration(TR)
c. C.O.P. of the chiller
d. Compressor input kW/TR
e. Indicate the operating parameters and the calculated values in a simple
schematic diagram
f. Find out the energy savings if the chilled water supply to one of the process
heat exchanger with a heat load of 90,000 kcal/hr operating for 8000 hrs in a
year is eliminated by process modification

_______________________
Bureau of Energy Efficiency
8
SUPPLEMENTARY Paper 4 – Set B

Solution
a. ∆T2 = To - TE = 7 – 2.5 = 4.5oC

∆T1 = ∆T2+ 5 = 4.5 + 5 = 9.5oC

∆T1 = Ti - TE = 9.5oC

Ti = 9.5 + 2.5 = 12oC

Ti – To = 12 – 7 = 5oC
∆T1 - ∆T2
LMTD of evaporator = -------------- = 6.69 oC
ln 9.5 / 4.5
---- 4 marks

b. Refrigeration load or evaporator heat load

Heat load = U(kcal/hr m2 C) x A (m2) x LMTD oC

= 542.42 x 250 x 6.69

= 9,07,197.45 kcal/hr

Refrigeration load = 9,07,197.45/3024

= 300 TR
---- 4 marks

c. Condenser heat load = 300 x 1.2 = 360 TR

d. Compressor input (heat energy) equivalent = 360 – 300 = 60 TR

COP = 300 / 60 = 5

Electrical input power to compressor motor

Input KW for compressor = 60 x 3024 / 860 = 210.98

Compressor chiller KW / TR = 210.98 / 300 = 0.703


---- 4 marks
_______________________
Bureau of Energy Efficiency
9
SUPPLEMENTARY Paper 4 – Set B

e.

Schematic of the Vapor Compression Water Chiler indicating the operating parameters
and the calculated values
---- 4 marks

f. Reduction in chiller load = 90,000.00 kcal/hr

Reduction in refrigeration load = (90000.00) / 3024 = 29.76 TR

Compressor input power is 0.703kw/TR, motor efficiency is 88%

Saving in electrical energy = (29.76 x 0.703) / 0.88 = 23.774 KW

Annual energy savings = 23.774 x 8000 = 1,90,192.00 kwh


---- 4 marks

N-4 Answer ANY ONE OF THE FOLLOWING among A, B, C and D

A) A steam power plant consisting of high pressure Turbine(HP Turbine) and low pressure
Turbine(LP Turbine) is operating on Reheat cycle(schematic of power plant is
_______________________
Bureau of Energy Efficiency
10
SUPPLEMENTARY Paper 4 – Set B

represented below).Steam from Boiler at a pressure of 150 bar(a) and a temperature of


5500C expands through the HP Turbine. The exhaust steam from HP Turbine is
reheatedin a Reheater at a constant pressure of 40 bar (a) to 550 0C and then expanded
through LP Turbine. The exhaust steam from LP Turbine is condensed in a condenser
at a pressure of 0.1 bar (a).The isentropic efficiency of HP Turbine and LP Turbine is
same and is 90%. Generator efficiency is 95%

The other data of the power plant is as given below:

Main steam flow rate : 228 TPH


Enthalpy of main steam : 3450 KJ/kg
Enthalpy of feed water : 990.3KJ/kg
Isentropic Enthalpy of cold reheat steam : 3050 KJ/kg
Enthalpy of hot reheat steam : 3560 KJ/kg
Condenser pressure and temperature : 0.1 bar(a) and 45.80C
Isentropic enthalpy of LP Turbine exhaust steam : 2300 KJ/kg
Enthalpy of dry saturated steam at 0.1 bar(a) and 45.8 0C : 2584.9KJ/kg
Enthalpy of water at 0.1 bar(a) and 45.8 0C:191.9 KJ/kg

_______________________
Bureau of Energy Efficiency
11
SUPPLEMENTARY Paper 4 – Set B

Based on the above data calculate the following parameters


(a) Power developed by the Generator
(b) Turbine heat rate
(c) Turbine cycle efficiency
(d)Dryness fraction of LP Turbine Exhaust steam
(e) Specific steam consumption of turbine cycle.

Ans SOLUTION:

(a) Power developed by the Generator: Turbine output x Generator efficiency--------(1)


Turbine out put = Q1 (H1 – h2) + Q2(H3 – h4)/860 MW -----------------(2)
Where, Q1=main steam flow rate =228 TPH
H1=main steam enthalpy=3450 KJ/Kg
h2=actual enthalpy at HP Turbine outlet= cold reheat enthalpy
Q2=steam flow through reheater=228TPH
H3=enthalpy of hot reheat steam=3560 KJ/kg
h4= actual enthalpy of LP turbine exhaust steam=?
---- 1 mark

HP Turbine isentropic efficiency= Actual enthalpy drop/isentropic enthalpy drop


0.9= (H1- h2)/(H1-h2is) , h2is=isentropic enthalpy of cold reheat steam=3050KJ/kg

0.9= (3450 –h2)/(3450—3050)


h2= 3090KJ/kg
---- 3 marks

LP Turbine isentropic efficiency= (H3—h4)/(H3—h4is), h4is=isentropic enthalpy of LP


Turbine Exhaust steam=2300KJ/kg

0.9=( 3560-h4)/(3560—2300)
h4= 2426 KJ/kg
---- 3 marks

Substituting the values in equation-2,we get

Turbine output = 228(3450—3090) + 228(3560—2426)/860 =75.73MW


Generator output= 75.73 x 0.95= 71.5 MW
---- 3 marks

(b) Turbine heat rate=Q1 (H1—hfw) +Q2(H3—h2)/Generator output =KJ/kwhr---------(3)


hfw=enthalpy of feed water=990.3KJ/kg
Substituting the values in the above equation-3, we get
_______________________
Bureau of Energy Efficiency
12
SUPPLEMENTARY Paper 4 – Set B

Turbine heat rate = 228 (3450—990.3) + 228(3560—3090)/71.5


= 9342 KJ/kWhr
---- 3 marks

(C) Turbine cycle efficiency= 860/Turbine heat rate


=860/9342=38.5%
---- (2 marks)

(d)Dryness fraction of steam at 0.1 bar(a) and 45.80C

Actual enthalpy of LP Exhaust steam= enthalpy of water + dryness fraction of steam x


L.H of vaporisation of steam
2426 = 191.9+ dryness fraction of steam x (2584.9—191.9)

Dryness fraction of steam= 93.35%


---- (3 marks)

(e) Specific steam consumption of cycle = Steam flow/generator output


= 228/71.5 = 3.19 tons/MWhr
---- (2 marks)

or
B) Stenter operations in a textile process were significantly improved to reduce inlet
moisture of from 60% to 55% in wet cloth while maintaining the same outlet moisture of
7% in the dried cloth . The Stenter was operated at 80 meters/min in both the cases.
The dried cloth weighs 0.1 kg /meter . Further steps were taken to improve the efficiency
of the fuel oil fired thermic fluid from 80% to 82%, which was supplying heat energy
from to the dryer. The other data and particulars are,

Latent heat of water evoprated = 540kcal/kg,


Inlet temperature of wetcloth = 28oC ,
Outlet temperature of dried cloth = 80 oC,
Dryer efficiency = 50% ,
G.C.V of fuel oil = 10,300.00 kcal/kg,
Yearly operation of the stenter = 7000 hours

Find out the % reduction in Dryer heat load ,


Estimate the overall yearly fuel savings in tonnes by reducing moisture and efficiency
improvement compared to the initial case. Assume only energy for moisture evaporation
only for dryer heat load

_______________________
Bureau of Energy Efficiency
13
SUPPLEMENTARY Paper 4 – Set B

Ans Solution:
Initial case: inlet moisture, 60%, outlet moisture 7%, dryer efficiency
50%,thermic fluid heater efficiency 80%

Output of stenter = 80 mts/min x 0.1 x 60


= 480 Kg/hr.

Moisture in the dried output cloth = 7%

Wt of bone- dry cloth = 480 X (1 – 0.07)


i.e. W = 446.4 Kg/hr.
---- (1.5 mark)
mo = moisture in outlet cloth= (480 – 446.4) /446.4
= 0.0753 Kg./Kg.bone-dry cloth
---- (1 mark)
Inlet moisture = 60%
Wt of inlet cloth = 446.4 / (1 – 0.60) = 1116.00 Kg/hr.

mi = moisture in inlet cloth


= (1160.0 - 446.4) ÷ 446.4 = 1.5 Kg/Kg bone- dry
cloth
---- (1 mark)

Inlet temperature of cloth Tin = 28oC


Final temperature of clothTout = 80oC

Heat load on the dryer = w x (mi – mo) X [(T out – Tin) + 540] Kcal/hr.

Heat load on the dryer = 446.4 (1.5 – 0.0753) X [(80 – 28) + 540]
= 3,76,503.76 Kcal/hr
---- (3 marks)

Efficiency of the dryer is50%, Efficiency of the thermic fluid heater is 80%

Fuel oil consumption in the thermic fluid heater


=3,76503.76/( 0.5x 0.8x10300) = 91.40 kg/hr

---- (2.5 marks)

Improve case: inlet moisture, 55%, outlet moisture 7%, dryer efficiency
50%,thermic fluid heater efficiency 82%

_______________________
Bureau of Energy Efficiency
14
SUPPLEMENTARY Paper 4 – Set B

Inlet moisture = 55%


Wt of inlet cloth = 446.4 / (1 – 0.55) = 992.00 Kg./hr.

mi = moisture in inlet cloth


= (992-446.4) ÷ 446.4 = 1.22 Kg/Kg bone-dry cloth

---- (1.5 marks)

Heat load on the dryer = w x (mi – mo) X [(T out – Tin) + 540] Kcal/hr.

Heat load on the dryer = 446.4 (1.22 – 0.0753) X [(80 – 28) + 540]
= 3,02508.00 Kcal/hr
---- (3 marks)

Efficiency of the dryer is50%, Efficiency of the thermic fluid heater is 82%

Fuel oil consumption in the thermic fluid heater in impoved case


=3,02,508.00/( 0.5x 0.82x10300) = 71.63 kg/hr
---- (2.5 marks)

% reduction in dryer load due to reduction inlet moisture


( 3,76,504-3,02,508)x 100
= -----------------------------------------
( 3,76,504)

= 19.65%
---- (2 marks)
Saving in fuel oil consumption in improved case
= 91.4- 71.63
= 19.77 kg/hr

Yearly fuel oil savings =19.77x7000 x1/1000


=138.39 tonnes
---- (2 marks)
or
C) In a steel industry, the composition of blast furnace gas by volume is as follows
CO – 27%, H2 - 2%, CO2 – 11%, N2 - 60%.

i) Calculate the stoichiometric air for combustion


ii) Calculate the gross calorific value of gas in kcal/Nm 3
iii) Calculate the net calorific value of gas in kcal/Nm3
iv) If 3,00,000 Nm3/hr of gas is available and is to be co-fired in a coal fired boiler. How much
_______________________
Bureau of Energy Efficiency
15
SUPPLEMENTARY Paper 4 – Set B

coal it can replace if the GCV of coal is 4300 kcal/kg.

Ans (i) Stoichiometric air for combustion:

C + O2 ------ CO2 + 8,084 kcal/kg Carbon


2C + O2 ------ 2 CO + 2,430 kcal/kg Carbon Available in Book-2
H2 + ½O2 -------H2O + 28,922 kcal/kg Hydrogen
CO + ½ O2 -------CO2 + 5,654 kcal/kg Carbon
---- (2 marks)
1 mole CO + 0.5 mole O2 ------ 1 mole CO2 + 5654 kCal/kg
For 27% CO, O2 required is (0.5/1) x 0.27 = 0.135 O 2
---- (2 marks)
1 mole H2 + 0.5 mole O2 ------ 1 mole H2O + 28922 Kcal/kg
For 2 % of H2, O2 required is (0.5/1) x 0.02 = 0.01 O2
---- (2 marks)
Total stoichiometric oxygen required = 0.135 + 0.01 = 0.145 O2
Stoichiometric air required = 100 x 0.145 = 0.690 m3 air / m3 blast furnace gas
21
---- (3 marks)
(ii) Gross calorific value of gas:

1 kg mole of any gas at STP occupies 22.4 m3 of volume.


---- (1 mark)
Therefore,

((5654 x 12) / 22.4) x 0.27 = 817,83 kCal/m3 (molecular weight of Carbon = 12)
---- (2 marks)
((28922 x 2) / 22.4) x 0.02 = 51.64 kCal/m3 (molecular weight of Hydrogen = 2)
---- (2 marks)
Gross Calorific Value = 817.83 + 51.64 = 869.5 kcal/m3
---- (1 mark)

(iii) Replacement of coal by blast furnace gas:

Gross calorific value of coal = 4300 kcal/kg (given)


Blast furnace gas available = 3,00,000 m3/hr (given)
_______________________
Bureau of Energy Efficiency
16
SUPPLEMENTARY Paper 4 – Set B

Heat content available from gas = 3,00,000 m3/hr x 869.5 kcal/m3


= 2608.5 x 105 kcal/hr
---- (2.5 marks)
If X is the coal quantity to be replaced, then
4300 kcal/kg x X = 2608.5 x 10 5 kcal/hr
X = 60663 kg/hr of coal can be replaced by gas of 3,00,000 m3/hr.
---- (2.5 marks)
or
D) As an energy auditor, auditing a cement plant, it is essential to assess the specific coal
consumption for the production of the clinker. With the following data available, calculate the
specific coal consumption (kgCoal/ KgClinker).

S.No Parameter Value

1. Reference temperature 20OC

2. Barometric pressure 10329 mmWC

3. Density of the Pre-heater at NTP 1.436kg/m3

4. Density of Air 1.293Kg/m3

5. Pitot Tube Constant 0.85

6. Clinker production rate 4127 TPD

7. Static Pressure of the gas at Pre-heater gas in the pre-heater duct 640mmWC

8. Dynamic pressure of the pre-heater gas in the duct 15.8mmWC

9. Temperature of the Pre-heater gas 320OC

10. Specific heat of the Pre-heater gas 0.247kCal/kg 0C

11. Area of the Pre-heater Duct 8.5 m2

12. Temperature of the exit clinker 128OC

13. Specific heat of the clinker 0.193 kCal/kg 0C

14. Static Pressure of the Cooler Exhaust gas in the duct 42mmWC

_______________________
Bureau of Energy Efficiency
17
SUPPLEMENTARY Paper 4 – Set B

15. Dynamic pressure of the Cooler Exhaust gas in the duct 15.5mmWC

16. Temperature of the Cooler Exhaust gas gas 290

17. Specific heat of the Cooler Exhaust gas 0.247kCal/kg 0C

18. Area of the Cooler exhaust duct 7.1m2

19. Heat of Formation of Clinker 405 Kcal/KgClinker

20. All other heat loss except heat loss through Pre-heater gas, 84.3 Kcal/Kg Clinker
exiting clinker and cooler exhaust gases
21. All heat inputs except heat due to Combustion of fuel (Coal) 29 Kcal/Kg Clinker

22. GCV of the Coal 6200Kcal/Kg

Ans Heat Lost along with the Exiting pre-heater gases:


QPH Gas = mphgas × Cpphgas × (tephgas-tr)
mphgas = VphgasX  Phgas
Vphgas = vph gas X A

Corrected density of the pre-heater gas:


10329  640 273
 Phgas = 1.436  
10334 273  320

= 0.6198 kg/ m3

Velocity (v) =Pt × √(2g(ΔPdynamic)avg /  Phgas) m/sec

2  9.81  15.8
= 0.85  m/sec
0.6198
= 19.0 m/sec

VPH gas = 19.0m3/s X 8.5 m2


= 161.5 m3/sec
= 5,81,400m3/hr

_______________________
Bureau of Energy Efficiency
18
SUPPLEMENTARY Paper 4 – Set B

Mph gas = 581400 m3/hr X 0.6198 kg/m3


= 3,60,351/72 Kg/hr

mphgas = 3,60,351 kg/hr / 1,71,958 kg/hr =2.095Kgph gas/ Kg clinker


QPH Gas = 2.095 X 0.247 X (320 -20)
= 155.24Kcal/KgClinker
----- 7 marks

Heat Lost along with the Exiting Hot Clinker:


QHot clinker = mclinker× Cpclinker× ( tclinker- tr)
= 1 x 0.193 x (128 – 20),
= 20.84 kCal/kgClinker
---- 2 marks
Heat Lost along with the Exiting Cooler Exhaust gases:
QCooler Exhaust Gas = mCooler Exhaust Gas × CpCooler Exhaust Gas × (tCooler Exhaust Gas-tr)
mCooler Exhaust Gas = VCooler Exhaust Gas X  Cooler Exhaust Gas
VCooler Exhaust Gas = vCooler Exhaust Gas X A

Corrected density of the pre-heater gas:


10329  42 273
 Cooler Exhaust gas = 1.293  
10334 273  290

= 0.624 kg/ m3

Velocity (v) = Pt × √(2g(ΔPdynamic)avg /  Cooler Exhausts) m/sec

2  9.81  15.5
= 0.85  m/sec
0.624
= 18.76 m/sec

VcoolerExhaustgas = 18.76m/s X 7.1 m2


= 133.196 m3/sec
= 4,79,505m3/hr

McoolerExhaustgas = 479505 m3/hr X 0.624 kg/m3


= 2,99,211 Kg/hr

mcoolerExhaustgas = 2,99,211 kg/hr / 1,71,958 kg/hr =1.74 KgcoolerExhaustgas/ Kg clinker


_______________________
Bureau of Energy Efficiency
19
SUPPLEMENTARY Paper 4 – Set B

QcoolerExhaustgas = 1.74 X 0.244 X (290 -20)


= 114.63Kcal/KgClinker
------- 7 marks

Heat Input = Heat output

Heat Inputcoal + Heat inputothers = HeatClikerfrmtn+ HeatPH gas + HeatCliker+ Heatcooler exhaust gas gas +
Heatothers

GCVcoalX m coal + 29 = 405 + 155.24 + 20.84 +114.63 + 84.3

mcoal = 751 / 6200


= 0.121 Kgcoal/Kgclinker
------- 4 marks

-------- End of Section - III ---------

_______________________
Bureau of Energy Efficiency
20
REGULAR Paper 4 – Set A

16th NATIONAL CERTIFICATION EXAMINATION


FOR
ENERGY MANAGERS & ENERGY AUDITORS – September, 2015

PAPER – 4:Energy Performance Assessment for Equipment and Utility Systems

Date: 20.09.2015 Timings: 14:00-16:00 HRS Duration: 2 HRS Max. Marks: 100

Section - I: BRIEF QUESTIONS Marks: 10 x 1 = 10

(i) Answer all Ten questions


(ii) Each question carries One mark

S-1 Why is the exhaust temperature of furnace oil fired systems limited to about 170 0C?
Ans Acid dew point due to presence of sulphur
S-2 The net present value of a energy conservation project is Rs.48,784/- and the initial
capital investment Rs,2,00,000/- calculate the profitability index of the project.
Ans PI = 48784 = 0.244
2,00,000

S-3 The dry bulb and wet bulb temperatures of air entering an air washer are 35 and 28
0
C respectively. If the saturation efficiency is 90 %, calculate the air temperature
leaving the air washer.

Ans 90% = 35 -Tout


35-28

Tout = 28.7oC

S-4 Other than exhaust gas what is the major source of waste heat recovery in a water
cooled DG set?
Ans Engine jacket cooling water
S-5 In poorly loaded motor, current measurements are not a right indicator of motor
loading. Why?
Ans PF will be low.
S-6 If the condenser back pressure is 76 mm Hg, calculate the condenser vacuum.
if the atmospheric pressure is 745 mmHg.

Ans Condenser vacuum, mmHg = (Atmospheric pressure, mmHg - Condenser back


pressure, mmHg)
= (745 - 76) = 669 mmHg.
REGULAR Paper 4 – Set A

S-7 If the coal GCV is 4000 kcal/kg and specific coal consumption is 0.65 kg/kWh, what is
the power station gross efficiency?
(860 /(4000 x 0.65)) x100 = 33.07%

S-8 For a process requiring indirect heating to 200 oC, thermic fluid is preferred to steam
as a heat carrier. Why?
Ans Because for steam to be heated to high temperatures, the pressure required will be
very high.
S-9 Between a natural gas fired boiler and oil fired boiler which will have a higher
percentage of hydrogen loss in flue gas? Why ?
Ans Gas fired boiler. Because the hydrogen percentage is more in natural gas compared
to oil.
S-10 After cleaning of choked AHU filter, AHU fan power increased. Why?

Ans Due to less resistance, the air flow increased.

…………. End of Section - I ………….

Section - II: SHORT NUMERICAL QUESTIONS Marks: 2 x 5 = 10

(i) Answer all Two questions


(ii) Each question carries Five marks

L-1 A pump is drawing water through a 150 mm diameter pipe with a suction head of 3.5
m below the pump centre line. Find out the pump efficiency if the actual power input
the motor is 17.6 kW at a motor efficiency of 90 %. The discharge pressure is 4.5
kg/cm2 and the velocity of water through the pipe as measured by an ultrasonic flow
meter is 1 m/s.
REGULAR Paper 4 – Set A

Ans Discharge Head, kg/cm2 = 4.5


Suction Head, m = - 3.5
Total Head = 45 - (-3.5)
48.5 m
Flow rate = (22/7 x D2/4) x 1 m/s
= (22/7 x 0.152 / 4) x 1 m/s
= 0.0177 m3/sec
….. 2 marks
Hydraulic Power = 0.0177 x 1000 x 9.81 x
48.5/1000

= 8.42 kW
….. 1 mark
Pump Efficiency 8.42/(17.6x0.9)
= 53.2 %
….. 2 marks
L-2 A luxury hotel is using a diesel fired heater with an efficiency of 70% for supplying
hot water at 55oC from an initial temperature of 20 oC. The hot water requirement is
24,000 litres per day.

The management is considering to install a specially designed electric heat pump for
the specific high hot water temperature requirement with a heat pump coefficient of
performance (C. O. P.) of 2. Find out the reduction in daily operating cost with heat
pump in place of diesel fired heater ignoring auxiliary energy consumption. The
following data are given.

Electricity cost = Rs.10/kWh


Diesel cost = Rs.50/litre
G.C.V. of diesel = 9100 kcal/litre
Ans Solution:

Diesel required
For hot water heater = [24000 LitHotwater/ day ) x (55-20oC) x (1 kcal/LitoC)]
(0.7 Effy x 9100 kcal/Lit diesel )

= 131.9 Litdiesel /day 1 Mark

Diesel cost / day = 131.9 x 50 = 6595 Rs./day


….. 1 mark
COP = Heat pump refrigeration effect / Input electrical energy

Or

Input electrical energy, kW = Heat pump refrigeration effect, kcal


COP x 1 kW
Or
REGULAR Paper 4 – Set A

Input electrical energy, kW = Heat pump refrigeration effect, kcal


COP x 860 kcal/hr

Electrical energy required with heat = 24000 x 1 x (55 – 20)


(2 x 860) ..1 Mark
pump of COP = 2

Energy input with heat pump = 488.372 kWh/day

Operating cost with heat pump = 488.372 x 10


= 4883.72 Rs./day
….. 1 mark
Reduction in operating cost = 6595 – 4883.72
= Rs.1711.28 /day

….. 1 mark

…………. End of Section - II ………….

Section - III: LONG NUMERICAL QUESTIONS Marks: 4 x 20 = 80

(i) Answer all Four questions


(ii) Each question carries Twenty marks

N-1 In an organic chemical industry 10 Tonne per hour steam is generated at 10


Kgf/cm2 in a 12 TPH natural gas fired smoke tube boiler. The % oxygen in the
exit flue gas was 3.5% and the flue gas temperature was 190oC. The following
data have been provided.

Ultimate analysis of natural gas per kg ,


Carbon = 0.72 kg/kg; Hydrogen = 0.236 kg/kg; Nitrogen = 0.03
kg/kg; Oxygen = 0.011 kg/kg;

Specific heat of flue gas = 0.297 Kcal/kgoC


Specific heat of superheated water vapor = 0.45 Kcal/kgoC
G.C.V. of natural gas = 9100 Kcal/m3
Density of natural gas = 0.7
Density of air = 1.12 kg/m3
Enthalpy of steam at 10 kg/cm2 = 665 Kcal/kg
Temperature of feed water at inlet to boiler = 95oC
Yearly hours of operation = 6000 hours

a. Find out the S/F (steam to fuel) ratio in kg steam/m3 gas


REGULAR Paper 4 – Set A

b. Estimate the annual reduction in carbon dioxide emission in tones/year


compared to the furnace oil fired boiler of 83% efficiency on G.C.V.
which was earlier used for delivering the same steam load. Assume
G.C.V. of furnace oil as 10300 Kcal/kg and 0.86 carbon per kg.
furnace oil.

Ans Ultimate analysis of natural gas per kg. of gas

Carbon = 0.72 kg/kg; Hydrogen = 0.236 kg/kg; Nitrogen = 0.03


kg/kg; Oxygen = 0.011 kg/kg;

Theoretical air required


= 11.6C + [34.8 (H2 – O2/8)] + 4.35S,
= 11.6 X 0.72 + [34.8 (0.236 – 0.011/8)]
(note S= sulfur in above composition is nil)
= 16.524 kg air/kg gas ….. 1 Mark

% Excess Air = [% O2 / (21 - % O2)] x 100


= [3.5 / (21 – 3.5)] x 100 = 20% ….. 1 Mark

Actual Air Supplied (AAS) = [1 + 0.2] x 16.524 = 19.83kg air / kg gas

…. (1 mark)

Mass of dry flue gas; mdfg = mass of combustion gases due to presence
C, N2,S in the fuel+mass of residual O2 in flue gas + mass of N2
supplied with air

= 0.72 X 44/12 + 0.03 + (19.83 – 16.524) x 0.23 + 19.83 x 0.77


= 18.70 kg dfg / kg gas
…. (1.5 marks)

L1 = % heat loss due to dry flue gases

mdfg x cpfg x (Tg – Ta)


= ------------------------------ X 100
G.C.V. of gas

Kcal / m3 9100
G.C.V. of gas = ----------- = --------- = 13000 Kcal/kg
Density 0.7

18.69 x 0.297 x (190 – 30)


= -------------------------------- X 100 = 6.84 %
13000
…..(2 marks)
REGULAR Paper 4 – Set A

L2 = Loss due to presence of hydrogen forming water vapor

9H [584 + Cps x (Tg – Ta)


= ---------------------------------- X 100
G.C.V.

9 x 0.236 [584 + 0.45 (190 – 30)


= --------------------------------------------- x 100
13000
L2 = 10.72 %
….(2 marks)

Radiation and unaccounted losses in the boiler (given) = 1.45%

Total losses = 6.84 + 10.72 + 1.45 = 19%

Efficiency of natural gas fired boiler on = 100 – 19 = 81%


G.C.V. by indicated method
…..(1.5 mark)

Steam to fuel ratio in kg steam/m3 gas= 0.81 x 9100 / (665 – 95) =


12.93
…. (2 marks)

Amount of gas required for generation = (10,000 / 12.93) x 0.7


10 tonne/hr of steam
= 541.38 kg/hour …(1.5 Marks)

CO2 emission with natural gas firing = 0.72 x 3.67 x 541.38

(1 kg carbon gives 44/12 i.e. 3.67 kg CO2)


= 1430.54 kg/hr
…. (1.5 marks)

Furnace oil required for 10000 kg steam = (10,000 x 570) / (0.83 x


10,300)
= 666.74 kg/hr … (1.5 Marks)

CO2 emission with furnace oil firing = 0.86 x 3.67 x 666.74

= 2104.36 kg/hr
….(1.5 marks)

Net reduction in CO2 emission with = 2104.36 – 1430.54


natural gas compared to furnace
oil firing
REGULAR Paper 4 – Set A

= 673.82 kg/hr
….. (1 mark)
Annual reduction in CO2 for 6000 hrs. = 673.82 x 6000
operation

= 4042.920 Tonnes
….. (1 mark)

N-2 A gas engine-based trigeneration plant operates in two modes:

• Power and heating mode (10 hours per day) :


Pel= 650 kW of electricity and 325 kg/h of steam with enthalpy addition of 530 kcal/kg of
steam & EUFheat = 0.85

• Power and cooling mode (14 hours per day) :


Pel = 650 kW of electricity and chilling load of 213 TR for absorption chillers &
EUFcool = 0.73

• Calorific value of natural gas = 8500 kcal/Sm3


• Average operating days/year = 330
• Alternator efficiency = 0.95
• The energy loss in the flue gas and that in the cooling water is same as engine power
output and other losses are negligible

Answer the following:

a. What is the average plant energy utilization factor


b. Calculate the useful energy produced daily by the trigeneration plant in Gcal
c. Determine the daily plant natural gas requirements based on average energy
utilization factor
d. The plant proposes to install a 60 TR hot water driven Vapour absorption chiller with
a COP of 0.5 using waste heat from jacket cooling water. Check if it is feasible with
supporting calculations.

Ans 1) Plant average energy utilization factor

Plant average energy utilization factor = (0.85 x 10 + 0.73 x 14)/24


= 0.78
….(3 marks)
2) The useful energy produced daily by the trigeneration plant in Gcal

PElect = 650 KW
QHeat = 325 x 530
= 172250 kcal/h
QCool = 213 x 3024
= 644112 kcal/h
REGULAR Paper 4 – Set A

….(2 marks)
Total daily useful energy production (650 x 860 x 24 +172250 x 10 +
=
of the plant 644112 x 14)
13416000 + 1722500 +
=
9017568
The useful energy produced daily = 24156068 kcal/day …(2 Marks)
The useful energy produced in 24156068x 330 / 106
=
Gcal
= 7971.5 Gcal
….(2 marks)
3)The daily plant natural gas requirements
Input heat = 24156068 / 0.78

= 3096931795 kcal/day (2 Marks)


Natural gas requirements = 3096931795 / 8500

= 3643 Sm3/day

.…(4 marks)
4) Justification for a 60 TR Vapour Absorption chiller from waste heat of the
jacket cooling water

Heat required for operating 60 TR at = 60 x 3024/0.5


COP of 0.5 = 362880 Kcal/hr
….(2 marks)
650 /0.95
Power output of the engine =
684.2 KW
….(2 marks)
= 684.2 x 860
Heat in the jacket cooling water
= 588412 kcal/hr
….(2 marks)
Since the heat requirement (362880 kcal/hr) is much less than heat
available (588412 kcal/hr) the proposal is feasible.
….(1 mark)
REGULAR Paper 4 – Set A

N-3 Hot effluent having a flow rate of 63450 Kg/hr at 80 OC from the process is sent to a
heat exchanger for cooling. The outlet temperature of effluent in the heat exchanger is
38 OC. Air having a flow rate of 370057 Kg/hr enters the heat exchanger at a
temperature of 30OC and leaves at 60 OC. Power drawn by the fan is 30 KW. The plant
works for 16 hours a day for 330 days per year.

Now plant has decided to replace air cooled heat exchanger with a water cooled
counter current Heat Exchanger.

Given that Pump Efficiency = 75%, Motor efficiency = 90 %, Effectiveness of water


cooled heat exchanger is 0.4, water is available at 25 OC & Pressure drop in plate heat
exchanger is 1.2 kg/cm2 , Over all heat transfer coefficient of heat exchanger is 22300
Kcal/m2/OC.

1. Calculate the savings due to replacement by water cooled heat exchanger

2. Calculate the heat transfer area of heat exchanger.


Ans Solution:

Heat Duty
Heat duty in hot fluid = M x Cph x (Ti - To)
= 63450 x 1 x (80 - 38)
= 2664900 kcal / Kg
…. (2 marks)
Heat duty in cold Air = M x Cpair x (to – ti)
= 370057 x 0.24 x (60 – 30)
= 2664410 Kcal / Kg
…. (2 marks)

In heat exchanger, Heat duty in hot fluid = Heat duty in cold Air

Effectiveness of water cooled heat exchanger = 0.4

Cold Water outlet – Cold water inlet


Effectiveness = Hot effluent inlet – Cold water inlet

Cold Water Outlet = (0.4 x (80 – 25)) + 25

= 47 OC
…. (2.5 marks)

Mass flow rate of cooling water (M) = Heat duty in hot fluid
Cp x (Cold water outlet – Cold water inlet)

= ___2664900
1 x (47 – 25) x 1000

= 121.13 m3 /hr
…. (2.5 marks)
REGULAR Paper 4 – Set A

Pressure drop in Plate Heat exchanger = 12 m

Hydraulic Power Requirement for one Cooling Water Pump:

= (Flow in m3/Hr x Head in m x Density in Kg/m3 x g in m/s2)


(1000 x 3600)

= (121.13 x 12 x 1000 x 9.81)


(1000 x 3600)

= 3.96 KW
…. (3 marks)

Pump Power Requirement at 75% pump efficiency = 3.96 KW


0.75
= 5.28 KW
…. (1 mark)

Motor Input Power Required at 90% Efficiency = 5.28


0.9
= 5.87 KW
…. (1 mark)

Savings = Power consumption by fans – Water Pumping Power


= 30 – 5.87
= 24.13 KW

Annual Saving in kWh = 24.13 KW x 16 Hrs x 330 Days = 127406 KWh/Annum


…. (2 marks)

Calculations for LMTD for Proposed HEx:

LMTD for counter current flow in HEx

= {(80-47) – (38-25)} / ln {(80-47) / (38-25)}


= 21.5 Deg C
…. (2 marks)

Considering overall heat transfer coefficient (U) = 22300 kcal/m2/0C

Heat transfer Area = Q


(U x ∆Tlm)

= 2664900
(22300 x 21.5)
REGULAR Paper 4 – Set A

= 5.6 m2 (Say 6 m2)


…. (2 marks)

N-4 Answer ANY ONE OF THE FOLLOWING among A, B, C and D

A) A steam power plant consisting of high pressure Turbine (HP Turbine) and low pressure
Turbine (LP Turbine) is operating on Reheat cycle(schematic of power plant is
represented below).Steam from Boiler at a pressure of 150 bar(a) and a temperature of
5500C expands through the HP Turbine. The exhaust steam from HP Turbine is
reheated in a Reheater at a constant pressure of 40 bar (a) to 550 0C and then expanded
through the LP Turbine. The exhaust steam from LP Turbine is condensed in a
condenser at a pressure of 0.1 bar (a).The isentropic efficiency of HP Turbine and LP
Turbine is same and is 90%. Generator efficiency is 95%

The other data of the power plant is as given below:

Main steam flow rate : 228 TPH


Enthalpy of main steam : 3450 KJ/kg
Enthalpy of feed water : 990.3KJ/kg
Isentropic Enthalpy of cold reheat steam : 3050 KJ/kg
Enthalpy of hot reheat steam : 3560 KJ/kg
Condenser pressure and temperature : 0.1 bar(a) and 45.80C
Isentropic enthalpy of LP Turbine exhaust steam : 2300 KJ/kg
Enthalpy of dry saturated steam at 0.1 bar(a) and 45.80C : 2584.9KJ/kg
Enthalpy of water at 0.1 bar(a) and 45.80C:191.9 KJ/kg
REGULAR Paper 4 – Set A

Based on the above data calculate the following parameters


(a) Power developed by the Generator
(b) Turbine heat rate
(c) Turbine cycle efficiency
(d)Dryness fraction of LP Turbine Exhaust steam
(e) Specific steam consumption of turbine cycle.

Ans SOLUTION:

(A) Power developed by the Generator: Turbine output x Generator efficiency--(1)

Turbine output = Q1 (H1 – h2) + Q2(H3 – h4)/860 MW ----------- (2)


Where, Q1= main steam flow rate =228 TPH
H1=main steam enthalpy=3450 KJ/Kg
h2=actual enthalpy at HP Turbine outlet (cold reheat enthalpy)
Q2=steam flow through reheater = 228TPH
H3=enthalpy of hot reheat steam = 3560 KJ/kg
h4= actual enthalpy of LP turbine exhaust steam
--- (1 mark)

HP Turbine isentropic efficiency= Actual enthalpy drop/isentropic enthalpy drop


0.9 = (H1- h2) / (H1-h2is)

h2is = isentropic enthalpy of cold reheat steam = 3050KJ/kg


0.9 = (3450 – h2) / (3450 - 3050)
h2= 3090 KJ/kg
---- (3 marks)
LP Turbine isentropic efficiency= (H3—h4)/(H3—h4is),

h4is = isentropic enthalpy of LP Turbine Exhaust steam = 2300 KJ/kg


0.9 = (3560 - h4) / (3560 - 2300)
h4 = 2426 KJ/kg
---- (3 marks)

Substituting the values in equation-2,we get

Turbine output = 228 x (3450 - 3090) + 228 x (3560 - 2426) / 860 = 75.73 MW
Generator output = 75.73 x 0.95 = 71.5 MW
---- (3 marks)

(B) Turbine heat rate = Q1(H1- hfw) + Q2(H3 - h2)/Generator output =KJ/kWhr---(3)
hfw = enthalpy of feed water = 990.3KJ/kg
Substituting the values in the above equation-3, we get

Turbine heat rate = 228 (3450 - 990.3) + 228(3560 - 3090) / 71.5


= 9342 KJ/kWhr
---- (3 marks)
(C) Turbine cycle efficiency = 860/Turbine heat rate
= 860x 4.18 / 9342 = 38.5%
REGULAR Paper 4 – Set A

---- (2 marks)
(D) Dryness fraction of steam at 0.1 bar(a) and 45.8 oC

Actual enthalpy of LP Exhaust steam = enthalpy of water + dryness fraction of


steam x L.H of vaporisation of steam
2426 = 191.9 + dryness fraction of steam x (2584.9 - 191.9)

Dryness fraction of steam = 93.35%


---- (3 marks)
(E) Specific steam consumption of cycle = Steam flow / generator output
= 228 / 71.5 = 3.19 tons/MWhr
--- (2 marks)
Or
B) Stenter operations in a textile process were significantly improved to reduce inlet
moisture from 60% to 55% in wet cloth while maintaining the same outlet moisture of 7%
in the dried cloth. The Stenter was operated at 80 meters/min in both the cases. The
dried cloth weighs 0.1 kg /meter. Further steps were taken to improve the efficiency of
the fuel oil fired thermic fluid heater from 80% to 82%, which was supplying heat energy
to the dryer. The other data and particulars are

Latent heat of water evaporated = 540kcal/kg,


Inlet temperature of wet cloth = 28oC ,
Outlet temperature of dried cloth = 80 oC,
Dryer efficiency = 50% ,
G.C.V of fuel oil = 10,300 kcal/kg,
Yearly operation of the stenter = 5000 hours

a) Find out the % reduction in Dryer heat load ,


b) Estimate the overall yearly fuel savings in tonnes by reducing moisture and
efficiency improvement compared to the initial case. Assume only energy for
moisture evaporation for dryer heat load

Solution:

Initial case: Inlet moisture, 60%, outlet moisture 7%, dryer efficiency 50%,
thermic fluid heater efficiency 80%

Output of stenter = 80 mts/min x 0.1 x 60


= 480 Kg/hr. …. (1 Mark)

Moisture in the dried output cloth = 7%

Wt. of bone- dry cloth = 480 X (1 – 0.07)


i.e. W = 446.4 Kg/hr.
---- (1 mark)
mo= moisture in outlet cloth = (480–446.4) /446.4
= 0.0753 Kg/Kg.bone-dry cloth
---- (1 mark)
REGULAR Paper 4 – Set A

Inlet moisture = 60%


Wt. of inlet cloth = 446.4 / (1 – 0.60) = 1116.00 Kg./hr.

mi = moisture in inlet cloth


= ((60/40) x 446.4)/446.4 = 1.5 Kg./Kg. bone- dry
cloth
---- (1 mark)

Inlet temperature of cloth Tin = 28oC


Final temperature of clothTout = 80oC

Heat load on the dryer = w x (mi – mo) X [(Tout – Tin) + 540] Kcal/hr.
.
. . Heat load on the dryer = 446.4 (1.5 – 0.0753) X [(80 – 28) + 540]
= 3,76,503.76 Kcal/hr
----(2.5 marks)

Efficiency of the dryer is 50%, Efficiency of the thermic fluid heater is 80%

Fuel oil consumption in the thermic fluid heater


= 3,76503.76/( 0.5x 0.8x10300) = 91.40 kg/hr

---- (2.5 marks)

Improved case: Inlet moisture, 55%, outlet moisture 7%, dryer efficiency 50%,
thermic fluid heater efficiency 82%

Inlet moisture = 55%


Wt of inlet cloth = 446.4 / (1 – 0.55) = 992.00 Kg./hr. ..(1 Mark)

mi = moisture in inlet cloth


= ((55/45) x 446.4)) / 446.4
= 1.22 Kg./Kg. bone-dry cloth
---- (1 mark)

Heat load on the dryer = w x (mi – mo) X [(Tout – Tin) + 540] Kcal/hr.
.
. . Heat load on the dryer = 446.4 (1.22 – 0.0753) X [(80 – 28) + 540]
= 3,02508.00 Kcal/hr
----(2.5 marks)

Efficiency of the dryer is 50%, Efficiency of the thermic fluid heater is 82%

Fuel oil consumption in the thermic fluid heater in impoved case


= 3,02,508.00/( 0.5x 0.82x10300) = 71.63 kg/hr

---- (2.5 marks)


REGULAR Paper 4 – Set A

(a) % reduction in dryer load due to reduction inlet moisture

(3,76,504 - 3,02,508) x 100


= -----------------------------------------
(3,76,504)

= 19.65%
---- (2 marks)
(b) Saving in fuel oil consumption in improved case
= 91.4 - 71.63
= 19.77 kg/hr

Yearly fuel oil savings = 19.77x 5000 x 1/1000

= 98.85 tonnes
--- (2 marks)
or
C) In a steel industry, the composition of blast furnace gas by volume is as follows
CO – 27%, H2 - 2%, CO2 – 11%, N2 - 60%.

i) Calculate the stoichiometric air for combustion


ii) Calculate the gross calorific value of gas in kcal/Nm 3
iii) Calculate the net calorific value of gas in kcal/Nm 3
iv) If 3,00,000 Nm3/hr of gas is available and is to be co-fired in a coal fired boiler.
How much coal it can replace if the GCV of coal is 4000 kcal/kg.

Ans (i) Stoichiometric air for combustion:

C + O2 ------ CO2 + 8,084 kcal/kg Carbon


2C + O2 ------ 2 CO + 2,430 kcal/kg Carbon
H2 + ½O2 -------H2O + 28,922 kcal/kg Hydrogen
CO + ½ O2 -------CO2 + 5,654 kcal/kg Carbon
---- (2 marks)

1 mole CO + 0.5 mole O2 ------ 1 mole CO2 + 5654 kCal/kg


For 27% CO, O2 required is (0.5/1) x 0.27 = 0.135 O 2
---- (2 marks)\

1 mole H2 + 0.5 mole O2 ------ 1 mole H2O + 28922 Kcal/kg


For 2 % of H2, O2 required is (0.5/1) x 0.02 = 0.01 O2
---- (2 marks)
Total stoichiometric oxygen required = 0.135 + 0.01 = 0.145 O2
REGULAR Paper 4 – Set A

Stoichiometric air required = 100 x 0.145 = 0.690 m3 air / m3 blast furnace gas
21
---- (3 marks)
(ii) Gross calorific value of gas:

1 kg mole of any gas at STP occupies 22.4 m3 of volume.


---- (1 mark)
Therefore,

((5654 x 12) / 22.4) x 0.27 = 817,83 kCal/m3 (molecular weight of Carbon = 12)
---- (2 marks)
((28922 x 2) / 22.4) x 0.02 = 51.64 kCal/m3 (molecular weight of Hydrogen = 2)
---- (2 marks)
Gross Calorific Value = 817.83 + 51.64 = 869.5 kcal/m3
---- (1 mark)

(iii) Replacement of coal by blast furnace gas:

Gross calorific value of coal = 4000 kcal/kg (given)


Blast furnace gas available = 3,00,000 m3/hr (given)

Heat content available from gas = 3,00,000 m3/hr x 869.5 kcal/m3


= 2608.5 x 105 kcal/hr
---- (2.5 marks)
If X is the coal quantity to be replaced, then
4000 kcal/kg x X = 2608.5 x 105 kcal/hr
X = 65212 kg/hr of coal can be replaced by gas of 3,00,000 m3/hr.
---- (2.5 marks)
or
D) As an energy auditor, auditing a cement plant, it is essential to assess the specific coal
consumption for the production of the clinker. With the following data available, calculate
the specific coal consumption (kgCoal/ KgClinker).

S.No Parameter Value

1. Reference temperature 20Oc

2. Barometric pressure 10329 mmWC

3. Density of the Pre-heater at NTP 1.436kg/m3


REGULAR Paper 4 – Set A

4. Density of Air 1.293Kg/m3

5. Pitot Tube Constant 0.85

6. Clinker production rate 4127 TPD

7. Static Pressure of the Pre-heater gas in the pre-heater duct 640mmWC

8. Dynamic pressure of the pre-heater gas in the duct 15.8mmWC

9. Temperature of the Pre-heater gas 320OC

10. Specific heat of the Pre-heater gas 0.247kCal/kg 0C

11. Area of the Pre-heater Duct 8.5 m2

12. Temperature of the exit clinker 128OC

13. Specific heat of the clinker 0.193 kCal/kg


0
C

14. Static Pressure of the Cooler Exhaust gas in the duct 42mmWC

15. Dynamic pressure of the Cooler Exhaust gas in the duct 15.5mmWC

16. Temperature of the Cooler Exhaust gas gas 290

17. Specific heat of the Cooler Exhaust gas 0.247kCal/kg 0C

18. Area of the Cooler exhaust duct 7.1m2

19. Heat of Formation of Clinker 405


Kcal/KgClinker

20. All other heat loss except heat loss through Pre-heater gas, exiting 84.3 Kcal/Kg
clinker and cooler exhaust gases Clinker

21. All heat inputs except heat due to Combustion of fuel (Coal) 29 Kcal/Kg Clinker

22. GCV of the Coal 5500Kcal/Kg

Ans Heat Lost in the along with the Exiting pre-heater gases:

QPH Gas = mphgas × Cpphgas × (tephgas-tr)


mphgas = VphgasX  Phgas
REGULAR Paper 4 – Set A

Vphgas = vph gas X A

Corrected density of the pre-heater gas:


10329  640 273
 Phgas = 1.436  
10334 273  320
…… (1 Mark)
= 0.6198 kg/ m3

Velocity (v) = Pt × [(2g(ΔPdynamic)avg /  Phgas)]0.5 m/sec

2  9.81  15.8
= 0.85  m/sec …..(2 Marks)
0.6198
= 19.0 m/sec

VPH gas = 19.0m3/s X 8.5 m2


= 161.5 m3/sec
= 5,81,400m3/hr ……(1 Mark)

Mph gas = 581400 m3/hr X 0.6198 kg/m3


=3,60,351/72 Kg/hr ……(1 Mark)

mphgas = 3,60,351 kg/hr / 1,71,958 kg/hr = 2.095Kgph gas/ Kg clinker

QPH Gas = 2.095 X 0.247 X (320 -20)


= 155.24Kcal/KgClinker
---- (1 mark)

Heat Lost in the along with the Exiting Hot Clinker:

QHot clinker = mclinker× Cpclinker× ( tclinker- tr)


= 1 x 0.193 x (128 – 20),
= 20.84 kCal/kgClinker
---- (2 marks)
Heat Lost in the along with the Exiting Cooler Exhaust gases:

QCooler Exhaust Gas = mCooler Exhaust Gas × CpCooler Exhaust Gas × (tCooler Exhaust Gas-tr)
mCooler Exhaust Gas = VCooler Exhaust Gas X  Cooler Exhaust Gas
VCooler Exhaust Gas = vCooler Exhaust Gas X A

Corrected density of the pre-heater gas:


REGULAR Paper 4 – Set A

10329  42 273
 Cooler Exhaust gas = 1.293  
10334 273  290
….. (1 Mark)
= 0.624 kg/ m3

Velocity (v) =Pt × √(2g(ΔPdynamic)avg /  Cooler Exhausts) m/sec

2  9.81  15.5
= 0.85  m/sec
0.624
= 18.76 m/sec ……(2 Marks)

VcoolerExhaustgas = 18.76m/s X 7.1 m2


= 133.196 m3/sec
= 4,79,505m3/hr ……..(1 Mark)

McoolerExhaustgas = 479505 m3/hr X 0.624 kg/m3


=2,99,211 Kg/hr ……(1 Mark)

mcoolerExhaustgas = 2,99,211 kg/hr / 1,71,958 kg/hr =1.74 KgcoolerExhaustgas/ Kg clinker

QcoolerExhaustgas = 1.74 X 0.244 X (290 -20) ……(1 Mark)


= 114.63Kcal/KgClinker

---- (1 mark)
Heat Input = Heat output

Heat Inputcoal + Heat inputothers = HeatClinker formation + HeatPH gas + HeatClinker+


Heatcooler exhaust gas + Heatothers

GCVcoalX mcoal + 29 = 405 + 155.24 + 20.84 +114.63 + 84.3

mcoal = 751 / 5500


= 0.137 Kgcoal/Kgclinker

---- (4 marks)

-------- End of Section - III ---------


REGULAR Paper 4 – Set B

16th NATIONAL CERTIFICATION EXAMINATION


FOR
ENERGY MANAGERS & ENERGY AUDITORS – September, 2015

PAPER – 4:Energy Performance Assessment for Equipment and Utility Systems

Date: 20.09.2015 Timings: 14:00-16:00 HRS Duration: 2 HRS Max. Marks: 100

Section - I: BRIEF QUESTIONS Marks: 10 x 1 = 10

(i) Answer all Ten questions


(ii) Each question carries One mark

S-1 The Net present value of a Energy Conservation Project is Rs.38784/- and the initial
capital investment Rs 1,50,000/- calculate the Profitability Index of the project.
Ans PI = 38784 = 0.258
1,50,000
S-2 Between a natural gas fired boiler and oil fired boiler which will have a higher
percentage of hydrogen loss in flue gas? Why?
Ans Gas fired boiler. Because the hydrogen percentage is more in natural gas compared
to oil.
S-3 If the condenser back pressure is 82 mm Hg, calculate the condenser vacuum
if the atmospheric pressure is 755 mmHg.
Ans Condenser vacuum, mmHg = (Atmospheric pressure, mmHg - Condenser back
pressure, mmHg)
= (755 - 82) = 673 mmHg.
S-4 For a process requiring indirect heating to 200 oC, thermic fluid is preferred to steam
as a heat carrier. Why ?
Ans Because for steam to be heated to high temperatures, the pressure required will be
very high.

S-5 After cleaning of choked AHU filter, AHU fan power increased. Why?
Ans Due to less resistance, the air flow increased.

S-6 Why is the exhaust temperature of furnace oil fired systems limited to about 170 0C?
Ans Acid dew point due to presence of sulphur
S-7 Other than exhaust gas what is the major source of waste heat recovery in a water
_______________________
Bureau of Energy Efficiency
1
REGULAR Paper 4 – Set B

cooled DG set?
Ans Engine jacket cooling water
S-8 In poorly loaded motor, current measurements are not a right indicator of motor
loading. Why?
Ans PF will be low.

S-9 If the coal GCV is 4500 kcal/kg and specific coal consumption is 0.60 kg/kWh, what is
the Power station Gross efficiency?
Ans (860 /(4500 x 0.60)) x100 = 31.85%

S-10 The dry bulb and wet bulb temperatures of air entering an air washer are 35 and 28
0
C respectively. If the saturation efficiency is 90 %, calculate the air temperature
leaving the air washer.

Ans 90% = 35 -Tout


35-28

Tout = 28.7oC

…………. End of Section - I ………….

Section - II: SHORT NUMERICAL QUESTIONS Marks: 2 x 5 = 10

(i) Answer all Two questions


(ii) Each question carries Five marks

L-1 A luxury hotel is using a diesel fired heater with an efficiency of 70% for supplying
hot water at 60oC from an initial temperature of 20oC. The hot water requirement is
24,000 litres per day.

The management is considering to install a specially designed electric heat pump for
the specific high hot water temperature requirement with a heat pump coefficient of
performance (C. O. P.) of 2.5. Find out the reduction in daily operating cost with heat
pump in place of diesel fired heater ignoring auxiliary energy consumption. The
following data are given.

Electricity cost = Rs.10/kWh


Diesel cost = Rs.50/litre
G.C.V. of diesel = 9100 kcal/litre
Ans
Diesel required
_______________________
Bureau of Energy Efficiency
2
REGULAR Paper 4 – Set B

For hot water heater = [24000 LitHotwater/ day ) x (60-20oC) x (1 kca/LitoC)]


(0.7 Effy x 9100 kcal/Lit diesel )

= 150.7 Lit diesel /day … 1 Mark

Diesel cost / day = 150.7 x 50 = 7535 Rs./day


…1 Mark
COP = Heat pump refrigeration effect / input electrical energy
or
Input electrical energy , kW = Heat pump refrigeration effect, kcal
COP x 1 kW
or
Input electrical energy , kW = Heat pump refrigeration effect, kcal
COP x 860 kcal/hr

Electrical energy required with heat = 24000 x 1 x (60 – 20)


(2.5 x 860)(1 Mark)
pump of COP = 2

Energy input with heat pump = 446.51 Kwh/day

Operating cost with heat pump = 446.51 x 10


= 4465.1 Rs./day
….. 1 mark
Reduction in operating cost = 7535 – 4465.1
= Rs.3069.9 /day
….. 1 mark
L-2 A pump is drawing water through a 150 mm diameter pipe with a suction head of 3
m below the pump centre line. Find out the pump efficiency if the actual power input
the motor is 16.7 kW at a motor efficiency of 90 %. The discharge pressure is 4.5
kg/cm2 and the velocity of water through the pipe as measured by an ultrasonic flow
meter is 1 m/s.

_______________________
Bureau of Energy Efficiency
3
REGULAR Paper 4 – Set B

Ans
Discharge Head, kg/cm2 = 4.5
Suction Head, m = -3
Total Head = 45 - (-3)
48 m
Flow rate = (22/7 x D2/4) x 1 m/s
= (22/7 x 0.152 / 4) x 1
m/s
= 0.0177 m3/sec
….. 2 marks
Hydraulic Power = 0.0177 x 1000 x 9.81 x
48/1000

= 8.33 kW
….. 1 mark
Pump Efficiency 8.33/(16.7x0.9)
= 55.2 %
….. 2 marks

…………. End of Section - II ………….

Section - III: LONG NUMERICAL QUESTIONS Marks: 4 x 20 = 80

(i) Answer all Four questions


(ii) Each question carries Twenty marks

N-1 In an organic chemical industry 15 Tonne per hour steam is generated at 10


kgf/cm2 in a 18 TPH natural gas fired smoke tube boiler. The % oxygen in the exit
flue gas was 3.1% and the flue gas temperature was 190oC. The following data
have been provided.

Ultimate analysis of natural gas per kg,


Carbon = 0.72 kg/kg; Hydrogen = 0.236 kg/kg; Nitrogen = 0.03 kg/kg;
Oxygen = 0.011 kg/kg;

Specific heat of flue gas = 0.297 kcal/kgoC


Specific heat of superheated water vapor = 0.45 kcal/kgoC
G.C.V. of natural gas = 9100 kcal/m3
Density of natural gas = 0.7
Density of air = 1.12 kg/m3
Enthalpy of steam at 10 kg/cm2 = 665 kcal/kg
Temperature of feed water at inlet to boiler = 95oC
Yearly hours of operation = 8000 hours
_______________________
Bureau of Energy Efficiency
4
REGULAR Paper 4 – Set B

a. Find out the S/F (steam to fuel) ratio in kg steam/m3 gas


b. Estimate the annual reduction in carbon dioxide emission in tones/year
compared to the furnace oil fired boiler of 83% efficiency on G.C.V. which
was earlier used for delivering the same steam load. Assume G.C.V. of
furnace oil as 10300 Kcal/kg and 0.86 carbon per kg furnace oil.

Ans Ultimate analysis of natural gas per kg. of gas

Carbon = 0.72 kg/kg; Hydrogen = 0.236 kg/kg; Nitrogen = 0.03


kg/kg; Oxygen = 0.011 kg/kg;

Theoretical air required


= 11.6C + [34.8 (H2 – O2/8)] + 4.35S,
= 11.6 X 0.72 + [34.8 (0.236 – 0.011/8)]
(note S= sulfur in above composition is nil)
= 16.524 kg air/kg gas … (1 Mark)

% Excess Air = [% O2 / (21 - % O2)] x 100


= [3.1 / (21 – 3.1)] x 100 = 17.3% … (1 Mark)

Actual Air Supplied (AAS) = [1 + 0.173] x 16.524 = 19.38 kg air / kg gas


…. (1 mark)

Mass of dry flue gas; mdfg = mass of combustion gases due to presence
C, N2,S in the fuel+mass of residual O2 in flue gas + mass of N2 supplied
with air

= 0.72 X 44/12 + 0.03 + (19.38 – 16.524) x 0.23 + 19.38 x 0.77


= 18.24 kg dfg / kg gas
…. (1.5 mark)

L1 = % heat loss due to dry flue gases

mdfg x cpfg x (Tg – Ta)


= ------------------------------ X 100
G.C.V. of gas

Kcal / m3 9100
G.C.V. of gas = ----------- = --------- = 13000 Kcal/kg
Density 0.7

_______________________
Bureau of Energy Efficiency
5
REGULAR Paper 4 – Set B

18.24 x 0.297 x (190 - 30)


= -------------------------------- X 100 = 6.67 %
13000
…. (2 marks)

L2 = Loss due to presence of hydrogen forming water vapor

9H [584 + Cps x (Tg – Ta)


= ---------------------------------- X 100
G.C.V.

9 x 0.236 [584 + 0.45 (190 – 30)


= --------------------------------------------- x 100
13000
L2 = 10.72 %
…. (2 marks)

Radiation and unaccounted losses in the boiler (given) = 1.45%

Total losses = 6.67 + 10.72 + 1.45 = 18.84%

Efficiency of natural gas fired boiler on = 100 – 18.84 = 81.16%


G.C.V. by indicated method
…. (1.5 marks)

Steam to fuel ratio in kg steam/m3 gas= 0.8116 x 9100 / (665 – 95) =


12.96
…. (2 marks)

Amount of gas required for generation = (15,000 / 12.96) x 0.7


15 tonne/hr of steam
= 810.19 kg/hour …(1.5 Marks)

CO2 emission with natural gas firing = 0.72 x 3.67 x 810.19


(1 kg carbon gives 44/12 i.e. 3.67 kg CO2)
= 2140.77 kg/hr
…. (1.5 marks)

Furnace oil required for 15TPH steam = (15,000 x 570) / (0.83 x


10,300)
= 1000.12 kg/hr … (1.5 Marks)

CO2 emission with furnace oil firing = 0.86 x 3.67 x 1000.12


= 3156.58 kg/hr …(1.5 Marks)
_______________________
Bureau of Energy Efficiency
6
REGULAR Paper 4 – Set B

Net reduction in CO2 emission with = 3156.58 – 2140.77


natural gas compared to furnace
oil firing = 1015.81 kg/hr
…. (1 mark)

Annual reduction in CO2 for 8000 hrs. = 1015.81 x 8000


operation
= 8126.140 Tonnes
…. (1 mark)

N-2 A gas engine-based trigeneration plant operates in two modes:

• Power and heating mode (10 hours per day) :


Pel= 650 kW of electricity and 325 kg/h of steam with enthalpy addition of 530 kcal/kg of
steam
EUFheat = 0.85

• Power and cooling mode (14 hours per day) :


Pel = 650 kW of electricity and chilling load of 250 TR for absorption chillers
EUFcool= 0.73

• Calorific value of natural gas = 8500 kcal/Sm3


• Average operating days/year = 330
• Alternator efficiency = 0.95
• The energy loss in the flue gas and that in the cooling water is same as engine power
output and other losses are negligible

Answer the following:

(1) What is the average plant energy utilization factor


(2) Calculate the useful energy produced daily by the trigeneration plant in Gcal
(3) Determine the daily plant natural gas requirements based on average energy
utilization factor
(4) The plant proposes to install a 60 TR hot water driven Vapour absorption chiller with
a COP of 0.5 using waste heat from jacket cooling water. Check if it is feasible with
supporting calculations.

_______________________
Bureau of Energy Efficiency
7
REGULAR Paper 4 – Set B

Ans 1) Plant average energy utilization factor

Plant average energy utilization factor = (0.85 x 10 + 0.73 x 14)/24


= 0.78
---- (3 marks)
2) The useful energy produced daily by the trigeneration plant in Gcal

PEle = 650 KW
QHeat = 325 x 530
= 172250 kcal/h
QCool = 250 x 3024
= 756000 kcal/h
---- (2 marks)
Total daily useful energy production (650 x 860 x 24 +172250 x 10 +
=
of the plant 756000 x 14)
13416000 + 1722500 +
=
10584000
The useful energy produced daily = 25722500 kcal/day (2 Marks)
The useful energy produced in 6
25722500x 330 / 10
=
Gcal/year
= 8488.43 Gcal
---- (2 marks)
3)The daily plant natural gas requirements
=
Input heat 25722500/ 0.78

= 32977564 kcal/day (2 Marks)


=
Natural gas requirements 32977564/ 8500

= 3879.7 Sm3/day

---- (2 marks)
4) Justification for a 60 TR Vapour Absorption chiller from waste heat
of the jacket cooling water
Heat required for operating 60 TR at = 60 x 3024/0.5
COP of 0.5 = 362880 Kcal/hr (2 Marks)

650 /0.95
Power output of the engine =
684.2 KW (2 Marks)

_______________________
Bureau of Energy Efficiency
8
REGULAR Paper 4 – Set B

= 684.2 x 860
Heat in the jacket cooling water
= 588412 kcal/hr (2 Marks)

Since the heat requirement (362880 Kcal/hr) is much less than heat
available (588412 kcal/hr) the proposal is feasible. ---- (1 mark)

N-3 Hot effluent having a flow rate of 56789 Kg/hr at 85OC from the process is sent to
a heat exchanger for cooling. The outlet temperature of effluent in the heat
exchanger is 38 OC. Air having a flow rate of 370057 Kg/hr enters the heat
exchanger at a temperature of 30 OC and leaves at 60 OC. Power drawn by the
fan is 30 KW. The plant works for 16 hours a day for 300 days per year.

Now plant has decided to replace air cooled heat exchanger with a water cooled
counter current Heat Exchanger.

Given that Pump Efficiency = 80%, Motor efficiency = 90 %, Effectiveness of


water cooled heat exchanger is 0.4 water is available at 25 OC & Pressure drop
in plate heat exchanger is 1.2 kg/cm2 , Over all heat transfer coefficient of heat
exchanger is 22300 Kcal/m2/OC.

1. Calculate the savings due to replacement by water cooled heat exchanger


2. Calculate the heat transfer area of heat exchanger.
Ans Heat Duty
Heat duty in hot fluid = M x Cph x (Ti - To)
= 56789 x 1 x (85 – 38)
= 2669083 Kcal / Kg
…. (2 marks)
Heat duty in cold Air = M x Cpair x (to – ti)
= 370057 x 0.24 x (60 – 30)
= 2664410 Kcal / Kg
…. (2 marks)
In heat exchanger, Heat duty in hot fluid = Heat duty in cold Air

Effectiveness of water cooled heat exchanger = 0.4

Cold Water outlet – Cold water inlet


Effectiveness = Hot effluent inlet – Cold water inlet

Cold Water Outlet = (0.4 x (85 – 25)) + 25

_______________________
Bureau of Energy Efficiency
9
REGULAR Paper 4 – Set B

= 49 OC
…. (2.5 marks)

Mass flow rate of cooling water (M) =Heat duty in hot fluid
Cpx(Cold Water outlet – Cold water inlet)

= 2669083
1 x (49 – 25) x 1000

= 111.21 m3 /Hr
…. (2.5 marks)

Pressure drop in Plate Heat exchanger = 12 m

Hydraulic Power Requirement for one Cooling Water Pump:

= (Flow in m3/Hr x Head in m x Density in Kg/m3 x g in m/s2)


(1000 x 3600)

= (111.21 x 12 x 1000 x 9.81)


(1000 x 3600)
= 3.64 KW

…. (3 marks)

Pump Power Requirement at 80% pump efficiency = 3.64 KW


0.8
= 4.55 KW
…. (1 mark)

Motor Input Power Required at 90% Efficiency = 4.55


0.9
= 5.06 KW
…. (1 mark)
Thus savings = Power consumption by fans – Water Pumping Power
= 30 – 5.06
= 24.94KW

Annual Saving in kWh = 24.94 KW x 16 Hrs x 300 Days = 119712


kWh/Annum
…. (2 marks)

_______________________
Bureau of Energy Efficiency
10
REGULAR Paper 4 – Set B

Calculations for LMTD for Proposed HEx:

LMTD for counter current flow in HEx

= {(85-49) – (38-25)} / ln {(85-49) / (38-25)}


= 22.5 Deg C
…. (2 marks)

Considering overall heat transfer coefficient (U) = 22300 kW/m2/0C

Heat transfer Area = Q


(U x ∆Tlm)
= 2669083
(22300 x 22.5)
= 5.32 m2 (Say 6 m2)
…. (2 marks)
N-4 Answer ANY ONE OF THE FOLLOWING among A, B, C and D

A) A steam power plant consisting of high pressure Turbine(HP Turbine) and low
pressure Turbine(LP Turbine) is operating on Reheat cycle(schematic of power
plant is represented below).Steam from Boiler at a pressure of 150 bar(a) and a
temperature of 5500C expands through the HP Turbine. The exhaust steam from
HP Turbine is reheated in a Reheater at a constant pressure of 40 bar (a) to
5500C and then expanded through LP Turbine. The exhaust steam from LP
Turbine is condensed in a condenser at a pressure of 0.1 bar (a).The isentropic
efficiency of HP Turbine and LP Turbine is same and is 0.9. Generator efficiency
is 95%

_______________________
Bureau of Energy Efficiency
11
REGULAR Paper 4 – Set B

The other data of the power plant is as given below:

Main steam flow rate : 228 TPH


Enthalpy of main steam : 3450 KJ/kg
Enthalpy of feed water : 990.3KJ/kg
Isentropic Enthalpy of cold reheat steam : 3050 KJ/kg
Enthalpy of hot reheat steam : 3560 KJ/kg
Condenser pressure and temperature : 0.1 bar(a) and 45.80C
Isentropic enthalpy of LP Turbine exhaust steam : 2300 KJ/kg
Enthalpy of dry saturated steam at 0.1 bar(a) and 45.80C : 2584.9KJ/kg
Enthalpy of water at 0.1 bar(a) and 45.80C:191.9 KJ/kg

Based on the above data calculate the following parameters


(a) Power developed by the Generator
(b) Turbine heat rate
(c) Turbine cycle efficiency
(d)Dryness fraction of LP Turbine Exhaust steam
(e) Specific steam consumption of turbine cycle.

_______________________
Bureau of Energy Efficiency
12
REGULAR Paper 4 – Set B

Ans (a) Power developed by the Generator: Turbine output x Generator efficiency--(1)
Turbine out put = Q1 (H1 – h2) + Q2(H3 – h4)/860 MW ----------- (2)
Where, Q1=main steam flow rate =228 TPH
H1=main steam enthalpy=3450 KJ/Kg
h2=actual enthalpy at HP Turbine outlet= ?=cold reheat enthalpy
Q2=steam flow through reheater=228TPH
H3=enthalpy of hot reheat steam=3560 KJ/kg
h4= actual enthalpy of LP turbine exhaust steam=?
…. (1 mark)

HP Turbine isentropic efficiency= Actual enthalpy drop/isentropic enthalpy drop


0.9= (H1- h2)/(H1-h2is) , h2is=isentropic enthalpy of cold reheat steam = 3050
KJ/kg
0.9= (3450 –h2)/(3450—3050)
h2= 3090KJ/kg
…. (3 marks)

LP Turbine isentropic efficiency= (H3—h4)/(H3—h4is), h4is=isentropic enthalpy of


LP Turbine Exhaust steam=2300KJ/kg
0.9=( 3560-h4)/(3560—2300)
h4= 2426 KJ/kg
…. (3 marks)

Substituting the values in equation-2,we get

Turbine output = 228(3450—3090) + 228(3560—2426)/860 =75.73MW


Generator output= 75.73 x 0.95= 71.5 MW
…. (3 marks)

(b) Turbine heat rate=Q1 (H1—hfw) +Q2(H3—h2)/Generator output =KJ/kwhr-----


--------(3)
hfw=enthalpy of feed water=990.3KJ/kg
Substituting the values in the above equation-3, we get

Turbine heat rate=228 (3450—990.3) + 228(3560—3090)/71.5


=9342 KJ/kWhr
…. (3 marks)

(C) Turbine cycle efficiency= 860/Turbine heat rate


=860/9342=38.5%
…. (2 marks)

(d)Dryness fraction of steam at 0.1 bar(a) and 45.8C

_______________________
Bureau of Energy Efficiency
13
REGULAR Paper 4 – Set B

Actual enthalpy of LP Exhaust steam= enthalpy of water + dryness fraction of


steam x L.H of vaporisation of steam
2426 = 191.9+ dryness fraction of steam x(2584.9—191.9)

Dryness fraction of steam= 93.35%


…. (3 marks)

(e) Specific steam consumption of cycle =Steam flow/generator output


=228/71.5=3.19 tons/MW hr
…. (2 marks)
Or
B) Stenter operations in a textile process were significantly improved to reduce inlet
moisture from 60% to 55% in wet cloth while maintaining the same outlet moisture of 7%
in the dried cloth . The Stenter was operated at 80 meters/min in both the cases. The
dried cloth weighs 0.1 kg /meter. Further steps were taken to improve the efficiency of the
fuel oil fired thermic fluid heater from 80% to 82%, which was supplying heat energy to
the dryer. The other data and particulars are

Latent heat of water evaporated = 540kcal/kg,


Inlet temperature of wet cloth = 28 oC ,
Outlet temperature of dried cloth = 80 oC,
Dryer efficiency = 50% ,
G.C.V of fuel oil = 10,300 kcal/kg,
Yearly operation of the stenter = 7000 hours

a) Find out the % reduction in Dryer heat load ,


b) Estimate the overall yearly fuel savings in tonnes by reducing moisture and
efficiency improvement compared to the initial case. Assume only energy for
moisture evaporation for dryer heat load

Ans Initial case: inlet moisture, 60%, outlet moisture 7%, dryer efficiency 50%,thermic fluid
heater efficiency 80%

Output of stenter = 80 mts/min x 0.1 x 60


= 480 Kg/hr (1 Mark)

Moisture in the dried output cloth = 7%

Wt. of bone- dry cloth,W = 480 X (1 – 0.07)


= 446.4 Kg/hr
---- (1 mark)

mo = moisture in outlet cloth = (480 – 446.4) /446.4


= 0.0753 Kg/Kg bone dry cloth (1 Mark)

_______________________
Bureau of Energy Efficiency
14
REGULAR Paper 4 – Set B

Inlet moisture = 60%


Wt of inlet cloth = 446.4 / (1 – 0.60) = 1116.00 Kg./hr.

mi = moisture in inlet cloth


= ((60/40) x 446.4)/446.4 = 1.5 Kg./Kg. bone- dry
cloth
---- (1 mark)

Inlet temperature of cloth Tin = 28oC


Final temperature of clothTout = 80oC

Heat load on the dryer = w x (mi – mo) X [(Tout – Tin) + 540] Kcal/hr.
.
. . Heat load on the dryer = 446.4 (1.5 – 0.0753) X [(80 – 28) + 540]
= 3,76,503.76 Kcal/hr
---- (2.5 marks)

Efficiency of the dryer is 50%, Efficiency of the thermic fluid heater is 80%

Fuel oil consumption in the thermic fluid heater


=3,76503.76/( 0.5x 0.8x10300) = 91.40 kg/hr

---- (2.5 marks)

Improve case: inlet moisture, 55%, outlet moisture 7%, dryer efficiency 50%,thermic fluid
heater efficiency 82%

Inlet moisture = 55%


Wt of inlet cloth = 446.4 / (1 – 0.55) = 992.00 Kg./hr. (1 Mark)

mi = moisture in inlet cloth


= ((55/45) x 446.4)) / 446.4
= 1.22 Kg./Kg. bone-dry cloth
---- (1 mark)

Heat load on the dryer = w x (mi – mo) X [(Tout – Tin) + 540] Kcal/hr.

Heat load on the dryer = 446.4 (1.22 – 0.0753) X [(80 – 28) + 540]
= 3,02508.00 Kcal/hr
---- (2.5 marks)

Efficiency of the dryer is50%, Efficiency of the thermic fluid heater is 82%

Fuel oil consumption in the thermic fluid heater in impoved case


= 3,02,508.00/( 0.5x 0.82x10300)
_______________________
Bureau of Energy Efficiency
15
REGULAR Paper 4 – Set B

= 71.63 kg/hr (2.5 Marks)

% reduction in dryer load due to reduction inlet moisture


(3,76,504-3,02,508) x 100
= -----------------------------------------
(3,76,504)

= 19.65%
---- (2 marks)

Saving in fuel oil consumption in improved case


= 91.4 - 71.63
= 19.77 kg/hr

Yearly fuel oil savings =19.77x7000 x1/1000


=138.390 tonnes
---- (2 marks)

or
C) In a steel industry, the composition of blast furnace gas by volume is as follows
CO – 27%, H2 - 2%, CO2 – 11%, N2 - 60%.

i) Calculate the stoichiometric air for combustion


ii) Calculate the gross calorific value of gas in kcal/m3
iii) Calculate the net calorific value of gas in kcal/Nm 3
iv) If 3,00,000 Nm3/hr of gas is available and is to be co-fired in a coal fired boiler.
How much coal it can replace if the GCV of coal is 4300 kcal/kg.

Ans (i) Stoichiometric air for combustion:

C + O2 ------ CO2 + 8,084 kcal/kg Carbon


2C + O2 ------ 2 CO + 2,430 kcal/kg Carbon
H2 + ½O2 -------H2O + 28,922 kcal/kg Hydrogen
CO + ½ O2 -------CO2 + 5,654 kcal/kg Carbon
---- (2 marks)
1 mole CO + 0.5 mole O2 ------ 1 mole CO2 + 5654 kCal/kg
For 27% CO, O2 required is (0.5/1) x 0.27 = 0.135 O 2
---- (2 marks)
1 mole H2 + 0.5 mole O2 ------ 1 mole H2O + 28922 Kcal/kg
For 2 % of H2, O2 required is (0.5/1) x 0.02 = 0.01 O2

_______________________
Bureau of Energy Efficiency
16
REGULAR Paper 4 – Set B

---- (2 marks)
Total stoichiometric oxygen required = 0.135 + 0.01 = 0.145 O2
Stoichiometric air required = 100 x 0.145 = 0.690 m3 air / m3 blast furnace gas
21
---- (3 marks)
(ii) Gross calorific value of gas:

1 kg mole of any gas at STP occupies 22.4 m3 of volume.


---- (1 mark)
Therefore,

((5654 x 12) / 22.4) x 0.27 = 817,83 kCal/m3 (molecular weight of Carbon = 12)
---- (2 marks)
((28922 x 2) / 22.4) x 0.02 = 51.64 kCal/m3 (molecular weight of Hydrogen = 2)
---- (2 marks)
Gross Calorific Value = 817.83 + 51.64 = 869.5 kcal/m3
---- (1 mark)

(iii) Replacement of coal by blast furnace gas:

Gross calorific value of coal = 4300 kcal/kg (given)


Blast furnace gas available = 3,00,000 m3/hr (given)

Heat content available from gas = 3,00,000 m3/hr x 869.5 kcal/m3


= 2608.5 x 105 kcal/hr
---- (2.5 marks)
If X is the coal quantity to be replaced, then
4300 kcal/kg x X = 2608.5 x 105 kcal/hr
X = 60663 kg/hr of coal can be replaced by gas of 3,00,000 m3/hr.
---- (2.5 marks)
or
D) As an energy auditor, auditing a cement plant, it is essential to assess the specific coal
consumption for the production of the clinker. With the following data available, calculate
the specific coal consumption (kgCoal/ KgClinker).

S.No Parameter Value

1. Reference temperature 20Oc

_______________________
Bureau of Energy Efficiency
17
REGULAR Paper 4 – Set B

2. Barometric pressure 10329 mmWC

3. Density of the Pre-heater at NTP 1.436kg/m3

4. Density of Air 1.293Kg/m3

5. Pitot Tube Constant 0.85

6. Clinker production rate 4127 TPD

7. Static Pressure of the Pre-heater gas in the pre-heater duct 640mmWC

8. Dynamic pressure of the pre-heater gas in the duct 15.8mmWC

9. Temperature of the Pre-heater gas 320OC

10. Specific heat of the Pre-heater gas 0.247kCal/kg 0C

11. Area of the Pre-heater Duct 8.5 m2

12. Temperature of the exit clinker 128OC

13. Specific heat of the clinker 0.193 kCal/kg


0
C

14. Static Pressure of the Cooler Exhaust gas in the duct 42mmWC

15. Dynamic pressure of the Cooler Exhaust gas in the duct 15.5mmWC

16. Temperature of the Cooler Exhaust gas gas 290

17. Specific heat of the Cooler Exhaust gas 0.247kCal/kg 0C

18. Area of the Cooler exhaust duct 7.1m2

19. Heat of Formation of Clinker 405


Kcal/KgClinker

20. All other heat loss except heat loss through Pre-heater gas, exiting 84.3
clinker and cooler exhaust gases Kcal/KgClinker

21. All heat inputs except heat due to Combustion of fuel (Coal) 29 Kcal/KgClinker

22. GCV of the Coal 6200Kcal/Kg

_______________________
Bureau of Energy Efficiency
18
REGULAR Paper 4 – Set B

Ans Solution:
Heat Lost in the along with the Exiting pre-heater gases:

QPH Gas = mphgas × Cpphgas × (tephgas-tr)


mphgas = VphgasX  Phgas
Vphgas = vph gas X A

Corrected density of the pre-heater gas:


10329  640 273
 Phgas = 1.436  
10334 273  320

= 0.6198 kg/ m3 (1 Mark)

Velocity (v) =Pt × √(2g(ΔPdynamic)avg /  Phgas) m/sec

2  9.81  15.8
= 0.85  m/sec
0.6198
= 19.0 m/sec (2 Marks)

VPH gas = 19.0m3/s X 8.5 m2


= 161.5 m3/sec
= 5,81,400m3/hr (1 Mark)

Mph gas = 581400 m3/hr X 0.6198 kg/m3


=3,60,351/72 Kg/hr (1 Mark)

mphgas = 3,60,351 kg/hr / 1,71,958 kg/hr =2.095Kgph gas/ Kg clinker (1 Mark)

QPH Gas = 2.095 X 0.247 X (320 -20)


= 155.24Kcal/KgClinker (1 Mark)

Heat Lost in the along with the Exiting Hot Clinker:


QHot clinker = mclinker× Cpclinker× ( tclinker- tr)
= 1 x 0.193 x (128 – 20),
= 20.84 kCal/kgClinker
---- (2 marks)
Heat Lost in the along with the Exiting Cooler Exhaust gases:
QCooler Exhaust Gas = mCooler Exhaust Gas × CpCooler Exhaust Gas × (tCooler Exhaust Gas-tr)
_______________________
Bureau of Energy Efficiency
19
REGULAR Paper 4 – Set B

mCooler Exhaust Gas = VCooler Exhaust Gas X  Cooler Exhaust Gas


VCooler Exhaust Gas = vCooler Exhaust Gas X A

Corrected density of the pre-heater gas:


10329  42 273
 Cooler Exhaust gas = 1.293  
10334 273  290

= 0.624 kg/ m3 (1 Mark)

Velocity (v) =Pt × √(2g(ΔPdynamic)avg /  Cooler Exhausts) m/sec

2  9.81  15.5
= 0.85  m/sec
0.624
= 18.76 m/sec (2 Marks)

VcoolerExhaustgas = 18.76m/s X 7.1 m2


= 133.196 m3/sec
= 4,79,505m3/hr (1 Mark)

McoolerExhaustgas = 479505 m3/hr X 0.624 kg/m3


=2,99,211 Kg/hr (1 Mark)
mcoolerExhaustgas = 2,99,211 kg/hr / 1,71,958 kg/hr =1.74 KgcoolerExhaustgas/ Kg clinker (1 Mark)

QcoolerExhaustgas = 1.74 X 0.244 X (290 -20)


= 114.63Kcal/KgClinker (1 Mark)

Heat Input = Heat output

Heat Inputcoal + Heat inputothers = HeatClikerfrmtn+ HeatPH gas + HeatCliker+ Heatcooler exhaust gas
+ Heatothers

GCVcoalX m coal + 29 = 405 + 155.24 + 20.84 +114.63 + 84.3

mcoal = 751 / 6200


= 0.121 Kgcoal/Kgclinker

---- (4 marks)

-------- End of Section - III ---------


_______________________
Bureau of Energy Efficiency
20
REGULAR Paper 4 – Set B

_______________________
Bureau of Energy Efficiency
21
Paper 4 – SET A

15th NATIONAL CERTIFICATION EXAMINATION


FOR
ENERGY AUDITORS – August, 2014

PAPER – 4: Energy Performance Assessment for Equipment and Utility Systems

Date: 24.8.2013 Timings: 14:00-16:00 Hrs Duration: 2 Hrs Max. Marks: 100

Section - I: BRIEF QUESTIONS Marks: 10 x 1 = 10

(i) Answer all Ten questions


(ii) Each question carries One mark

S-1 Which loss is not considered while evaluating boiler efficiency by “Indirect Method”?

Ans Blow down loss

S-2 What will be the synchronous speed of a VFD driven 4-pole induction motor
operating at 40 Hz ?
Ns = 120 x f/P
Ans
= 120 x 40/4= 1200 RPM
What is the refrigerant used in a vapour absorption system with lithium bromide as
S-3
an absorbent?
Ans Water
Other than rated kW of motor and the actual power drawn, what other parameter is
S-4
required to determine the percentage loading of the motor ?
Ans Motor Efficiency or rated motor efficiency
Inclined tube manometer is used for measuring gas flow in a duct when the air
S-5
velocity is very high: True or False?
False.
Ans
A pump will cavitate if the NPSHrequired is _________ than the NPSHavailable.
S-6
More
Ans
To determine the effectiveness of the cooling tower, it is required to measure cooling
S-7
water inlet, outlet and _____temperatures.

Ambient Wet bulb


Ans

S-8 The ratio of actual heat transfer to the heat that could be transferred by heat
exchanger of infinite size is termed as ………

1
_______________________
Bureau of Energy Efficiency
Paper 4 – SET A

Ans Effectiveness

S-9 If the unit heat rate of a power plant is 3070 kcal/kWh ,what is the power plant
efficiency ?

Ans (860/3070) x 100 = 28 %

S-10
The difference between GCV and NCV of hydrogen fuel is Zero: True or False
False
Ans

…………. End of Section - I ………….

Section - II: SHORT NUMERICAL QUESTIONS Marks: 2 x 5 = 10

(i) Answer all Two questions


(ii) Each question carries Five marks

Hot water at 80 OC is used for room heating in a 5 Star hotel for 4 months in a year.
L-1
About 200 litres per minute of hot water is maintained in circulation with the return
temperature at 50 OC. The hot water is generated using a „hot waste stream‟,
through a Plate Heat Exchanger (PHE). The hot stream enters the PHE in
counterflow direction at 95 OC and leaves at 60 OC. The area of the heat exchanger
is 20 m2.

Calculate the LMTD and the overall heat transfer coefficient.

Heat load, Q = 200 * 60 * (80 – 50) = 360000 Kcals/hr (or) 418.7 kW


Ans
(95 – 80)/(60 – 50)
LMTD (for counter flow) = ------------------------ = 3.7 OC
ln (15/10)

Overall Heat Transfer Coefficient, U = Q/( AxLMTD)

= 418.7/(20 x 3.7) = 5.66 kW/m2.OC


(OR)
= 4864.8 kcal/hr.m2.OC

A gas turbine generator is delivering an output of 20 MW in an open cycle with a


L-2
heat rate of 3440 kcal/kWh. It is converted to combined cycle plant by adding heat
recovery steam generator and a steam turbine raising the power generation output
to 28 MW. However, with this retrofitting and increased auxiliary consumption, the
fuel consumption increases by 5% in the gas turbine.

2
_______________________
Bureau of Energy Efficiency
Paper 4 – SET A

Calculate the combined cycle gross heat rate and efficiency.

Gas turbine output = 20 MW


Ans
Combined cycle output = 28 MW

Heat rate in GT open cycle for 20 MW = 3440 kcal/kwh

Increase in fuel consumption in = 5%


combined cycle operation

Combined cycle heat rate = (3440 X 1.05) X( 20 / 28)


= 2580 kcal/kwh

Combined cycle plant efficiency = (860 / 2580) X 100


= 33.33%

…………. End of Section - II ………….

Section - III: LONG NUMERICAL QUESTIONS Marks: 4 x 20 = 80

(i) Answer all Four questions

N-1 The steam requirement of an export oriented unit is met by a 6 TPH oil fired package
boiler generating steam at 10 kg/cm2. The monthly steam consumption of the unit is
3000 tonnes.
Other data are given below:

Fuel oil composition:


Carbon = 86%; Hydrogen = 12%; Oxygen= 0.5%; Sulphur =1.5%

Specific heat of flue gases, Cp = 0.27 kcal/kgoC


G.C.V. of fuel oil = 10,000 kcal/kg
Sp.heat of super heated water vapour = 0.45 kcal/kgoC
Enthalpy of steam at 10 kg/cm2 = 665kcal/kg
Feed water temperature = 85 oC
% O2 in dry flue gas = 6%
Flue gas temperature at boiler outlet = 240 oC
Ambient temperature = 30oC
Cost of fuel oil = Rs.43 per kg.
Radiation and other unaccounted losses = 2.45%

The export oriented unit is costing its steam cost based on the fuel consumption cost
with additional 15% to account for the auxiliary and consumables.

A neighbouring continuous process plant now offers to supply the required steam at 10
3
_______________________
Bureau of Energy Efficiency
Paper 4 – SET A

kg/cm2 to the export oriented unit at a cost of Rs 3300 per tonne with a condition that
all the condensate will be returned back.

Calculate the following:

a) Boiler efficiency
b) Cost advantage per tonne of availing steam from neighbouring plant in place of
in-house generation and also monthly monetary saving.

Ans
First calculate the efficiency of Boiler (in EOU)

Theoretical air required =


= 11.6 C + 34.8 (H – O/8) + 4.35 S
= [11.6 X 86 + 34.8 (12 – 0.5/8) + 4.35 X 1.5] x 1/100
= 14.195 = Say 14.2

% Excess Air = [% O2 / (21 - % O2)] X 100


= [6 / (21 – 6)] X 100 = 40%

AAS = Actual amount of air supplied = 14.2 X 1.4


= 19.88 kg per kg. of fuel oil

Mass of dry flue gas mdfg = Mass of combustion gases due to presence of C,H,S
+Mass of N2 supplied
= (0.86 X 44/12) + (0.015 X 64 / 32) + [(19.88 – 14.2) X 23 / 100] + (19.88 X 77/100)
= 19.797
Mass dry flue gas ,say = 19.8 Kg / kg fuel
Or
Alternatively mass of dry flue gas = (AAS + 1) – 9 H
= (19.88 + 1) – 9 X 0.12 = 19.8 Kg./Kg. fuel

L1 = % heat loss in dry flue gas = [mdfg x Cp x (Tq – Ta) / GCV] x 100

19.8 X 0.27 x (240 – 30)


= --------------------------------- x 100
10,000
L1 = 11.23%
L2 = Loss due to presence of hydrogen forming water vapour
9 x H [584 + Cps (Tg – Ta)]
= ------------------------------------------ x 100
GCV
9 X 0.12 [584 + 0.45 (240 – 30)]
= ------------------------------------------- x 100
10000
L2 = 7.33%
L3 = Radiation and other unaccounted losses = 2.45%
Total losses = L1 + L2 + L3

4
_______________________
Bureau of Energy Efficiency
Paper 4 – SET A

= 11.23 + 7.33 + 2.45 = 21.05 %


Efficiency of the EOU boiler by indirect method
= 100 – 21.05 =78.99 %
= Say 79 %

Secondly calculate the cost of steam in the EOU plant


Evaporation Ratio = [(n X GCV) / (hg – hf)] X 100
= [(0.79 X 10000) / (665 – 85)] X 100
= 13.62 kg Steam / kg. Fuel

Fuel oil consumption = 1000 / 13.62 kg. per tonne of steam


Fuel oil consumption = 73.42 kg./tonne of steam gen

Cost of fuel oil. = Rs. 43 per kg


Cost of steam in EOU = Fuel cost + 15% fuel cost
= 73.42 x 1.15 x 43
= Rs.3,599 per tonne
Say = Rs 3600 per tonne

Selling cost of steam from neighboring plant = Rs 3300 per tonne

Cost advantage = 3600 – 3300 = Rs.300 per tonne

Annual Savings = Rs.300 per tonne x 3000tonne/month X 12 month


= Rs.108 Lacs
N-2
a) The operating parameters of a Vapour Compression Refrigeration system are
indicated below.

Parameter Chiller side Condenser side


Water Flow (m3/hr) 89 87
Inlet Temperature (OC) 10.1 32.3
Outlet Temperature (OC) 6.8 36.6
Density (kg/m3) 1000 990

Find the COP of the Refrigeration system ignoring heat losses.

b) A 6 pole, 415 volt, 3 , 50 Hz induction motor delivers 22 kW power at rotor shaft


at a speed of 950 rpm with PF of 0.88. The total loss in the stator including core,
copper and other losses, is 2 kW. Calculate the following.
i) i) Slip
ii) ii) Rotor Copper Loss
iii) iii) Total Input to motor
iv) ivi) Line current at 415 V and motor pf of 0.88
v) v) Motor operating efficiency

a) Refrigeration Effect = 89 x 1000 x (10.1 – 6.8)

5
_______________________
Bureau of Energy Efficiency
Paper 4 – SET A

Ans = 293700 kcal/hr


Condenser load = 87 x 990 x (36.6 – 32.3)
= 370359 kcal/hr
Compressor work = Condenser load – Refrigeration effect
= 370359 – 293700
= 76659 Kcal/hr
C.O.P. = Refrigeration Effect/ Compressor work
= 293700/76659 = 3.83

b) Synchronous Speed = (120 x 50 / 6 ) = 1000 rpm


Motor Speed = 950 rpm
(i) Slip = (1000 – 950 ) / 1000 = 5 %
Power input to rotor = { ( 22 / ( 1 – 0.05 ) } = 23.16 kW

(ii) Rotor Copper Loss = ( 0.05 x 23.16 ) = 1 . 1 5 8 k W


Or = 23.16-22 =1.16 kW

(iii) Total Input to motor = ( 23.16 + 2 ) = 25.16 kW

(iv) Line Current = ( 25.16 x 1000) /( 3 x 415 x0.88 )


= 39.75 Amps

(v) Motor Efficiency = ( 22 / 25.16 ) = 87.44 %

N-3 A common plant facility is installed to provide steam and power to textile and paper
plant with a co-generation system. The details and operating parameters are given
below:
60 T/hr.80 bar at 5000C

Enthalpy- 810 kcal/kg

To paper
Boiler industry
Turbine G
Coal
Alternator To textile
Boiler efficiency- 80% industry
GCV of Coal-5000 kcal/kg
Boiler feed water temp.- 800C
Steam 60T/hr. at 10 bar
Enthalpy- 660 kcal/kg

Steam 20 T/hr Steam Header Steam 40 T/hr


To textile industry To paper industry

Other data:
- Turbine, alternator and other losses = 8%
- Specific steam consumption in paper industry= 5 Tons/Ton of paper
- Specific power consumption in paper industry= 600 kWh/Ton of paper
Calculate:

6
_______________________
Bureau of Energy Efficiency
Paper 4 – SET A

i. Coal consumption in boiler per hour or per day.


ii. Power generation from co-generation plant
iii. If 10% is auxiliary power consumption in co-generation plant, how much power
is consumed by the textile industry per hour?
iv. What is the gross heat rate of turbine?

Ans
i. i) Boiler efficiency = Steam production ( steam enthalpy- Feed water enthalpy) /
Quantity of coal x G.C.V. of coal
Quantity of coal = 60,000 (810-80)/ 0.8 x 5000
= 10.95 tons/hr.

ii) Gross power generation from co-generation plant

Total enthalpy input to turbine = 60,000 x 810 = 48.6 Million kcal.


Total enthalpy out put through back pressure= 60,000* 660 = 39.6 Million kcal
Enthalpy difference = 48.6- 39.6 = 9 Million kcal/hr
Turbine,alternator and other losses =8% or 9x0.08 = 0.72 Million kcal/hr
Useful energy for power generation = 9- 0.72 = 8.28 Million kcal/hr
6
Power generation from co-generation plant = 8.28 x 10 /860 = 9628kWh

iii) If 10% is auxiliary power consumption in co-generation plant, power consumed by


textile industry

10% of total power generation = 9628 x 0.10 = 962.8kWh

Total power consumed by industries = 9628 – 962.8 = 8665.2kWh

Total steam consumption in paper plant 40 tons/hr. and specific steam consumption
5 ton/ton of paper. So Paper production per hour is 8 tons.

Specific power consumption = 600kWh/ton.


Total power consumption in paper industry = 8 x 600 = 4800kWh
Total power consumption in textile industry = 8665.2- 4800 = 3865.2 kWh

iv) Gross heat rate= Input enthalpy – output enthalpy/ gross generation
=( 48.6- 39.6) 106/ 9628 = 934.7 kCal/kWh

N-4 To attempt ANY ONE OF THE FOLLOWING among A, B, C and D

A A captive thermal plant is delivering an output of 29 MW at the generator terminal. The


generator efficiency is 96%. The steam generated in a utility boiler with an efficiency of
86% at 105 ata and 485oC is fed to the turbine. The turbine exhausts steam to
condenser maintained at 0.1 ata and 45.5oC. The feed water temperature at inlet to the
boiler is 105oC.

7
_______________________
Bureau of Energy Efficiency
Paper 4 – SET A

The other data pertaining to captive power plant are,

Enthalpy of steam at 105 ata 485oC = 795 kcal/kg.


Dryness fraction of steam at inlet to condenser = 0.9
Enthalpy of dry saturated steam at 0.1ata =618 kcal/Kg.
o
Enthalpy of water at 0.1 ata & at 45.5 C = 45.5 kcal/Kg.
Loss in the gear box connecting turbine and generator = 1100 kW
Enthalpy of feed water at inlet to the boiler , = 105 kcal/Kg.

Based on the above data determine:


i. Output of the steam turbine in kW
ii. Steam flow through the turbine
iii. Turbine heat rate
iv. Unit heat rate

Ans
Enthalpy of steam at turbine exhaust = 45.5 +0.9 (618 – 45.5)
i.e. h3 = 560.75 Kcal/Kg.

Generator electric output = 29000 KW


Generator input = 29000 / 0.96 = 30208.33 KW

Loss in gear box = 1100 KW


Output of steam turbine = Generator input + Gear box loss
= 30208.33 + 1100
= 31308.33 KW

i) Output of the steam turbine Say = 31308 KW

ms = Steam flow through turbine

8
_______________________
Bureau of Energy Efficiency
Paper 4 – SET A

= (Turbine output x 860)


(h2 – h3) Turbine enthalpy drop

h2 = Enthalpy at turbine inlet = 795 kcal/kg


h3 = Enthalpy at turbine exhaust = 560.75 kcal/Kg.
ms = 31308 x 860 / 795 – 560.75) = 114940.78 kg/Hr.
= 114.94 TPH
ii) Steam flow through the turbine Say = 115 TPH

iii) Turbine heat rate = Heat input to turbine / Generator output


= [ms (h2 – h1)] / 29000
= 115000 (795 – 105) / 29000
= 2736.2 kcal/ kWh

iv)Unit heat rate = Turbine heat rate / Efficiency of boiler


Unit heat rate = 2736.2 / 0.86 = 3181.63 kcal/ kWh

Or
B In a textile unit a stenter is delivering 80 meters/min of dried cloth at 5%
moisture. The moisture of wet cloth at inlet is 50%. The stenter is heated by
steam at 7 kg/cm2 with inlet enthalpy of 660 kcal/kg. and condensate exits the
stenter at 135 kcal/kg.

Other data
Latent heat of water evaporated from the wet cloth = 540 kcal/kg
Weight of 10 meters of dried cloth = 1 kg
Inlet temperature of wet cloth = 27oC
Outlet temperature of dried cloth at stenter outlet = 80oC.

i) Estimate the steam consumption in the stenter considering a dryer


efficiency of 48%.
ii) Determine the specific steam consumption kg/kg of dried cloth

Ans Output of stenter = 80 mts/min.


= 80 x 60 /10 = 480 Kg/hr.

Moisture in the dried output cloth = 5%


Wt of bone dry cloth = 480 X (1 – 0.05)
i.e. W = 456 Kg/hr.

mo =moisture in outlet cloth


=(480 – 456)/456 =0.0526 Kg./Kg. of bone dried cloth
Inlet moisture = 50%
Wt of inlet cloth = 456 / (1 – 0.50) = 912 Kg./hr.
mi = moisture in inlet cloth
= 912 X 0.5 / 456 = 1.00 Kg./Kg. bone dried cloth
Inlet temperature of cloth = 27oC
9
_______________________
Bureau of Energy Efficiency
Paper 4 – SET A

Final temperature of cloth = 80oC

Heat load on the dryer = w x (mi – mo) X [(Tout – Tin) + 540] Kcal/hr.
.
Heat load on the dryer = 456 (1 – 0.0526) X [(80 – 27) + 540]
= 2,56,184.5 Kcal/hr

Efficiency of the dryer = 48%

Heat input to the stenter = 2,56,184.5 / 0.48 = 5,33,717.71 Kcal/hr

Steam consumption in
the stenter = 5,33,717.71 / (660 – 135)
= 1016.61 Kg/hr

Steam consumption per Kg. of dried at stenter outlet cloth


= 1016.61 / 480
= 2.12 Kg./Kg. dried cloth

Or

C Determine the cooling load of a commercial building for the following given data.
Outdoor conditions :
DBT = 35 C ; WBT = 25 C; Humidity = 18 g of water / kg of dry air
Desired indoor conditions :
DBT = 25.6 C ; RH = 50 %; Humidity = 10 g of water / kg of dry air
Total area of wall = 40 m2
Total area of window = 20m2
U – Factor ( Wall ) = 0.33 W / m2K
U – Factor ( Roof ) = 0.323 W / m2K
U – factor [ fixed windows with aluminum frames and a thermal break ] = 3.56 W / m2K
15 m x 25 m roof constructed of 100 mm concrete with 90 mm insulation & steel
decking.
CLTD at 17:00 h :Details : Wall = 12 C Roof = 44 C Glass Window = 7 C
SCL at 17 : 00 h :Details : Glass Window = 605 W/ m2
Shading coefficient of Window = 0.74
Space is occupied from 8:00 to 17:00 h by 25 people doing moderately active work.
Sensible heat gain / person = 75 W ; Latent heat gain / person = 55 W ; CLF for
people = 0.9

10
_______________________
Bureau of Energy Efficiency
Paper 4 – SET A

Fluorescent light in space = 21.5 W/m2 FLF for lighting = 0.9


Ballast factor details = 1.2 for fluorescent lights & 1.0 for incandescent lights
Computers and office equipment in space produces 5.4 W/m2 of sensible heat
One coffee maker produces 1050 W of sensible heat and 450 W of latent heat.
Air changes / hr of infiltration = 0.3
Height of building = 3.6 m

Ans
I External Heat Gain

(i) Conduction heat gain through the wall =U – factor x net area of wall x CLTD
=[ 0.33 x 40 x 12 ] = 158.4 W
(ii) Conduction heat gain through the roof =U – factor x net area of roof x CLTD
=0.323 x ( 15 x 25 ) x 44 = 5 329.5 W
(iii) Conduction heat gain through the windows =U – factor x net area of windows x CLTD
= (3.56 x 20 x 7) = 498.4 W
(i) Solar radiation through glass = Surface area c Shading coefficient x SCL

=(20 x 0.74 x 605) = 8 954 W

II Internal Heat Gain

(i) Heat gain from people =Sensible heat gain + Latent heat gain

Sensible heat gain =(No.of people x Sensible heat gain / person x CLF)
=(25 x 75 x 0.9) = 1 687.5 W
Latent heat gain =No.of people x Latent heat gain / person
=(25 x 55 ) = 1 375 W
Therefore, Heat gain from people=(1687.5 + 1375 ) = 3 062.5 W

(ii) Heat gain from lighting =( Energy input x Ballast factor x CLF )

Energy input =( Amount of lighting in space / unit area ) x Floor area


=21.5 x ( 15 x 25 ) =8 062.5 W
Therefore, heat gain from lighting =( 8062.5 x 1.2 x 0.9 ) =8 707.5 W

11
_______________________
Bureau of Energy Efficiency
Paper 4 – SET A

(iii) Heat generated by equipment :

Sensible heat generated by coffee maker =1050 W


Latent heat generated by coffee maker =450 W
Sensible heat gain by computers and office equipment = 5.4 x 375 = 2025 W
Therefore, Heat generated by equipment = 3 525 h

(iv) Heat gain through air infiltration =( Sensible heat gain + Latent heat gain )

Sensible heat gain =(1210 x airflow x T )


Airflow =( Volume of space x air change rate ) / 3600
={ (15 x 25 x 3.6 ) x 0.3 } / 3600
=0.1125 m3 / s
Therefore, sensible heat gain =1210 x 0.1125 x ( 35 – 25.6 ) = 1 279.58 W
Latent heat gain =3010 x 0.1125 x ( 18 – 10 ) = 2 709 W

No Space Load Components Sensible Heat Load (W) Latent Heat Load (W)

1 Conduction through exterior wall 158.4 ------

2 Conduction through roof 5 329.5 ------

3 Conduction through windows 498.4 ------

4 Solar radiation through windows 8954 ------

5 Heat gained from people 1 687.5 1 375

6 Heat gained from lighting 8 707.5 ------

7 Heat gained from equipment 3 075 450

8 Heat gained by air infiltration 1 279.58 2 709

Total space cooling load 29 689.88 4 534

Or
D During heat balance of a 5 stage preheater Kiln in a cement plant, the following data
was measured at Preheater (PH) Fan Inlet and clinker cooler vent air fan inlet:

12
_______________________
Bureau of Energy Efficiency
Paper 4 – SET A

Parameter Temperature Static Avg. Specific Gas Duct


measured Pressure Dynamic heat Density Area
Pressure at STP
o
Unit C (Ps) mm (Pd) mm kcal/kg kg/m3 m2
o
WC WC C
PH Exit Gas 316 -650 28.6 0.248 1.4 2.27
at PH fan Inlet
Clinker cooler 268 -56 9.7 0.24 1.29 2.01
vent air at
cooler Stack
Fan Inlet

Note: take Pitot tube constant as 0.85, reference temperature 20 oC and atmospheric pressure
9908 mm WC.

Other Data
Clinker Designed specific NCV of Cost of Annual
Production volume of PH gas Coal coal Operation
TPH Nm3/kg clinker kcal/kg Rs./ton hrs
45.16 1.75 5500 6500 8000

Calculate the following:


i. Specific volume of PH gas as well as cooler vent air (Nm3/kg clinker)

ii. Heat loss in pre-heater exit gas (kcal/kg clinker)

iii. Heat loss in cooler vent air (kcal/kg clinker)

iv. If the measured specific volume of PH gas (Nm3/kg clinker) exceeds the design value,
calculate the heat loss (kcal/kg clinker) and annual monetary loss due to excessive
specific volume of PH gas.

Ans i. Density of Pre-heater gas at PH Fan Inlet at prevailing temp., pressure


conditions:

273 (9908 PS )
T,P STP
(273 T ) 10334
273 (9908 650)
T,P 1.40 = 0.581 kg/m3
(273 316) 10334
Velocity of PH gas
2 g Pd
v Pt
T,P

13
_______________________
Bureau of Energy Efficiency
Paper 4 – SET A

2 9.8 28.6
v 0.85 = 26.4 m/sec
0.581
Volumetric flow rate of PH gas = velocity X duct cross-sectional area
= 26.4 X 2.27
= 59.9 m3/sec
= 59.9 X 3600
= 215640 m3/hr
Specific volume of PH gas = 215640 X 0.58/1.4
= 89491 Nm3/hr
= 89491/45160 = 1.98 Nm3/kg clinker

Similarly density of cooler vent air at cooler vent air fan Inlet at prevailing temp.,
pressure conditions:
273 (9908 PS )
T,P STP
(273 T ) 10334
273 (9908 56)
T,P 1.29 = 0.62 kg/m3
(273 268) 10334
Velocity of cooler vent air in the fan inlet duct
2 g Pd
v Pt
T,P

2 9.8 9.7
v 0.85 = 14.88 m/sec
0.62
Volumetric flow rate of PH gas = velocity X duct cross-sectional area
= 14.88 X 2.01
= 29.9 m3/sec
= 29.9 X 3600
= 107640 m3/hr
Specific volume of cooler vent air = 107640 X 0.62/1.29
= 51734 Nm3/hr
= 51734/45160 = 1.15 Nm3/kg clinker

ii)Heat loss in PH exit gas

Q1 = m ph c p T (Cp of PH gas = 0.248 kcal/kg oC)


Q1= 1.98 X 1.4 X 0.248 x (316-20)
= 203.5 kcal/kg clinker

iii) Heat loss in cooler vent air

Q2 = mCA c p T (Cp of cooler vent air = 0.24 kcal/kg oC)


Q2= 1.15 x 1.29 x 0.24 x (268-20)
= 88.3 kcal/kg clinker
14
_______________________
Bureau of Energy Efficiency
Paper 4 – SET A

iv) Heat Loss due to excess specific volume of PH gas

Vexcess =1.98 – 1.75 = 0.23 Nm3/kg clinker


Heat loss Q = 0.23 x 1.4 x 0.248 x (316-20) = 23.6 kcal/kg clinker

Equivalent coal saving = 23.6/5500 = 0.0043 kg coal/kg clinker or ton of coal/ton of


clinker
Coal saving in one hour = 0.0043 x 45.16 = 0.194 TPH
Annual Coal Saving = 0.194 x 8000= 1552 tons of coal per annum
Annual Monitory Saving = 1552 x 6500 = Rs. 100.88 lakhs

-------- End of Section - III ---------

15
_______________________
Bureau of Energy Efficiency
Paper 4 – SET B

15th NATIONAL CERTIFICATION EXAMINATION


FOR
ENERGY AUDITORS – August, 2014

PAPER – 4: Energy Performance Assessment for Equipment and Utility Systems

Date: 24.8.2013 Timings: 14:00-16:00 Hrs Duration: 2 Hrs Max. Marks: 100

Section - I: BRIEF QUESTIONS Marks: 10 x 1 = 10

(i) Answer all Ten questions


(ii) Each question carries One mark

S-1 Which loss is not considered while evaluating boiler efficiency by “Indirect Method”?

Ans Blow down loss

S-2 What will be the synchronous speed of a VFD driven 4-pole induction motor
operating at 60 Hz ?
Ns = 120 x f/P
Ans
= 120 x 60/4= 1800 RPM
What is the refrigerant used in a vapour absorption system with lithium bromide as
S-3
an absorbent?
Ans Water
Other than rated kW of motor and the actual power drawn, what other parameter is
S-4
required to determine the percentage loading of the motor ?
Ans Motor Efficiency or rated motor efficiency
Inclined tube manometer is used for measuring gas flow in a duct when the air
S-5
velocity is very high: True or False?
False.
Ans
A pump will cavitate if the NPSHavailable. is _________ than the NPSHrequired
S-6
Less
Ans
To determine the effectiveness of the cooling tower, it is required to measure cooling
S-7
water inlet, outlet and _____temperatures.

Ambient Wet bulb


Ans

S-8 The ratio of actual heat transfer to the heat that could be transferred by heat
exchanger of infinite size is termed as ………

1
_______________________
Bureau of Energy Efficiency
Paper 4 – SET B

Ans Effectiveness

S-9 If the unit heat rate of a power plant is 2866 kcal/kWh ,what is the power plant
efficiency ?

Ans (860/2866) x 100 = 30 %

S-10
The difference between GCV and NCV of hydrogen fuel is Zero: True or False
False
Ans

…………. End of Section - I ………….

Section - II: SHORT NUMERICAL QUESTIONS Marks: 2 x 5 = 10

(i) Answer all Two questions


(ii) Each question carries Five marks
Hot water at 80 OC is used for room heating in a 5 Star hotel for 4 months in a year.
L-1
About 200 litres per minute of hot water is maintained in circulation with the return
temperature at 50 OC. The hot water is generated using a „hot waste stream‟,
through a Plate Heat Exchanger (PHE). The hot stream enters the PHE in
counterflow direction at 95 OC and leaves at 60 OC. The area of the heat exchanger
is 22 m2.

Calculate the LMTD and the overall heat transfer coefficient.

Heat load, Q = 200 * 60 * (80 – 50) = 360000 Kcals/hr (or) 418.7 kW


Ans
(95 – 80)/(60 – 50)
LMTD (for counter flow) = ------------------------ = 3.7 OC
ln (15/10)

Overall Heat Transfer Coefficient, U = Q/( AxLMTD)

= 418.7/(22 x 3.7) = 5.14 kW/m2.OC


(OR)
= 4420.4 kcal/hr.m2.OC

A gas turbine generator is delivering an output of 20 MW in an open cycle with a


L-2
heat rate of 3440 kcal/kWh. It is converted to combined cycle plant by adding heat
recovery steam generator and a steam turbine raising the power generation output
to 28 MW. However, with this retrofitting and increased auxiliary consumption, the
fuel consumption increases by 3% in the gas turbine.

Calculate the combined cycle gross heat rate and efficiency.


2
_______________________
Bureau of Energy Efficiency
Paper 4 – SET B

Gas turbine output = 20 MW


Ans
Combined cycle output = 28 MW

Heat rate in GT open cycle for 20 MW = 3440 kcal/kwh

Increase in fuel consumption in = 3%


combined cycle operation

Combined cycle heat rate = (3440 X 1.03) X( 20 / 28)


= 2530.8 kcal/kwh

Combined cycle plant efficiency = (860 / 2530.8) X 100


= 33.98%

…………. End of Section - II ………….

Section - III: LONG NUMERICAL QUESTIONS Marks: 4 x 20 = 80


(i) Answer all Four questions

N-1 The steam requirement of an export oriented unit is met by a 6 TPH oil fired package
boiler generating steam at 10 kg/cm2. The monthly steam consumption of the unit is
3000 tonnes.
Other data are given below:

Fuel oil composition:


Carbon = 86%; Hydrogen = 12%; Oxygen= 0.5%; Sulphur =1.5%

Specific heat of flue gases, Cp = 0.27 kcal/kgoC


G.C.V. of fuel oil = 10,000 kcal/kg
Sp.heat of super heated water vapour = 0.45 kcal/kgoC
Enthalpy of steam at 10 kg/cm2 = 665kcal/kg
Feed water temperature = 85 oC
% O2 in dry flue gas = 6%
Flue gas temperature at boiler outlet = 240 oC
Ambient temperature = 30oC
Cost of fuel oil = Rs.43 per kg.
Radiation and other unaccounted losses = 2.45%

The export oriented unit is costing its steam cost based on the fuel consumption cost
with additional 10% to account for the auxiliary and consumables.

A neighbouring continuous process plant now offers to supply the required steam at 10
kg/cm2 to the export oriented unit at a cost of Rs 3300 per tonne.with a condition that
all the condensate will be returned back.

3
_______________________
Bureau of Energy Efficiency
Paper 4 – SET B

Calculate the following:

a) Boiler efficiency
b) Cost advantage per tonne of availing steam from neighbouring plant in place of
in-house generation and also monthly monetary saving.

Ans First calculate the efficiency of Boiler (in EOU)

Theoretical air required =


= 11.6 C + 34.8 (H – O/8) + 4.35 S
= [11.6 X 86 + 34.8 (12 – 0.5/8) + 4.35 X 1.5] x 1/100
= 14.195 = Say 14.2

% Excess Air = [% O2 / (21 - % O2)] X 100


= [6 / (21 – 6)] X 100 = 40%

AAS = Actual amount of air supplied = 14.2 X 1.4


= 19.88 kg per kg. of fuel oil

Mass of dry flue gas mdfg = Mass of combustion gases due to presence of C,H,S
+Mass of N2 supplied
= (0.86 X 44/12) + (0.015 X 64 / 32) + [(19.88 – 14.2) X 23 / 100] + (19.88 X 77/100)
= 19.797
Mass dry flue gas ,say = 19.8 Kg / kg fuel
Or
Alternatively mass of dry flue gas = (AAS + 1) – 9 H
= (19.88 + 1) – 9 X 0.12 = 19.8 Kg./Kg. fuel

L1 = % heat loss in dry flue gas = [mdfg x Cp x (Tq – Ta) / GCV] x 100

19.8 X 0.27 x (240 – 30)


= --------------------------------- x 100
10,000
L1 = 11.23%
L2 = Loss due to presence of hydrogen forming water vapour
9 x H [584 + Cps (Tg – Ta)]
= ------------------------------------------ x 100
GCV
9 X 0.12 [584 + 0.45 (240 – 30)]
= ------------------------------------------- x 100
10000
L2 = 7.33%
L3 = Radiation and other unaccounted losses = 2.45%
Total losses = L1 + L2 + L3
= 11.23 + 7.33 + 2.45 = 21.05 %
Efficiency of the EOU boiler by indirect method
= 100 – 21.05 =78.99 %

4
_______________________
Bureau of Energy Efficiency
Paper 4 – SET B

= Say 79 %

Secondly calculate the cost of steam in the EOU plant


Evaporation Ratio = [(n X GCV) / (hg – hf)] X 100
= [(0.79 X 10000) / (665 – 85)] X 100
= 13.62 kg Steam / kg. Fuel

Fuel oil consumption = 1000 / 13.62 kg. per tonne of steam


Fuel oil consumption = 73.42 kg./tonne of steam gen

Cost of fuel oil. = Rs. 43 per kg


Cost of steam in EOU = Fuel cost + 10% fuel cost
= 73.42 x 1.10 x 43
= Rs.3472.8 per tonne

Selling cost of steam from neighboring plant = Rs 3300 per tonne

Cost advantage = 3472.8 – 3300 = Rs.172.8 per tonne

Annual Savings = Rs.172.8 per tonne x 3000tonne/month X 12 month


= Rs.62.2 Lacs
N-2
a) The operating parameters of a Vapour Compression Refrigeration system are
indicated below.

Parameter Chiller side Condenser side


Water Flow (m3/hr) 89 87
Inlet Temperature (OC) 12.2 33.3
Outlet Temperature (OC) 8.9 37.6
Density (kg/m3) 1000 990

Find the COP of the Refrigeration system ignoring heat losses.

b) A 6 pole, 415 volt, 3 , 50 Hz induction motor delivers 22 kW power at rotor shaft


at a speed of 950 rpm with PF of 0.9. The total loss in the stator including core,
copper and other losses, is 2 kW. Calculate the following.
i) i) Slip
ii) ii) Rotor Copper Loss
iii) iii) Total Input to motor
iv) ivi) Line current at 415 V and motor pf of 0.9
v) v) Motor operating efficiency

a) Refrigeration Effect = 89 x 1000 x (12.2 – 8.9)


Ans = 293700 kcal/hr
Condenser load = 87 x 990 x (37.6 – 33.3)
= 370359 kcal/hr
5
_______________________
Bureau of Energy Efficiency
Paper 4 – SET B

Compressor work = Condenser load – Refrigeration effect


= 370359 – 293700
= 76659 Kcal/hr
C.O.P. = Refrigeration Effect/ Compressor work
= 293700/76659 = 3.83

b) Synchronous Speed = (120 x 50 / 6 ) = 1000 rpm


Motor Speed = 950 rpm

(i) Slip = (1000 – 950 ) / 1000 = 5 %


Power input to rotor = { ( 22 / ( 1 – 0.05 ) } = 23.16 kW

(ii) Rotor Copper Loss = ( 0.05 x 23.16 ) = 1 . 1 5 8 k W


Or = 23.16-22 =1.16 kW

(iii) Total Input to motor = ( 23.16 + 2 ) = 25.16 kW


(iv) Line Current = ( 25.16 x 1000) /( 3 x 415 x0.9 )
= 38.86 Amps

(v) Motor Efficiency = ( 22 / 25.16 ) = 87.44 %

N-3 A common plant facility is installed to provide steam and power to textile and paper
plant with a co-generation system. The details and operating parameters are given
below:
60 T/hr.80 bar at 5000C

Enthalpy- 810 kcal/kg

To paper
Boiler industry
Turbine G
Coal
Alternator To textile
Boiler efficiency- 80% industry
GCV of Coal-5000 kcal/kg
Boiler feed water temp.- 800C
Steam 60T/hr. at 10 bar
Enthalpy- 660 kcal/kg

Steam 20 T/hr Steam Header Steam 40 T/hr


To textile industry To paper industry

Other data:
- Turbine, alternator and other losses = 8%
- Specific steam consumption in paper industry= 5 Tons/Ton of paper
- Specific power consumption in paper industry= 550 kWh/Ton of paper
Calculate:
i. Coal consumption in boiler per hour or per day.
ii. Power generation from co-generation plant
iii. If 10% is auxiliary power consumption in co-generation plant, how much power
6
_______________________
Bureau of Energy Efficiency
Paper 4 – SET B

is consumed by the textile industry per hour?


iv. What is the gross heat rate of turbine?

Ans
i. i) Boiler efficiency = Steam production ( steam enthalpy- Feed water enthalpy) /
Quantity of coal x G.C.V. of coal
Quantity of coal = 60,000 (810-80)/ 0.8 x 5000
= 10.95 tons/hr.

ii) Gross power generation from co-generation plant

Total enthalpy input to turbine = 60,000 x 810 = 48.6 Million kcal.


Total enthalpy out put through back pressure= 60,000* 660 = 39.6 Million kcal
Enthalpy difference = 48.6- 39.6 = 9 Million kcal/hr
Turbine,alternator and other losses =8% or 9x0.08 = 0.72 Million kcal/hr
Useful energy for power generation = 9- 0.72 = 8.28 Million kcal/hr
6
Power generation from co-generation plant = 8.28 x 10 /860 = 9628kWh

iii) If 10% is auxiliary power consumption in co-generation plant, power consumed by


textile industry

10% of total power generation = 9628 x 0.10 = 962.8kWh

Total power consumed by industries = 9628 – 962.8 = 8665.2kWh

Total steam consumption in paper plant 40 tons/hr. and specific steam consumption
5 ton/ton of paper. So Paper production per hour is 8 tons.

Specific power consumption = 550 kWh/ton.


Total power consumption in paper industry = 8 x 550 = 4400kWh
Total power consumption in textile industry = 8665.2- 4400 = 4265.2 kWh

iv) Gross heat rate= Input enthalpy – output enthalpy/ gross generation
=( 48.6- 39.6) 106/ 9628 = 934.7 kCal/kWh

N-4 To attempt ANY ONE OF THE FOLLOWING among A, B, C and D

A A captive thermal plant is delivering an output of 29 MW at the generator terminal. The


generator efficiency is 96%. The steam generated in a utility boiler with an efficiency of
86% at 105 ata and 485oC is fed to the turbine. The turbine exhausts steam to
condenser maintained at 0.1 ata and 45.5oC. The feed water temperature at inlet to the
boiler is 105oC.

7
_______________________
Bureau of Energy Efficiency
Paper 4 – SET B

The other data pertaining to captive power plant are,

Enthalpy of steam at 105 ata 485oC = 795 kcal/kg.


Dryness fraction of steam at inlet to condenser = 0.9
Enthalpy of dry saturated steam at 0.1ata =618 kcal/Kg.
o
Enthalpy of water at 0.1 ata & at 45.5 C = 45.5 kcal/Kg.
Loss in the gear box connecting turbine and generator = 1100 kW
Enthalpy of feed water at inlet to the boiler , = 105 kcal/Kg.

Based on the above data determine:


i. Output of the steam turbine in kW
ii. Steam flow through the turbine
iii. Turbine heat rate
iv. Unit heat rate

Ans
Enthalpy of steam at turbine exhaust = 45.5 +0.9 (618 – 45.5)
i.e. h3 = 560.75 Kcal/Kg.

Generator electric output = 29000 KW


Generator input = 29000 / 0.96 = 30208.33 KW

Loss in gear box = 1100 KW


Output of steam turbine = Generator input + Gear box loss
= 30208.33 + 1100
= 31308.33 KW

i) Output of the steam turbine Say = 31308 KW

ms = Steam flow through turbine


8
_______________________
Bureau of Energy Efficiency
Paper 4 – SET B

= (Turbine output x 860)


(h2 – h3) Turbine enthalpy drop
h2 = Enthalpy at turbine inlet = 795 kcal/kg
h3 = Enthalpy at turbine exhaust = 560.75 kcal/Kg.
ms = 31308 x 860 / 795 – 560.75) = 114940.78 kg/Hr.
= 114.94 TPH
ii) Steam flow through the turbine Say = 115 TPH

iii) Turbine heat rate = Heat input to turbine / Generator output


= [ms (h2 – h1)] / 29000
= 115000 (795 – 105) / 29000
= 2736.2 kcal/ kWh

iv)Unit heat rate = Turbine heat rate / Efficiency of boiler


Unit heat rate = 2736.2 / 0.86 = 3181.63 kcal/ kWh

Or
B In a textile unit a stenter is delivering 80 meters/min of dried cloth at 5%
moisture. The moisture of wet cloth at inlet is 50%. The stenter is heated by
steam at 7 kg/cm2 with inlet enthalpy of 660 kcal/kg. and condensate exits the
stenter at 105 kcal/kg.

Other data
Latent heat of water evaporated from the wet cloth = 540 kcal/kg
Weight of 10 meters of dried cloth = 1 kg
Inlet temperature of wet cloth = 27oC
Outlet temperature of dried cloth at stenter outlet = 80oC.

i) Estimate the steam consumption in the stenter considering a dryer


efficiency of 48%.
ii) Determine the specific steam consumption kg/kg of dried cloth
Ans Output of stenter = 80 mts/min.
= 80 x 60 /10 = 480 Kg/hr.

Moisture in the dried output cloth = 5%


Wt of bone dry cloth = 480 X (1 – 0.05)
i.e. W = 456 Kg/hr.

mo =moisture in outlet cloth


=(480 – 456)/456 =0.0526 Kg./Kg. of bone dried cloth
Inlet moisture = 50%
Wt of inlet cloth = 456 / (1 – 0.50) = 912 Kg./hr.
mi = moisture in inlet cloth
= 912 X 0.5 / 456 = 1.00 Kg./Kg. bone dried cloth
Inlet temperature of cloth = 27oC
Final temperature of cloth = 80oC

9
_______________________
Bureau of Energy Efficiency
Paper 4 – SET B

Heat load on the dryer = w x (mi – mo) X [(Tout – Tin) + 540] Kcal/hr.
.
Heat load on the dryer = 456 (1 – 0.0526) X [(80 – 27) + 540]
= 2,56,184.5 Kcal/hr

Efficiency of the dryer = 48%

Heat input to the stenter = 2,56,184.5 / 0.48 = 5,33,717.71 Kcal/hr

Steam consumption in
the stenter = 5,33,717.71 / (660 – 105)
= 961.7 Kg/hr

Steam consumption per Kg. of dried cloth at stenter outlet cloth


= 961.7 / 480
= 2 Kg./Kg. dried cloth

Or

C Determine the cooling load of a commercial building for the following given data.
Outdoor conditions :
DBT = 35 C ; WBT = 25 C; Humidity = 18 g of water / kg of dry air
Desired indoor conditions :
DBT = 25.6 C ; RH = 50 %; Humidity = 10 g of water / kg of dry air
Total area of wall = 40 m2
Total area of window = 20m2
U – Factor ( Wall ) = 0.33 W / m2K
U – Factor ( Roof ) = 0.323 W / m2K
U – factor [ fixed windows with aluminum frames and a thermal break ] = 3.56 W / m2K
15 m x 25 m roof constructed of 100 mm concrete with 90 mm insulation & steel
decking.
CLTD at 17:00 h :Details : Wall = 12 C Roof = 44 C Glass Window = 7 C
SCL at 17 : 00 h :Details : Glass Window = 605 W/ m2
Shading coefficient of Window = 0.74
Space is occupied from 8:00 to 17:00 h by 25 people doing moderately active work.
Sensible heat gain / person = 75 W ; Latent heat gain / person = 55 W ; CLF for
people = 0.9
Fluorescent light in space = 21.5 W/m2 FLF for lighting = 0.9

10
_______________________
Bureau of Energy Efficiency
Paper 4 – SET B

Ballast factor details = 1.2 for fluorescent lights & 1.0 for incandescent lights
Computers and office equipment in space produces 5.4 W/m2 of sensible heat
One coffee maker produces 1050 W of sensible heat and 450 W of latent heat.
Air changes / hr of infiltration = 0.3
Height of building = 3.6 m

Ans I External Heat Gain


(i) Conduction heat gain through the wall =U – factor x net area of wall x CLTD
=[ 0.33 x 40 x 12 ] = 158.4 W
(ii) Conduction heat gain through the roof =U – factor x net area of roof x CLTD
=0.323 x ( 15 x 25 ) x 44 = 5 329.5 W
(iii) Conduction heat gain through the windows =U – factor x net area of windows x CLTD
= (3.56 x 20 x 7) = 498.4 W
(i) Solar radiation through glass = Surface area c Shading coefficient x SCL

=(20 x 0.74 x 605) = 8 954 W

II Internal Heat Gain


(i) Heat gain from people =Sensible heat gain + Latent heat gain

Sensible heat gain =(No.of people x Sensible heat gain / person x CLF)
=(25 x 75 x 0.9) = 1 687.5 W
Latent heat gain =No.of people x Latent heat gain / person
=(25 x 55 ) = 1 375 W
Therefore, Heat gain from people=(1687.5 + 1375 ) = 3 062.5 W

(ii) Heat gain from lighting =( Energy input x Ballast factor x CLF )

Energy input =( Amount of lighting in space / unit area ) x Floor area


=21.5 x ( 15 x 25 ) =8 062.5 W
Therefore, heat gain from lighting =( 8062.5 x 1.2 x 0.9 ) =8 707.5 W

(iii) Heat generated by equipment :

Sensible heat generated by coffee maker =1050 W


Latent heat generated by coffee maker =450 W
11
_______________________
Bureau of Energy Efficiency
Paper 4 – SET B

Sensible heat gain by computers and office equipment = 5.4 x 375 = 2025 W
Therefore, Heat generated by equipment = 3 525 h

(iv) Heat gain through air infiltration =( Sensible heat gain + Latent heat gain )

Sensible heat gain =(1210 x airflow x T )


Airflow =( Volume of space x air change rate ) / 3600
={ (15 x 25 x 3.6 ) x 0.3 } / 3600
=0.1125 m3 / s
Therefore, sensible heat gain =1210 x 0.1125 x ( 35 – 25.6 ) = 1 279.58 W
Latent heat gain =3010 x 0.1125 x ( 18 – 10 ) = 2 709 W

No Space Load Components Sensible Heat Load (W) Latent Heat Load (W)
1 Conduction through exterior wall 158.4 ------
2 Conduction through roof 5 329.5 ------
3 Conduction through windows 498.4 ------
4 Solar radiation through windows 8954 ------
5 Heat gained from people 1 687.5 1 375
6 Heat gained from lighting 8 707.5 ------
7 Heat gained from equipment 3 075 450
8 Heat gained by air infiltration 1 279.58 2 709
Total space cooling load 29 689.88 4 534

Or
D During heat balance of a 5 stage preheater Kiln in a cement plant, the following data
was measured at Preheater (PH) Fan Inlet and clinker cooler vent air fan inlet:

Parameter Temperature Static Avg. Specific Gas Duct


measured Pressure Dynamic heat Density Area
Pressure at STP
o
Unit C (Ps) mm (Pd) mm kcal/kg kg/m3 m2
o
WC WC C
PH Exit Gas 316 -650 28.6 0.248 1.4 2.27
at PH fan Inlet
Clinker cooler 268 -56 9.7 0.24 1.29 2.01
vent air at
cooler Stack
Fan Inlet

Note: take Pitot tube constant as 0.85, reference temperature 20 oC and atmospheric pressure
9908 mm WC.

12
_______________________
Bureau of Energy Efficiency
Paper 4 – SET B

Other Data
Clinker Designed specific NCV of Cost of Annual
Production volume of PH gas Coal coal Operation
TPH Nm3/kg clinker kcal/kg Rs./ton hrs
45.16 1.75 5500 5500 8000

Calculate the following:


i. Specific volume of PH gas as well as cooler vent air (Nm3/kg clinker)

ii. Heat loss in pre-heater exit gas (kcal/kg clinker)

iii. Heat loss in cooler vent air (kcal/kg clinker)

iv. If the measured specific volume of PH gas (Nm3/kg clinker) exceeds the design value,
calculate the heat loss (kcal/kg clinker) and annual monetary loss due to excessive
specific volume of PH gas.

Ans i. Density of Pre-heater gas at PH Fan Inlet at prevailing temp., pressure


conditions:

273 (9908 PS )
T,P STP
(273 T ) 10334
273 (9908 650)
T,P 1.40 = 0.581 kg/m3
(273 316) 10334
Velocity of PH gas
2 g Pd
v Pt
T,P

2 9.8 28.6
v 0.85 = 26.4 m/sec
0.581
Volumetric flow rate of PH gas = velocity X duct cross-sectional area
= 26.4 X 2.27
= 59.9 m3/sec
= 59.9 X 3600
= 215640 m3/hr
Specific volume of PH gas = 215640 X 0.58/1.4
= 89491 Nm3/hr
= 89491/45160 = 1.98 Nm3/kg clinker

Similarly density of cooler vent air at cooler vent air fan Inlet at prevailing temp.,
pressure conditions:
273 (9908 PS )
T,P STP
(273 T ) 10334

13
_______________________
Bureau of Energy Efficiency
Paper 4 – SET B

273 (9908 56)


T,P 1.29 = 0.62 kg/m3
(273 268) 10334
Velocity of cooler vent air in the fan inlet duct
2 g Pd
v Pt
T,P

2 9.8 9.7
v 0.85 = 14.88 m/sec
0.62
Volumetric flow rate of PH gas = velocity X duct cross-sectional area
= 14.88 X 2.01
= 29.9 m3/sec
= 29.9 X 3600
= 107640 m3/hr
Specific volume of cooler vent air = 107640 X 0.62/1.29
= 51734 Nm3/hr
= 51734/45160 = 1.15 Nm3/kg clinker

ii)Heat loss in PH exit gas

Q1 = m ph c p T (Cp of PH gas = 0.248 kcal/kg oC)


Q1= 1.98 X 1.4 X 0.248 x (316-20)
= 203.5 kcal/kg clinker

iii) Heat loss in cooler vent air

Q2 = mCA c p T (Cp of cooler vent air = 0.24 kcal/kg oC)


Q2= 1.15 x 1.29 x 0.24 x (268-20)
= 88.3 kcal/kg clinker

iv) Heat Loss due to excess specific volume of PH gas

Vexcess =1.98 – 1.75 = 0.23 Nm3/kg clinker


Heat loss Q = 0.23 x 1.4 x 0.248 x (316-20) = 23.6 kcal/kg clinker

Equivalent coal saving = 23.6/5500 = 0.0043 kg coal/kg clinker or ton of coal/ton of


clinker
Coal saving in one hour = 0.0043 x 45.16 = 0.194 TPH
Annual Coal Saving = 0.194 x 8000= 1552 tons of coal per annum
Annual Monitory Saving = 1552 x 5500 = Rs. 85.36 lakhs

-------- End of Section - III ---------

14
_______________________
Bureau of Energy Efficiency
Paper 4 – SET B KEY

14th NATIONAL CERTIFICATION EXAMINATION


FOR
ENERGY AUDITORS – August, 2013

PAPER – 4: Energy Performance Assessment for Equipment and Utility Systems

Date: 25.8.2013 Timings: 14:00-16:00 Hrs Duration: 2 Hrs Max. Marks: 100

Section - I: BRIEF QUESTIONS Marks: 10 x 1 = 10

(i) Answer all Ten questions


(ii) Each question carries One mark

S-1 If EER of a 1.5 TR window airconditioner is 3 what will be the power input?

Ans 1.5 x 3.5163 = 1.758 kW

What is the significance of monitoring dew point of compressed air for


S-2
pneumatic instruments application?

Ans To check the moisture level/dryness in instrument air

For a thermal power plant, the percentage auxiliary consumption of a 110 MW


S-3
unit is ____than that of a 500 MW unit.

Ans More

Between one kg of ‘liquid hydrogen’ and one litre of ‘liquid gasoline’ which will
S-4
have a higher heat content?

Ans Liquid hydrogen

Why is the COP of a vapour absorption refrigeration system always less than
S-5
one?

COP is given by (heat taken by evaporator/ heat given to generator). The heat given
Ans to generator of VAR is always more than heat taken away in the evaporator
(refrigeration effect)

S-6 Regenerators utilising waste heat are widely used in _________furnaces

Ans Glass melting or Open hearth furnaces

S-7 Why small bypass lines are provided in a centrifugal pump?

1
_______________________
Bureau of Energy Efficiency
Paper 4 – SET B KEY

Ans To avoid pump running at zero flow

If the speed of a reciprocating pump is reduced by 50 %, what will be its effect


S-8
on the head?

Ans The head will remain the same

As the ‘approach’ decreases, the other parameters remaining constant, the


S-9
effectiveness of cooling tower will _____________

Ans Increase

In a DG set, waste heat is used for steam generation. This type of


S-10
cogeneration is called _________cycle.

Ans Topping

…………. End of Section - I ………….

Section - II: SHORT NUMERICAL QUESTIONS Marks: 2 x 5 = 10

(i) Answer all Two questions


(ii) Each question carries Five marks

L-1 An automobile plant has a maximum demand of 5000 kVA at a PF of 0.95. The
plant has shifted its electric annealing furnace with a steady resistive load of
500 kW to its foundry unit in a nearby location after suitable modifications.
What will be the new PF of the automobile plant without the electric annealing
furnace?

Ans
Existing maximum demand in kW, 5000 x 0.95 = 4750 KW
Existing reactive power load in the plant

(KVAR)2 = KVA2 – KW 2 = (5000)2 – (4750)2

KVAR = 1561

Electrical load after shifting 500 KW annealing furnace = 4750 – 500 = 4250 KW

However, KVAR load will remain same as 500 kW annealing furnace did not impose
any kVAr loading.

KVA = SQRT [(4250)2 + (1561)2] = 4528

PF = 4250 / 4528 = 0.938

2
_______________________
Bureau of Energy Efficiency
Paper 4 – SET B KEY

L-2 In a medium sized engineering industry a 340 m3/hr reciprocating compressor


is operated to meet compressed air requirement at 7 bar. The compressor is in
loaded condition for 80% of the time. The compressor draws 32 kW during
load and 7 kW during unload cycle.
After arresting the system leakages the loading time of the compressor came
down to 60%.
Calculate the annual energy savings at 6000 hours of operation per year.

Ans
Average power consumption with 80% loading =
= [0.8 x 32 + 0.2 x 7] = 27 KW

Average power consumption with 60% loading after leakage reduction =


= [0.6 x 32 + 0.4 x 7] = 22 kW

Saving in electrical power = 5 KW


Yearly savings = 5 x 6000
= 30,000 kWH

…………. End of Section - II ………….

Section - III: LONG NUMERICAL QUESTIONS Marks: 4 x 20 = 80

(i) Answer all Four questions

N1 A multi-product chemical plant has an oil fired boiler for meeting its steam requirements for
process heating. The average fuel oil consumption for the boiler was found to be 950 litres per
hour. Calculate the cost of steam per tonne considering only the fuel cost.

The performance and other associated data are given below:

O2 in the flue gas (dry) at boiler exit = 6%


Temperature of the flue gas at boiler exit = 200oC
Enthalpy of steam = 665 kcal/kg
Enthalpy of feed water = 80 kcal/kg
Steam is dry saturated.

Fuel analysis data:


Carbon (C) = 85% Hydrogen (H2) = 12% Nitrogen (N2) = 0.5%
Oxygen (O2) = 1% Sulfur (S) = 1.5%

Gross calorific value of fuel oil = 10,000 kcal/kg


Specific gravity of fuel oil = 0.9
Cost of fuel oil per KL = Rs.40,850/-

Specific heat of flue gas = 0.262 kcal/kgoC

3
_______________________
Bureau of Energy Efficiency
Paper 4 – SET B KEY

Specific heat of superheated vapour in flue gas = 0.43 kcal/kgoC


Humidity in combustion air = 0.025 kg/kg dry air
Ambient air temperature = 30oC
Radiation & convection loss from boiler = 1.8%

ANS
Calculate boiler efficiency by indirect method

Calculate evaporation ratio Kg steam / Kg fuel oil

And then compute fuel cost of steam

Boiler efficiency by indirect method:

Theoretical air required for complete combustion of fuel oil

= {11.6. C + [34.8 (H2 – O2/8)] + 4.35 S} / 100


= {11.6 x 85 [34.8 (12 – 1/8)] + 4.35 x 1.5} / 100
= 14.05 Kg/Kg fuel oil

% O2 in fuel gas = 6
% Excess air = [%O2 / (21 - % O2)] x 100
= [6 / (21 – 6)] x 100
= 40%

Actual Air Supplied (ASS) = (1 + 0.4) x 14.05 = 19.67 Kg/Kg fuel oil

Mass of dry flue gas = mdfg

Mass of dry flue gas = mass of combustion gases due to presence C, S, O2, N2
+ mass of N2 in air supplied

Mdfg = 0.85 x (44 / 12) + 0.015 x (64 / 32) + .005 + [(19.67 – 14.05) x (23 / 100)] + 19.67 x (77/100)
Mdfg = 19.59 Kg/Kg fuel oil

Alternatively Mdfg = (AAS+1) – (9xH2) = (19.67+1) – (9 x 0.12) = 19.59 kg/kg fuel oil

% heat loss in dry flue gas = mdfg x Cpf x (Tg – Ta) / GCV of fuel
Tg = flue gas temperature = 200oC
Ta = ambient temperature = 30oC
Cp = SP ht of flue gas = 0.26 Kcal/KgoC
GCV = Gross Calorific Value of fuel oil = 10,000 Kcal/kg

L1 = % heat loss in dry flue gases = [(19.59 x 0.262 x (200-30))/10,000] X 100 = 8.73 %

Heat loss due to evaporation of water due to H2 in fuel


= {9 x H2 [584 + CPS (Tg – Ta)]} / GCV
CPS = Specific heat of superheated steam
= 0.43 Kcal/KgoC

4
_______________________
Bureau of Energy Efficiency
Paper 4 – SET B KEY

L2 = {9 x 0.12 [ 584 + 0.43 (200 – 30)] / 10000} x 100 = 7.09%


L3 = % heat loss due to moisture in fuel = 0
As % moisture in fuel is nil (not given)

% heat loss due to moisture in air

L4 = AAS x humidity factor x CPS x (Tg – Ta) / GCV


Humidity factor = 0.025 Kg/Kg dry air

L4 = {[19.67 x 0.025 x 0.43 (200-30)] / 10000} x 100 = 0.36%

L5 = Radiation and convection loss from the boiler = 1.8% (given data)

Total losses in the boiler in % = L1 + L2 + L3 + L4 + L5


= 8.73 + 7.09 + 0 + 0.36 + 1.8
= 17.98 say 18 %
Efficiency of boiler by indirect method = 100 - % total loss = 100 – 18 = 82%

Boiler n = E.R. (hs – hw) / GCV x 100


ER = Evaporation Ratio = Kg steam / Kg fuel oil
hs = Enthalpy of Steam = 665 Kcal/Kg
hw = feed water enthalpy = 80 Kcal/Kg
Boiler Efficiency = 82%

ER = 0.82 x 10000 / (665-80) = 14.02 = Say 14 Kg steam / Kg fuel oil

Cost of fuel oil per KL = Rs.40,850/-


S.G. of fuel oil = 0.9
Cost of fuel oil per tonne = 40,850 / 0.9 = Rs.45,389/-

Fuel cost of steam per tonne = 45,389 / 14 = Rs.3242

N2 In a food processing unit, 24,000 litres of water per day is to be heated from 25oC to 55oC.
Presently this requirement is met by an electrical heater. The management is planning to install a
vapour compression heat pump system having a COP of 2.3 which includes the compressor motor
losses. The schematic of the heat pump hot water system is given below:

5
_______________________
Bureau of Energy Efficiency
Paper 4 – SET B KEY

Hours of operation of water circulation pump = 24 hours/day


Evaporator fan operation = 20 hours/day
Energy consumption of water circulation pump
and evaporator fan per day = 50 kWh
Compressor motor efficiency = 88 %
Annual operating days of heat pump = 330 days
Cost of electrical energy = Rs.10/kWh
Heat loss in the condenser and hot water tank
in addition to the heat load = 5%
Investment for heat pump = Rs.15 Lakhs
The compressor and evaporator fan are interlocked in operation.

Find out
i) Heat pump capacity in TR in terms of heat delivered
ii) The payback period of investment towards heat pump
iii) Evaporator capacity in TR
i)
Hot water requirement per day = 24000 litres = 24000 kgs.
ANS
Inlet water temperature = 25oC
Outlet water temperature = 55oC

Energy required for electrical heating per day = 24000 (55 – 25) / 860 = 837.2 Kwh

Heat load on the condenser per day = 24000 (55 – 25) x 1.05
including 5% loss (in the condenser & hot water tank)
= 756000 Kcals/day
Heat pump capacity based on delivered heat =756000/(24 x 3024)
= 10.4 TR
Electrical energy equivalent of heat delivered = 756000 / 860 = 879.07 Kwh

Daily energy consumption in the heat pump with a COP of 2.3 = 879.07 / 2.3
= 382.2 Kwh

Daily energy consumption in the circulating = 50 Kwh (given data)


water pump and evaporator fan

Total energy consumption for operation of = 432.2 Kwh


heat pump per day

Energy saving with heat pump compared = 837.2 – 432.2 = 405 Kwh
to electrical heating per day

Cost of electricity per kwh = Rs.10


Monetary saving per day = 405 x 10 = Rs.4,050/-

Annual savings with 330 days operation = 4050 x 330 = Rs.13,36,500/-


= Rs.13.365 lacs

Investment for heat pump = Rs.15 lacs


6
_______________________
Bureau of Energy Efficiency
Paper 4 – SET B KEY

Simple pay back period = 15 / 13.365 = 13 months

The investment is attractive and justifiable.

Evaporator capacity in tonne refrigeration


= Heat delivered at the condenser – Input energy to the compressor
= [{879 – (382.17 x 0.88)} x 860] / (3024 x 20)
= 7.72 TR

N-3 In an organic chemical industry 10 tonne per hour of hot oil is to be cooled from 210 oC to 105oC by
DM water. The DM water enters the heat exchanger at 25oC and exits at 85oC after which it is fed to
the feed water storage tank of the boiler.

i. Depict the heat exchanger process on a schematic for the parallel and counter flow
indicating the hot and cold stream temperatures along with terminal temperature difference.
ii. Find out the LMTD for parallel and counter flow heat exchange and justify the choice of the
heat exchanger.
iii. Estimate the DM water flow rate through the heat exchanger. The specific heat of oil is 0.8
kcal/kgoC.

Ans I)

ii)
Δ t1 - Δ t2
LMTD parallel flow = -----------------
Ln Δt1 / Δt2

= (185 – 20) / ln (185 / 20) = 74.19oC

LMTD Counter flow = (125 – 80) / ln (125/80) = 100.9oC

Counter flow heat exchange will yield higher LMTD and hence heat exchanger area will be less and hence
preferred.
iii)
mc = mass flow rate of DM water
mc x 1 x (85 – 25) = mh x 0.8 x (210 – 105)
mc x 1 x (85 – 25) = 10000 x 0.8 x (210 – 105)
mc = 14,000 kg/hour

7
_______________________
Bureau of Energy Efficiency
Paper 4 – SET B KEY

N-4 To attempt ANY ONE OF THE FOLLOWING among A, B, C and D

A An energy audit was conducted on a 110 MW thermal power generating unit. The details of
design parameters and operating parameters observed during the audit are given below.
Parameters Design Operating
Generator output 110 MW 110 MW
Boiler outlet superheated steam temperature 540oC 520oC
Boiler outlet steam pressure 140 kg/cm2(a) 130 kg/cm2(a)
Feed water inlet temperature to Boiler 120 oC 120 oC
Feed water enthalpy 120 kcal/kg 120 kcal/kg
Boiler efficiency 87% 87%
GCV of coal 3650 kcal/Kg 3650 kcal/Kg
Turbine exhaust steam pressure 0.09 kg/cm2(a) 0.12 kg/cm2(a)
Dryness fraction of exhaust steam 88% 88%
Unit gross heat rate 2815 kcal/kWh ?
Efficiency of turbine & generator (including gear box) - 90 %

Steam properties are as under:

Enthalpy of steam at 520oC and 130 kg/cm2(a) is 808.4 kcal/kg


Enthalpy of Exhaust steam at 0.12 kg/cm2(a) is 550 kcal/kg

For the changed current operating parameters calculate the following..


I. Steam flow rate to the Turbine
II. Specific steam consumption of Turbine
III. Specific coal consumption and unit gross heat rate
IV. Additional quantity of coal required based on 8000 hours/year of operation of the plant
V. Increase in annual coal cost due to increase in coal consumption at a cost of Rs. 4000 per
tonne of coal.

Ans
(i) Calculation of Steam flow rate to Turbine

Turbine output, KW = m x (Hs – Hf)/ 860

Where, m = steam flow to turbine, Kg/hr


Hs = Enthalpy of steam at 520 Deg.C& 130 Kg/cm2 = 808.4 Kcal/Kg
Hf = Enthalpy of turbine exhaust steam =550 Kcal/kg

Turbine output = Generator output/ Efficiency of Turbine & Generator


= 110 / 0.9 =122.2 MW
8
_______________________
Bureau of Energy Efficiency
Paper 4 – SET B KEY

122.2 x 1000 = (m x (808.4 – 550))/860


Steam flow rate to Turbine, m = 406.7 Tonnes/hr

(ii) Calculation of specific steam consumption of Turbine, kg/kwh

Specific steam consumption = Steam flow to turbine, Kg/hr / Generator output, Kw


= 406.7 x 1000/( 110 x 1000)
= 3.697 Kg/Kwh
(OR)
ALTERNATE PROCEDURE
Specific steam consumption = 860/((Hs – Hf) x Efficiency of Turbine & Generator)
= 860/(( 808.4 – 550) x 0.9)
= 3.697 Kg/Kwh

(iii) Calculation of specific coal consumption, Kg/kwh


Boiler efficiency = m kg/hr ( Hs – Hw) Kcal/kg / ( Q x GCV)------------(2)
0.87 = (406.7 x1000 ( 808.4 – 120 ))/ ( Q X 3650 )
Coal consumption Q = 88166 Kg/hr

Specific coal consumption = Coal consumption, Kg/hr / Generator output, Kw


Specific coal consumption = 88166/ (110 x 1000)= 0.801 Kg /Kwh
Unit gross Heat rate= 0.801x3650=2923.6 kcal/kWh

( iv) Additional quantity of coal required


Specific coal consumption at design conditions = unit heat rate / GCV of coal
2815/3650 =0.771 kg/kwh

Additional coal consumption/year = (0.801 – 0.771) x 110 x 1000 x 8000 =26400000 kg


Additional quantity of coal required /year = 26400 Tonnes

(v) Annual increase in coal cost


Additional cost of coal = 26400Tonnes x 4000 Rs/Tonne
=Rs. 10,56,00,000

B
During the conduct of heat balance of a 5 stage inline calciner Kiln of a cement plant , the following data
were measured at preheater outlet using pitot tube and flue gas analyser.

Temp Static Pressure Avg. Dynamic Pressure Oxygen CO2 CO Duct Area
o
C (Ps) mm WC (Pd) mm WC % (v/v) dry % (v/v) dry % (v/v) dry m2
350 -435 16.9 6.0 19.2 0.06 3.098

Note: take Pitot tube constant as 0.85, reference temperature as 20 oC and atmospheric pressure same
as at sea level i.e. 10334 mm WC.

Other Data obtained


Kiln Clinker Return Dust in PH NCV of Cost of Annual Operation
Feed Production gas Coal coal
9
_______________________
Bureau of Energy Efficiency
Paper 4 – SET B KEY

TPH TPH % of Kiln Feed kcal/kg INR hrs


(Rs)/tonne
88.5 55 6.8 5356 6950 8000

Cp of PH gas = 0.25 kcal/kg oC) , (Cp of return dust = 0.23 kcal/kg oC)

Calculate the following:

a. Specific volume of Preheater gas (Nm3/kg clinker)


b. Heat loss in pre-heater exit gas (kcal/kg clinker)
c. Heat loss in pre-heater return dust (kcal/kg clinker)
d. Heat loss due to CO formation (kcal/kg clinker)
e. Reduction in above mentioned heat losses (kcal/kg clinker) and the annual thermal monitory
savings if the Preheater exit gas temperature is reduced to 330 oC and there is no CO formation in
the system.

Ans :
a. Density of Pre-heater gas at STP:

(O2 %  MWO2 )  (CO2 %  MWCO2 )  ((N 2  CO)%  MWCO )


 STP 
22.4 100
(6.0  32)  (19.2 44)  ((74.74 0.06)  28)
 STP  = 1.398 kg/Nm3
22.4 100

273 (10334 PS )
T , P   STP 
(273 T ) 10334
273 (10334 435)
T , P 1.393 = 0.587 kg/m3
(273 350) 10334

Velocity of Preheater gas


2 g Pd
v  Pt
T , P
2  9.8 16.9
v  0.85 = 20.19 m/sec
0.585
Volumetric flow rate of PH gas = velocity x duct cross-sectional area
= 20.22 x 3.098
= 62.55 m3/sec
= 62.55 x 3600
= 225180 m3/hr
= 225180 x 0.587/1.398
= 94550 Nm3/hr
Specific volume of PH gas = 94550/55000 = 1.72 Nm3/kg clinker

10
_______________________
Bureau of Energy Efficiency
Paper 4 – SET B KEY

b. Heat loss in pre-heater exit gas

Q1 = m ph c p T (Cp of PH gas = 0.25 kcal/kg oC)


Q1= 1.72 x 1.398 x 0.25 (350-20)
= 198.37 kcal/kg clinker

c. Heat loss in return dust

Q2 = mdust c p T (Cp of return dust = 0.23 kcal/kg oC)


mdust = clinker factor x % return dust/100
= (88.5/55) x (6.8/100)
= 1.609 x 0.068
= 0.1094 kg dust/ kg clinker
Q2= 0.1094 x 0.23 x (350-20)
= 8.3 kcal/kg clinker

d. Heat Loss due to CO Formation

Q3= mco X 67636


CO%
mco= X sp. volum eof PH Gas
22.4 X 100
= [0.06/(22.4 x100)] x 1.72
Q3= 2.68 x 10-5 x 1.72 x 67636
= 3.12 kcal/kg clinker

e. At exit temperature 330 oC the above losses would be

Q1’= 1.72 x 1.398 x 0.25 x (330-20)


= 186.35 kcal/clinker
Thermal saving = Q1- Q1’= 197.95 – 186.35 = 12.02 kcal/kg clinker

Q2’= 0.1094 x 0.23 x (330-20)


= 7.8 kcal/kg clinker
Thermal Saving = Q2 – Q2’ = 8.3 – 7.8 = 0.5 kcal/kg clinker
Overall Thermal Savings = 12.02 + 0.5 + 3.12
= 15.64 kcal/kg clinker

Equivalent coal saving = 14.56/5356 = 0.0029 kg coal/kg clinker


Coal saving in one hour = 0.0029 x 55 = 0.1595 TPH
Annual Coal Saving = 0.1595 x 8000= 1276 tons of coal

Annual Monitory Saving = 1276 x 6950 = INR 88,68,200

or

11
_______________________
Bureau of Energy Efficiency
Paper 4 – SET B KEY

C In a textile process house a new stenter is being installed with a feed rate of 1000 kg/hr of wet
cloth having a moisture content of 55%. The outlet (final) moisture of the dried cloth is 7%. The
inlet and outlet temperature of the cloth is 25oC and 75oC respectively. The drying efficiency of the
stenter is 50%. It is proposed to connect the stenter to the existing thermic fluid heater of
20,00,000 kcal/hr capacity, which is already loaded to 60% of its capacity. The thermic fluid heater
has an efficiency of 75%. Check whether the thermic fluid heater will be able to cater to the input
heat requirements of the stenter.

Ans Feed rate of wet cloth to the stenter = 1000 Kg/hr


Initial moisture = 55%
Quantity of moisture at inlet = 0.55 x 1000 = 550 Kg/hr
Wt of bone dry cloth = 1000 – 550 = 450 Kg/hr
Final moisture in outlet cloth = 7%
Quantity of moisture (final) at outlet = (450 / 0.93)– 450 = 483.87 – 450 = 33.87 Kg/hr
mi = inlet moisture in bone dry cloth = 550 / 450 = 1.22 kg moisture / kg cloth
mo = Outlet (final) moisture in bone dry cloth = 33.87 / 450 = 0.0753 Kg moisture/Kg cloth

Heat load of the stenter for drying process = W x (mi – mo) x [(Tin – Tout) + 540]
W = Wt of bone dry cloth Kg/hr
Tin = Inlet temperature of cloth to stenter
Tout = Outlet temperature of cloth from stenter
Latent heat of evaporation of water = 540 Kcal/Kg

Heat load of the stenter for drying = 450 (1.22 – 0.0753) x [(75 – 25) + 540]
= 303917.85 Kcal/hr.

Input heat requirement of the stenter = 303917.85 / 0.5 = 607835.7 Kcal/hr.


with 50% efficiency for drying
(heat to be supplied)

Capacity of the thermic flue heater = 2000000 Kcal/hr


Existing load = 60% x 2000000 = 12,00,000 Kcal/hr
Balance capacity = 8,00,000 Kcal/hr.

The thermic fluid heater capacity is sufficient to cater to the input heat requirement of the new stenter.

D In a steel plant, daily sponge iron production is 500 tons. The sponge iron is further processed in a steel
melting shop for production of ingots. The yield from converting sponge iron into ingots is 88%. The plant
has a coal fired captive power station to meet the entire power demand of the steel plant.

The base year (2011) and current year (2012) energy consumption data are given below:

Parameters Base Year (2011) Current Year (2012)


Sponge iron production 500 T/day 500 T/day
Specific coal consumption for sponge 1.2 T/ T of Sponge Iron 1.1 T/T of Sponge Iron
iron production

12
_______________________
Bureau of Energy Efficiency
Paper 4 – SET B KEY

Specific power consumption for sponge 120 kWh/ T of Sponge 100 kWh/ T of Sponge
iron production Iron Iron
Yield ,in converting sponge iron into 91% 91%
ingot in steel melting shop
Specific power consumption in steel 950 kWh / T of Ingot 900 kWh / T of Ingot
melting shop to produce ingots
Captive power station heat rate 3500 kcal/ kWh 3200 kcal / kWh
GCV of coal 5000 kcal /kg 5000 kcal /kg

i) Calculate the specific energy consumption of the plant in Million kcals / Ton of finished product
(Ingot) for the base year as well as for the current year
ii) Reduction in Coal consumption per day in current year compared to base year for the plant

Ans i) specific energy consumption of the plant


For Base Year

Specific energy consumption for sponge iron = 1200 kgx 5000 + 120 Kwhx 3500
= 6.42 million K Cal/ Ton of SI
Total energy consumption for sponge iron /day 6.42 X 500=3210 million K Cal/day
Actual production considering 88% yield from sponge = 500 Tons x 0.91 = 455 Tons / day
iron to ingot conversion
Specific energy consumption for ingot = 950 Kwhx 3500
= 3.325 million Kcal/ ton of ingot
Total energy consumption for ingot production per day 3.325X 455= 1512.87 million K Cal/day
Plant specific energy consumption for production of = (3210+1512.87)/455
finished product ( ingot) during base year = 10.38million Kcal/ ton

For Current Year

Specific energy consumption for sponge iron = 1100 kgx 5000 + 100 Kwhx 3200
= 5.82 million K Cal Ton of SI
Total energy consumption for sponge iron /day 5.82 X 500=2910 million KCal/day
Actual production considering 88% yield from sponge = 500 T X 0.91 = 455 Tons / day
iron to ingot conversion
Specific energy consumption for ingot = 900Kwh x 3200 = 2.88 million
Kcal/ ton of ingot
Total energy consumption for ingot production per day 2.88 X 455= 1310.4 million
KCal/day
Plant specific energy consumption for production of =(2910+1310.4)/455
finished product ( ingot) during current year = 9.27 million Kcal/ ton

ii) Reduction in coal consumption

13
_______________________
Bureau of Energy Efficiency
Paper 4 – SET B KEY

Energy saving in sponge iron plant = (6.42-5.82) x500 = 300 million Kcals/day
Energy saving in steel melting plant = ( 3.325-2.88) x 455 = 202.5 million Kcal/day
Total energy saving = 300 + 202.5 = 502.5 million Kcals/day
Equivalent coal reduction(saving) = 495.8 x 106 /5000 = 100.5 Tons per day

-------- End of Section - III ---------

14
_______________________
Bureau of Energy Efficiency
Paper 4 – SET A KEY

14th NATIONAL CERTIFICATION EXAMINATION


FOR
ENERGY AUDITORS – August, 2013

PAPER – 4: Energy Performance Assessment for Equipment and Utility Systems

Date: 25.8.2013 Timings: 14:00-16:00 Hrs Duration: 2 Hrs Max. Marks: 100

Section - I: BRIEF QUESTIONS Marks: 10 x 1 = 10

(i) Answer all Ten questions


(ii) Each question carries One mark

S-1 If EER of a 1.5 TR window airconditioner is 2.5 what will be the power input?

Ans 1.5 x 3.516/2.5 = 2.11 kW

What is the significance of monitoring dew point of compressed air for


S-2
pneumatic instruments application?

Ans To check the moisture level/dryness in instrument air

For a thermal power plant, the percentage auxiliary consumption of a 500 MW


S-3
unit is ____than that of a 110 MW unit.

Ans Less

Between one litre of ‘liquid hydrogen’ and one litre of ‘liquid gasoline’ which
S-4
will have a higher heat content?

Ans Liquid gasoline

Why is the COP of a vapour absorption refrigeration system always less than
S-5
one?

COP is given by (heat taken by evaporator/ heat given to generator). The heat given
Ans to generator of VAR is always more than heat taken away in the evaporator
(refrigeration effect)

S-6 Regenerators utilising waste heat are widely used in _________furnaces

Ans Glass melting or Open hearth furnaces

S-7 Why small bypass lines are provided in a centrifugal pump?

1
_______________________
Bureau of Energy Efficiency
Paper 4 – SET A KEY

Ans To avoid pump running at zero flow

If the speed of a reciprocating pump is reduced by 50 %, what will be its effect


S-8
on the head?

Ans The head will remain the same

As the ‘approach’ increases, the other parameters remaining constant, the


S-9
effectiveness of cooling tower will _____________

Ans Decrease

In a DG set, waste heat is used for steam generation. This type of


S-10
cogeneration is called _________cycle.

Ans Topping

…………. End of Section - I ………….

Section - II: SHORT NUMERICAL QUESTIONS Marks: 2 x 5 = 10

(i) Answer all Two questions


(ii) Each question carries Five marks

L-1 An automobile plant has a maximum demand of 5000 kVA at a PF of 0.95. The
plant has shifted its electric annealing furnace with a steady resistive load of
600 kW to its foundry unit in a nearby location after suitable modifications.
What will be the new PF of the automobile plant without the electric annealing
furnace?

Ans
Existing maximum demand in kW, 5000 x 0.95 = 4750 KW
Existing reactive power load in the plant

(KVAR)2 = KVA2 – KW 2 = (5000)2 – (4750)2

KVAR = 1561

Electrical load after shifting 600 KW annealing furnace = 4750 – 600 = 4150 KW

However, KVAR load will remain same as 600 kW annealing furnace did not impose
any kVAr loading.

KVA = SQRT [(4150)2 + (1561)2] = 4434

PF = 4150 / 4434 = 0.936

2
_______________________
Bureau of Energy Efficiency
Paper 4 – SET A KEY

L-2 In a medium sized engineering industry a 340 m3/hr reciprocating compressor


is operated to meet compressed air requirement at 7 bar. The compressor is in
loaded condition for 80% of the time. The compressor draws 32 kW during
load and 7 kW during unload cycle.
After arresting the system leakages the loading time of the compressor came
down to 60%.
Calculate the annual energy savings at 6000 hours of operation per year.

Ans
Average power consumption with 80% loading =
= [0.8 x 32 + 0.2 x 7] = 27 KW

Average power consumption with 60% loading after leakage reduction =


= [0.6 x 32 + 0.4 x 7] = 22 KW

Saving in electrical power = 5 KW


Yearly savings = 5 x 6000
= 30,000 kWH

…………. End of Section - II ………….

Section - III: LONG NUMERICAL QUESTIONS Marks: 4 x 20 = 80

(i) Answer all Four questions

N1 A multi-product chemical plant has an oil fired boiler for meeting its steam requirements for
process heating. The average fuel oil consumption for the boiler was found to be 950 litres per
hour. Calculate the cost of steam per tonne considering only the fuel cost.

The performance and other associated data are given below:

O2 in the flue gas (dry) at boiler exit = 6%


Temperature of the flue gas at boiler exit = 200oC
Enthalpy of steam = 665 kcal/kg
Enthalpy of feed water = 80 kcal/kg
Steam is dry saturated.

Fuel analysis data:


Carbon (C) = 85% Hydrogen (H2) = 12% Nitrogen (N2) = 0.5%
Oxygen (O2) = 1% Sulfur (S) = 1.5%

Gross calorific value of fuel oil = 10,000 kcal/kg


Specific gravity of fuel oil = 0.95
Cost of fuel oil per KL = Rs.40,850/-

Specific heat of flue gas = 0.262 kcal/kgoC

3
_______________________
Bureau of Energy Efficiency
Paper 4 – SET A KEY

Specific heat of superheated vapour in flue gas = 0.43 kcal/kgoC


Humidity in combustion air = 0.025 kg/kg dry air
Ambient air temperature = 30oC
Radiation & convection loss from boiler = 1.8%

ANS
Calculate boiler efficiency by indirect method

Calculate evaporation ratio Kg steam / Kg fuel oil

And then compute fuel cost of steam

Boiler efficiency by indirect method:

Theoretical air required for complete combustion of fuel oil

= {11.6. C + [34.8 (H2 – O2/8)] + 4.35 S} / 100


= {11.6 x 85 [34.8 (12 – 1/8)] + 4.35 x 1.5} / 100
= 14.05 Kg/Kg fuel oil

% O2 in fuel gas = 6
% Excess air = [%O2 / (21 - % O2)] x 100
= [6 / (21 – 6)] x 100
= 40%

Actual Air Supplied (ASS) = (1 + 0.4) x 14.05 = 19.67 Kg/Kg fuel oil

Mass of dry flue gas = mdfg

Mass of dry flue gas = mass of combustion gases due to presence C, S, O2, N2
+ mass of N2 in air supplied

Mdfg = 0.85 x (44 / 12) + 0.015 x (64 / 32) + .005 + [(19.67 – 14.05) x (23 / 100)] + 19.67 x (77/100)
Mdfg = 19.59 Kg/Kg fuel oil

Alternatively Mdfg = (AAS+1) – (9xH2) = (19.67+1) – (9 x 0.12) = 19.59 kg/kg fuel oil

% heat loss in dry flue gas = mdfg x Cpf x (Tg – Ta) / GCV of fuel
Tg = flue gas temperature = 200oC
Ta = ambient temperature = 30oC
Cp = SP ht of flue gas = 0.26 Kcal/KgoC
GCV = Gross Calorific Value of fuel oil = 10,000 Kcal/kg

L1 = % heat loss in dry flue gases = [(19.59 x 0.262 x (200-30))/10,000] X 100 = 8.73 %

Heat loss due to evaporation of water due to H2 in fuel


= {9 x H2 [584 + CPS (Tg – Ta)]} / GCV
CPS = Specific heat of superheated steam
= 0.43 Kcal/KgoC

4
_______________________
Bureau of Energy Efficiency
Paper 4 – SET A KEY

L2 = {9 x 0.12 [ 584 + 0.43 (200 – 30)] / 10000} x 100 = 7.09%


L3 = % heat loss due to moisture in fuel = 0
As % moisture in fuel is nil (not given)

% heat loss due to moisture in air

L4 = AAS x humidity factor x CPS x (Tg – Ta) / GCV


Humidity factor = 0.025 Kg/Kg dry air

L4 = {[19.67 x 0.025 x 0.43 (200-30)] / 10000} x 100 = 0.36%

L5 = Radiation and convection loss from the boiler = 1.8% (given data)

Total losses in the boiler in % = L1 + L2 + L3 + L4 + L5


= 8.73 + 7.09 + 0 + 0.36 + 1.8
= 17.98 say 18 %
Efficiency of boiler by indirect method = 100 - % total loss = 100 – 18 = 82%

Boiler n = E.R. (hs – hw) / GCV x 100


ER = Evaporation Ratio = Kg steam / Kg fuel oil
hs = Enthalpy of Steam = 665 Kcal/Kg
hw = feed water enthalpy = 80 Kcal/Kg
Boiler Efficiency = 82%

ER = 0.82 x 10000 / (665-80) = 14.02 = Say 14 Kg steam / Kg fuel oil

Cost of fuel oil per KL = Rs.40,850/-


S.G. of fuel oil = 0.95
Cost of fuel oil per tonne = 40,850 / 0.95 = Rs.43,000/-

Fuel cost of steam per tonne = 43000 / 14 = Rs.3071.45

N2 In a food processing unit, 24,000 litres of water per day is to be heated from 25oC to 55oC.
Presently this requirement is met by an electrical heater. The management is planning to install a
vapour compression heat pump system having a COP of 2.3 which includes the compressor motor
losses. The schematic of the heat pump hot water system is given below:

5
_______________________
Bureau of Energy Efficiency
Paper 4 – SET A KEY

Hours of operation of water circulation pump = 24 hours/day


Evaporator fan operation = 20 hours/day
Energy consumption of water circulation pump
and evaporator fan per day = 50 kWh
Compressor motor efficiency = 88 %
Annual operating days of heat pump = 330 days
Cost of electrical energy = Rs.10/kWh
Heat loss in the condenser and hot water tank
in addition to the heat load = 5%
Investment for heat pump = Rs.15 Lakhs
The compressor and evaporator fan are interlocked in operation.

Find out
i) Heat pump capacity in TR in terms of heat delivered
ii) The payback period of investment towards heat pump
iii) Evaporator capacity in TR
i)
Hot water requirement per day = 24000 litres = 24000 kgs.
ANS
Inlet water temperature = 25oC
Outlet water temperature = 55oC

Energy required for electrical heating per day = 24000 (55 – 25) / 860 = 837.2 Kwh

Heat load on the condenser per day = 24000 (55 – 25) x 1.05
including 5% loss (in the condenser & hot water tank)
= 756000 Kcals/day
Heat pump capacity based on delivered heat =756000/(24 x 3024)
= 10.4 TR
Electrical energy equivalent of heat delivered = 756000 / 860 = 879.07 Kwh

Daily energy consumption in the heat pump with a COP of 2.3 = 879.07 / 2.3
= 382.2 Kwh

Daily energy consumption in the circulating = 50 Kwh (given data)


water pump and evaporator fan

Total energy consumption for operation of = 432.2 Kwh


heat pump per day

Energy saving with heat pump compared = 837.2 – 432.2 = 405 Kwh
to electrical heating per day

Cost of electricity per kwh = Rs.10


Monetary saving per day = 405 x 10 = Rs.4,050/-

Annual savings with 330 days operation = 4050 x 330 = Rs.13,36,500/-


= Rs.13.365 lacs

Investment for heat pump = Rs.15 lacs


6
_______________________
Bureau of Energy Efficiency
Paper 4 – SET A KEY

Simple pay back period = 15 / 13.365 = 13 months

The investment is attractive and justifiable.

Evaporator capacity in tonne refrigeration


= Heat delivered at the condenser – Input energy to the compressor
= [{879 – (382.17 x 0.88)} x 860] / (3024 x 20)
= 7.72 TR

N-3 In an organic chemical industry 10 tonne per hour of hot oil is to be cooled from 210 oC to 105oC by
DM water. The DM water enters the heat exchanger at 25oC and exits at 85oC after which it is fed to
the feed water storage tank of the boiler.

i. Depict the heat exchanger process on a schematic for the parallel and counter flow
indicating the hot and cold stream temperatures along with terminal temperature difference.
ii. Find out the LMTD for parallel and counter flow heat exchange and justify the choice of the
heat exchanger.
iii. Estimate the DM water flow rate through the heat exchanger. The specific heat of oil is 0.5
kcal/kgoC.

Ans I)

ii)
Δ t1 - Δ t2
LMTD parallel flow = -----------------
Ln Δt1 / Δt2

= (185 – 20) / ln (185 / 20) = 74.19oC

LMTD Counter flow = (125 – 80) / ln (125/80) = 100.9oC

Counter flow heat exchange will yield higher LMTD and hence heat exchanger area will be less and hence
preferred.
iii)
mc = mass flow rate of DM water
mc x 1 x (85 – 25) = mh x 0.5 x (210 – 105)
mc x 1 x (85 – 25) = 10000 x 0.5 x (210 – 105)
mc = 8750 kg/hour

7
_______________________
Bureau of Energy Efficiency
Paper 4 – SET A KEY

N-4 To attempt ANY ONE OF THE FOLLOWING among A, B, C and D

A An energy audit was conducted on a 110 MW thermal power generating unit. The details of
design parameters and operating parameters observed during the audit are given below.
Parameters Design Operating
Generator output 110 MW 110 MW
Boiler outlet superheated steam temperature 540oC 520oC
Boiler outlet steam pressure 140 kg/cm2(a) 130 kg/cm2(a)
Feed water inlet temperature to Boiler 120 oC 120 oC
Feed water enthalpy 120 kcal/kg 120 kcal/kg
Boiler efficiency 87% 87%
GCV of coal 3650 kcal/Kg 3650 kcal/Kg
Turbine exhaust steam pressure 0.09 kg/cm2(a) 0.12 kg/cm2(a)
Dryness fraction of exhaust steam 88% 88%
Unit gross heat rate 2815 kcal/kWh ?
Efficiency of turbine & generator (including gear box) - 90 %

Steam properties are as under:

Enthalpy of steam at 520oC and 130 kg/cm2(a) is 808.4 kcal/kg


Enthalpy of Exhaust steam at 0.12 kg/cm2(a) is 550 kcal/kg

For the changed current operating parameters calculate the following..


I. Steam flow rate to the Turbine

II. Specific steam consumption of Turbine

III. Specific coal consumption and unit gross heat rate

IV. Additional quantity of coal required based on 8000 hours/year of operation of the plant

V. Increase in annual coal cost due to increase in coal consumption at a cost of Rs. 3400 per
tonne of coal.

Ans
(i) Calculation of Steam flow rate to Turbine

Turbine output, KW = m x (Hs – Hf)/ 860

Where, m = steam flow to turbine, Kg/hr


Hs = Enthalpy of steam at 520 Deg.C& 130 Kg/cm2 = 808.4 Kcal/Kg

8
_______________________
Bureau of Energy Efficiency
Paper 4 – SET A KEY

Hf = Enthalpy of turbine exhaust steam =550 Kcal/kg

Turbine output = Generator output/ Efficiency of Turbine & Generator


= 110 / 0.9 =122.2 MW
122.2 x 1000 = (m x (808.4 – 550))/860
Steam flow rate to Turbine, m = 406.7 Tonnes/hr

(ii) Calculation of specific steam consumption of Turbine, kg/kwh

Specific steam consumption = Steam flow to turbine, Kg/hr / Generator output, Kw


= 406.7 x 1000/( 110 x 1000)
= 3.697 Kg/Kwh
(OR)

ALTERNATE PROCEDURE

Specific steam consumption = 860/((Hs – Hf) x Efficiency of Turbine & Generator)


= 860/(( 808.4 – 550) x 0.9)
= 3.697 Kg/Kwh

(iii) Calculation of specific coal consumption, Kg/kWh


Boiler efficiency = m kg/hr ( Hs – Hw) Kcal/kg / ( Q x GCV)------------(2)
0.87 = (406.7 x1000 ( 808.4 – 120 ))/ ( Q X 3650 )
Coal consumption Q = 88166 Kg/hr

Specific coal consumption = Coal consumption, Kg/hr / Generator output, Kw


Specific coal consumption = 88166/ (110 x 1000)= 0.801 Kg /Kwh
Unit gross Heat rate= 0.801x3650=2923.6 kcal/kWh

( iv) Additional quantity of coal required


Specific coal consumption at design conditions = unit heat rate / GCV of coal
2815/3650 =0.771 kg/kwh
Additional coal consumption/year = (0.801 – 0.771) x 110 x 1000 x 8000 =26400000 kg
Additional quantity of coal required /year = 26400 Tonnes

(v) Annual increase in coal cost


Additional cost of coal = 26400Tonnes x 3400 Rs/Tonne
=Rs. 8,97,60,000
B

During the conduct of heat balance of a 5 stage inline calciner Kiln of a cement plant , the following data
were measured at preheater outlet using pitot tube and flue gas analyser.

Temp Static Pressure Avg. Dynamic Pressure Oxygen CO2 CO Duct Area
o
C (Ps) mm WC (Pd) mm WC % (v/v) dry % (v/v) dry % (v/v) dry m2
350 -435 16.9 6.0 19.2 0.06 3.098

Note: take Pitot tube constant as 0.85, reference temperature as 20 oC and atmospheric pressure same
as at sea level i.e. 10334 mm WC.
9
_______________________
Bureau of Energy Efficiency
Paper 4 – SET A KEY

Other Data obtained


Kiln Clinker Return Dust in PH NCV of Cost of Annual Operation
Feed Production gas Coal coal
TPH TPH % of Kiln Feed kcal/kg INR hrs
(Rs)/tonne
88.5 55 6.8 5356 6950 8000

Cp of PH gas = 0.25 kcal/kg oC) , (Cp of return dust = 0.23 kcal/kg oC)

Calculate the following:

a. Specific volume of Preheater gas (Nm3/kg clinker)

b. Heat loss in pre-heater exit gas (kcal/kg clinker)

c. Heat loss in pre-heater return dust (kcal/kg clinker)

d. Heat loss due to CO formation (kcal/kg clinker)

e. Reduction in above mentioned heat losses (kcal/kg clinker) and the annual thermal monitory
savings if the Preheater exit gas temperature is reduced to 330 oC and there is no CO formation in
the system.

Ans :
a. Density of Pre-heater gas at STP:

(O2 %  MWO2 )  (CO2 %  MWCO2 )  ((N 2  CO)%  MWCO )


 STP 
22.4 100
(6.0  32)  (19.2 44)  ((74.74 0.06)  28)
 STP  = 1.398 kg/Nm3
22.4 100

273 (10334 PS )
T , P   STP 
(273 T ) 10334
273 (10334 435)
T , P 1.393 = 0.587 kg/m3
(273 350) 10334

Velocity of Preheater gas


2 g Pd
v  Pt
T , P
2  9.8 16.9
v  0.85 = 20.19 m/sec
0.585
Volumetric flow rate of PH gas = velocity x duct cross-sectional area
= 20.22 x 3.098
= 62.55 m3/sec
10
_______________________
Bureau of Energy Efficiency
Paper 4 – SET A KEY

= 62.55 x 3600
= 225180 m3/hr
= 225180 x 0.587/1.398
= 94550 Nm3/hr
Specific volume of PH gas = 94550/55000 = 1.72 Nm3/kg clinker

b. Heat loss in pre-heater exit gas

Q1 = m ph c p T (Cp of PH gas = 0.25 kcal/kg oC)


Q1= 1.72 x 1.398 x 0.25 (350-20)
= 198.37 kcal/kg clinker

c. Heat loss in return dust

Q2 = mdust c p T (Cp of return dust = 0.23 kcal/kg oC)


mdust = clinker factor x % return dust/100
= (88.5/55) x (6.8/100)
= 1.609 x 0.068
= 0.1094 kg dust/ kg clinker
Q2= 0.1094 x 0.23 x (350-20)
= 8.3 kcal/kg clinker

d. Heat Loss due to CO Formation

Q3= mco X 67636


CO%
mco= X sp. volum eof PH Gas
22.4 X 100
= [0.06/(22.4 x100)] x 1.72
Q3= 2.68 x 10-5 x 1.72 x 67636
= 3.12 kcal/kg clinker

e. At exit temperature 330 oC the above losses would be

Q1’= 1.72 x 1.398 x 0.25 x (330-20)


= 186.35 kcal/clinker
Thermal saving = Q1- Q1’= 197.95 – 186.35 = 12.02 kcal/kg clinker

Q2’= 0.1094 x 0.23 x (330-20)


= 7.8 kcal/kg clinker
Thermal Saving = Q2 – Q2’ = 8.3 – 7.8 = 0.5 kcal/kg clinker
Overall Thermal Savings = 12.02 + 0.5 + 3.12
= 15.64 kcal/kg clinker

Equivalent coal saving = 14.56/5356 = 0.0029 kg coal/kg clinker


Coal saving in one hour = 0.0029 x 55 = 0.1595 TPH
Annual Coal Saving = 0.1595 x 8000= 1276 tons of coal
11
_______________________
Bureau of Energy Efficiency
Paper 4 – SET A KEY

Annual Monitory Saving = 1276 x 6950 = INR 88,68,200

or

C In a textile process house a new stenter is being installed with a feed rate of 1000 kg/hr of wet
cloth having a moisture content of 55%. The outlet (final) moisture of the dried cloth is 7%. The
inlet and outlet temperature of the cloth is 25oC and 75oC respectively. The drying efficiency of the
stenter is 50%. It is proposed to connect the stenter to the existing thermic fluid heater of
10,00,000 kcal/hr capacity, which is already loaded to 60% of its capacity. The thermic fluid heater
has an efficiency of 75%. Check whether the thermic fluid heater will be able to cater to the input
heat requirements of the stenter.

Ans Feed rate of wet cloth to the stenter = 1000 Kg/hr


Initial moisture = 55%
Quantity of moisture at inlet = 0.55 x 1000 = 550 Kg/hr
Wt of bone dry cloth = 1000 – 550 = 450 Kg/hr
Final moisture in outlet cloth = 7%
Quantity of moisture (final) at outlet = (450 / 0.93)– 450 = 483.87 – 450 = 33.87 Kg/hr
mi = inlet moisture in bone dry cloth = 550 / 450 = 1.22 kg moisture / kg cloth
mo = Outlet (final) moisture in bone dry cloth = 33.87 / 450 = 0.0753 Kg moisture/Kg cloth

Heat load of the stenter for drying process = W x (mi – mo) x [(Tin – Tout) + 540]
W = Wt of bone dry cloth Kg/hr
Tin = Inlet temperature of cloth to stenter
Tout = Outlet temperature of cloth from stenter
Latent heat of evaporation of water = 540 Kcal/Kg

Heat load of the stenter for drying = 450 (1.22 – 0.0753) x [(75 – 25) + 540]
= 303917.85 Kcal/hr.

Input heat requirement of the stenter = 303917.85 / 0.5 = 607835.7 Kcal/hr.


with 50% efficiency for drying
(heat to be supplied)

Capacity of the thermic flue heater = 1000000 Kcal/hr


Existing load = 60% x 1000000 = 600000 Kcal/hr
Balance capacity = 400000 Kcal/hr.

The thermic fluid heater capacity is not sufficient to cater to the input heat requirement of the new stenter.

D In a steel plant, daily sponge iron production is 500 tons. The sponge iron is further processed in a steel
melting shop for production of ingots. The yield from converting sponge iron into ingots is 88%. The plant
has a coal fired captive power station to meet the entire power demand of the steel plant.

The base year (2011) and current year (2012) energy consumption data are given below:

Parameters Base Year (2011) Current Year (2012)


12
_______________________
Bureau of Energy Efficiency
Paper 4 – SET A KEY

Sponge iron production 500 T/day 500 T/day


Specific coal consumption for sponge 1.2 T/ T of Sponge Iron 1.1 T/T of Sponge Iron
iron production
Specific power consumption for sponge 120 kWh/ T of Sponge 100 kWh/ T of Sponge
iron production Iron Iron
Yield ,in converting sponge iron into 88% 88%
ingot in steel melting shop
Specific power consumption in steel 950 kWh / T of Ingot 900 kWh / T of Ingot
melting shop to produce ingots
Captive power station heat rate 3500 kcal/ kWh 3200 kcal / kWh
GCV of coal 5000 kcal /kg 5000 kcal /kg

i) Calculate the specific energy consumption of the plant in Million kcals / Ton of finished product
(Ingot) for the base year as well as for the current year

ii) Reduction in Coal consumption per day in current year compared to base year for the plant

Ans i) specific energy consumption of the plant

For Base Year

Specific energy consumption for sponge iron = 1200 kgx 5000 + 120 Kwhx 3500
= 6.42 million K Cal/ Ton of SI
Total energy consumption for sponge iron /day 6.42 X 500 = 3210 million kCal
Actual production considering 88% yield from sponge = 500 Tons x 0.88 = 440 Tons / day
iron to ingot conversion
Specific energy consumption for ingot = 950 kWhx 3500
= 3.325 million Kcal/ ton of ingot
Total energy consumption for ingot production per day 3.325X 440=1463 million kCal
Plant specific energy consumption for production of (3210+1463)/440
finished product ( ingot) during base year = 10.62 million kCal/ ton

For Current Year

Specific energy consumption for sponge iron = 1100 kgx 5000 + 100 Kwhx 3200
= 5.82 million K Cal Ton of SI
Total energy consumption for sponge iron /day 5.82 X 500 = 2910 million kCal
Actual production considering 88% yield from sponge = 500 T X 0.88 = 440 Tons / day
iron to ingot conversion
Specific energy consumption for ingot = 900Kwh x 3200 = 2.88 million
Kcal/ ton of ingot
Total energy consumption for ingot production per day 2.88 X 440 = 1267.2 million kCal
Plant specific energy consumption for production of = (2910+1267.2)/440
13
_______________________
Bureau of Energy Efficiency
Paper 4 – SET A KEY

finished product ( ingot) during current year = 9.49 million Kcal/ ton

ii) Reduction in coal consumption

Energy saving in sponge iron plant = (6.42-5.82) x500 = 300 million Kcals/day
Energy saving in steel melting plant = ( 3.325-2.88) x 440 = 195.8 million Kcal/day
Total energy saving = 300 + 195.8 = 495.8 million Kcals/day
Equivalent coal reduction(saving) = 495.8 x 106 /5000 = 99.16 Tons per day

-------- End of Section - III ---------

14
_______________________
Bureau of Energy Efficiency
Click Here & Upgrade
Expanded Features
PDF Unlimited Pages
Documents
Complete Paper 4 – SET A KEY

13th NATIONAL CERTIFICATION EXAMINATION


FOR
ENERGY AUDITORS – September, 2012

PAPER – 4: Energy Performance Assessment for Equipment and Utility


Systems

Date: 16.9.2012 Timings: 14:00-16:00 HRS Duration: 2 HRS Max. Marks: 100

Section - I: BRIEF QUESTIONS Marks: 10 x 1 = 10

(i) Answer all Ten questions


(ii) Each question carries One mark

S-1 In a vapour compression refrigeration system, why the heat rejected in the
condenser is more than the heat absorbed in the evaporator ?

Ans Because heat of compression is also added to it

S-2 If the unit heat rate is 3120 kcal/kWh and the turbine heat rate is 2808
kCal/kWh what is the boiler efficiency ?

Ans (2808/3120) x 100 = 90 %

S-3 A rise in conductivity of boiler feed water indicates ____ .

Rise in the TDS level of feed water


Ans

S-4 Why is it preferable to measure the flow at the inlet side of the fan?

Ans Less turbulence

S-5 The critical point of steam occurs at ____bar and _______ oC

Ans 221.2 bar and 374.15oC

S-6 In a heat exchanger _______ is the ratio of actual heat transfer rate to the
maximum heat transfer rate.

1
_______________________
Bureau of Energy Efficiency
Click Here & Upgrade
Expanded Features
PDF Unlimited Pages
Documents
Complete Paper 4 – SET A KEY

Ans Effectiveness

S-7 In an integrated steel plant pig iron is produced from _________furnace?

Ans Blast furnace

S-8
If the PLF of a 210 MW power plant is 80% , what is the annual gross generation in
MWh

Ans 1,471,680 MWH

S-9 A pump operates on water with a total head of 12 m. If water is replaced by


brine with a specific gravity of 1.2 what will be the total head developed by the
pump ?

Ans 12 m or same

S-10 A draft system in a boiler which uses both FD and ID fan is called…….

Ans Balanced Draft

. End of Section - I .

Section - II: SHORT NUMERICAL QUESTIONS Marks: 2 x 5 = 10

(i) Answer all Two questions


(ii) Each question carries Five marks

L-1 Calculate pressure drop in meters when pipe diameter is increased from
250 mm to 300 mm for a length of 600 meters. Water velocity is 2 m/s in
the 250 mm diameter pipe and friction factor is 0.005.

Ans Pressure drop = 4fLV2


--------------
2gD

Velocity of water in pipe of 300 mm diameter = (0.25 x 0.25 x 2) /(0.3 x 0.3)


= 1.39 m/s

Pressure drop with 300 mm = 4 x 0.005 x 600 x 1.392 / (2 x 9.81 x 0.300)


= 3.94 m

2
_______________________
Bureau of Energy Efficiency
Click Here & Upgrade
Expanded Features
PDF Unlimited Pages
Documents
Complete Paper 4 – SET A KEY

L-2 A three phase 37 kW four pole induction motor operating at 49.8 Hz is rated for 415
V, 50 Hz and 1440 RPM. The actual measured speed is 1460 RPM. Find out the
percentage loading of the motor if the voltage applied is 410 V.
Ans

% Loading = Slip x 100%


(Ss – Sr) x (Vr / V)2

Synchronous speed = 120 x 49.8 / 4 = 1494 rpm

Slip = Synchronous Speed – Measured speed in rpm.


= 1494 – 1460 = 34 rpm.

% Loading = 34 x 100% = 61.45%


(1494 - 1440) x (415/410)2

. End of Section - II .

3
_______________________
Bureau of Energy Efficiency
Click Here & Upgrade
Expanded Features
PDF Unlimited Pages
Documents
Complete Paper 4 – SET A KEY

Section - III: LONG NUMERICAL QUESTIONS Marks: 4 x 20 = 80

(i) Answer all Four questions

• Refer Original question paper for questions

N1 Key

a) Theoretical air required for complete combustion


   O  
(11.6 x C ) + 34.8 x  H 2 −  2    + (4.35 x S )
   8  
=
100
   32.52  
(11.6 x 33.95) + 34.8 x  5.01 −    + (4.35 x 0.09)
=    8   = 4.27 kg / kg of paddy husk
100

 77 
4.27 ×  
 100   0.0091 
Moles of N2 = +  = 0.1178
28  28 

Moles of C
% CO2 theoretical =
Moles of N 2 + Moles of C + Moles of S

 0.3395 
 
=  12 
 0.3395   0.0009 
0.1178 +   +  
 12   32 

Max theoretical ( CO2 )t = 19.36 %

Actual CO2 measured in flue gas = 14.0%


7900 x [( CO 2 ) t − (CO 2 ) a
b) % Excess air supplied = = 37.5 %
(CO 2 ) a x [100 − ( CO 2 ) t ]

c) Actual mass of air supplied = {1 + EA/100} x theoretical air


= {1 + 37.5/100} x 4.27
= 5.87 kg/kg of coal

4
_______________________
Bureau of Energy Efficiency
Click Here & Upgrade
Expanded Features
PDF Unlimited Pages
Documents
Complete Paper 4 – SET A KEY

0 .3395 × 44 5 .87 × 77 (5 .87 − 4 .27 ) × 23


Mass of dry flue gas = + 0 .0091 + +
12 100 100

= 6.15 kg / kg of coal

(or)
(actual mass of air supplied + 1) – mass of H20

(5.87 + 1) – (9H + M) = 6.87 – (9x.05 + 0.1079)= 6.87 – 0.5579 = 6.31 kg/kg of coal

m x C P x ( Tf − Ta )
% Heat loss in dry flue gas = x 100
GCV of fuel
6.15 x 0.23 x (160 − 32 )
= x 100
3568
= 5.07 %

Loss due to CO =

= 0.35 x 0.3395 x 5654


(0.35+14) x 3568

L2 = 1.31 %

Heat Loss in ash

% heat loss due to unburnt flyash

% ash in paddy husk = 16.73


Ratio of bottom ash to flyash = 10:90
GCV of flyash = 450 kcal/kg
Amount of flyash in 1 kg of husk = 0.9 x 0.1673
= 0.15 kg
Heat loss in flyash = 0.15 x 450
= 67.5 kcal/kg of husk
GCV of bottom ash = 800 kcal/kg

Amount of bottom ash in 1 kg of husk = 0.1 x 0.1673


= 0.01673 kg
Heat loss in bottom ash = 0.01673 x 800
= 13.4 kcal/kg of husk

5
_______________________
Bureau of Energy Efficiency
Click Here & Upgrade
Expanded Features
PDF Unlimited Pages
Documents
Complete Paper 4 – SET A KEY

Total heat loss in ash = 67.5 + 13.4


= 80.9 kcal/kg
% loss in ash = 80.9/3568
= 2.26 %

Total losses = 100 – (5.07 + 1.31 +2.26) – (15.4)

Boiler efficiency = 100 – 8.64 – 15.4 = 75.96 %

N-2 KEY

Hot Water use per day : 20,000 L/day

Water in = 200C
Water out = 600C
Temp. diff. = 400C
Total Heat required = mCpdt
= 20000 x 1 x 40 = 8,00,000 kcal/day

1. Energy Requirement for 20KL/day of water for a temperature differential of 40


deg.C in an Electric Boiler/Geyser

Energy Requirement (for 20 KL/day)


= Total heat required (800000)
860 kcal/kWh x 0.99 ( efficiency of electric heating))
= 939.6 kWh/day

2. For 20 KL/day, of water flow with 400C Temperature Diff. Energy to be drawn by
Heat Pump

= 8,00,000 = 391.68 Kwh/day


860x0.95x2.5

Energy drawn by circulation pump = 3.74 x 24 hr = 89.76 kWh/day


Energy drawn by evaporator fan = 1.4 kW x 16 hr = 22.4 kWh/day

Total Energy drawn by heat pump system = 391.68 +89.76+22.4


= 503.8 kWh /day

SAVINGS IN COMPARISON TO ELECTRIC WATER HEATER

= 939.6 – 503.8 = 435.75 Kwh/day


= 1,52,516 kWh/year ( @ 350 days/year)
= 12.20 lakhs ( @ Rs8.0 per kWh)

6
_______________________
Bureau of Energy Efficiency
Click Here & Upgrade
Expanded Features
PDF Unlimited Pages
Documents
Complete Paper 4 – SET A KEY

3. SIMPLE PAY BACK PERIOD = Rs.16.0 LAKHS Investment/ Rs.12.20 lakhs per
year savings
= 1.30 years or 16 months

N-3 KEY

Ans
Power generation from cogen plant = 5000X 0.9 X 8000 = 360 lac Kwh/yr

Auxiliary power = 1%
Net power generation = 0.99 X 360 = 356.4 lac Kwh

Natural gas requirement for = 360 X 3050 / 9500 = 115.57 lac sm3
power generation

Cost of fuel per annum = 115.57 X 8 = Rs.924.56 lacs

Annual expenditure for interest, = 500 + 200 = 700 lacs


depreciation and O&M

Total cost of generation = Rs.1624.56 lacs.

Cost of cogeneration power = 1624.56 X 105 / 356.4 X 105

= Rs.4.56 / Kwh.

Gas consumption in existing gas = [10000 (665 – 85) / (0.86 X 9500)]


fired boiler = 710 Sm3/hr
= 710 x 24 = 17040 sm3/day

Cost of steam from existing boiler = 710*Rs. 8 x8000


= Rs. 454.4 Lacs /yr

Cost of power generation after giving = 1624.56 – 454.4 = Rs.1170.16 lacs


credit for steam generation

Cost of power generation after accounting = 1170.16 X 105 / 356.4 X 105


for steam cost
= Rs. 3.28 / Kwh

Grid power cost = Rs. 4.5 / Kwh

Cost advantage for cogen plant = 4.5 – 3.28 = Rs.1.22 / Kwh


generation

Daily gas requirement for operating = 5000 X 0.9 X 3050 X 24


GT cogen plant 9500
= 34673.68 Sm3 / day
7
_______________________
Bureau of Energy Efficiency
Click Here & Upgrade
Expanded Features
PDF Unlimited Pages
Documents
Complete Paper 4 – SET A KEY

Additional gas requirement for = 34673.68 – 17040 = 17633.68 Sm3/day


co-gen plant

N-4 To attempt ANY ONE OF THE FOLLOWING among A, B, C and D

N4 A KEY

Ans i) Turbine power output kW =

Steam flow to turbine kg/hr x enthalpy drop across the turbine kcal/kg
-------------------------------------------------------------------------------------------------------
860

Inlet enthalpy of steam =794.4 kcal/kg

Enthalpy of exhaust steam is calculated as given below


exhaust steam dryness fraction = 90%
enthalpy of exhaust steam = (45.9 + 0.9 x 572.5) = 561 kcal/kg
turbine out put = ((120 x 1000 kg/hr x (794.4 – 561) kcal/kg) /860
turbine output = 32567.4 kW

ii) generator output kW = turbine output x combined efficiency of mechanical, gear


transmission & generator
= 32567.4 x 0.92
=29962 kW

iii) turbine heat rate = heat input in to the turbine/ generator out put
=q x (h1 – hw)/generator out put

Where q = steam inflow to turbine kg/hr


h1= enthalpy of turbine inlet steam =794.4 kcal/kg
hw= enthalpy of feed water to boiler = 100 kcal/kg

Turbine heat rate = ((120 x 1000 kg/hr) x (794.4 – 100) kcal/kg))/ 29962 kw
= 2781 kcal/kwh

iv) unit heat rate = turbine heat rate /boiler efficiency


= 2781 / 0.88 = 3160 kcal/ kwh

v) turbine cycle efficiency = (860 / turbine heat rate) x 100


= 860 /2781 =0.309
=0.309 x 100 = 30.9%

vi) condenser heat load = m x cp x dt

Where m = cooling water flow through condenser, kg/hr

8
_______________________
Bureau of Energy Efficiency
Click Here & Upgrade
Expanded Features
PDF Unlimited Pages
Documents
Complete Paper 4 – SET A KEY

note: density of water is given as 0.95 g /cubic centimetre = 950 kg/ cubic meter
cp = specific heat of cooling water, kcal/ kg. oC = 0.98 kcal /kg. oC
dt = cooling water temperature rise, oC = 10

Condenser heat load =6318 x 950 x 0.98 x 10 = 5,88,20,580 kcal /hr

vii) specific steam consumption of turbine = 860 / (enthalpy drop x combined efficiency)
= 860/ ((794.4 – 561) x 0.92))
=860 / (233.4 x 0.92) =4.0 kg/kwh
= 4.0 kg / kwh

N4-B KEY
Ans
Volumetric flow rate of PH gas at NTP = 1.47 x 125 x 1000 = 183750 [Nm3/hr]

Mass flow rate of PH gas = 183750 x 1.42 = 260925 [kg/hr]

Calculation for 4 stage pre-heater kiln

Heat loss in PH Gas = m x cp x T [kcal/hr]


= 260925 x 0.244 x 370 = 23556309 [kcal/hr]
Equivalent coal wasted = 23556309 = 4.252 [tons of coal/hr]
5540 Χ 1000

Electrical Energy consumption of PH Fan

Volumetric flow rate of PH Gas at 370 oC temperature and -400 mm WC static pressure:

V = 183750 Χ ( 273 + 370) Χ 10333 = 450216 [m3/hr]


273 Χ (10333 − 400 )
or V = 450216/3600 = 125 [m3/sec]

Pressure difference across PH fan = 50 – (- 400) = 450 [mm WC]


Power consumption of PH fan
125 Χ 450 = 806.24 [kW]
P=
102 Χ 0.72 Χ 0 .95

Calculation for 6 stage pre-heater kiln

Heat loss in PH Gas = m x cp x T [kcal/hr]


= 260925 x 0.244 x 295 = 18781381 [kcal/hr]

Equivalent coal wasted = 18781381 = 3.39 [tons of coal/hr]


5540 Χ 1000

Electrical Energy consumption of PH Fan


Volumetric flow rate of PH Gas at 295 oC temperature and -600 mm WC static pressure:
V = 183750 Χ ( 273 + 295) Χ 10333 = 405875 [m3/hr]
273 Χ (10333 − 600)

Or V = 405875/3600 = 112.75 [m3/sec]


9
_______________________
Bureau of Energy Efficiency
Click Here & Upgrade
Expanded Features
PDF Unlimited Pages
Documents
Complete Paper 4 – SET A KEY

Pressure difference across PH fan = 50 – (- 600) = 650 [mm WC]


Power consumption of PH fan
P = 112.75 Χ 650 = 1050.4 [kW]
102 Χ 0.72 Χ 0.95

The above kilns can be compared as follows:


Item 6 Stage PH Kiln 4 stage PH kiln
PH Gas heat loss (kcal/hr) 18781381 23556309
Equivalent coal wasted (tons of coal) 3.39 4.252
Power consumption in PH Gas (kW) 1050 806.24

Calculation for annual Monetary savings

Coal savings in 6 stage PH Kiln = 4.252 – 3.39 = 0.862 [ton of coal/hr]

Annual monetary savings (Thermal) = 0.862 x 8000 x 6150 = 4,24,10,400 [Rs.]

Additional Electrical energy requirement for 6 stage PH Kiln = 1050.4 – 806.24 = 244.16 [kW]

Annual additional electrical cost = 244.16 x 8000 x 5 = 97,66,400 [Rs.]

It is obvious that in monetary terms, thermal energy saving in 6 stage pre-heater kiln is higher than the additional
electrical energy cost in 4 stage kiln. Therefore, 6 stage pre-heater kiln is better option than 4 stage pre-heater
kiln.

So the net annual monetary saving in case of 6 stage pre-heater kiln is


= 4,24,10,400 – 97,66,400 = 3,26,44,000 [Rs.]

N4-C KEY

Ans a)
Before insulation
Surface heat loss, S = [ 10 + (TS-Ta)/20] x (Ts –Ta)
Total heat Loss =SxA where A= Surface area, m2
Surface heat loss , S = [ 10 + ( 110-25)/20] x (110-25) = 1211.25 K.Cal/m2/hr
Total heat loss = 1211.25 x 20 m2 = 24225 kCal/hr

After insulation
Surface heat loss ,S = [ 10 + (55-25)/20] x (55-25) =345 K.Cal/m2/hr
Total heat loss = 345 x 20 m2 = 6900 kCal/hr
Heat reduction per hour after proper insulation = 24225- 6900 = 17325 kCal/hr
Annual heat loss reduction = 17325 x 8000 = 138600000
= 138.6 million kCal/year

Steam distribution loss = 20%


Heat loss = 138.6 million kCal/ 0.8 = 173.25 million kcal/year
Boiler efficiency = 70%

Equivalent coal consumption reduction = 173.25 x106 /0.7 x 4800 = 51.56 Ton /year
10
_______________________
Bureau of Energy Efficiency
Click Here & Upgrade
Expanded Features
PDF Unlimited Pages
Documents
Complete Paper 4 – SET A KEY

Monetary Cost savings per year = 51.5 x 5000 = Rs 2.575 lacs


Investment @ Rs 1000 per M2 = 20 x 1000 = Rs 20000

Condensate recovery
Reduction in coal consumption through
heat recovered from condensate return = 2000 x 1 x (80 – 40) / 0.7 x 4800
= 23.8 kg of coal per hour
Annual coal savings = 23.8 x 8000/1000
= 190.4 ton / year
Annual savings = 23.8 x 8000 x Rs.5/kg coal
= Rs. 9.52 lacs

b)Simple payback period


Total savings from both the measures = 2.575 + 9.52 = 12.1 lakhs
Total investment = Rs. 20,000 + Rs 2 lakhs = Rs.2.2 lakhs
Simple payback period (combined) = 2.2/12.1= 2.2 months

c)GHG emission reduction


Carbon content in the coal = 40% by weight
Total Coal saving /year = 51.5 + 190.4 = 241.9 Ton per year
CO2 reduction = 241.9 x 0.4 x 44/12 = 355 Ton of CO2/year

N4-D KEY

Ans Theoretical air required for complete combustion

=[(11.6x85.9)x(34.8x(12-0.7/8))+4.35x0.5]/100
=996.44+414.12+2.175/100
=14.1 kg/kg of oil

Existing oxygen % in flue gas =6%

% excess air supplied =6 x100/(21-6) =40%


Actual mass of air supplied =(1+Excess air/100)x Theoretical air
=(1+40/100)x 14.1
=19.74 kg/kg of oil

After modification, oxygen % in flue gas =3%


% excess air supplied =3 x100/(21-3) =16.67%

Actual mass of air supplied =(1+Excess air/100)x Theoretical air


=(1+16.67/100)x 14.1
=16.45 kg/kg of oil

a) Heat loss reduction through actual mass of air supplied

11
_______________________
Bureau of Energy Efficiency
Click Here & Upgrade
Expanded Features
PDF Unlimited Pages
Documents
Complete Paper 4 – SET A KEY

Actual mass of air supplied before WHR =19.74 kg/kg of oil


Actual mass of air supplied AFTER WHR =16.45 kg/kg of oil

Existing oil consumption per hour = 25 ton/hr x 60kg/ton = 1500 kg of oil /hr

Flue gas loss before WHR = [1500 kg oil + (1500 x 19.74 kg air)] x 0.24 x (600-30)
= 4255848 kcal/hr

Flue gas loss after WHR = [1500 kg oil + (1500 x 16.45 kg air)] x 0.24 x (300-30)
= 1696140 kcal/hr

Flue gas heat loss reduction after WHR implementation = 4255848-1696140


= 2559708 kcal/hr

Reduction in fuel oil consumption after installing


Waste heat recovery and reduction in excess air = 256 kg/hr

Furnace efficiency after WHR = 25000 x 0.12 x (1200-40) x 100


[(1500-256) x10000)]
= 28 %

b) Calculate fuel oil reduction after charging hot ingot in reheating furnace

Ingot charging temperature is increased from 40 oC to 500 oC

Fuel oil reduction due to increased charge temperature =


= 25 x 1000 x 0.12 x(500-40)/0.28 x10,000
= 492.86 kg/hr = 493 kg/hr

c ) Specific oil and power consumption after implementing both the above measure

Fuel oil reduction after implementation of both measures


= 256 + 493 = 749 kg oil/hr

Fuel oil consumption after implementation of both measures


= 1500 – 749 = 751 kg oil/hr

Yield improvement = 3%
Production after implementation = 25 x 1.03 = 25.75 ton/hr
of both measures

Specific oil consumption = 751/ 25.75 = 29.2 kg/Ton

Specific power consumption = 25x90 / 25.75 = 87.37 kWh/ton

-------- End of Section - III ---------

12
_______________________
Bureau of Energy Efficiency
Click Here & Upgrade
Expanded Features
PDF Unlimited Pages
Documents
Complete

Paper -4 Set B Solutions

13th NATIONAL CERTIFICATION EXAMINATION


FOR
ENERGY AUDITORS – September, 2012

PAPER – 4: Energy Performance Assessment for Equipment and Utility


Systems

Date: 16.9.2012 Timings: 14:00-16:00 HRS Duration: 2 HRS Max. Marks: 100

Section - I: BRIEF QUESTIONS Marks: 10 x 1 = 10

(i) Answer all Ten questions


(ii) Each question carries One mark

A draft system in a boiler which uses both FD and ID fan is called…….


S-1
Balanced Draft
Ans

S-2 In a vapour compression refrigeration system, why the heat rejected in the
condenser is more than the heat absorbed in the evaporator ?

Ans Because heat of compression is also added to it


If the unit heat rate is 3191 kcal/kWh and the turbine heat rate is 2808
S-3
kCal/kWh what is the boiler efficiency ?
Ans (2808/3191) x 100 = 88 %

S-4 A rise in conductivity of boiler feed water indicates ____ .


Rise in the TDS level of feed water
Ans

S-5 Why is it preferable to measure the flow at the inlet side of the fan?

Ans Less turbulence

S-6 The critical point of steam occurs at ____bar and _______ oC

Ans 221.2 bar and 374.15oC

S-7 In a heat exchanger _______ is the ratio of actual heat transfer rate to the
maximum heat transfer rate.

___________________________
Bureau of Energy Efficiency
Click Here & Upgrade
Expanded Features
PDF Unlimited Pages
Documents
Complete

Ans Effectiveness

S-8
If the In an integrated steel plant pig iron is produced from _________furnace?

Ans Blast furnace

S-9 PLF of a 210 MW power plant is 85% , what is the annual gross generation in
MWh

Ans 1,563,660 MWH


A pump operates on water with a total head of 10 m. If water is replaced by
S-10
ethylene glycol with a specific gravity of 1.12 what will be the total head
developed by the pump ?
Ans 10 m

. End of Section - I .

Section - II: SHORT NUMERICAL QUESTIONS Marks: 2 x 5 = 10

(i) Answer all Two questions


(ii) Each question carries Five marks

L-1 Calculate pressure drop in meters when pipe diameter is increased from 250
mm to 300 mm for a length of 600 meters. Water velocity is 1.5 m/s in the 250
mm diameter pipe and friction factor is 0.005 for both the pipes.

Ans Pressure drop = 4fLV2


--------------
2gD

Velocity of water in pipe of 300 mm diameter = (0.25 x 0.25 x 1.5) /(0.3 x 0.3)
= 1.04 m/s

Pressure drop with 300 mm = 4 x 0.005 x 600 x 1.042 / (2 x 9.81 x 0.300)


= 2.2 m

L-2 A three phase 37 kW four pole induction motor operating at 49.8 Hz is rated for 415
V, 50 Hz and 1440 RPM. The actual measured speed is 1480 RPM. Find out the
percentage loading of the motor if the voltage applied is 410 V.
Ans % Loading = Slip x 100%
(Ss – Sr) x (Vr / V)2

___________________________
Bureau of Energy Efficiency
Click Here & Upgrade
Expanded Features
PDF Unlimited Pages
Documents
Complete

Synchronous speed = 120 x 49.8 / 4 = 1494 rpm

Slip = Synchronous Speed – Measured speed in rpm.


= 1494 – 1480 = 14 rpm.

% Loading = 14 x 100% = 25.8 %


(1494 - 1440) x (415/410)2

. End of Section - II .

Section - III: LONG NUMERICAL QUESTIONS Marks: 4 x 20 = 80

(i) Answer all Four questions


(ii) Each question carries Twenty marks

N-1 KEY

a) Theoretical air required for complete combustion


  O  
(11.6 x C ) + 34.8 x  H 2 −  2   + (4.35 x S )
   8  
=
100
   32.52  
(11.6 x 33.95) + 34.8 x  5.01 −    + (4.35 x 0.09)
=    8   = 4.27 kg / kg of paddy husk
100

 77 
4.27 ×  
 100   0.0091 
Moles of N2 = +  = 0.1178
28  28 

Moles of C
% CO2 theoretical =
Moles of N 2 + Moles of C + Moles of S

 0.3395 
 
 12 
=
 0.3395   0.0009 
0.1178 +   +  
 12   32 
___________________________
Bureau of Energy Efficiency
Click Here & Upgrade
Expanded Features
PDF Unlimited Pages
Documents
Complete

Max theoretical ( CO2 )t = 19.36 %

Actual CO2 measured in flue gas = 14.0%

7900 x [( CO 2 ) t − (CO 2 ) a
b) % Excess air supplied = = 37.5 %
(CO 2 ) a x [100 − ( CO 2 ) t ]

c) Actual mass of air supplied = {1 + EA/100} x theoretical air


= {1 + 37.5/100} x 4.27
= 5.87 kg/kg of coal

0 .3395 × 44 5 . 87 × 77 (5 .87 − 4 . 27 ) × 23
Mass of dry flue gas = + 0 .0091 + +
12 100 100

= 6.15 kg / kg of coal

(or)

(actual mass of air supplied + 1) – mass of H20

(5.87 + 1) – (9H + M) = 6.87 – (9x.05 + 0.1079)= 6.87 – 0.5579 = 6.31 kg/kg of coal

m x C P x ( Tf − Ta )
% Heat loss in dry flue gas = x 100
GCV of fuel
6.15 x 0 .23 x (140 − 32 )
= x 100
3568
= 4.28 %

Loss due to CO =

= 0.35 x 0.3395 x 5654


(0.35+14) x 3568

= 1.31 %

Heat Loss in ash

% heat loss due to unburnt flyash

% ash in paddy husk = 16.73


Ratio of bottom ash to flyash = 10:90
___________________________
Bureau of Energy Efficiency
Click Here & Upgrade
Expanded Features
PDF Unlimited Pages
Documents
Complete

GCV of flyash = 450 kcal/kg


Amount of flyash in 1 kg of husk = 0.9 x 0.1673
= 0.15 kg
Heat loss in flyash = 0.15 x 450
= 67.5 kcal/kg of husk
GCV of bottom ash = 800 kcal/kg

Amount of bottom ash in 1 kg of husk = 0.1 x 0.1673


= 0.01673 kg
Heat loss in bottom ash = 0.01673 x 800
= 13.4 kcal/kg of husk

Total heat loss in ash = 67.5 + 13.4


= 80.9 kcal/kg
% loss in ash = 80.9/3568
= 2.26 %

Total losses = 100 – (4.28 + 1.31+2.26) – (14.33)

Boiler efficiency = 100 – 7.85 – 14.33 = 78 %


N-2 KEY

Ans
Hot Water use per day : 20,000 L/day

Water in = 200C
Water out = 600C
Temp. diff. = 400C
Total Heat required = mCpdt
= 20000 x 1 x 40 = 8,00,000 kcal/day

a) Energy Requirement for 20KL/day of water for a temperature differential of 40


deg.C in an Electric Boiler/Geyser

Energy Requirement (for 20 KL/day)


= Total heat required (800000)
860 kcal/kWh x 0.99 ( efficiency of electric heating))
= 939.6 kWh/day

b) For 20 KL/day, of water flow with 400C Temperature Diff. Energy to be drawn by
Heat Pump
= 8,00,000 = 391.68 Kwh/day
860x0.95x2.5

___________________________
Bureau of Energy Efficiency
Click Here & Upgrade
Expanded Features
PDF Unlimited Pages
Documents
Complete

Energy drawn by circulation pump = 2.8 x 24 hr = 67.2 kWh/day


Energy drawn by evaporator fan = 1.4 kW x 16 hr = 22.4 kWh/day

Total Energy drawn by heat pump system = 391.68 +67.2+22.4


= 481.28 kWh /day

SAVINGS IN COMPARISON TO ELECTRIC WATER HEATER

= 939.6 – 481.28 = 458.32 Kwh/day


= 1,60,412 kWh/year ( @ 350 days/year)
= 12.83 lakhs ( @ Rs8.0 per kWh)

SIMPLE PAY BACK PERIOD = Rs.16.0 LAKHS Investment/ Rs.12.83 lakhs per
year savings
= 1.25 years or 15 months

N-3 KEY

Ans Power generation from cogen plant = 5000X 0.9 X 8000 = 360 lac Kwh/yr
Auxiliary power = 1%
Net power generation = 0.99 X 360 = 356.4 lac Kwh

Natural gas requirement for = 360 X 3050 / 9500 = 115.57 lac sm3
power generation

Cost of fuel per annum = 115.57 X 8 = Rs.924.56 lacs

Annual expenditure for interest, = 500 + 200 = 700 lacs


depreciation and O&M

Total cost of generation = Rs.1624.56 lacs.

Cost of cogeneration power = 1624.56 X 105 / 356.4 X 105

= Rs.4.56 / Kwh.

Gas consumption in existing gas = [10000 (665 – 85) / (0.82 X 9500)]


fired boiler = 744.6 Sm3/hr
= 744.6 x 24 = 17870 sm3/day

___________________________
Bureau of Energy Efficiency
Click Here & Upgrade
Expanded Features
PDF Unlimited Pages
Documents
Complete

Cost of steam from existing boiler = 744.6*Rs. 8 x8000


= Rs. 476.5 Lacs /yr

Cost of power generation after giving = 1624.56 – 476.5 = Rs.1148.06 lacs


credit for steam generation

Cost of power generation after accounting = 1148.06 X 105 / 356.4 X 105


for steam cost
= Rs. 3.22 / kWh

Grid power cost = Rs. 4.5 / Kwh


Cost advantage for cogen plant = 4.5 – 3.22 = Rs.1.28 / Kwh
generation

Daily gas requirement for operating = 5000 X 0.9 X 3050 X 24


GT cogen plant 9500
= 34673.68 Sm3 / day

Additional gas requirement for = 34673.68 – 17870 = 16803.68 Sm3/day


co-gen plant

N-4 To attempt ANY ONE OF THE FOLLOWING among A, B, C and D

N4A KEY

Ans i) Turbine power output kW =

Steam flow to turbine kg/hr x enthalpy drop across the turbine kcal/kg
-------------------------------------------------------------------------------------------------------
860

Inlet enthalpy of steam =794.4 kcal/kg

Enthalpy of exhaust steam is calculated as given below


exhaust steam dryness fraction = 90%
enthalpy of exhaust steam = (45.9 + 0.9 x 572.5) = 561 kcal/kg
turbine output = ((120 x 1000 kg/hr x (794.4 – 561) kcal/kg) /860
turbine output = 32567.4 kW

ii) generator output kW = turbine output x combined efficiency of mechanical, gear


transmission & generator
= 32567.4 x 0.92
=29962 kW

iii) turbine heat rate = heat input in to the turbine/ generator out put
=q x (h1 – hw)/generator out put

___________________________
Bureau of Energy Efficiency
Click Here & Upgrade
Expanded Features
PDF Unlimited Pages
Documents
Complete

Where q = steam inflow to turbine kg/hr


h1= enthalpy of turbine inlet steam =794.4 kcal/kg
hw= enthalpy of feed water to boiler = 100 kcal/kg

Turbine heat rate = ((120 x 1000 kg/hr) x (794.4 – 100) kcal/kg))/ 29962 kw
= 2781 kcal/kwh

iv) unit heat rate = turbine heat rate /boiler efficiency


= 2781 / 0.88 = 3160 kcal/ kwh

3 Marks

v) turbine cycle efficiency = (860 / turbine heat rate) x 100


= 860 /2781 =0.309
=0.309 x 100 = 30.9%

vi) condenser heat load = m x cp x dt

Where m = cooling water flow through condenser, kg/hr

note: density of water is given as 0.95 g /cubic centimetre = 950 kg/ cubic meter
cp = specific heat of cooling water, kcal/ kg. oC = 0.98 kcal /kg. oC
dt = cooling water temperature rise, oC = 10

Condenser heat load =6318 x 950x 0.98 x 10 = 5,88,20,580 kcal /hr

vii) specific steam consumption of turbine = 860 / (enthalpy drop x combined efficiency)
= 860/ ((794.4 – 561) x 0.92))
=860 / (233.4 x 0.92) =4.0 kg/kwh
= 4.0 kg / kwh

N4B KEY
Ans
Volumetric flow rate of PH gas at NTP = 1.47 x 125 x 1000 = 183750 [Nm3/hr]

Mass flow rate of PH gas = 183750 x 1.42 = 260925 [kg/hr]

Calculation for 4 stage pre-heater kiln

Heat loss in PH Gas = m x cp x T [kcal/hr]

= 260925 x 0.244 x 370 = 23556309 [kcal/hr]

Equivalent coal wasted = 23556309 = 4.252 [tons of coal/hr]


5540 Χ 1000

Electrical Energy consumption of PH Fan

___________________________
Bureau of Energy Efficiency
Click Here & Upgrade
Expanded Features
PDF Unlimited Pages
Documents
Complete

Volumetric flow rate of PH Gas at 370oC temperature and -400 mm WC static pressure:

V = 183750 Χ (273 + 370 ) Χ 10333 = 450216 [m3/hr]


273 Χ (10333 − 400 )

or V = 450216/3600 = 125 [m3/sec]

Pressure difference across PH fan = 50 – (- 400) = 450 [mm WC]

Power consumption of PH fan


125 Χ 450 = 806.24 [kW]
P=
102 Χ 0.72 Χ 0 .95

Calculation for 6 stage pre-heater kiln

Heat loss in PH Gas = m x cp x T [kcal/hr]


= 260925 x 0.244 x 295 = 18781381 [kcal/hr]

Equivalent coal wasted = 18781381 = 3.39 [tons of coal/hr]


5540 Χ 1000

Electrical Energy consumption of PH Fan

Volumetric flow rate of PH Gas at 295 oC temperature and -600 mm WC static pressure:
V = 183750 Χ ( 273 + 295) Χ 10333 = 405875 [m3/hr]
273 Χ (10333 − 600)
Or V = 405875/3600 = 112.75 [m3/sec]

Pressure difference across PH fan = 50 – (- 600) = 650 [mm WC]

Power consumption of PH fan


P = 112.75 Χ 650 = 1050.4 [kW]
102 Χ 0.72 Χ 0.95

The above kilns can be compared as follows:


Item 6 Stage PH 4 stage PH kiln
Kiln
PH Gas heat loss (kcal/hr) 18781381 23556309
Equivalent coal wasted (tons of coal) 3.39 4.252
Power consumption in PH Gas (kW) 1050 806.24

Calculation for annual Monetary savings


Coal savings in 6 stage PH Kiln = 4.252 – 3.39 = 0.862 [ton of coal/hr]

Annual monetary savings (Thermal) = 0.862 x 8000 x 6150 = 4,24,10,400 [Rs.]

Additional Electrical energy requirement for 6 stage PH Kiln = 1050.4 – 806.24 = 244.16 [kW]

Annual additional electrical cost = 244.16 x 8000 x 5 = 97,66,400 [Rs.]

It is obvious that in monitory terms, thermal energy saving in 6 stage pre-heater kiln is higher than the additional
electrical energy cost in 4 stage kiln. Therefore, 6 stage pre-heater kiln is better option than 4 stage pre-heater

___________________________
Bureau of Energy Efficiency
Click Here & Upgrade
Expanded Features
PDF Unlimited Pages
Documents
Complete

kiln.

So the net annual monetary saving in case of 6 stage pre-heater kiln is


= 4,24,10,400 – 97,66,400 = 3,26,44,000 [Rs.]

N4C KEY

Ans a)
Before insulation
Surface heat loss, S = [ 10 + (TS-Ta)/20] x (Ts –Ta)
Total heat Loss =SxA where A= Surface area, m2
Surface heat loss , S = [ 10 + ( 110-25)/20] x (110-25) = 1211.25 K.Cal/m2/hr
Total heat loss = 1211.25 x 20 m2 = 24225 kCal/hr

After insulation
Surface heat loss ,S = [ 10 + (55-25)/20] x (55-25) =345 K.Cal/m2/hr
Total heat loss = 345 x 20 m2 = 6900 kCal/hr
Heat reduction per hour after proper insulation = 24225- 6900 = 17325 kCal/hr
Annual heat loss reduction = 17325 x 8000 = 138600000
= 138.6 million kCal/year

Steam distribution loss = 20%


Heat loss = 138.6 million kCal/ 0.8 = 173.25 million kcal/year

Boiler efficiency = 70%


Equivalent coal consumption reduction = 173.25 x106 /0.7 x 4800 = 51.56 Ton /year
Monetary Cost savings per year = 51.5 x 5000 = Rs 2.575 lacs

Investment @ Rs 1000 per M2 = 20 x 1000 = Rs 20000

Condensate recovery
Reduction in coal consumption through
heat recovered from condensate return = 2000 x 1 x (80 – 40) / 0.7 x 4800
= 23.8 kg of coal per hour
Annual coal savings = 23.8 x 8000/1000
= 190.4 ton / year
Annual savings = 23.8 x 8000 x Rs.5/kg coal
= Rs. 9.52 lacs

b)Simple payback period


Total savings from both the measures = 2.575 + 9.52 = 12.1 lakhs
Total investment = Rs. 20,000 + Rs 2 lakhs = Rs.2.2 lakhs
Simple payback period (combined) = 2.2/12.1= 2.2 months

c)GHG emission reduction


Carbon content in the coal = 40% by weight
___________________________
Bureau of Energy Efficiency
Click Here & Upgrade
Expanded Features
PDF Unlimited Pages
Documents
Complete

Total Coal saving /year = 51.5 + 190.4 = 241.9 Ton per year
CO2 reduction = 241.9 x 0.4 x 44/12 = 355 Ton of CO2/year

N4D KEY

Ans Theoretical air required for complete combustion

=[(11.6x85.9)x(34.8x(12-0.7/8))+4.35x0.5]/100
=996.44+414.12+2.175/100
=14.1 kg/kg of oil

Existing oxygen % in flue gas =6%

% excess air supplied =6 x100/(21-6) =40%


Actual mass of air supplied =(1+Excess air/100)x Theoretical air
=(1+40/100)x 14.1
=19.74 kg/kg of oil

After modification, oxygen % in flue gas =3%


% excess air supplied =3 x100/(21-3) =16.67%

Actual mass of air supplied =(1+Excess air/100)x Theoretical air


=(1+16.67/100)x 14.1
=16.45 kg/kg of oil

a) Heat loss reduction through actual mass of air supplied

Actual mass of air supplied before WHR =19.74 kg/kg of oil


Actual mass of air supplied AFTER WHR =16.45 kg/kg of oil

Existing oil consumption per hour = 25 ton/hr x 60kg/ton = 1500 kg of oil /hr

Flue gas loss before WHR = [1500 kg oil + (1500 x 19.74 kg air)] x 0.24 x (600-30)
= 4255848 kcal/hr

Flue gas loss after WHR = [1500 kg oil + (1500 x 16.45 kg air)] x 0.24 x (300-30)
= 1696140 kcal/hr

Flue gas heat loss reduction after WHR implementation = 4255848-1696140


= 2559708 kcal/hr

Reduction in fuel oil consumption after installing


Waste heat recovery and reduction in excess air = 256 kg/hr

Furnace efficiency after WHR = 25000 x 0.12 x (1200-40) x 100


___________________________
Bureau of Energy Efficiency
Click Here & Upgrade
Expanded Features
PDF Unlimited Pages
Documents
Complete

[(1500-256) x10000)]

= 28 %

b) Calculate fuel oil reduction after charging hot ingot in reheating furnace

Ingot charging temperature is increased from 40 oC to 500 oC

Fuel oil reduction due to increased charge temperature =


= 25 x 1000 x 0.12 x(500-40)/0.28 x10,000
= 492.86 kg/hr = 493 kg/hr

c ) Specific oil and power consumption after implementing both the above measure

Fuel oil reduction after implementation of both measures


= 256 + 493 = 749 kg oil/hr

Fuel oil consumption after implementation of both measures


= 1500 – 749 = 751 kg oil/hr

Yield improvement = 3%
Production after implementation = 25 x 1.03 = 25.75 ton/hr
of both measures

Specific oil consumption = 751/ 25.75 = 29.2 kg/Ton

Specific power consumption = 25x90 / 25.75 = 87.37 kWh/ton

-------- End of Section - III ---------

___________________________
Bureau of Energy Efficiency
Paper 4 – Energy Auditor – Set A Key

12th NATIONAL CERTIFICATION EXAMINATION


FOR
ENERGY AUDITORS – October, 2011

PAPER – 4: Energy Performance Assessment for Equipment and Utility Systems

Date: 16.10.2011 Timings: 14:00-16:00 HRS Duration: 2 HRS Max. Marks: 100

General instructions:
o Please check that this question paper contains 12 printed pages
o Please check that this question paper contains 16 questions
o The question paper is divided into three sections
o All questions in all three sections are compulsory
o All parts of a question should be answered at one place

Section - I: BRIEF QUESTIONS Marks: 10 x 1 = 10

(i) Answer all Ten questions


(ii) Each question carries One mark

S-1 When you do a walk through energy audit of a cooling tower, which two
parameters will you quickly spot check for indication of the cooling tower
performance?

Ans Cold well temperature and wet bulb temperature


(Range and approach may also be considered)

S-2 Which component of a cooling tower enhances heat transfer by maximizing


water and air contact?

Ans Fill
S-3 What is the index used to express the harmonics level in an electrical system?

Ans Total Harmonic Distortion (THD)


S-4 The EER of an air conditioner as indicated in BEE Star labeling scheme is
represented in_________

Ans Watts/Watts
S-5 For the determination of which boiler losses by indirect method, the specific
heat of superheated water vapour is used?

Ans Loss due to moisture in air, loss due to hydrogen in fuel and loss due to moisture in

1
_______________________
Bureau of Energy Efficiency
Paper 4 – Energy Auditor – Set A Key

fuel
(Full marks may be given for any two right answers)
S-6 While reducing excess air in a boiler, what two parameters should be closely
monitored in the exit flue gases?
Ans CO and O2
S-7 Between back pressure turbine and condensing turbine which will have more
power generation efficiency?
Ans Condensing turbine

S-8 Calculate the boiler efficiency where the Turbine heat rate is 1930 kCal/ kWh
and the generating unit heat rate is 2250 kCal/kWh
Ans Boiler Efficiency=( Turbine Heat Rate*100)/Unit Heat Rate= 85.7%
S-9 In a power plant boiler, if there is air ingress in the flue gas duct, which auxiliary
equipment would be most affected?

Ans Induced Draft Fan


S-10 The loading and unloading of a reciprocating compressor is carried out based
on______

Ans Pressure

( Also full marks may be given If a candidate writes ‘Air Demand’)

…………. End of Section - I ………….

Section - II: SHORT NUMERICAL QUESTIONS Marks: 2 x 5 = 10

(i) Answer all Two questions


(ii) Each question carries Five marks

L-1 An open cycle gas turbine was running with naphtha as fuel. The following are
the data collected during the gas turbine operation:

Fuel (Naphtha) consumption :180 kg/hr


GCV of naphtha fuel :11500 kCal/kg
Overall Efficiency of gas turbine which includes
air compressor and alternator : 25%
Cost of naphtha fuel : Rs.40,000/Ton

Find out the cost of fuel for generating one unit of electricity.

Ans Heat input to turbine = 11500 x 180


= 2070000 kCal/hr
1 Mark
2
_______________________
Bureau of Energy Efficiency
Paper 4 – Energy Auditor – Set A Key

Efficiency of gas turbine = 25%


Gas turbine output = 2070000 x 0.25/ 860
= 601.7 kWh
2 Marks

Cost of generating 601.7 units of electricity = 180 kg x Rs.40


= Rs.7200/hr
Cost of One unit of Electricity generation = 7200/601.7
= Rs.12/kWh
2 Marks
L-2 In an air conditioning duct 0.5 m x 0.5 m, the average velocity of air measured
by vane anemometer is 28 m/s. The static pressure at suction of the fan is -20
mmWC and at the discharge is 30 mmWC. The three phase induction motor
draws 10.8 A at 415 V with a power factor of 0.9. Find out the efficiency of the
fan if motor efficiency = 90% (Neglect air density correction).

Ans Volume flow rate of the fan, Q = Velocity x Area


= 28 x (0.5 x 0.5)
= 7 m3/s
1 Mark
Power input to the fan shaft = Motor input power x motor efficiency
= (Sqrt 3 x 0.415 x 10.8 x 0.9) x 0.9
= 6.3 kW
1 Mark
Fan efficiency = Volume in m3 / Sec x total pressure in mmwc
102 x Power input to the shaft in (kW)
1 Mark for writing the formula
= 7 x (30-(-20)) x 100
102 x 6.3

Fan efficiency = 54.5%


2 Marks

…………. End of Section - II ………….

Section - III: LONG NUMERICAL QUESTIONS Marks: 4 x 20 = 80

(i) Answer all Four questions


(ii) Each question carries Twenty marks

N-1 Calculate the efficiency of the Atmospheric Fluidised Bed Combustion Boiler
by indirect method using the following data:

Analysis of blended coal (% by mass)

Carbon : 53.9 %
Hydrogen : 3.1 %
Nitrogen : 1.1 %
Sulphur : 0.3 %
3
_______________________
Bureau of Energy Efficiency
Paper 4 – Energy Auditor – Set A Key

Ash : 23.8 %
Oxygen : 10.5 %
Moisture : 7.3 %
GCV : 5060 kCal / kg

The boiler operating parameters are given below.

Steam pressure : 62.0 kg / cm2g


Steam temperature : 470 oC
Actual air supplied : 8.91 kg/kg of coal
Mass of dry flue gas : 9.31 kg/kg of coal
Specific heat of flue gas : 0.23 kCal/kg oC
Flue gas temperature : 160 oC
CO2 in flue gas : 14.7 %
CO in flue gas : 325 ppm

GCV of bottom ash : 800 kCal/kg


GCV of fly ash : 452.5 kCal/kg

Ratio of bottom ash to fly ash : 15 : 85

Ambient temperature : 32.4 oC

Loss due to hydrogen in fuel : 3.54 %


Loss due to moisture in fuel : 0.93 %
Loss due to moisture in air : 0.2 %
Surface heat losses : 2%
(as assessed)

Ans To find all losses


1. % Heat loss in dry flue gas (L1) = L1

= m x C P x ( Tf − Ta )
x 100
GCV of fuel
= 9.31 x 0.23 x (160 − 32.4)
x 100
5060
= 5.40 %
4 Marks
2. % Heat loss due to partial conversion of C to CO (L2)

= %CO x C x 5654
x 100
% CO + (%CO2 )a  x GCVof fuel

0.0325 x 0.539 x 5654


= x 100
0.0325+14.7 x 5060
=0.13 %
4 Marks
3. % Heat loss due to unburnt in fly ash

4
_______________________
Bureau of Energy Efficiency
Paper 4 – Energy Auditor – Set A Key

% Ash in coal = 23.8


Ratio of bottom ash to fly ash = 15:85
GCV of fly ash = 452.5 Kcal/kg
Amount of fly ash in 1 kg of coal = 0.85 x 0.238 = 0.2023 kg
Heat loss in fly ash = 0.2023 x 452.5= 91.54 kCal / kg of coal
% heat loss in fly ash L3 = 91.54 x 100 / 5060 = 1.81 %
4 Marks
4. % Heat loss due to unburnt in bottom ash

GCV of bottom ash = 800 Kcal/kg


Amount of bottom ash
in 1 kg of coal = 0.15 x 0.238 = 0.0357 kg
Heat loss in bottom ash = 0.0357 x 800 = 28.56 kCal/kg of coal
% Heat loss in bottom ash L4 = 28.56 x 100 / 5060 = 0.56 %
4 Marks
5. Loss due to hydrogen in fuel L5 = 3.54% (given)
6. Loss due to moisture in fuel L6 = 0.93% (given)
7. Loss due to moisture in air L7 = 0.2% (given)
8. Surface heat losses L8 = 2% (given)

Boiler efficiency by indirect method = 100 – (L1+ L2+ L3+ L4+ L5+ L6+ L7+ L8)
= 100-(5.40 + 0.13+ 1.81+ 0.56+ 3.54+0.93+0.2+2)
= 100-14.57 = 85.43 %
4 Marks
N-2 a) For a VFD retrofit in a compressed air system with an initial investment of
Rs.2.55 lakhs the annual savings are Rs.58,000/-

The NPV of the project over a six year period for 8% discount rate is
Rs.13,134.
The NPV of the project over a six year period for 10% discount rate is Rs.(-)
2468

Calculate the exact discount rate for NPV to be zero.

b) A counter current Heat Exchanger with total heat transfer area of 52 m2 is


used to exchange heat between a hot effluent stream of specific heat 0.33
kCal/kgoC and cooling water stream.

The monitored parameters are given below:

Parameters Unit Inlet Outlet


Hot fluid flow kg/hr 86532 86532
o
Hot fluid temperature C 84 57
o
Cold fluid temperature C 45 54

Assuming LMTD correction factor of 0.89 for plate heat exchanger, calculate

i) The corrected LMTD


ii) Overall heat transfer coefficient, U (kCal/hr.m2.oC)
Ans a)

5
_______________________
Bureau of Energy Efficiency
Paper 4 – Energy Auditor – Set A Key

The positive and negative NPV's which are closest to zero discount rates are Rs
(+)13,134 corresponding to 8% discount rate and Rs (-) 2,468 corresponding to10%
discount rate .

Hence the exact discount rate = (0.08+(0.1-0.08)*(13134)/(13134-(-2468))*100


= 9.68 %
6 Marks
b)
i)
LMTD, Counter flow = {(84-54) – (57-45)}/ {ln (84-54) / (57-45)}
= 19.64 0 C
8 Marks
Correction Factor, F = 0.89 (given)

Corrected LMTD = F x LMTD


= 0.89 x 19.64 = 17.48 0 C
2 Marks
ii)
Overall heat transfer coefficient, U = Q / (A x Corrected LMTD)
U = 771000 / (52 x 17.48) = 848 kCal/hr.m2.0C
4 Marks
N-3 a) A centrifugal water pump operates at 30 m3/hr and at 1440 RPM. The pump
operating efficiency is 65% and motor efficiency is 89%. The discharge
pressure gauge shows 3.4 kg/cm2. The suction is 3 m below the pump
centerline. If the speed of the pump is reduced by 25 %, estimate the
following:

i) pump flow,
ii) pump head and
iii) motor power.

Assume motor and pump efficiency remains same at the reduced speed.

b) In a 75 kW four pole induction motor operating at 49.8 Hz and rated for 415
V and 1440 RPM, the actual measured speed is 1470 RPM. Find out the
percentage loading of the motor if the voltage applied is 428 V.

Ans a)
Flow = 30 m3/hr
Head developed by the pump = 34 – (-3) = 37 m
Power drawn by the pump = (30/3600) x 37 x 1000 x 9.81/(1000 x 0.65)
= 4.65 kW
3 Marks
i)
Flow at 75 % speed = 30 / Q2 = 1440/1080
= 22.5 m3/hr
4 Marks
ii)
Head at 75 % speed = 37 / H2 = (1440/1080)2

6
_______________________
Bureau of Energy Efficiency
Paper 4 – Energy Auditor – Set A Key

= 20.81 m
4 Marks

iii)
Shaft Power at 75 % speed = 4.65/kW2 = (1440)3 / (1080 ) 3
= 1.96 kW
Power drawn by motor = 1.96 / 0.89
= 2.2 kW
4 Marks
b)
% Loading = Slip x 100%
(Ss – Sr) x (Vr / V)2

1 Mark
Synchronous speed = 120 x 49.8 / 4 = 1494 rpm

Slip = Synchronous Speed – Measured speed in rpm.


= 1494 – 1470 = 24 rpm.
2 Marks
% Loading = 24 x 100% = 47.27%
(1494 - 1440) x (415/428)2
2 Marks
N-4 Answer ANY ONE OF THE FOLLOWING among A, B, C and D

A) A cement kiln exhaust gas has the following composition on dry basis :
CO2 – 24.7%, O2 – 5.1%, CO - 0.1%, N2 – 70.1%. The static pressure and
temperature measured in the duct are -710 mmWC and 3000C respectively. The
velocity pressure measured with a pitot tube is 20.5 mmWC. The atmospheric
pressure at the site is 10350 mmWC.

Determine the volumetric flow rate of exhaust gas flowing through a duct of
3200 mm diameter ( Pitot tube constant = 0.89 )

Ans Molecular weight exhaust gas (dry basis) M


= %CO2xMCO2 + %O2xMO2 + %COxMCO + %N2 x MN2
= {(24.7 x 44) + (5.1 x 32) + (0.1 x 28) + (70.1 x 28)}/100
= 32.16 kg/kg mole
5 Marks
Exhaust Gas density at operating temperature= γ = [ PM / RT ]
= [ (10350 – 710) x 32.16 ) / { 847.84 x (273+300) }
= 0.638 kg/m3
5 Marks
Duct Area= 3.14 x( 3.2/2) 2= 8.05 m2
2 Marks
Volume flow rate
=A Cp (2 x g x ∆P / γ)1/2 = 8.05 x 0.89 (2 x 9.81x 20.5/0.638)1/2
= 179.89 m3/s
Volume flow rate = 6 47 596 m3/ h
8 Marks
or

7
_______________________
Bureau of Energy Efficiency
Paper 4 – Energy Auditor – Set A Key

B)
(i) If the heat rate of a power plant is 2900 kCal/kWh, what is its efficiency?
(ii) What is the condenser vacuum in millibar, if the condenser back pressure
is 0.89 kg/cm2?
(iii) Explain how Terminal Temperature Difference (TTD ) can also be negative
(iv) Calculate the % auxiliary power consumption for a Thermal power station
if Gross Heat Rate is 3200 kCal/kWh and Net Heat rate is 3500 kCal/kWh.

Ans (i) The efficiency of a power plant is inverse of heat rate


Efficiency = ((1*860)/2900)*100
= 29.7%
5 Marks
(ii) Condenser vaccum, kg/cm (a) = Atmospheric pressure – Condenser back
2

pressure
5 Marks
Condenser Vacuum = 1 – 0.89
Calculated condenser vacuum = 0.11 kg/cm2 (a) = 110 millibar

(iii) In some of the cases, because of the de-superheating zone in the feed water
heater, the feed water temperature leaving the heater may be higher than the
saturation temperature of the condensing zone. Therefore, the heater may have a
negative TTD
5 Marks
(iv) Auxiliary Power(%)= (1-(Gross Heat Rate/Net Heat rate))*100
= (1-(3200/3500))*100
= 8.5%
5 Marks
or
C) In an oil fired furnace following are the operating parameters:

Capacity of furnace - 10 T/hr


Daily production operating at 10 hours a day - 100 T/day
Specific fuel consumption - 65 litres /T of finished product
Flue gas temperature at the exit of furnace - 600 oC
Ambient temperature - 30 oC
G.C.V of oil - 10,000 kCal/kg
Theoretical air required for combustion - 14 kg of air/ kg of fuel
Specific heat of flue gas - 0.26 kCal/kgoC
Specific heat of air - 0.24 kCal/kgoC
Oxygen in flue gas - 8%

The management is planning to install a recuperator to preheat the


combustion air upto 200oC

Yield without the recuperator - 90%


Yield after installing the recuperator - 95%

Calculate

8
_______________________
Bureau of Energy Efficiency
Paper 4 – Energy Auditor – Set A Key

(i) the percentage heat reduction in flue gas after installation of recuperator
(ii) the increase in daily production due to yield improvement
(iii) specific fuel consumption after installing the heat recovery recuperator
(assuming 1 % fuel saving for every 20oC rise in combustion air
temperature)
Ans (i)
% excess air supplied = 8/ (21-8)
= 61.5 %
2 Marks
Actual mass of air supplied = [1 + (EA/100)] x theoretical air
= [1 + (61.5/100)] x 14
=22.61 kg of air/kg of fuel
2 Marks
Daily fuel consumption =65 x 100 = 6500 kg/day

( In the question the Sp. Gravity of fuel is not given. If the candidate has calculated as
above ie 65 litres full marks may be given.

If a candidate assumes a sp.gravity and multiplies 65 x assumed sp.gravity again full


marks may be given.)

Heat in flue gas = {6500 + (6500 x 22.61)} x 0.26 x (600-30)


= 22743513 kCal/day

Heat in preheated combustion air = 6500 x 22.61 x 0.24 x (200 - 30)


= 5996172 kCal/day

Percentage heat reduction in flue gas =(5996172 / 22743513)


= 26.4 %
5 Marks
(ii)

Daily additional production due to yield improvement


= 100 x 95/90
= 105.5 Tonnes/day

Additional production = 105.5 – 100


= 5.5 T/day
5 Marks
(iii)
Reduction in fuel consumption = (200 - 30)/20 = 8.5 %
Fuel consumption after waste heat recovery = 6500 – (6500 x 0.085)
= 5947.5
Specific fuel consumption

Daily oil consumption = 5947.5 kg/day


Production = 105.5 T/day
Specific fuel consumption = 5947.5/ 105.5
= 56.37 kg/tone
6 Marks

9
_______________________
Bureau of Energy Efficiency
Paper 4 – Energy Auditor – Set A Key

or
D) Determine the cooling load of a commercial building for the following given
data:

Outdoor conditions : DBT = 35C, WBT = 25C, Humidity = 18 g of water / kg


of dry air

Desired indoor conditions : DBT = 25.6C, RH = 50 %, Humidity = 10 g of water


/ kg of dry air

Total area of wall = 40 m2, Total area of window = 20 m2

U – Factor ( Wall ) = 0.33 W / m2K

U – Factor ( Roof ) = 0.323 W / m2K

U – factor [ fixed windows with aluminium frames and a thermal break ] = 3.56
W / m2K

Other data:

• 15 m x 25 m roof constructed of 100 mm concrete with 90 mm


insulation & steel decking.
• CLTD at 17:00 hr : Details : Wall = 12C; Roof = 44C; Glass Window =
7C
• SCL at 17 : 00 hr : Details : Glass Window = 605 W/ m2
• Shading coefficient of Window = 0.74
• Space is occupied from 8:00 to 17:00 hr by 25 people doing moderately
active work.
• Sensible heat gain / person = 75 W ; Latent heat gain / person = 55 W ;
CLF for people = 0.9
• Fluorescent light in space = 21.5 W/m2 ; CLF for lighting = 0.9
• Ballast factor details = 1.2 for fluorescent lights & 1.0 for incandescent
lights
• Computers and office equipment in space produces 5.4 W/m2 of
sensible heat
• One coffee maker produces 1050 W of sensible heat and 450 W of latent
heat.
• Air changes / hr of infiltration = 0.3
• Height of building = 3.6 m

10
_______________________
Bureau of Energy Efficiency
Paper 4 – Energy Auditor – Set A Key

Ans
I External Heat Gain

(i) Conduction heat gain through the wall = U – factor x net area of wall x CLTD
= 0.33 x (40-20) x 12 ] = 79.2 W
2 Marks
(ii) Conduction heat gain through the roof = U – factor x net area of roof x CLTD
= 0.323 x ( 15 x 25 ) x 44
= 5 329.5 W
2 Marks
(iii) Conduction heat gain through the windows = U – factor x net area of windows x
CLTD
= (3.56 x 20 x 7) = 498.4 W
2 Marks
(iv) Solar radiation through glass
= Surface area x Shading coefficient x SCL
= (20 x 0.74 x 605) = 8954 W
2 Marks
II Internal Heat Gain

(i) Heat gain from people = Sensible heat gain + Latent heat gain

Sensible heat gain = (No. of people x Sensible heat gain / person x CLF)
= (25 x 75 x 0.9) = 1687.5 W

Latent heat gain = No. of people x Latent heat gain / person


= (25 x 55 ) = 1375 W
Therefore, Heat gain from people = (1687.5 + 1375 ) = 3062.5 W

(ii) Heat gain from lighting = ( Energy input x Ballast factor x CLF )
Energy input = ( Amount of lighting in space / unit area ) x Floor area
= 21.5 x ( 15 x 25 ) = 8062.5 W

Therefore, heat gain from lighting = ( 8062.5 x 1.2 x 0.9 ) = 8707.5 W

3 Marks
(iii) Heat generated by equipment :

Sensible heat generated by coffee maker = 1050 W


Latent heat generated by coffee maker = 450 W
Sensible heat gain by computers and office equipment = 5.4 x 375 = 2025 W
Therefore, Heat generated by equipment = 3525 W
2 Marks
(iv)Heat gain through air infiltration = ( Sensible heat gain + Latent heat gain )

Sensible heat gain = (1210 x airflow x ∆T )


Airflow =( Volume of space x air change rate ) / 3600
= { (15 x 25 x 3.6 ) x 0.3 } / 3600
= 0.1125 m3 / s

Therefore, sensible heat gain= 1210 x 0.1125 x ( 35 – 25.6 ) =1279.58 W


11
_______________________
Bureau of Energy Efficiency
Paper 4 – Energy Auditor – Set A Key

Latent heat gain = 3010 x 0.1125 x ( 18 – 10 ) = 2709 W

3 Marks

No Space Load Components Sensible Heat Load (W) Latent Heat Load (W)
1 Conduction through exterior wall 158.4 ------
2 Conduction through roof 5 329.5 ------
3 Conduction through windows 498.4 ------
4 Solar radiation through windows 8954 ------
5 Heat gained from people 1 687.5 1 375
6 Heat gained from lighting 8 707.5 ------
7 Heat gained from equipment 3 075 450
8 Heat gained by air infiltration 1 279.58 2 709
Total space cooling load 29 689.88 4 534

Total Cooling Load = 34,223.88 W


4 Marks

-------- End of Section - III ---------

12
_______________________
Bureau of Energy Efficiency
Paper 4 – Energy Auditor – Set B Key

12th NATIONAL CERTIFICATION EXAMINATION


FOR
ENERGY AUDITORS – October, 2011

PAPER – 4: Energy Performance Assessment for Equipment and Utility Systems

Date: 16.10.2011 Timings: 14:00-16:00 HRS Duration: 2 HRS Max. Marks: 100

General instructions:
o Please check that this question paper contains 12 printed pages
o Please check that this question paper contains 16 questions
o The question paper is divided into three sections
o All questions in all three sections are compulsory
o All parts of a question should be answered at one place

Section - I: BRIEF QUESTIONS Marks: 10 x 1 = 10

(i) Answer all Ten questions


(ii) Each question carries One mark

S-1 The loading and unloading of a reciprocating compressor is carried out based
on______

Ans
Pressure
( Also full marks may be given If a candidate writes ‘Air Demand’)

S-2 In a power plant boiler, if there is air ingress in the flue gas duct, which auxiliary
equipment would be most affected?

Ans
Induced Draft Fan

S-3 Calculate the boiler efficiency where the Turbine heat rate is 1930 kCal/ kWh
and the generating unit heat rate is 2250 kCal/kWh
Ans
Boiler Efficiency=( Turbine Heat Rate*100)/Unit Heat Rate= 85.7%

S-4 Between back pressure turbine and condensing turbine which will have more
power generation efficiency?
Ans
Condensing turbine
1
_______________________
Bureau of Energy Efficiency
Paper 4 – Energy Auditor – Set B Key

S-5 While reducing excess air in a boiler, what two parameters should be closely
monitored in the exit flue gases?
Ans
CO and O2

S-6 For the determination of which boiler losses by indirect method, the specific
heat of superheated water vapour is used?

Ans
Loss due to moisture in air, loss due to hydrogen in fuel and loss due to moisture in
fuel
(Full marks may be given for any two right answers)

S-7 The EER of an air conditioner as indicated in BEE Star labeling scheme is
represented in_________

Ans
Watts/Watts

S-8 What is the index used to express the harmonics level in an electrical system?

Ans
Total Harmonic Distortion (THD)

S-9 Which component of a cooling tower enhances heat transfer by maximizing


water and air contact?

Ans
Fill

S-10 When you do a walk through energy audit of a cooling tower, which two
parameters will you quickly spot check for indication of the cooling tower
performance?

Ans
Cold well temperature and wet bulb temperature
(Range and approach may also be considered)

…………. End of Section - I ………….

Section - II: SHORT NUMERICAL QUESTIONS Marks: 2 x 5 = 10

(i) Answer all Two questions


(ii) Each question carries Five marks

2
_______________________
Bureau of Energy Efficiency
Paper 4 – Energy Auditor – Set B Key

L-1 An open cycle gas turbine was running with naphtha as fuel. The following are
the data collected during the gas turbine operation:

Fuel (Naphtha) consumption : 180 kg/hr


GCV of naphtha fuel : 11500 kCal/kg
Overall Efficiency of gas turbine which includes
air compressor and alternator : 28%
Cost of naphtha fuel : Rs.40,000/Ton

Find out the cost of fuel for generating one unit of electricity.
Ans Heat input to turbine = 11500 x 180
= 2070000 kCal/hr
1 Mark
Efficiency of gas turbine = 28%
Gas turbine output = 2070000 x 0.28/ 860
= 674 kWh
2 Marks
Cost of generating 674 units of electricity = 180 kg x Rs.40
= Rs.7200/hr

Cost of One unit of Electricity generation = 7200/674


= Rs.10.7 / kWh
2 Marks
L-2 In an air conditioning duct 0.5 m x 0.5 m, the average velocity of air measured
by vane anemometer is 28 m/s. The static pressure at suction of the fan is -25
mmWC and at the discharge is 30 mmWC. The three phase induction motor
draws 10.8 A at 415 V with a power factor of 0.9. Find out the efficiency of the
fan if motor efficiency = 90% (Neglect air density correction).

Ans Volume flow rate of the fan, Q = Velocity x Area


= 28 x (0.5 x 0.5)
= 7 m3/s
1 Mark
Power input to the fan shaft = Motor input power x motor efficiency
= (Sqrt 3 x 0.415 x 10.8 x 0.9) x 0.9
= 6.3 kW
1 Mark
Fan efficiency = Volume in m3 / Sec x total pressure in mmwc
102 x Power input to the shaft in (kW)
1 Mark for writing the formula

= 7 x (30-(-25)) x 100
102 x 6.3

Fan efficiency = 59.91%


2 Marks
…………. End of Section - II ………….
3
_______________________
Bureau of Energy Efficiency
Paper 4 – Energy Auditor – Set B Key

Section - III: LONG NUMERICAL QUESTIONS Marks: 4 x 20 = 80

(i) Answer all Four questions


(ii) Each question carries Twenty marks

N-1 Calculate the efficiency of the Atmospheric Fluidised Bed Combustion Boiler by
indirect method using the following data:

Analysis of blended coal (% by mass)

Carbon : 53.9 %
Hydrogen : 3.1 %
Nitrogen : 1.1 %
Sulphur : 0.3 %
Ash : 23.8 %
Oxygen : 10.5 %
Moisture : 7.3 %
GCV : 5060 kCal / kg

The boiler operating parameters are given below.

Steam pressure : 62.0 kg / cm2g


Steam temperature : 470 oC
Actual air supplied : 8.91 kg/kg of coal
Mass of dry flue gas : 9.31 kg/kg of coal
Specific heat of flue gas : 0.23 kCal/kg oC
Flue gas temperature : 160 oC
CO2 in flue gas : 14.7 %
CO in flue gas : 325 ppm

GCV of bottom ash : 800 kCal/kg


GCV of fly ash : 452.5 kCal/kg

Ratio of bottom ash to fly ash : 10 : 90

Ambient temperature : 32.4 oC

Loss due to hydrogen in fuel : 3.54 %


Loss due to moisture in fuel : 0.93 %
Loss due to moisture in air : 0.2 %
Surface heat losses : 2%
(as assessed)

Ans To find all losses


1. % Heat loss in dry flue gas (L1) = L1

= m x C P x ( Tf − Ta )
x 100
GCV of fuel
= 9.31 x 0.23 x (160 − 32.4)
x 100
5060
4
_______________________
Bureau of Energy Efficiency
Paper 4 – Energy Auditor – Set B Key

= 5.40 %
4 Marks
2. % Heat loss due to partial conversion of C to CO (L2)

= %CO x C x 5654
x 100
% CO + (%CO2 )a  x GCVof fuel

0.0325 x 0.539 x 5654


= x 100
0.0325+14.7 x 5060
=0.13 %
4 Marks
3. % Heat loss due to unburnt in fly ash

% Ash in coal = 23.8


Ratio of bottom ash to fly ash = 10:90
GCV of fly ash = 452.5 Kcal/kg
Amount of fly ash in 1 kg of coal = 0.9 x 0.238 = 0.2142 kg
Heat loss in fly ash = 0.2142 x 452.5= 97 kCal / kg of coal
% heat loss in fly ash L3 = 97 x 100 / 5060 = 1.92 %
4 Marks
4. % Heat loss due to unburnt in bottom ash

GCV of bottom ash = 800 Kcal/kg


Amount of bottom ash
in 1 kg of coal = 0.10 x 0.238 = 0.0238 kg
Heat loss in bottom ash = 0.0238 x 800 = 19.04 kCal/kg of coal
% Heat loss in bottom ash L4 = 19.04 x 100 / 5060 = 0.38 %
4 Marks
5. Loss due to hydrogen in fuel L5 = 3.54% (given)
6. Loss due to moisture in fuel L6 = 0.93% (given)
7. Loss due to moisture in air L7 = 0.2% (given)
8. Surface heat losses L8 = 2% (given)

Boiler efficiency by indirect method = 100 – (L1+ L2+ L3+ L4+ L5+ L6+ L7+ L8)
= 100-(5.40 + 0.13+ 1.92+ 0.38+ 3.54+0.93+0.2+2)
= 100-14.57 = 85.5 %
4 Marks
N-2 a) For a VFD retrofit in a compressed air system with an initial investment of
Rs.2.55 lakhs the annual savings are Rs.58,000/-

The NPV of the project over a six year period for 8% discount rate is Rs.13,134.
The NPV of the project over a six year period for 10% discount rate is Rs.(-) 2468

Calculate the exact discount rate for NPV to be zero.

b) A counter current Heat Exchanger with total heat transfer area of 52 m2 is used
to exchange heat between a hot effluent stream of specific heat 0.33 kCal/kgoC
and cooling water stream.

5
_______________________
Bureau of Energy Efficiency
Paper 4 – Energy Auditor – Set B Key

The monitored parameters are given below:

Parameters Unit Inlet Outlet


Hot fluid flow kg/hr 86532 86532
o
Hot fluid temperature C 84 57
o
Cold fluid temperature C 45 54

Assuming LMTD correction factor of 0.91 for plate heat exchanger, calculate

i) The corrected LMTD


ii) Overall heat transfer coefficient, U (kCal/hr.m2.oC)
Ans a)

The positive and negative NPV's which are closest to zero discount rates are Rs
(+)13,134 corresponding to 8% discount rate and Rs (-) 2,468 corresponding to10%
discount rate .

Hence the exact discount rate = (0.08+(0.1-0.08)*(13134)/(13134-(-2468))*100


= 9.68 %
6 Marks
b)
i)
LMTD, Counter flow = {(84-54) – (57-45)}/ {ln (84-54) / (57-45)}
= 19.64 0 C
8 Marks
Correction Factor, F = 0.91 (given)

Corrected LMTD = F x LMTD


= 0.91 x 19.64 = 17.9 0 C
2 Marks
ii)
Overall heat transfer coefficient, U = Q / (A x Corrected LMTD)
U = 771000 / (52 x 17.9) = 828 kCal/hr.m2.0C
4 Marks

N-3 a) A centrifugal water pump operates at 30 m3/hr and at 1440 RPM. The pump
operating efficiency is 65% and motor efficiency is 89%. The discharge pressure
gauge shows 3.4 kg/cm2. The suction is 3 m below the pump centerline. If the
speed of the pump is reduced by 20 %, estimate the following:

i) pump flow,
ii) pump head and
iii) motor power.

Assume motor and pump efficiency remains same at the reduced speed.

b) In a 75 kW four pole induction motor operating at 49.8 Hz and rated for 415 V and
1440 RPM, the actual measured speed is 1470 RPM. Find out the percentage
loading of the motor if the voltage applied is 410 V.

6
_______________________
Bureau of Energy Efficiency
Paper 4 – Energy Auditor – Set B Key

Ans a)
Flow = 30 m3/hr
Head developed by the pump = 34 – (-3) = 37 m
Power drawn by the pump = (30/3600) x 37 x 1000 x 9.81/(1000 x 0.65)
= 4.65 kW
i)
Flow at 80 % speed = 30 / Q2 = 1440/1152
= 24 m3/hr
ii)
Head at 80% speed = 37 / H2 = (1440/1152)2
= 23.68 m
iii)
Shaft Power at 80% speed = 4.65/kW2 = (1440)3 / (1152 ) 3
= 2.38 kW
Power drawn by motor = 2.38 / 0.89
= 2.67 kW

b)
% Loading = Slip x 100%
(Ss – Sr) x (Vr / V)2

1 Mark
Synchronous speed = 120 x 49.8 / 4 = 1494 rpm

Slip = Synchronous Speed – Measured speed in rpm.


= 1494 – 1470 = 24 rpm.
2 Marks
% Loading = 24 x 100% = 43.37%
(1494 - 1440) x (415/410)2
2 Marks
N-4 Answer ANY ONE OF THE FOLLOWING among A, B, C and D

A) A cement kiln exhaust gas has the following composition on dry basis :
CO2 – 24.7%, O2 – 5.1%, CO - 0.1%, N2 – 70.1%. The static pressure and
temperature measured in the duct are -710 mmWC and 3000C respectively. The
velocity pressure measured with a pitot tube is 20.5 mmWC. The atmospheric
pressure at the site is 10350 mmWC.

Determine the volumetric flow rate of exhaust gas flowing through a duct of 3200
mm diameter ( Pitot tube constant = 0.89 )

Ans Molecular weight exhaust gas (dry basis) M


= %CO2xMCO2 + %O2xMO2 + %COxMCO + %N2 x MN2
= {(24.7 x 44) + (5.1 x 32) + (0.1 x 28) + (70.1 x 28)}/100
= 32.16 kg/kg mole
5 Marks
Exhaust Gas density at operating temperature= γ = [ PM / RT ]
= [ (10350 – 710) x 32.16 ) / { 847.84 x (273+300) }
= 0.638 kg/m3
5 Marks
Duct Area= 3.14 x( 3.2/2) 2= 8.05 m2
7
_______________________
Bureau of Energy Efficiency
Paper 4 – Energy Auditor – Set B Key

2 Marks
Volume flow rate
=A Cp (2 x g x ∆P / γ)1/2 = 8.05 x 0.89 (2 x 9.81x 20.5/0.638)1/2
= 179.89 m3/s
Volume flow rate = 6 47 596 m3/ h
8 Marks
or
B) Determine the cooling load of a commercial building for the following given data:

Outdoor conditions : DBT = 35C, WBT = 25C, Humidity = 18 g of water / kg of


dry air

Desired indoor conditions : DBT = 25.6C, RH = 50 %, Humidity = 10 g of water /


kg of dry air

Total area of wall = 40 m2, Total area of window = 20 m2

U – Factor ( Wall ) = 0.33 W / m2K

U – Factor ( Roof ) = 0.323 W / m2K

U – factor [ fixed windows with aluminium frames and a thermal break ] = 3.56 W /
m2K

Other data:

• 15 m x 25 m roof constructed of 100 mm concrete with 90 mm insulation &


steel decking.
• CLTD at 17:00 hr : Details : Wall = 12C; Roof = 44C; Glass Window = 7C
• SCL at 17 : 00 hr : Details : Glass Window = 605 W/ m2
• Shading coefficient of Window = 0.74
• Space is occupied from 8:00 to 17:00 hr by 25 people doing moderately
active work.
• Sensible heat gain / person = 75 W ; Latent heat gain / person = 55 W ;
CLF for people = 0.9
• Fluorescent light in space = 21.5 W/m2 ; CLF for lighting = 0.9
• Ballast factor details = 1.2 for fluorescent lights & 1.0 for incandescent lights
• Computers and office equipment in space produces 5.4 W/m2 of sensible
heat
• One coffee maker produces 1050 W of sensible heat and 450 W of latent
heat.
• Air changes / hr of infiltration = 0.3
• Height of building = 3.6 m

8
_______________________
Bureau of Energy Efficiency
Paper 4 – Energy Auditor – Set B Key

Ans
I External Heat Gain

(i) Conduction heat gain through the wall = U – factor x net area of wall x CLTD
= 0.33 x (40-20) x 12 ] = 79.2 W
2 Marks
(ii) Conduction heat gain through the roof = U – factor x net area of roof x CLTD
= 0.323 x ( 15 x 25 ) x 44
= 5 329.5 W
2 Marks
(iii) Conduction heat gain through the windows = U – factor x net area of windows x
CLTD
= (3.56 x 20 x 7) = 498.4 W
2 Marks
(iv) Solar radiation through glass
= Surface area x Shading coefficient x SCL
= (20 x 0.74 x 605) = 8954 W
2 Marks
II Internal Heat Gain

(i) Heat gain from people = Sensible heat gain + Latent heat gain

Sensible heat gain = (No. of people x Sensible heat gain / person x CLF)
= (25 x 75 x 0.9) = 1687.5 W

Latent heat gain = No. of people x Latent heat gain / person


= (25 x 55 ) = 1375 W
Therefore, Heat gain from people = (1687.5 + 1375 ) = 3062.5 W

(ii) Heat gain from lighting = ( Energy input x Ballast factor x CLF )
Energy input = ( Amount of lighting in space / unit area ) x Floor area
= 21.5 x ( 15 x 25 ) = 8062.5 W

Therefore, heat gain from lighting = ( 8062.5 x 1.2 x 0.9 ) = 8707.5 W

3 Marks
(iii) Heat generated by equipment :

Sensible heat generated by coffee maker = 1050 W


Latent heat generated by coffee maker = 450 W
Sensible heat gain by computers and office equipment = 5.4 x 375 = 2025 W
Therefore, Heat generated by equipment = 3525 W
2 Marks
(iv)Heat gain through air infiltration = ( Sensible heat gain + Latent heat gain )

Sensible heat gain = (1210 x airflow x ∆T )


Airflow =( Volume of space x air change rate ) / 3600
= { (15 x 25 x 3.6 ) x 0.3 } / 3600
9
_______________________
Bureau of Energy Efficiency
Paper 4 – Energy Auditor – Set B Key

= 0.1125 m3 / s

Therefore, sensible heat gain= 1210 x 0.1125 x ( 35 – 25.6 ) =1279.58 W

Latent heat gain = 3010 x 0.1125 x ( 18 – 10 ) = 2709 W

3 Marks

No Space Load Components Sensible Heat Load (W) Latent Heat Load (W)
1 Conduction through exterior wall 158.4 ------
2 Conduction through roof 5 329.5 ------
3 Conduction through windows 498.4 ------
4 Solar radiation through windows 8954 ------
5 Heat gained from people 1 687.5 1 375
6 Heat gained from lighting 8 707.5 ------
7 Heat gained from equipment 3 075 450
8 Heat gained by air infiltration 1 279.58 2 709
Total space cooling load 29 689.88 4 534

Total Cooling Load = 34,223.88 W


4 Marks
or
C)
(i) If the heat rate of a power plant is 2900 kCal/kWh, what is its efficiency?
(ii) What is the condenser vacuum in millibar, if the condenser back pressure is 0.89
kg/cm2?
(iii) Explain how Terminal Temperature Difference (TTD ) can also be negative
(iv) Calculate the % auxiliary power consumption for a Thermal power station if
Gross Heat Rate is 3200 kCal/kWh and Net Heat rate is 3500 kCal/kWh.

Ans (i) The efficiency of a power plant is inverse of heat rate


Efficiency = ((1*860)/2900)*100
= 29.7%
5 Marks
(ii) Condenser vaccum, kg/cm2 (a) = Atmospheric pressure – Condenser back
pressure
5 Marks
Condenser Vacuum = 1 – 0.89
Calculated condenser vacuum = 0.11 kg/cm2 (a) = 110 millibar

(iii) In some of the cases, because of the de-superheating zone in the feed water
heater, the feed water temperature leaving the heater may be higher than the
saturation temperature of the condensing zone. Therefore, the heater may have a
negative TTD
5 Marks
(iv) Auxiliary Power(%)= (1-(Gross Heat Rate/Net Heat rate))*100
= (1-(3200/3500))*100
= 8.5%
5 Marks
or

10
_______________________
Bureau of Energy Efficiency
Paper 4 – Energy Auditor – Set B Key

D) In an oil fired furnace following are the operating parameters:

Capacity of furnace - 10 T/hr


Daily production operating at 10 hours a day - 100 T/day
Specific fuel consumption - 65 litres /T of finished product
Flue gas temperature at the exit of furnace - 600 oC
Ambient temperature - 30 oC
G.C.V of oil - 10,000 kCal/kg
Theoretical air required for combustion - 14 kg of air/ kg of fuel
Specific heat of flue gas - 0.26 kCal/kgoC
Specific heat of air - 0.24 kCal/kgoC
Oxygen in flue gas - 8%

The management is planning to install a recuperator to preheat the combustion air


upto 200oC

Yield without the recuperator - 90%


Yield after installing the recuperator - 95%

Calculate

(i) the percentage heat reduction in flue gas after installation of recuperator
(ii) the increase in daily production due to yield improvement
(iii) specific fuel consumption after installing the heat recovery recuperator
(assuming 1 % fuel saving for every 20oC rise in combustion air temperature)

Ans i)
% excess air supplied = 8/ (21-8)
= 61.5 %
2 Marks
Actual mass of air supplied = [1 + (EA/100)] x theoretical air
= [1 + (61.5/100)] x 14
=22.61 kg of air/kg of fuel
2 Marks
Daily fuel consumption =65 x 100 = 6500 kg/day

( In the question the Sp. Gravity of fuel is not given. If the candidate has calculated as
above ie 65 litres full marks may be give.

If a candidate assumes a sp.gravity and multiplies 65 x assumed sp.gravity again full


marks may be given.)

Heat in flue gas = {6500 + (6500 x 22.61)} x 0.26 x (600-30)


= 22743513 kCal/day

Heat in preheated combustion air = 6500 x 22.61 x 0.24 x (200 - 30)


= 5996172 kCal/day

Percentage heat reduction in flue gas =(5996172 / 22743513)


= 26.4 %
5 Marks
11
_______________________
Bureau of Energy Efficiency
Paper 4 – Energy Auditor – Set B Key

(ii)

Daily additional production due to yield improvement


= 100 x 95/90
= 105.5 Tonnes/day

Additional production = 105.5 – 100


= 5.5 T/day
5 Marks
(iii)
Reduction in fuel consumption = (200 - 30)/20 = 8.5 %
Fuel consumption after waste heat recovery = 6500 – (6500 x 0.085)
= 5947.5
Specific fuel consumption

Daily oil consumption = 5947.5 kg/day


Production = 105.5 T/day
Specific fuel consumption = 5947.5/ 105.5
= 56.37 kg/tone
6 Marks

-------- End of Section - III ---------

12
_______________________
Bureau of Energy Efficiency
Paper 4 – Energy Auditor – Set A Key

11th NATIONAL CERTIFICATION EXAMINATION – February, 2011


FOR
ENERGY AUDITORS

PAPER – 4: Energy Performance Assessment For Equipment and Utility Systems

Date: 06.02.2011 Timings: 1400-1600 HRS Duration: 2 HRS Max. Marks: 100

General instructions:
o Please check that this question paper contains 12 printed pages
o Please check that this question paper contains 16 questions
o The question paper is divided into three sections
o All questions in all three sections are compulsory
o All parts of a question should be answered at one place

Section - I: SHORT DESCRIPTIVE QUESTIONS Marks: 10 x 1 = 10

(i) Answer all Ten questions


(ii) Each question carries One mark

S-1 In what type of furnace, a top pressure recovery turbine is used?

Ans: blast furnace


S-2 In a heat exchanger, the hot fluid inlet and outlet temperatures are 110 °C and 70 °C.
The cold fluid inlet and outlet temperatures are 30 °C and 45 °C. The effectiveness of
heat exchanger is

Ans:
= 15/80 = 0.1875
S-3 Name two types of discounted cash-flow techniques used in the financial evaluation of
energy saving projects

Ans:
1 NPV: Net present value
2 IRR: Internal rate of return
S-4 Name two areas of major thermal energy consumption in a cement plant

Ans: Rotary kiln and precalciner


S-5 For which fuel the difference between GCV and LCV will be higher, Coal or Natural
Gas?

Ans: Natural Gas


1
_______________________
Bureau of Energy Efficiency
Paper 4 – Energy Auditor – Set A Key

S-6 What are the two major functions of coke in a blast furnace?

Ans: Coke is used in Blast Furnace (BF) both as a reductant (reduction of ore to liquid
metal )and as a source of thermal energy.
S-7 For a thermal power plant, which type of heat rate (Gross or Net) has a higher value
for the same generator output?

Ans: Net heat rate


S-8 Why humidification is required in spinning and weaving sections of textile processing?

Ans: Humidity is required to prevent yarn breakage and minimize build up of static
charge to reduce dust and fibre fly (Fluff)
S-9 What is the purpose of ‘reheat’ in a thermal power plant cycle?

Ans: There is a gain in net work and because of which the efficiency is enhanced.
S-10 In a rotary kiln of cement plant, why % CO2 in exhaust gases cannot be an indicator of
excess air?

Ans: Because the process also emits CO2

…………. End of Section - I ………….

Section - II: LONG DESCRIPTIVE QUESTIONS Marks: 2 x 5 = 10

(i) Answer all Two questions


(ii) Each question carries Five marks

L-1 The following are the operating parameters of a regenerative feedwater heater in a
thermal power plant

Saturation temperature of steam to heater - 84.3 oC


Inlet feedwater temperature - 44 oC
Outlet feedwater temperature - 75 oC
Drain outlet temperature - 58.4 oC

Calculate the Terminal Temperature Difference (TTD) and Drain Cooler Approach
(DCA).

Ans : TTD = 84.3 – 75

= 9.3 oC

DCA = 58.4 – 44
= 14.4 oC

2
_______________________
Bureau of Energy Efficiency
Paper 4 – Energy Auditor – Set A Key

L-2 In a 30 kW four pole induction motor operating at 49.9 Hz and rated for 415 V and
1470 RPM, the actual measured speed is 1485 RPM. Find out the percentage
loading of the motor if the voltage applied is 425 V.

Ans:
% Loading = Slip x 100%
(Ss – Sr) x (Vr / V)2

Synchronous speed = 120 x 49.9 / 4 = 1497 rpm

Slip = Synchronous Speed – Measured speed in rpm.


= 1497– 1485 = 12 rpm.

% Loading = 12 x 100% = 46.6%


( 1497- 1470) x (415/425)2

…………. End of Section - II ………….

Section - III: NUMERICAL QUESTIONS Marks: 4 x 20 = 80

(i) Answer all Four questions


(ii) Each question carries Twenty marks

N-1 Flow rates of the hot and the cold water streams flowing through a heat exchanger
are 12 and 30 kg/min, respectively. Hot and cold water stream inlet temperatures are
72 °C and 27 °C, respectively. The exit temperature of the hot stream is required to
be 52 °C. The specific heat of water is 4.179 kJ/kg K. The overall heat transfer
coefficient is 800 W/m2 K.

Neglecting the effect of fouling, calculate the heat transfer area for

a) Parallel-flow

b) Counter-flow

a)
Rate of heat transfer M x cp x delt T
(12/60) × 4.179 x 1000 × (72 – 52) =
16716 W

Cold water exit temperature 27 + [16716 / (4.179 x 1000 × (30/60))]


27 + 8 = 35 °C

Terminal temperature differences for (72 – 27) and (52 – 35) 0C i.e., 45 °C
3
_______________________
Bureau of Energy Efficiency
Paper 4 – Energy Auditor – Set A Key

parallel flow heat exchangers and 17 °C respectively.

LMTD (45 – 17)/ln(45/17) = 28.76

Overall heat transfer coefficient U 800 W/m2 K


Heat transfer area required for parallel [16716 / (800 × 28.76)]
flow
0.72 m2

b)
Terminal temperature differences for (72 – 35) and (52 – 27) 0C i.e., 37 °C
counter flow heat exchangers and 25 °C respectively.

LMTD (37 – 25)/ln(37/25) = 30.6

Overall heat transfer coefficient U 800 W/m2 K


Heat transfer area required for counter [16716 / (800 × 30.6)]
flow
0.684 m2

N-2 Two energy conservation projects have been proposed.

For the first project, a capital investment of Rs.1,50,000/- is required and the net
annual saving is Rs. 50,000/- for 5 years. The salvage value at the end of 5 years for
the first project is Nil.

For the second project, a capital investment of Rs. 1,50,000/- yields savings of Rs.
50,000/- for first 2 years each and Rs. 60,000/- for next 3 years each. The salvage
value at the end of 5 years for the second project is Rs. 10,000/-. Determine:

a) Net present value for both the projects with a discount factor of 9%.
b) Profitability index for both the projects with a discount factor of 9%.
c) Internal rate of return for both the projects.

Ans:

a)
NPV for project 1 = – 150000 + 50000/1.09 + 50000/(1.09)2 + 50000/(1.09)3 +
50000/(1.09)4 + 50000/(1.09)5 = + 44483

NPV for project 2 = – 150000 + 50000/1.09 + 50000/(1.09)2 + 60000/(1.09)3 +


60000/(1.09)4 + 60000/(1.09)5 + 10000/(1.09)5 = + 72287

b)
Profitability index for project 1 = 44483 / 150000 = 0.297

Profitability index for project 2 = 72287 / 150000 = 0.481

4
_______________________
Bureau of Energy Efficiency
Paper 4 – Energy Auditor – Set A Key

c)
IRR for project 1 = 19.86 %

IRR for project 2 = 25.07 %

N-3 In an air conditioning system of a food processing industry, the cold air flow rate is
20,000 m3/hr at a density of 1.2 kg/m3 .The inlet and outlet enthalpy of the air are 105
kJ/kg and 80 kJ/kg. The COP of the existing vapour compression system is 3.75. The
efficiency of the motor coupled with the compressor is 90%.

The management wants to install a Vapour Absorption System (VAR).The saturated


steam for VAR will be supplied either from a new waste heat boiler to be installed with
the existing DG sets or from the existing FO fuel fired boiler. The plant is operating for
8000 hr/annum. The investment of VAR system is Rs. 20 lakhs. The investment for
waste heat boiler is Rs. 6 lakhs. The power cost is Rs. 6/kWh.

As an energy auditor which one of the following options will you recommend to the
management?

Option1: Supply steam from the existing FO fuel fired boiler to VAR system and avoid
the investment of waste heat boiler

Option2 - Supply steam from the waste heat boiler, which needs an investment in
addition to VAR system

The steam consumption per TR will be 5.5 kg/TR. The cost of FO is Rs.32,000/
tonne. The evaporation ratio of the existing FO fired boiler is 14. Neglect losses in
transmission of steam and chilled water.

Ans.
Existing Base Case VCR System
TR Rating = (20,000 m3/hr x 1.2 kg/m3) (105-80) kJ/kg
------------------------------------------------------
3024 x 4.187
= 47.38 TR

COP = 3.75
COP = Refrigeration effect kCal/hr
-----------------------------------------
Power Input kCal/hr
Compressor power input = 47.38 x 3024 kCal
------------------------------
3.75 x 860
= 44.43 kW
Motor input power = 44.43/0.9 = 49.37 kW
Annual Energy Consumption = 49.37 x 8000 = 3.95 Lakhs kWh
Annual cost in VCR system (Base =3.95 x 6 = Rs. 23.7 Lakhs
Case)

5
_______________________
Bureau of Energy Efficiency
Paper 4 – Energy Auditor – Set A Key

Option – 1 : VAR System with Steam Supply from Existing Boiler


Steam Consumption /TR = 5.5 kg/TR
Steam Consumption per hr = 5.5 x 47.38 = 261 kg/hr

Evaporation Ratio IN THE EXISTING = 14


BOILER
1 ton of steam cost = Rs.32000
-------------- = Rs.2.29 /kg of steam
14
Investment for VAR system = Rs.20.00 Lakhs
Electricity cost saving per hr = 49.37 x 6 =Rs. 296.22
Steam cost per hr = 261 x 2.29 = Rs. 598
Since the steam cost per hour is higher than electricity cost this option is not feasible

Option-2: With VAR & steam supply from WHR steam boiler of DG set

Total Investment = Rs. 20.00 + 6.00 = 26.00 Lakhs


Annual Savings = Rs.23.7 lakhs
Simple Pay back period = 26/23.7 = 1.09 years.
Solution : Option 2 should be selected

N -4 The candidate may answer ANY ONE OF THE FOLLOWING among A, B, C and D

A) The following are the data obtained from a pulverized coal fired thermal power plant

Main steam pressure and temperature : 155 kg/cm2 (g), 540 oC


Main steam flow rate : 624 TPH
Enthalpy of main steam : 815.2 kCal/kg
Feed water temperature : 229 oC
Cold reheat steam pressure and temperature : 36 kg/cm2(g), 330 oC
Enthalpy of cold reheat steam : 730 kCal/kg
Hot reheat steam pressure and temperature : 35 kg/cm2(g), 540 oC
Enthalpy of hot reheat steam : 844.5 kCal/kg
Reheat steam flow : 563 TPH
Generator output : 207.3 MW
Boiler efficiency : 85.5%
Back pressure : 0.9 kg/cm2 (g)
Condenser CW inlet temperature : 26 oC
Condenser CW outlet temperature : 37 oC
Exhaust steam saturation temperature : 45.4 oC
Enthalpy of wet steam at vacuum : 508 kCal/kg
Condenser CW flow : 24800 m3/hr

Calculate
a) Turbine heat rate, unit heat rate and turbine cycle efficiency
b) Condenser heat load, effectiveness and calculated condenser vacuum in millibar

Ans:
6
_______________________
Bureau of Energy Efficiency
Paper 4 – Energy Auditor – Set A Key

Turbine heat rate and unit heat rate


624000 (815.2 − 229) + 563000 (844.5 − 730)
Turbine heat rate, kCal/kWh =
207300

Turbine heat rate = 2075.5 kCal/kWh


2075.5
Unit heat rate = = 2427.5 kCal/kWh
0.855

Turbine cycle efficiency (thermal efficiency)

860
Turbine cycle efficiency, % =  100
Turbine heat rate

860
Turbine cycle efficiency, % = 100
2075.5

Turbine cycle efficiency= 41.4 %

Condenser heat load, MCal/hr = Q  T  Cp

Q -Water flow rate, kg/hr


T - Average CW temperature rise, oC
Cp - Specific heat, kcal/kg oC

Condenser heat load = 24800 (37 − 26) 1


Condenser heat load = 272800MCal/hr

Calculated condenser vacuum, kg/cm2 (a)


= Atmospheric pressure − Condenser back pressure

Calculated condenser vacuum = 1 − 0.9


Calculated condenser vacuum = 0.1kg/cm 2 (a) = 98.06 millibar

Condenser Effectiveness,%
Rise in cooling water temperature
= 100
Saturationtemperature of steam − Cooling water inlettemperature

37 - 26
Condenser Effectiveness = 100 = 56.7 %
45.4 − 26

or

7
_______________________
Bureau of Energy Efficiency
Paper 4 – Energy Auditor – Set A Key

B) a) What are the major advantages of using sinter in a Blast furnace


b) A furnace is fired with blast furnace gas having an analysis by volume as follows

CO2 – 13 %, CO – 25 %, H2 - 3.5 %, N2 – 58.5 %

Calculate the percentage of excess air when the dry product of combustion contains
3.5 % O2

Ans:

a) The major advantages of using sinter in Blast Furnaces are :

• Use of iron ore fines, coke breeze, metallurgical wastes, lime, dolomite for hot
metal production
• Better reducibility and other high temperature properties
• Increased BF productivity
• Improved quality of hot metal
• Reduction in coke rate in blast furnaces

b)

Basis : 100 kg mol. of blast furnace gas

Oxygen demand and flue gas formed are computed as under

Amount, kg Oxygen required kg Flue gas obtained, kg


Constituents
mol. mol mol.
CO2 13.0 0 13.0 (CO2)
CO 25.0 12.50 25.0 (CO2)
H2 3.5 1.75 3.5 (H2)
N2 58.5 0 58.5 (N2)
Total 100.0 14.25 100.0

Let y1 = kg mol of dry flue gas obtained


Z = kg mol of combustion air used

N2 from combustion air = 0.79x


O2 from combustion air = 0.21x

O2 excess in flue gas = 0.21 z – 14.25


Dry flue gas = (100 – water vapour) + 0.79z + (0.21z-14.25)
= (100-3.5) +z-14.25= 82.25 + z = y1

Oxygen / dry flue gas = (0.21z-14.25)/ (82.25+z) = 3.5/100

Solving z= 97.88 kg mol

8
_______________________
Bureau of Energy Efficiency
Paper 4 – Energy Auditor – Set A Key

Oxygen from air = 97.88 x 0.21 = 20.65

Oxygen excess = 20.55 – 14.25 = 6.3 kg mol

% Excess air = % excess oxygen = 6.3/14.25 x100 = 44.2

or

C) a) 125 kg of fabric is to be dyed in a jigger. The dye liquor is heated from 30 0C to 90 0C.
Calculate steam (steam enthalpy 660 kCal/kg) requirement per batch and specific
steam consumption (kg of steam per kg of cloth), if liquor ratio is 1:6.5; allowing 10%
margin for losses.

b) In a textile mill, a thermic fluid heater of 15 lakh kCal/hr capacity is meeting process
heat requirements.

The observed parameters of thermic fluid heater are:

Thermic fluid circulation rate - 100 m3/hr


Outlet temperature of fluid - 270 0C
Return temperature of fluid - 256 0C
Specific heat of fluid - 0.55 kCal/kg 0C
Density of fluid - 830 kg/m3
Present coal consumption - 300 kg /hr
GCV of coal - 3500 kCal/hr

i) What is the % loading of the thermic fluid heater?


ii) What is the existing thermal efficiency?
iii) The management is proposing to add a five chamber, (each chamber 1 Lakh kCal/hr
duty) to the heater. Will it be able to take the load?

Ans:

a)
Heat energy required / batch = 125 kg x 6.5 liq.ratio x 1.10 (including margin)
x (90 – 30)
= 53625 kCal/batch

= steam quantity needed / batch


= 53625/660
= 81.25 kg
Specific steam consumption = 81.25/125 = 0.65 kg/kg

Ans:
9
_______________________
Bureau of Energy Efficiency
Paper 4 – Energy Auditor – Set A Key

i)
Heat duty = 100 m3/hr x 830 kg/m3 x 0.55 x 14
= 6,39,100 kCal/hr
% loading = 6,39,100/15,00,000
= 42.60 %

ii)
Input energy = 300 kg/hr x 3500 kCal
= 10,50,000 kCal/hr
Efficiency of the heater = 6,39,100/10,50,000
= 60.86%

iii)
New chamber heat duty = 5 x 1,00,000
= 5,00,000 kCal/hr

The total heat duty with new stenter loading = 6,39,100 + 5,00,000
= 11,39,100 kCal/hr

It is possible to comment new stenter since this is within 15 Lakh kCal/hr capacity of
the TFH

or

D) a) The following are the data obtained from the energy audit of a cement plant

Clinker analysis (loss free basis)

Constituents Determined SiO2 Fe2O3 Al2O3 CaO MgO


(%) 20.54 4.14 5.53 63.79 3.8

Preheater exit gas analysis

Oxygen (% v/v) CO CO2


(% v/v) (% v/v)
5.6 0.1 23.2

Preheater exit temperature – 328oC


Atomospheric pressure at site – 10.318 m
Static pressure – (-684 mmwg)

i) Calculate the heat of formation of clinker


ii) Calculate the density of preheater exit gases

b) An energy audit of a coal mill fan was carried out. It was observed that fan was
delivering 1,60,000 Nm3/hr of air at static pressure rise of 65 mm WC. The power
measurement of the 3-phase induction motor coupled with the fan recorded 20 kW/
10
_______________________
Bureau of Energy Efficiency
Paper 4 – Energy Auditor – Set A Key

phase on an average. The motor operating efficiency was assessed as 0.90 from the
motor performance curves. What would be the fan static efficiency?

Ans:

a)

i) Heat of formation of Clinker

 HR = 2.22Al2O3+6.48MgO+7.646CaO–5.116SiO2–0.59Fe2O3

= (2.22 x 5.53)+(6.48 x 3.8)+(7.646 x 63.79)–(5.116x 20.54)–(0.59 x 4.14)


= 417.11 kCal/kg Clinker

ii) Density of gases at preheater exit

(O2% x MW) + (CO2% x MW) + ((N2 + CO)% x MW)


 stp = kg/Nm3
22.4 x 100

5.6 x 32) + (23.2 x 44) + (71.2 x 28)


 stp = kg/Nm3
22.4 x 100
3
= 1.4257 kg/Nm

10318* −684S 273


t , p = 1.4257   kg/m3
10318 273 + 328e
(* as per the altitude of the plant above MSL)
10334 − 655 273
t , p = 1.436   kg/m3
10334 273 + 316
= 0.623 kg/m3

b)

Q = 1,60000 Nm3 / hr.= 44.44 m3/sec ,


SP = 65 mmWC,
St = ?,

Power input to 3 phase motor= 20 x 3 = 60 kW


Power input to fan shaft = 60 x 0.90 = 54 kW

Fan static  = Volume in m3/sec x Pst in mmWc x 100


102 x Power I/p to shaft

11
_______________________
Bureau of Energy Efficiency
Paper 4 – Energy Auditor – Set A Key

= 44.44 x 65 x 100
102 x 54
= 0.524 x 100
= 52.4%

-------- End of Section - III ---------

12
_______________________
Bureau of Energy Efficiency
Paper 4 – Energy Auditor – Set B Key

Regn No: _________________


Name: ___________________
(To be written by the candidate)

11th NATIONAL CERTIFICATION EXAMINATION – February, 2011


FOR
ENERGY AUDITORS

PAPER – 4: Energy Performance Assessment For Equipment and Utility Systems

Date: 06.02.2011 Timings: 1400-1600 HRS Duration: 2 HRS Max. Marks: 100

General instructions:
o Please check that this question paper contains 12 printed pages
o Please check that this question paper contains 16 questions
o The question paper is divided into three sections
o All questions in all three sections are compulsory
o All parts of a question should be answered at one place

Section - I: SHORT DESCRIPTIVE QUESTIONS Marks: 10 x 1 = 10

(i) Answer all Ten questions


(ii) Each question carries One mark

S-1 Why humidification is required in spinning and weaving sections of textile processing?

Ans: Humidity is required to prevent yarn breakage and minimize build up of static
charge to reduce dust and fibre fly (Fluff)
S-2 What is the purpose of ‘reheat’ in a thermal power plant cycle?

Ans: There is a gain in net work and because of which the efficiency is enhanced.
S-3 What are the two major functions of coke in a blast furnace?

Ans: Coke is used in Blast Furnace (BF) both as a reductant (reduction of ore to liquid
metal )and as a source of thermal energy.
S-4 Name two types of discounted cash-flow techniques used in the financial evaluation of
energy saving projects

Ans:
1 NPV: Net present value
2 IRR: Internal rate of return
S-5 Name two areas of major thermal energy consumption in a cement plant

Ans: Rotary kiln and precalciner


1
_______________________
Bureau of Energy Efficiency
Paper 4 – Energy Auditor – Set B Key

S-6 In what type of furnace, a top pressure recovery turbine is used?

Ans: blast furnace


S-7 In a rotary kiln of cement plant, why % CO2 in exhaust gases cannot be an indicator of
excess air?

Ans: Because the process also emits CO2


S-8 In a heat exchanger, the hot fluid inlet and outlet temperatures are 110 °C and 70 °C.
The cold fluid inlet and outlet temperatures are 30 °C and 45 °C. The effectiveness of
heat exchanger is

Ans:
= 15/80 = 0.1875
S-9 For which fuel the difference between GCV and LCV will be higher, Coal or Natural
Gas?

Ans: Natural Gas


S-10 For a thermal power plant, which type of heat rate (Gross or Net) has a higher value
for the same generator output?

Ans: Net heat rate

…………. End of Section - I ………….

Section - II: LONG DESCRIPTIVE QUESTIONS Marks: 2 x 5 = 10

(i) Answer all Two questions


(ii) Each question carries Five marks

L-1 In a 30 kW four pole induction motor operating at 49.9 Hz and rated for 415 V and
1470 RPM, the actual measured speed is 1475 RPM. Find out the percentage
loading of the motor if the voltage applied is 425 V.

Ans:
% Loading = Slip x 100%
(Ss – Sr) x (Vr / V)2

Synchronous speed = 120 x 49.9 / 4 = 1497 rpm

Slip = Synchronous Speed – Measured speed in rpm.


= 1497– 1475 = 22 rpm.

% Loading = 22 x 100% = 85.43%


( 1497- 1470) x (415/425)2

2
_______________________
Bureau of Energy Efficiency
Paper 4 – Energy Auditor – Set B Key

L-2 The following are the operating parameters of a regenerative feedwater heater in a
thermal power plant

Saturation temperature of steam to heater - 80.8 oC


Inlet feedwater temperature - 46 oC
Outlet feedwater temperature - 73 oC
Drain outlet temperature - 59.2 oC

Calculate the Terminal Temperature Difference (TTD) and Drain Cooler Approach
(DCA).

Ans : TTD = 80.8 – 73

= 7.8 oC

DCA = 59.2 – 46
= 13.2 oC

…………. End of Section - II ………….

Section - III: NUMERICAL QUESTIONS Marks: 4 x 20 = 80

(i) Answer all Four questions


(ii) Each question carries Twenty marks

N-1 Flow rates of the hot and the cold water streams flowing through a heat exchanger
are 12 and 30 kg/min, respectively. Hot and cold water stream inlet temperatures are
72 °C and 27 °C, respectively. The exit temperature of the hot stream is required to
be 52 °C. The specific heat of water is 4.179 kJ/kg K. The overall heat transfer
coefficient is 900 W/m2 K.

Neglecting the effect of fouling, calculate the heat transfer area for

a) Parallel-flow

b) Counter-flow

a)
Rate of heat transfer M x cp x delt T
(12/60) × 4.179 x 1000 × (72 – 52) =
16716 W

Cold water exit temperature 27 + [16716 / (4.179 x 1000 × (30/60))]


27 + 8 = 35 °C

3
_______________________
Bureau of Energy Efficiency
Paper 4 – Energy Auditor – Set B Key

Terminal temperature differences for (72 – 27) and (52 – 35) 0C i.e., 45 °C
parallel flow heat exchangers and 17 °C respectively.

LMTD (45 – 17)/ln(45/17) = 28.76

Overall heat transfer coefficient U 900 W/m2 K


Heat transfer area required for parallel [16716 / (900 × 28.76)]
flow
0.64 m2

b)
Terminal temperature differences for (72 – 35) and (52 – 27) 0C i.e., 37 °C
counter flow heat exchangers and 25 °C respectively.

LMTD (37 – 25)/ln(37/25) = 30.6

Overall heat transfer coefficient U 900 W/m2 K


Heat transfer area required for counter [16716 / (900 × 30.6)]
flow
0.601 m2

N-2 Two energy conservation projects have been proposed.

For the first project, a capital investment of Rs.2,00,000/- is required and the net
annual saving is Rs. 50,000/- for 5 years. The salvage value at the end of 5 years for
the first project is Nil.

For the second project, a capital investment of Rs. 2,00,000/- yields savings of Rs.
50,000/- for first 2 years each and Rs. 60,000/- for next 3 years each. The salvage
value at the end of 5 years for the second project is Rs. 10,000/-. Determine:

a) Net present value for both the projects with a discount factor of 9%.
b) Profitability index for both the projects with a discount factor of 9%.
c) Internal rate of return for both the projects.

Ans:

a)
NPV for project 1 = - 200000 + 50000/1.09 + 50000/(1.09)2 + 50000/(1.09)3 +
50000/(1.09)4 + 50000/(1.09)5 = - 5517.4

NPV for project 2 = – 200000 + 50000/1.09 + 50000/(1.09)2 + 60000/(1.09)3 +


60000/(1.09)4 + 60000/(1.09)5 + 10000/(1.09)5 = + 22287.3

b)
Profitability index for project 1 = -5517.4 / 200000 = -0.0276
4
_______________________
Bureau of Energy Efficiency
Paper 4 – Energy Auditor – Set B Key

Profitability index for project 2 = 22287.3 / 200000 = 0.111

c)
IRR for project 1 = 7.93 %

IRR for project 2 = 12.96 %

N-3 In an air conditioning system of a food processing industry, the cold air flow rate is
20,000 m3/hr at a density of 1.2 kg/m3 .The inlet and outlet enthalpy of the air are 105
kJ/kg and 80 kJ/kg. The COP of the existing vapour compression system is 3.75. The
efficiency of the motor coupled with the compressor is 90%.

The management wants to install a Vapour Absorption System (VAR).The saturated


steam for VAR will be supplied either from a new waste heat boiler to be installed with
the existing DG sets or from the existing FO fuel fired boiler. The plant is operating for
8000 hr/annum. The investment of VAR system is Rs. 22 lakhs. The investment for
waste heat boiler is Rs. 8 lakhs. The power cost is Rs. 6/kWh.

As an energy auditor which one of the following options will you recommend to the
management?

Option1: Supply steam from the existing FO fuel fired boiler to VAR system and avoid
the investment of waste heat boiler

Option2 - Supply steam from the waste heat boiler, which needs an investment in
addition to VAR system

The steam consumption per TR will be 5.5 kg/TR. The cost of FO is Rs.32,000/
tonne. The evaporation ratio of the existing FO fired boiler is 14. Neglect losses in
transmission of steam and chilled water.

Ans.
Existing Base Case VCR System
TR Rating = (20,000 m3/hr x 1.2 kg/m3) (105-80) kJ/kg
------------------------------------------------------
3024 x 4.187
= 47.38 TR

COP = 3.75
COP = Refrigeration effect kCal/hr
-----------------------------------------
Power Input kCal/hr
Compressor power input = 47.38 x 3024 kCal
------------------------------
3.75 x 860
= 44.43 kW

5
_______________________
Bureau of Energy Efficiency
Paper 4 – Energy Auditor – Set B Key

Motor input power = 44.43/0.9 = 49.37 kW

Annual Energy Consumption = 49.37 x 8000 = 3.95 Lakhs kWh


Annual cost in VCR system (Base =3.95 x 6 = Rs. 23.7 Lakhs
Case)

Option – 1 : VAR System with Steam Supply from Existing Boiler


Steam Consumption /TR = 5.5 kg/TR
Steam Consumption per hr = 5.5 x 47.38 = 261 kg/hr

Evaporation Ratio IN THE EXISTING = 14


BOILER
1 ton of steam cost = Rs.32000
-------------- = Rs.2.29 /kg of steam
14
Investment for VAR system = Rs.22.00 Lakhs
Electricity cost saving per hr = 49.37 x 6 =Rs. 296.22
Steam cost per hr = 261 x 2.29 = Rs. 598
Since the steam cost per hour is higher than electricity cost this option is not feasible

Option-2: With VAR & steam supply from WHR steam boiler of DG set

Total Investment = Rs. 22.00 + 8.00 = 30.00 Lakhs


Annual Savings = Rs.23.7 lakhs
Simple Pay back period = 30/23.7 = 1.27 years.
Solution : Option 2 should be selected

N -4 The candidate may answer ANY ONE OF THE FOLLOWING among A, B, C and D

A) The following are the data obtained from a pulverized coal fired thermal power plant

Main steam pressure and temperature : 155 kg/cm2 (g), 540 oC


Main steam flow rate : 624 TPH
Enthalpy of main steam : 815.2 kCal/kg
Feed water temperature : 229 oC
Cold reheat steam pressure and temperature : 36 kg/cm2(g), 330 oC
Enthalpy of cold reheat steam : 730 kCal/kg
Hot reheat steam pressure and temperature : 35 kg/cm2(g), 540 oC
Enthalpy of hot reheat steam : 844.5 kCal/kg
Reheat steam flow : 563 TPH
Generator output : 207.3 MW
Boiler efficiency : 85.5%
Back pressure : 0.9 kg/cm2 (g)
Condenser CW inlet temperature : 26 oC
Condenser CW outlet temperature : 37 oC
Exhaust steam saturation temperature : 45.4 oC
Enthalpy of wet steam at vacuum : 508 kCal/kg
Condenser CW flow : 24800 m3/hr
6
_______________________
Bureau of Energy Efficiency
Paper 4 – Energy Auditor – Set B Key

Calculate
a) Turbine heat rate, unit heat rate and turbine cycle efficiency
b) Condenser heat load, effectiveness and calculated condenser vacuum in millibar

Ans:

Turbine heat rate and unit heat rate


624000 (815.2 − 229) + 563000 (844.5 − 730)
Turbine heat rate, kCal/kWh =
207300

Turbine heat rate = 2075.5 kCal/kWh


2075.5
Unit heat rate = = 2427.5 kCal/kWh
0.855

Turbine cycle efficiency (thermal efficiency)

860
Turbine cycle efficiency, % =  100
Turbine heat rate

860
Turbine cycle efficiency, % = 100
2075.5

Turbine cycle efficiency= 41.4 %

Condenser heat load, MCal/hr = Q  T  Cp

Q -Water flow rate, kg/hr


T - Average CW temperature rise, oC
Cp - Specific heat, kcal/kg oC

Condenser heat load = 24800 (37 − 26) 1


Condenser heat load = 272800MCal/hr

Calculated condenser vacuum, kg/cm2 (a)


= Atmospheric pressure − Condenser back pressure

Calculated condenser vacuum = 1 − 0.9


Calculated condenser vacuum = 0.1kg/cm 2 (a) = 98.06 millibar

7
_______________________
Bureau of Energy Efficiency
Paper 4 – Energy Auditor – Set B Key

Condenser Effectiveness,%
Rise in cooling water temperature
= 100
Saturationtemperature of steam − Cooling water inlettemperature

37 - 26
Condenser Effectiveness = 100 = 56.7 %
45.4 − 26

or

B) a) What are the major advantages of using sinter in a Blast furnace


b) A furnace is fired with blast furnace gas having an analysis by volume as follows

CO2 – 13 %, CO – 25 %, H2 - 3.5 %, N2 – 58.5 %

Calculate the percentage of excess air when the dry product of combustion contains
3.5 % O2

Ans:

a) The major advantages of using sinter in Blast Furnaces are :

• Use of iron ore fines, coke breeze, metallurgical wastes, lime, dolomite for hot
metal production
• Better reducibility and other high temperature properties
• Increased BF productivity
• Improved quality of hot metal
• Reduction in coke rate in blast furnaces

b)

Basis : 100 kg mol. of blast furnace gas

Oxygen demand and flue gas formed are computed as under

Amount, kg Oxygen required kg Flue gas obtained, kg


Constituents
mol. mol mol.
CO2 13.0 0 13.0 (CO2)
CO 25.0 12.50 25.0 (CO2)
H2 3.5 1.75 3.5 (H2)
N2 58.5 0 58.5 (N2)
Total 100.0 14.25 100.0

Let y1 = kg mol of dry flue gas obtained


Z = kg mol of combustion air used

8
_______________________
Bureau of Energy Efficiency
Paper 4 – Energy Auditor – Set B Key

a) N2 from combustion air = 0.79x


O2 from combustion air = 0.21x

O2 excess in flue gas = 0.21 z – 14.25


Dry flue gas = (100 – water vapour) + 0.79z + (0.21z-14.25)
= (100-3.5) +z-14.25= 82.25 + z = y1

Oxygen / dry flue gas = (0.21z-14.25)/ (82.25+z) = 3.5/100

Solving z= 97.88 kg mol

Oxygen from air = 97.88 x 0.21 = 20.65

Oxygen excess = 20.55 – 14.25 = 6.3 kg mol

% Excess air = % excess oxygen = 6.3/14.25 x100 = 44.2

or

C) a) 125 kg of fabric is to be dyed in a jigger. The dye liquor is heated from 30 0C to 90 0C.
Calculate steam (steam enthalpy 660 kCal/kg) requirement per batch and specific
steam consumption (kg of steam per kg of cloth), if liquor ratio is 1:6.5; allowing 10%
margin for losses.

b) In a textile mill, a thermic fluid heater of 15 lakh kCal/hr capacity is meeting process
heat requirements.

The observed parameters of thermic fluid heater are:

Thermic fluid circulation rate - 100 m3/hr


Outlet temperature of fluid - 270 0C
Return temperature of fluid - 256 0C
Specific heat of fluid - 0.55 kCal/kg 0C
Density of fluid - 830 kg/m3
Present coal consumption - 300 kg /hr
GCV of coal - 3500 kCal/hr

i) What is the % loading of the thermic fluid heater?


ii) What is the existing thermal efficiency?
iii) The management is proposing to add a five chamber, (each chamber 1 Lakh kCal/hr
duty) to the heater. Will it be able to take the load?

Ans:

a)

9
_______________________
Bureau of Energy Efficiency
Paper 4 – Energy Auditor – Set B Key

Heat energy required / batch = 125 kg x 6.5 liq.ratio x 1.10 (including margin)
x (90 – 30)
= 53625 kCal/batch

= steam quantity needed / batch


= 53625/660
= 81.25 kg
Specific steam consumption = 81.25/125 = 0.65 kg/kg

b)

Ans:

i)
Heat duty = 100 m3/hr x 830 kg/m3 x 0.55 x 14
= 6,39,100 kCal/hr

% loading = 6,39,100/15,00,000
= 42.60 %

ii)
Input energy = 300 kg/hr x 3500 kCal
= 10,50,000 kCal/hr
Efficiency of the heater = 6,39,100/10,50,000

= 60.86%

iii)
New chamber heat duty = 5 x 1,00,000
= 5,00,000 kCal/hr

The total heat duty with new stenter loading = 6,39,100 + 5,00,000
= 11,39,100 kCal/hr

It is possible to comment new stenter since this is within 15 Lakh kCal/hr capacity of
the TFH

or

D) a) The following are the data obtained from the energy audit of a cement plant

Clinker analysis (loss free basis)

Constituents Determined SiO2 Fe2O3 Al2O3 CaO MgO


10
_______________________
Bureau of Energy Efficiency
Paper 4 – Energy Auditor – Set B Key

(%) 20.54 4.14 5.53 63.79 3.8

Preheater exit gas analysis

Oxygen (% v/v) CO CO2


(% v/v) (% v/v)
5.6 0.1 23.2

Preheater exit temperature – 328oC


Atomospheric pressure at site – 10.318 m
Static pressure – (-684 mmwg)

i) Calculate the heat of formation of clinker


ii) Calculate the density of preheater exit gases

b) An energy audit of a coal mill fan was carried out. It was observed that fan was
delivering 1,60,000 Nm3/hr of air at static pressure rise of 65 mm WC. The power
measurement of the 3-phase induction motor coupled with the fan recorded 20 kW/
phase on an average. The motor operating efficiency was assessed as 0.90 from the
motor performance curves. What would be the fan static efficiency?

Ans:

a)

i) Heat of formation of Clinker

 HR = 2.22Al2O3+6.48MgO+7.646CaO–5.116SiO2–0.59Fe2O3

= (2.22 x 5.53)+(6.48 x 3.8)+(7.646 x 63.79)–(5.116x 20.54)–(0.59 x 4.14)


= 417.11 kCal/kg Clinker

ii) Density of gases at preheater exit

(O2% x MW) + (CO2% x MW) + ((N2 + CO)% x MW)


 stp = kg/Nm3
22.4 x 100

(5.6 x 32) + (23.2 x 44) + (71.2 x 28)


 stp = kg/Nm3
22.4 x 100

11
_______________________
Bureau of Energy Efficiency
Paper 4 – Energy Auditor – Set B Key

= 1.4257 kg/Nm3

10318* −684S 273


t , p = 1.4257   kg/m3
10318 273 + 328e
(* as per the altitude of the plant above MSL)
10334 − 655 273
t , p = 1.436   kg/m3
10334 273 + 316
= 0.623 kg/m3

b)

Q = 1,60000 Nm3 / hr.= 44.44 m3/sec ,


SP = 65 mmWC,
St = ?,

Power input to 3 phase motor= 20 x 3 = 60 kW


Power input to fan shaft = 60 x 0.90 = 54 kW

Fan static  = Volume in m3/sec x Pst in mmWc x 100


102 x Power I/p to shaft
= 44.44 x 65 x 100
102 x 54
= 0.524 x 100
= 52.4%

-------- End of Section - III ---------

12
_______________________
Bureau of Energy Efficiency
Paper 4 – Energy Auditor – Set A Key

Regn No: _________________


Name: ___________________
(To be written by the candidates)

8th NATIONAL CERTIFICATION EXAMINATION


FOR
ENERGY AUDITORS – May, 2009

PAPER – 4: ENERGY PERFORMANCE ASSESSMENT FOR EQUIPMENT AND


UTILITY SYSTEMS

Date: 24.05.2009 Timings: 1400-1600 HRS Duration: 2 HRS Max. Marks: 100

General instructions:
o Please check that this question paper contains 6 printed pages
o Please check that this question paper contains 16 questions
o The question paper is divided into three sections
o All questions in all three sections are compulsory
o All parts of a question should be answered at one place

Section - I: SHORT DESCRIPTIVE QUESTIONS Marks: 10 x 1 = 10

(i) Answer all Ten questions


(ii) Each question carries One mark
(iii) Answer should not exceed 50 words

S-1 Between a thermal power plant and a cogeneration plant with a back pressure
turbine, which will have a higher heat rate?

Ans: A cogeneration plant will have a higher heat rate

S-2 Which loss is considered the most unreliable or complicated to measure in electric
motor efficiency testing?

Ans. The stray load loss, because this loss is only estimated and not measured

S-3 How is the Overall Heat transfer Coefficient related to surface area?

Ans Inversely proportional.

S-4 The inclined manometer connected to a pitot tube is sensing which pressure in a gas
stream?

1
_____________________
Bureau of Energy Efficiency
Paper 4 – Energy Auditor – Set A Key

Ans: The difference between total and static pressure (also called velocity pressure)

S-5 The more fouling fluid should be on which side of a shell & tube heat exchanger and
why?

Ans. Tube side, because it is easier to clean

S-6 For which fuel the sulphur dew point of the flue gases is lower: Furnace oil or Natural
gas.

Ans : Natural gas , because the sulphur content is less

S-7 What is the range of conversion efficiency of the gasification process using biomass?

Ans. 60 - 70 %

S-8 Calculate the annual power generated from a 100 kW wind turbine generator with a
capacity factor of 20% ?

Ans : 100 x .20 x 8760 = 175200 kWhr

S-9 Define Profitability Index?

Ans. Profitability Index is defined as the Sum of the discounted net savings to the
Capital Cost

S-10 In a power plant boiler if there is air ingress in the flue duct, which auxiliary system
would be most affected?

Ans: Induced Draft Fan

-------- End of Section - III ---------

Section - II: Long Questions Marks: 2 x 5 = 10

(i) Answer all Two questions


(ii) Each question carries Five marks

L-1 The suction head of a pump is 5 m below the pump centerline. The discharge
pressure is 4 kg/cm2. The flow rate of water is 100 m3 /hr. Find out the pump
efficiency if the actual power input at the shaft is 15 kW.

Ans: Discharge Head = 4 kg/cm2

= 40 metre head.
Suction Head = - 5 metre.
2
_____________________
Bureau of Energy Efficiency
Paper 4 – Energy Auditor – Set A Key

Total Head = 40 – (-5) = 45 metre

Hydraulic Power = (100/3600) x 1000 x 9.81 x 45/1000


= 12.26 kW
Pump Efficiency = 100 x 12.26/15 = 81.7 %

L-2. Calculate the blow down rate for a boiler with an evaporation rate of 3 tons/hr, if the
maximum permissible TDS in boiler water is 3000 ppm. The make up water addition
rate is 10 % and the feed water TDS is around 250 ppm.
Ans.
Feed water TDS x % Make up
Blow down (%) =
Permissible TDS in Boiler − Feed water TDS

250 X 10
Percentage blow down = = 0.91%
3000 − 250

If boiler evaporation rate is 3000 kg/hr then required blow down rate is:
3000 X 0.91
= = 27.3 kg / hr
100

OR

Blow down (%) =Feed water TDS x % Makeup/ Permissible TDS in Boiler
Percentage blow down = 250 x 10/ 3000 = 0.83%
If boiler evaporation rate is 3000 kg/hr then required blow down rate is:
= 3000 x .83/100 = 24.9 kg/hr

-------- End of Section - II ---------

Section - III: Numerical Questions Marks: 4 x 20 = 80

(i) Answer all Four questions


(ii) Each question carries Twenty marks

N-1 Government of India has undertaken various schemes to promote energy efficiency
in the country. In the last one year implementation of these schemes have resulted in
savings as given below:

S. Name of the scheme Energy Source Units Quantity Saved


No.
1 Energy Efficiency in Electricity from Million kWh 62
Buildings grid
3
_____________________
Bureau of Energy Efficiency
Paper 4 – Energy Auditor – Set A Key

2 Energy Efficiency in Electricity from Million kWh 1216


Industries Grid
Electricity from Million kWh 1000
Captive Diesel
Generation
Fuel Oil Lakh kilo 1.85
liter
Coal Lakh tons 3.5
Natural Gas Lakh Sm3 15728
3 Domestic Appliance Electricity from Million kWh 1374
Labelling Scheme grid
4 Various other schemes Electricity from Million kWh 635
as reported by different grid
States exclusive of GOI
schemes
Thermal Energy MTOE 80702
Saved

Given that:

1 kWhr = 860 kCal


GCV of Coal = 4000 kCal/ kg
GCV of Natural Gas = 8500 k Cal/Sm3
Assuming GCV of fuel Oil & Diesel = 10000 kCal/kg
Specific gravity of fuel Oil = 0.94, & for diesel 0.85
Assume average Transmission and Distribution Losses in India = 20%
Average Plant Load Factor = 78%
Specific diesel consumption = 3.5 kWh/ltr

Calculate : i) Total Energy Saved in MTOE (metric ton oil equivalent)


ii) Total Generation Capacity Avoided (in MW) in the country because of energy
savings due to grid connected power.

Ans: Total Electricity Saved from grid = 62 + 1216 + 1374 + 635 = 3287 million kWh

MTOE due to electrical savings = ( 3287 x 10 6 x 860 ) / 10 7


= 282682 MTOE

MTOE due to coal savings= ( 3.5 x 10 5 x 1000 x 4000 ) / 10 7

= 140000 MTOE

MTOE due to natural gas savings= ( 15728 x 10 5 x 8500 ) / 10 7

= 1336880 MTOE
MTOE due to Fuel oil savings = ( 1.85 x 10 x 0.94 x 1000 x 10000 ) / 10 7
5

= 173900 MTOE

MTOE due to captive diesel generation = 1000 x 106 x 0.85 x 10000 / 3.5 x 107
= 242857 MTOE

4
_____________________
Bureau of Energy Efficiency
Paper 4 – Energy Auditor – Set A Key

Total MTOE = 282682 + 140000 + 1336880 + 173900 + 80702 + 242857 = 2257021 MTOE

ii) Grid Electricity Saved = (62 +1216 + 1374 + 635 ) million k Wh =

= 3287 x 10 6 kWh

Therefore Avoided Capacity = 3287 x 10 6 /( 0.78 x .80 x 365 x 24 x 1000 )

= 601.3 MW

N-2 A pharma unit had installed a centralized refrigeration system of 120 TR Capacity
several years ago. The refrigeration system operators 24 hours a day, 200 days per
annum and the average electricity cost is Rs. 4.5/ kWh. The following are the key
operational parameters.

• Compressor operating current and power factor : 153 amps. 0.9 pf


• Condenser pump operating current and power factor: 43 amps, 0.88 pf
• Chiller pump operating current and power factor : 25 amps, 0.9 pf
• CT fan operating current and power factor : 20 amps. 0.65 pf
• ΔT across the chiller (evaporator) : 3.5OC
• Chilled water flow : 23 Lit / Sec
• Total head developed by chiller pump : 35 mtrs.
• Condenser water flow : 41 Lit / Sec
• Total head developed by condenser pump : 30 mtrs.

PS: all the motors operate at 415 Volts:

Calculate:
• The power consumed by the compressor, condenser pump, chiller pump and CT fan.
• TR developed by the system
• Specific power consumption i.e. overall kW/TR and COP and Energy Efficiency ratio
(EER)
• Combined efficiency (motor and pump) of condenser and chiller pumps

The unit proposes to replace the existing condenser and chilled water pumps with efficient
pumps having a combined efficiency of 65%. Also the unit goes in for condenser cleaning by
which the power consumption of compressor has reduced by 10%.

Calculate:
• The envisaged power consumption of the compressor, condenser and chiller pump
• Hourly energy savings (compressor, condenser and chilled water pump)
• Annual energy and equivalent monetary savings (compressor, condenser and chilled
water pump)
• Specific power consumption i.e. overall kW/ TR and COP and Energy Efficiency ratio
(EER)

Answer

Present Condition:

5
_____________________
Bureau of Energy Efficiency
Paper 4 – Energy Auditor – Set A Key

Compressor Power : 99 kW
Condenser Pump Power : 27.2 kW
Chiller Pump Power : 16.2 kW
CT Fan : 9.4 kW
Total Power : 151.8 kW
TR Devp : (23 * 3600 * 3.5 / 3024) = 95.83
Sp. Power : 1.58 kW/ TR
Compressor kW/TR : 99/95.83
: 1.03 kW/tR

COP : 3.516/kW/TR = 3.41


EER : 12/kW/TR
: 11.65
Condenser pump efficiency : 44.4%
Chiller pump efficiency : 48.8%

Proposed condition:

Compressor Power : 89 kW
Condenser Pump Power : 18.6 KW
Chiller Pump Power : 12.2 kW
CT Fan : 9.4 kW
Total Power : 129.2 kW
TR Devp : 95.83 TR i.e. (23 LPS * 3600 Sec * 3.5 / 3024)
Sp. Power : 1.35 kW/ TR
Compressor kW/TR : 89/95.83 = 0.93 kW/TR
COP : 3.516 / 0.93 = 3.78
EER : 12 / 0.93 = 12.90

Compressor Condenser Chiller


Pump Pump
Hourly energy savings – kWh (99-89) = 10 (27.2-18.6) = (16.2–12.2)
8.6 = 4.0
Annual energy savings – kWh (4800 hrs) 48000 41280 19200
Annual monetary savings (Rs) @ Rs. 4.5/ 216000 185760 86400
kWh

N-3 A fertilizer plant consuming 100TPH of saturated steam at 45 kg/sq.cm pressure has
been using Indian coal as fuel to the boiler and is now switching over to Imported
coal.
Typical ultimate analysis of the two types of coals:

------------------------------------------------------------------------------------------------
Parameters Indian coal Imported coal
% %
-------------------------------------------------------------------------------------------------
Carbon 41.11 58.96
Hydrogen 2.76 4.16
Nitrogen 1.22 1.02
Oxygen 9.89 11.88

6
_____________________
Bureau of Energy Efficiency
Paper 4 – Energy Auditor – Set A Key

Sulphur 0.41 0.56


Moisture 5.98 9.43
Ash 38.63 13.99
---------------------------------------------------------------------------------------------------
GCV (kCal/kg) 4,000 5,900

Determine:

(i) Coal requirement in each case


(ii) Calculate % dry flue gas losses in both cases

Assume: in both cases


Flue gas temperature = 200oC
Ambient temperature = 30oC
Enthalpy of steam = 668 kCal/kg
Feed water temperature = 80oC
Specific heat of flue gases = 0.23

Boiler efficiency with Indian coal = 75%


Boiler efficiency with Imported coal = 82%
Oxygen content in flue gases with Indian coal = 10%
Oxygen content in flue gases with Imported coal = 4%

Answer
Coal requirement

Steam (q) x (hg – hf)


Q=
Efficiency x GCV

Indian coal
Imported coal
100,000 x (668 – 80)
Q= 100,000 x (668 – 80)
0.75 x 4000 Q=
0.82 x 5900
19.6 T/hr
12.1 T/hr
Find theoretical air requirement

Indian coal
= [(11.6 x C) + {34.8 x(H2 - O2/8)} + (4.35 x S)] kg / kg of coal
100
7
_____________________
Bureau of Energy Efficiency
Paper 4 – Energy Auditor – Set A Key

For Indian coal


= [(11.6 x 41.11) + {34.8 x(2.76 – 9.89/8)} + (4.35 x 0.41)] = 5.3 kg / kg of coal
100

For Imported Coal

= [(11.6 x 58.96) + {34.8 x(4.16 – 11.88/8)} + (4.35 x .56)] = 7.79 kg / kg of coal


100

Excess air percentage in Indian coal = 10 x 100/ 21 - 10 = 90.9%

Excess air percentage in Imported coal = 4 x 100/ 21 – 4 = 23.5%

Actual mass of air (AAS)

Kg/ kg of coal = 5.3 x 1.9 = 10.07 kg (Indian coal)

= 7.79 x 1.235= 9.62 kg (Imported coal)

Heat loss in dry flue gas = M x CP (Tf – Ta) x 100


GCV
- Indian coal = (10.07+1) x 0.23 x (200 – 30) x 100
4000

= 10.8%

- Imported coal = (9.62+1) x 0.23 x (200 – 30) x 100


5900
= 7.04%
Alternatively:

Indian Coal : Mass of dry flue gas = Mass of (CO2 + SO2 + N2 + O2) in flue gas
+ N2 in air we supply
= 0.4111x44 + 0.0041x64 + 0.0122 + 10.07x77 + {(10.07-5.3)x23/100}
12 32 100
= 10.38 kg/kg of fuel.

Imported Coal:
= 0.5896x44 + 0.0056x64 + 0.0102 + 9.62x77 + {(9.62- 7.79)x23/100}
12 32 100
= 10.01 kg/kg of fuel.

Heat loss dry flue gas:


Indian Coal: = 10.38 x 0.23 x (200 – 30) x 100 = 10.1%
4000
8
_____________________
Bureau of Energy Efficiency
Paper 4 – Energy Auditor – Set A Key

Imported Coal: = 10.01 x 0.23 x (200 – 30) x 100 = 6.63%


5900

N-4 The cost and estimated savings data for an energy saving retrofit project is given in
table below.

Retrofit cost Energy & demand Maintenance cost savings


savings
Rs. 1,00,000 6000 kWh/year & Annual maintenance cost savings
Rs.3800/year as will be Rs. 2000/-.
demand charges

• The key data is given below:


• Energy savings are based on Rs 3.00/kWh
• No changes in energy rates for 10 years
• The project has a 10 year life period
Calculate NPV for the upgrade option against 12% discount rate.

Cost of Energy & demand savings per year = 6000 x 3 + 3800 = Rs 21,800
Ans
The cash flow detail for ten year duration is given below:

Year Retrofi Cost of Maintenan Cash Discou Present


t cost Energy & ce cost flow nt Value
demand savings Factor
savings for 12%
0 100000 0 0 -100000 1.000 -100000
1 0 21800 +2000 23800 0.893 21253.4
2 0 21800 +2000 23800 0.797 18968.6

3 0 21800 +2000 23800 0.712 16945.6

4 0 21800 +2000 23800 0.636 15136.8

5 0 21800 +2000 23800 0.567 13494.6

6 0 21800 +2000 23800 0.507 12066.6

7 0 21800 +2000 23800 0.452 10757.6

8 0 21800 +2000 23800 0.404 9615.2

9 0 21800 +2000 23800 0.361 8591.8

10 0 21800 +2000 23800 0.322 7663.6

9
_____________________
Bureau of Energy Efficiency
Paper 4 – Energy Auditor – Set A Key

Total 20,000 1,38,000 34,493.


8

The NPV for the upgrade option against 12% interest rate = Rs 34,494/-

-------- End of Section - III ---------

10
_____________________
Bureau of Energy Efficiency
Paper 4 – Energy Auditor – Set B Key

Regn No: _________________


Name: ___________________
(To be written by the candidates)

8th NATIONAL CERTIFICATION EXAMINATION


FOR
ENERGY AUDITORS – May, 2009

PAPER – 4: ENERGY PERFORMANCE ASSESSMENT FOR EQUIPMENT AND


UTILITY SYSTEMS

Date: 24.05.2009 Timings: 1400-1600 HRS Duration: 2 HRS Max. Marks: 100

General instructions:
o Please check that this question paper contains 6 printed pages
o Please check that this question paper contains 16 questions
o The question paper is divided into three sections
o All questions in all three sections are compulsory
o All parts of a question should be answered at one place

Section - I: SHORT DESCRIPTIVE QUESTIONS Marks: 10 x 1 = 10

(i) Answer all Ten questions


(ii) Each question carries One mark
(iii) Answer should not exceed 50 words

S-1 Between a thermal power plant and a cogeneration plant with a back pressure
turbine, which will have a higher heat rate?

Ans: A cogeneration plant will have a higher heat rate

S-2 Which loss is considered the most unreliable or complicated to measure in electric
motor efficiency testing?

Ans. The stray load loss, because this loss is only estimated and not measured

S-3 How is the Overall Heat transfer Coefficient related to surface area?

Ans Inversely proportional.

S-4 The inclined manometer connected to a pitot tube is sensing which pressure in a gas
stream?

1
_____________________
Bureau of Energy Efficiency
Paper 4 – Energy Auditor – Set B Key

Ans: The difference between total and static pressure (also called velocity pressure)

S-5 The more fouling fluid should be on which side of a shell & tube heat exchanger and
why?

Ans. Tube side, because it is easier to clean

S-6 For which fuel the sulphur dew point of the flue gases is lower: Furnace oil or Natural
gas.

Ans : Natural gas , because the sulphur content is less

S-7 What is the range of conversion efficiency of the gasification process using biomass?

Ans. 60 - 70 %

S-8 Calculate the annual power generated from a 100 kW wind turbine generator with a
capacity factor of 20% ?

Ans : 100 x .20 x 8760 = 175200 kWhr

S-9 Define Profitability Index?

Ans. Profitability Index is defined as the Sum of the discounted net savings to the
Capital Cost

S-10 In a power plant boiler if there is air ingress in the flue duct, which auxiliary system
would be most affected?

Ans: Induced Draft Fan

-------- End of Section - III ---------

Section - II: Long Questions Marks: 2 x 5 = 10

(i) Answer all Two questions


(ii) Each question carries Five marks

L-1. Calculate the blow down rate for a boiler with an evaporation rate of 4 tons/hr, if the
maximum permissible TDS in boiler water is 3000 ppm. The make up water addition
rate is 10 % and the feed water TDS is around 250 ppm.
Ans.
Feed water TDS x % Make up
Blow down (%) =
Permissible TDS in Boiler − Feed water TDS

2
_____________________
Bureau of Energy Efficiency
Paper 4 – Energy Auditor – Set B Key

250 X 10
Percentage blow down = = 0.91%
3000 − 250

If boiler evaporation rate is 4000 kg/hr then required blow down rate is:
= 4000 x 0.91/ 100

= 36.4 kg/hr

OR

Blow down (%) =Feed water TDS x % Makeup/ Permissible TDS in Boiler
Percentage blow down = 250 x 10/ 3000 = 0.83%
If boiler evaporation rate is 4000 kg/hr then required blow down rate is:
= 4000 x .83/100 = 33.2 kg/hr

L-2 The suction head of a pump is 5 m below the pump centre line. The discharge
pressure is 3 kg/cm2. The flow rate of water is 100 m3 /hr. Find out the pump
efficiency if the actual power input at the shaft is 15 kW.

Ans: Discharge Head = 3 kg/cm2

= 30 metre head.
Suction Head = - 5 metre.
Total Head = 30 – (-5) = 35 metre

Hydraulic Power = (100/3600) x 1000 x 9.81 x 35/1000


= 9.53 kW
Pump Efficiency = 100 x 9.53 / 15 = 63.5 %

-------- End of Section - II ---------

Section - III: Numerical Questions Marks: 4 x 20 = 80

(i) Answer all Four questions


(ii) Each question carries Twenty marks

N-1 A fertilizer plant consuming 100TPH of saturated steam at 45 kg/sq.cm pressure has
been using Indian coal as fuel to the boiler and is now switching over to imported
coal.

3
_____________________
Bureau of Energy Efficiency
Paper 4 – Energy Auditor – Set B Key

Typical ultimate analysis of the two types of coals:

------------------------------------------------------------------------------------------------
Parameters Indian coal Imported coal
% %
-------------------------------------------------------------------------------------------------
Carbon 41.11 58.96
Hydrogen 2.76 4.16
Nitrogen 1.22 1.02
Oxygen 9.89 11.88
Sulphur 0.41 0.56
Moisture 5.98 9.43
Ash 38.63 13.99
---------------------------------------------------------------------------------------------------
GCV (kCal/kg) 4,000 5,900

Determine:

(i) Coal requirement in each case


(ii) Calculate % dry flue gas losses in both cases

Assume: in both cases


Flue gas temperature = 200oC
Ambient temperature = 30oC
Enthalpy of steam = 668 kCal/kg
Feed water temperature = 80oC
Specific heat of flue gases = 0.23

Boiler efficiency with Indian coal = 70%


Boiler efficiency with Imported coal = 75%
Oxygen content in flue gases with Indian coal = 10%
Oxygen content in flue gases with Imported coal = 4%

Answer
Coal requirement

Steam (q) x (hg – hf)


Q=
Efficiency x GCV

4
_____________________
Bureau of Energy Efficiency
Paper 4 – Energy Auditor – Set B Key

Indian coal
Imported coal
Q= 100,000 x (668 – 80)
100,000 x (668 – 80)
0.70 x 4000 Q=
0.75 x 5900
21.0 T/hr
13.28 T/hr

Find theoretical air requirement

= [(11.6 x C) + {34.8 x(H2 - O2/8)} + (4.35 x S)] kg / kg of coal


100

For Indian coal


= [(11.6 x 41.11) + {34.8 x(2.76 – 9.89/8)} + (4.35 x 0.41)] = 5.3 kg / kg of coal
100

For Imported Coal

= [(11.6 x 58.96) + {34.8 x(4.16 – 11.88/8)} + (4.35 x .56)] = 7.79 kg / kg of coal


100

Excess air percentage in Indian coal = 10 x 100/ 21 - 10 = 90.9%

Excess air percentage in Imported coal = 4 x 100/ 21 – 4 = 23.5%

Actual mass of air (AAS)

Kg/ kg of coal = 5.3 x 1.9 = 10.07 kg (Indian coal)

= 7.79 x 1.235= 9.62 kg (Imported coal)

Heat loss in dry flue gas = M x CP (Tf – Ta) x 100


GCV
- Indian coal = (10.07+1) x 0.23 x (200 – 30) x 100
4000

= 10.8%

- Imported coal = (9.62+1) x 0.23 x (200 – 30) x 100


5900
= 7.04%

Alternatively:

Indian Coal : Mass of dry flue gas = Mass of (CO2 + SO2 + N2 + O2) in flue gas
5
_____________________
Bureau of Energy Efficiency
Paper 4 – Energy Auditor – Set B Key

+ N2 in air we supply

= 0.4111x44 + 0.0041x64 + 0.0122 + 10.07x77 + {(10.07-5.3)x23/100}


12 32 100

= 10.38 kg/kg of fuel.

Imported Coal:

= 0.5896x44 + 0.0056x64 + 0.0102 + 9.62x77 + {(9.62- 7.79)x23/100}


12 32 100

= 10.01 kg/kg of fuel.

Heat loss dry flue gas:

Indian Coal: = 10.38 x 0.23 x (200 – 30) x 100 = 10.1%


4000

Imported Coal: = 10.01 x 0.23 x (200 – 30) x 100 = 6.63%


5900

N-2 The cost and estimated savings data for an energy saving retrofit project is given in
table below.

Retrofit cost Energy & demand Maintenance cost savings


savings
Rs. 1,20,000 6000 kWh/year & Annual maintenance cost savings
Rs.3800/year as will be Rs. 2000/-.
demand charges

• The key data is given below:


• Energy savings are based on Rs 3.00/kWh
• No changes in energy rates for 10 years
• The project has a 10 year life period

Calculate NPV for the upgrade option against 12% discount rate.

6
_____________________
Bureau of Energy Efficiency
Paper 4 – Energy Auditor – Set B Key

Cost of Energy & demand savings per year = 6000 x 3 + 3800 = Rs 21,800
Ans
The cash flow detail for ten year duration is given below:

Year Retrofit Cost of Maintenan Cash Discount Present


cost Energy & ce cost flow Factor Value
demand savings for 12%
savings
0 120000 0 0 -120000 1.000 -120000
1 0 21800 +2000 23800 0.893 21253.4
2 0 21800 +2000 23800 0.797 18968.6

3 0 21800 +2000 23800 0.712 16945.6

4 0 21800 +2000 23800 0.636 15136.8

5 0 21800 +2000 23800 0.567 13494.6

6 0 21800 +2000 23800 0.507 12066.6

7 0 21800 +2000 23800 0.452 10757.6

8 0 21800 +2000 23800 0.404 9615.2

9 0 21800 +2000 23800 0.361 8591.8

10 0 21800 +2000 23800 0.322 7663.6

Total 20,000 1,38,000 14,493.8

The NPV for the upgrade option against 12% interest rate = Rs 14,494/-

N-3 A pharma unit had installed a centralized refrigeration system of 120 TR Capacity
several years ago. The refrigeration system operators 24 hours a day, 300 days per
annum and the average electricity cost is Rs. 4.5/ kWh. The following are the key
operational parameters.

• Compressor operating current and power factor : 153 amps. 0.9 pf


• Condenser pump operating current and power factor: 43 amps, 0.88 pf
• Chiller pump operating current and power factor : 25 amps, 0.9 pf
• CT fan operating current and power factor : 20 amps. 0.65 pf
• ΔT across the chiller (evaporator) : 3.5OC
• Chilled water flow : 23 Lit / Sec
• Total head developed by chiller pump : 35 mtrs.
• Condenser water flow : 41 Lit / Sec
• Total head developed by condenser pump : 30 mtrs.

PS: all the motors operate at 415 Volts:

7
_____________________
Bureau of Energy Efficiency
Paper 4 – Energy Auditor – Set B Key

Calculate:
• The power consumed by the compressor, condenser pump, chiller pump and CT fan.
• TR developed by the system
• Specific power consumption i.e. overall kW/TR and COP and Energy Efficiency ratio
(EER)
• Combined efficiency (motor and pump) of condenser and chiller pumps

The unit proposes to replace the existing condenser and chilled water pumps with efficient
pumps having a combined efficiency of 65%. Also the unit goes in for condenser cleaning by
which the power consumption of compressor has reduced by 10%.

Calculate:
• The envisaged power consumption of the compressor, condenser and chiller pump
• Hourly energy savings (compressor, condenser and chilled water pump)
• Annual energy and equivalent monetary savings (compressor, condenser and chilled
water pump)
• Specific power consumption i.e. overall kW/ TR and COP and Energy Efficiency ratio
(EER)

Answer

Present Condition:

Compressor Power : 99 kW
Condenser Pump Power : 27.2 kW
Chiller Pump Power : 16.2 kW
CT Fan : 9.4 kW
Total Power : 151.8 kW
TR Devp : (23 * 3600 * 3.5 / 3024) = 95.83
Sp. Power : 1.58 kW/ TR
Compressor kW/TR : 99/95.83 = 1.03 kW/TR

COP : 3.516 / 1.03 = 3.41


EER : 12 / 1.03 = 11.65
Condenser pump efficiency : 44.4%
Chiller pump efficiency : 48.8%

Proposed condition:

Compressor Power : 89 kW
Condenser Pump Power : 18.6 KW
Chiller Pump Power : 12.2 kW
CT Fan : 9.4 kW
Total Power : 129.2 kW
TR Devp : 95.83 TR i.e. (23 LPS * 3600 Sec * 3.5 / 3024)
Sp. Power : 1.35 kW/ TR
Compressor kW/TR : 89/95.83 = 0.93 kW/TR

COP : 3.516 / 0.93 = 3.78


EER : 12 / 0.93 = 12.90

8
_____________________
Bureau of Energy Efficiency
Paper 4 – Energy Auditor – Set B Key

Compressor Condenser Chiller Pump


Pump
Hourly energy savings – kWh (99-89) = 10 (27.2-18.6) = (16.2–12.2) =
8.6 4.0
Annual energy savings – kWh (7200 hrs) 72000 61920 28800
Annual monetary savings (Rs) @ Rs. 4.5/ 324000 278640 129600
kWh

N-4 Government of India has undertaken various schemes to promote energy efficiency
in the country. In the last one year implementation of these schemes have resulted in
savings as given below:

S. Name of the scheme Energy Source Units Quantity Saved


No.
1 Energy Efficiency in Electricity from Million kWh 62
Buildings grid
2 Energy Efficiency in Electricity from Million kWh 1216
Industries Grid
Electricity from Million kWh 1000
Captive Diesel
Generation
Fuel Oil Lakh kilo 1.85
liter
Coal Lakh tons 3.5
Natural Gas Lakh Sm3 15728
3 Domestic Appliance Electricity from Million kWh 1374
Labelling Scheme grid
4 Various other schemes Electricity from Million kWh 635
as reported by different grid
States exclusive of GOI
schemes
Thermal Energy MTOE 80702
Saved

Given that:

1 kWhr = 860 kCal


GCV of Coal = 4000 kCal/ kg
GCV of Natural Gas = 8500 k Cal/Sm3
Assuming GCV of fuel Oil & Diesel = 10000 kCal/kg
Specific gravity of fuel Oil = 0.94, & for diesel 0.85
Assume average Transmission and Distribution Losses in India = 20%
Average Plant Load Factor = 70%
Specific diesel consumption = 3.5 kWh/ltr

Calculate : i) Total Energy Saved in MTOE (metric ton oil equivalent)


ii) Total Generation Capacity Avoided (in MW) in the country because of energy
savings due to grid connected power.

Ans: Total Electricity Saved from grid = 62 + 1216 + 1374 + 635 = 3287 million kWh
9
_____________________
Bureau of Energy Efficiency
Paper 4 – Energy Auditor – Set B Key

MTOE due to electrical savings = ( 3287 x 10 6 x 860 ) / 10 7


= 282682 MTOE

MTOE due to coal savings= ( 3.5 x 10 5 x 1000 x 4000 ) / 10 7

= 140000 MTOE

MTOE due to natural gas savings= ( 15728 x 10 5 x 8500 ) / 10 7

= 1336880 MTOE
MTOE due to Fuel oil savings = ( 1.85 x 105 x 0.94 x 1000 x 10000 ) / 10 7

= 173900 MTOE

MTOE due to captive diesel generation = 1000 x 106 x 0.85 x 10000 / 3.5 x 107
= 242857 MTOE

Total MTOE = 282682 + 140000 + 1336880 + 173900 + 80702 + 242857 = 2257021 MTOE

ii) Grid Electricity Saved = (62 +1216 + 1374 + 635 ) million k Wh =

= 3287 x 10 6 kWh

Therefore Avoided Capacity = 3287 x 10 6 / ( 0.70 x .80 x 365 x 24 x 1000 )

= 670 MW

-------- End of Section - III ---------

10
_____________________
Bureau of Energy Efficiency
Paper 4 – Energy Auditor – Set A Solutions
Regn No: _________________
Name: ___________________
(To be written by the candidate)

9TH NATIONAL CERTIFICATION EXAMINATION– December, 2009


FOR
ENERGY AUDITORS

PAPER – 4: Energy Performance Assessment For Equipment and Utility


Systems

Date: 20.12.2009 Timings: 1400-1600 HRS Duration: 2 HRS Max. Marks: 100

General instructions:
o Please check that this question paper contains 8 printed pages
o Please check that this question paper contains 16 questions
o The question paper is divided into three sections
o All questions in all three sections are compulsory
o All parts of a question should be answered at one place

Section - I: SHORT DESCRIPTIVE QUESTIONS Marks: 10 x 1 = 10

(i) Answer all Ten questions


(ii) Each question carries One mark

S-1 A cogeneration plant with a back pressure turbine has a constant steam demand and
fluctuating power demand. What is the common option to meet the fluctuating power
demand?

Ans
Parallel operation with grid
S-2 What are the two major sources of waste heat available from a water-cooled Diesel
Generator set?

Ans
Exhaust flue gases and jacket cooling water
S-3 For determining heat loss in flue gases due to incomplete combustion which flue gas
constituent needs to be measured?

Ans
Carbon monoxide

1
_______________________
Bureau of Energy Efficiency
Paper 4 – Energy Auditor – Set A Solutions

S-4 Which parameter needs to be measured to assess the percentage loading of a motor
by slip method neglecting voltage correction?

Ans
Motor speed
S-5 How many volt-amperes (VA) does a 100 Watt incandescent light require?

Ans
100 VA
S-6
In the indirect method of boiler efficiency evaluation, list any two additional losses
computed for solid fuel fired boilers as compared to liquid and gas fired boilers?
Ans.
Unburnt losses in fly ash (Carbon)
Unburnt losses in bottom ash (Carbon)

S-7 Why do biomass combustion projects qualify for CDM benefits even though they emit
carbon dioxide?

Ans:
Because it absorbs the same amount of carbon in growing as it releases when
consumed as fuel
Or
Biomass is carbon neutral

S-8 Name two most common bio fuels used for transportation

Ans
1. Biodiesel
2. Ethanol
S-9 Which loss is assumed in the determination of electric motor efficiency?

Ans
The stray load loss is estimated and not measured for testing electric motor
efficiency.

S-10 In a shell and tube heat exchanger, engaged in heat transfer between fouling fluid and
clear fluid, the fouling fluid should be put on shell side or tube side?

Ans
Tube side

…………. End of Section - I ………….

2
_______________________
Bureau of Energy Efficiency
Paper 4 – Energy Auditor – Set A Solutions

Section - II: LONG DESCRIPTIVE QUESTIONS Marks: 2 x 5 = 10

(i) Answer all Two questions


(ii) Each question carries Five marks

L-1 The steam flow to a process plant is 5000 kg/hr. 2000 kg/hr of condensate at 173oC is
returned to boiler feed water tank due to its own pressure. However there is 10% heat
loss in transit to boiler feed tank. The balance is made up as feed water at 30oC. The
final feed water temperature observed was 95oC. Comment on the feed water
temperature. The plant personnel are sure that the temperature gauge is in order.

Ans Heat in condensate (less 10%) + Heat in make up water = Heat in feed water

2000 X 173 X 0.9 + 3000 X 30 = 5000 X Tf


311400 + 90000 = 5000 X Tf

Tf = 80.28oC
(3 Marks)

The feed water temperature cannot be more than 80.28oC. The high temperature of
95oC is mostly due to live steam leakage, as the temperature gauge is OK.
(2 Marks)

L-2 The maximum demand registered by an automobile plant is 5000 KVA and the power
factor is 0.95. The plant management converts the existing electrical resistance heated
furnace with an average load of 750 kW to gas heating as a cost reduction measure.
What will be effect on maximum demand and power factor with this conversion?

Ans. Registered maximum demand = 5000 KVA


Electrical load (real power) = 5000 X 0.95
= 4750 KW
KVAR2 = KVA2 – KW2
KVAR2 = (5000)2 – (4750)2

KVAR = 100 (502) – (47.5)2


= 1561
(3 Marks)
KVAR in the system will remain same.
Reduction in real power by conversion is 750 KW.
Real Power = 4750 – 750 = 4000 KW

KVA2 = KW2 + KVAR2


= (4000)2 + (1561)2
KVA = 4294

Power factor = 4000 / 4294 = 0.932

Reduction in Electrical Demand = 5000 – 4294


= 706 KVA

3
_______________________
Bureau of Energy Efficiency
Paper 4 – Energy Auditor – Set A Solutions
Reduction in Power Factor = 0.95 – 0.932
= 0.018
(2 Marks)

…………. End of Section - II ………….

Section - III: NUMERICAL QUESTIONS Marks: 4 x 20 = 80

(i) Answer all Four questions


(ii) Each question carries Twenty marks

N -1 Given below is a set of curves for a centrifugal fan. At its Best Efficiency Point (BEP)
determine to the nearest approximation the following:

a) Static pressure in mmwc


b) Flow in m3/hr
c) Shaft power in kW
d) Work out the static efficiency of the fan by calculation
e) Power drawn by the motor if the motor operating efficiency is 90%

Shaft power (HP)

Ans
Static pressure in inches H2O 36
(2 Marks)

Static pressure in mmwc 914.4 mmwc


(2 Marks)
Flow in CFM 2750

4
_______________________
Bureau of Energy Efficiency
Paper 4 – Energy Auditor – Set A Solutions
(2 Marks)
Flow in m3/hr 4672.28 m3/hr ie 1.3 m3/sec
(2 Marks)
Shaft power in hp 29
(2 Marks)
Shaft power in kW 21.6 kW
(2 Marks)
Fan Efficiency Q x dP/(102 x kW)
1.3 x 914.4 /(102 x 21.6)
54 %
(5 Marks)
Motor input power at 90% 21.6 / 0.9
= 24 kW
(3 Marks)
The data is read from the graph and hence a deviation of +/- 10% in values may be
given full marks.

N-2 In a Continuous Process Industry 5 Tonne per hour hot oil on process stream has to be
cooled from 230oC to 110oC by DM water at 25oC heated to 80oC on its route to boiler
de-arator.
(i) Depict the heat exchange process on a schematic for both parallel and counter flow heat
exchanger clearly indicating inlet and outlet temperature and terminal temperature
difference.
(ii) Find out LMTD for parallel and counter flow heat exchangers and comment on the
preference of the heat exchanger.

(iii) Find out the DM water flow rate through the heat exchanger. Assume specific heat of hot
oil to be 0.5 kCal / kgoC.

Ans.

Hot Oil Hot Oil


230oC 110 Co
230 Co
110oC
Parallel Flow
TTD TTD TTD Counter Flow TTD
t1 = 205 t2 = 30 t1 = 150oC t2 = 85

25oC 80oC 80oC 25oC


Cold DM Water Cold DM Water

t1 - t2
LMTD parallel flow = ------------------------
Ln t1 / t2

5
_______________________
Bureau of Energy Efficiency
Paper 4 – Energy Auditor – Set A Solutions
205 - 30
= ------------------ = 91.15oC
Ln 205 / 30
(8 Marks)

t1 - t2
LMTD counter-flow = ------------------------
Ln t1 / t2

150 - 85
= ------------------ = 114.44oC
Ln 150 / 85

(8 Marks)

Counter flow heat exchanger is preferred as the LMTD is larger and hence heat
exchanger area will be less and compact.

Mc X 1 X (80 – 25) = mh X Cph X (230 – 110)


Mc = 5000 X 0.5 X (230 – 110) / 55
= 5454.54 Kg / hr

DM water flow rate = 5454.54 Kg / hr


(4 Marks)

N-3 The following are the data collected for a boiler using furnace oil as the fuel. Determine
the boiler efficiency based on GCV by indirect method ignoring radiation and convection
losses.
Ultimate chemical analysis (% weight) : Carbon : 84, Hydrogen : 12, Nitrogen : 0.5,
Oxygen : 1.5, Sulphur : 1.5, Moisture : 0.5, GCV of fuel 10,397 kCal/kg and humidity
0.015 kg moisture/kg of dry air.
Flue gas analysis: CO2 : 12% volume, flue gas temperature : 180oC and ambient
temperature : 20oC

Ans:
a) Theoretical Air = 11.43 x 0.84 + 34.5 x (0.12 – 0.015/8) + 4.32 x 0.015 = 13.74

(or)

Theoretical Air = 11.6 x 0.84 + 34.8x (0.12 – 0.015/8) + 4.35 x 0.015 = 13.92
(2 Marks)

b) Theoretical % CO2 = Moles of C/(Moles of N2 + Moles of C + Moles of Sulphur)


= 100 x (0.84/12)/(13.74 x 0.77/28 + 0.005/28 +0.84/12 + 0.015/32) = 15.62%
(or)

6
_______________________
Bureau of Energy Efficiency
Paper 4 – Energy Auditor – Set A Solutions

Theoretical % CO2 = Moles of C/(Moles of N2 + Moles of C + Moles of Sulphur)


= 100 x (0.84/12)/(13.92 x 0.77/28 + 0.005/28 +0.84/12 + 0.015/32) = 15.44%

(2 Marks)
Depending on the variation in the above values (a and b) the subsequent calculations will
also have a minor variation in the end result, which can be ignored.

c) Excess air supplied = 7900 x(15.62 -12)/(12(100-15.62)) = 28.24%


(2 Marks)

d) Actual mass of air supplied = (1+0.2824) x 13.74 = 17.62 kg/kg of oil.


(2 Marks)
e) Actual mass of dry flue gas =
0.84 x 44/12 + 0.005 + 17.62 x 0.77 + 0.015 x 64/32 + 3(17.62-13.74) x 0.23
= 19.3596 kg/kg oil
(2 Marks)
f) % loss by dry flue gas = 19.3596 x 0.23 x (180-20)/10,397 = 0.0685 or 6.85%
(2 Marks)

g) % loss by formation of water from Hydrogen in the fuel


9 x 0.12 x (584 + 0.45 (180-20))/10,397 = 0.681 or 6.81%
(2 Marks)

h) % loss due to moisture in fuel = 0.005 x (584 +0.45 (180-20))/10,397 = 0.03%


(2 Marks)

i) % loss due to moisture in air = 17.62 x 0.015 x 0.45 (180-20)/10,397 = 0.183%


(2 Marks)

j) Boiler efficiency = 100- 6.85 – 6.81 – 0.03 – 0.183 = 86.127%.


(2 Marks)

N-4 A process plant is installing a 5 MW gas turbine cogeneration system with 12 TPH waste
heat boiler to meet the power and steam demand of the plant. The plant will operate at
90% of capacity, meeting the entire power requirement of the plant, which is presently
drawn from grid supply. The co-gen plant will also meet the steam requirement of 10
TPH, which is presently generated in a gas fired boiler with 86% efficiency on N.C.V.
basis. Calculate the differential cost between cogenerated power and grid power per unit
and also the additional natural gas requirement per day based on the following data.

Capacity of gas turbine = 5000 kW


Plant load factor = 90%
Auxiliary power consumption = 1%
Operating hrs. per annum = 8000
Net calorific value of natural gas = 9500 kCal / Sm3
Cost of natural gas = Rs.8 / Sm3
Steam produced by co-gen waste heat = 10 TPH boiler
Annual expenditure towards depreciation
and interest = Rs 500 lacs
Annual expenditure for operation &
7
_______________________
Bureau of Energy Efficiency
Paper 4 – Energy Auditor – Set A Solutions
maintenance of co-gen plant = Rs 200 lacs
Heat Rate of gas turbine on NCV = 3050 kCal / kWh
Cost of electric power from grid supply = Rs. 4.5/ kWh
Enthalpy of steam = 665 kCal/ kg
Feed water temperature = 85oC

Ans Power generation from cogen plant = 5000X 0.9 X 8000 = 360 lac Kwh
Auxiliary power = 1%
Net power generation = 0.99 X 360 = 356.4 lac Kwh
Natural gas requirement for = 360 X 3050 / 9500 = 115.57 lac Sm3
power generation
(4 Marks)
Cost of fuel per annum = 115.57 X 8 = Rs.924.56 lacs
Annual expenditure for interest, = 500 + 200 = 700 lacs
depreciation and O&M
Total cost of generation = Rs.1624.56 lacs.

Cost of cogeneration power =1624.56 X 105 / 356.4 X 105

=Rs.4.56 / Kwh.
(4 Marks)
Gas consumption in existing gas = [10000 (665 – 85) / (0.86 X 9500)]
fired boiler = 710 Sm3/hr
= 17040 Sm3/day

Cost of steam = 710*Rs. 8/10


= Rs. 568/Ton
Annual savings by avoiding steam= 10 X 568 X 8000
In the existing gas fired Boiler = Rs. 454.4 Lacs.

Cost of generation after giving = 1624.56 – 454.4 = Rs.1170.16 lacs


credit for steam generation
Cost of generation after accounting = 1170.16 X 105 / 356.4 X 105
for steam cost
= Rs. 3.28 / Kwh
(5 Marks)
Grid power cost = Rs. 4.5 / Kwh
Cost advantage for cogen plant = 4.5 – 3.28 = Rs.1.22 / Kwh
generation

Daily gas requirement for operating = 5000 X 0.9 X 3050 X 24


GT cogen plant 9500
= 34673.68 Sm3 / day
(3 Marks)
Additional gas requirement for = 34673.68 – 17040 = 17633.68 Sm3/day
co-gen plant
(4 Marks)
-------- End of Section - III ---------

8
_______________________
Bureau of Energy Efficiency
Paper 4 – Set A, Energy Auditor Key

Regn No: _________________


Name: ___________________
(To be written by the candidates)

7th NATIONAL CERTIFICATION EXAMINATION


FOR
ENERGY AUDITORS – Nov., 2008

PAPER – 4: Energy Performance Assessment for Equipment and Utility


Systems
Date: 23.11.2008 Timings: 1400-1600 HRS Duration: 2 HRS Max. Marks: 100

Section - I: SHORT DESCRIPTIVE QUESTIONS Marks: 10 x 1 = 10

(i) Answer all Ten questions


(ii) Each question carries One mark
(iii) Answer should not exceed 50 words

S-1 What will be the synchronous speed of a VFD driven 4-pole induction motor operating at
38 Hz ?

Ans. Ns = 120 x f/P


= 120 x 38/4
= 1140 RPM

S-2 If the power consumed by a refrigeration compressor is 2 kW per ton of refrigeration, what
is the energy efficiency ratio?

Ans.
12000 Btu
EER = 2000 W = 6

S-3 Explain why heat rate of back pressure turbine is greater than that of a condensing
turbine.

Ans. As it does not take into account of the heat content of the exhaust steam used in the
process.

S-4 Why line current method used for estimating loading of a motor is not applicable for motor
loading less than 75%.

Ans. At lower loadings, power factor of a motor degrades significantly and ampere-load curve
becomes nonlinear

S-5 Explain why actual air delivered is always converted to (FAD) while measuring delivered
air volume flow rates in an air compressor.

Ans. As air is compressible, its volume flow rate will vary with pressure on delivery side and
hence for comparison purposes the volume flow rates are always converted to their value
at standard atmospheric pressure.

1
_________________________
Bureau of Energy Efficiency
Paper 4 – Set A, Energy Auditor Key

S-6 What is the minimum wind speed which is acceptable for viable power generation from a
wind turbine?

Ans. 15 kmph

S-7 If the dry bulb temperature of air is 35oC and the wet bulb temperature is 35oC what will be
the relative humidity %.
Ans. 100 %

S-8 For which fuel the difference between GCV and NCV will be smaller, Coal or Natural Gas?
Ans. Coal

S-9 What is the conversion efficiency range of a biomass gasifier ?

Ans. 60 – 70 %

S-10 How many units of energy will be generated by a wind turbine of 250 kW operating at a
capacity factor 0.25 in 8760 hours ?

Ans. 250 x 0.25 x 8760 =

5,47,500 kWh

-------- End of Section - I ---------

Section - II: LONG DESCRIPTIVE QUESTIONS Marks: 2 x 5 = 10

(i) Answer all Two questions


(ii) Each question carries Five marks

L-1 A trial for finding out the actual capacity of a reciprocating instrument air
compressor of nominal capacity of 900 Nm^3/Hr was done.

The following observations were made :


Atmospheric pressure : 1.033 kg/sq.cm
Ambient temperature : 30 deg. C
Receiver capacity : 12 m^3
Additional hold-up volume : 10% of receiver volume
Initial pressure (after bleeding) : 0.2 kg/sq.cm g
Final pressure (after pump-up) : 7.0 kg/sq.cm g
Pump-up time : 5 min:30sec
Motor power(avg) : 105 kW (as per power analyzer)
Discharge temperature : 45 deg. C
Calculate:
i) The actual compressor capacity
ii) The specific power consumption in kW/nm3/hr

Ans.
i) Actual capacity,FAD Q= ( P2 – P1) *V * (273+t1) Nm^3/mte
P0 T (273+t2)

2
_________________________
Bureau of Energy Efficiency
Paper 4 – Set A, Energy Auditor Key

where ,
P2 = final pressure at receiver after pump-up ,kg/sq.cm a
P1 = initial pressure at receiver after bleeding, kg/sq.cm a
P0 = atmospheric pressure, kg./sq.cm a
V = total storage volume , m^3
T = pump-up time ,mte

Q = (8.033-1.233) * 12*1.1/5.5 *303/318


1.033

= 15.05 Nm3/min, say 15 Nm3/min

ii) specific power consumption:= 105 kW/Nm3


15*60
= 0.117 kW/Nm3

L-2 The following parameters were observed during the performance testing of
pump.
Flow rate of fluid :900m3/hr.
Density of fluid :950kg/m3
Discharge pressure : 5.0kg/cm2(a)
Suction head :5 metre above the pump centerline.
Measured power :180kW
Motor efficiency :90%
Calculate the pump efficiency.

Ans. Hydraulic power = (900/3600) x 45 x 950 x 9.81/ 1000


= 104.7 kW

Pump shaft power= 180 x 0.9

= 162 kW

Pump efficiency = 104.7/162


= 64.6 %

-------- End of Section - II ---------

Section - III: Numerical Questions Marks: 4 x 20 = 80

(i) Answer all Four questions


(ii) Each question carries Twenty marks

3
_________________________
Bureau of Energy Efficiency
Paper 4 – Set A, Energy Auditor Key

N-1 A furnace oil fired boiler is generating steam 20 t/hr @10 kg/cm2 ( enthalpy – 650
kcal/kg & feed water temp-80 0C) The evaporation ratio of the oil fired boiler is
14. The GCV of the fuel is 10,200 kCal/kg. Due to high furnace oil cost the
management wants to covert from oil firing to Agro residue briquettes firing with a
GCV of 3200 kcal/kg. The expected efficiency of the new Briquette fired boiler is
75%. The cost of furnace oil is Rs.28000/t and briquette cost is Rs.4000/t. The
annual operating hrs of the boiler is 7000 hrs. The emission factor for furnace oil
is 3 t CO2/ton.

a. Find out the annual savings for the company by shifting to Briquettes.
b. In addition the management wants to claim carbon credits for fuel switch.
Calculate the estimated carbon credits for this measure.

Ans.

Parameter Unit F.Oil Briquettes


Steam Generation T/hr 20 20
Steam Enthalpy kcal/kg 650 650
o
Feed water temp C 80 80
Evap. Ratio t/t 14
Efficiency % 0.75
GCV kcal/kg 10200 3,200
20(650-80)
Fuel Consumption t/hr (20/14) /(3200*0.75)
1.43 4.75
fuel cost Rs/T 28000.00 4000
Cost of operation (1.43 x 28000) (4.75x 4000)
Rs./hr 40000 19000
Energy Cost
Savings (40000-19000)
21000
annual operating
hrs hrs 7000 7000
annual cost (21000 x 7000)
Savings Rs. Lakh/hr /100000=
1470
Carbon Credits
t
Emission Factor CO2/Ton 3
annual F.Oil
savings Ton/year (1.43x 7000)
10000
Expected Carbon
Credits CERs 10000 x 3
30000

4
_________________________
Bureau of Energy Efficiency
Paper 4 – Set A, Energy Auditor Key

N-2 The following are the operating parameters of rerolling mill furnace

Weight of input material - 10 T/hr


Furnace oil consumption - 600 litres/hr
Specific gravity of oil - 0.92
Final material temperature - 1200oC
Initial material temperature - 40oC
Outlet flue gas temperature - 650oC
Specific heat of the material – 0.12 kCal/kg/oC
GCV of oil - 10,000 kCal/kg
Percentage yield - 92 %

a. Calculate furnace efficiency by direct method


b. Calculate Specific fuel consumption on finished product basis

The management installed a recuperator to preheat combustion air from 40oC to


300oC resulted in following benefits:

Increase in material input by 10 %


Reduction in fuel consumption by 13 %
Yield improvement from 92% to 96%

c. Calculate the furnace efficiency after the modifications


d. Reduction in specific fuel consumption after installing the waste heat
recovery

Ans.
a) Furnace efficiency by direct method

Heat input 600 lit/hr x 0.92 x 10000


55,20,000 kCal/hr
Heat output 10,000 x 0.12 x (1200 – 40)
1,39,2000 kcal/hr
Efficiency 1,39,2000 /55,20,000
25.21 %

b) Specific fuel consumption on finished product basis

Weight of finished products 10 x 0.92


9.2 T/hr
Furnace oil consumption 600 litres/hr
Specific fuel consumption 600/9.2
65.2 litres/ton

c) Furnace efficiency with 10 % increase in input material


Fuel consumption after 600 x 0.87
modification
522 litres/hr
Production after modification 10 + 10 x 0.1

5
_________________________
Bureau of Energy Efficiency
Paper 4 – Set A, Energy Auditor Key

11 T/hr
Heat input 522 lit/hr x 0.92 x 10000
48,02,400 kCal/hr
Heat output 11,000 x 0.12 x (1200 – 40)
15,31,200 kcal/hr
Efficiency 15,31,200/48,02,400
31.9 %

d) Reduction in Specific fuel


consumption
Yield of finished product 11 x 0.96
10.56 T/hr
Fuel consumption 522 litres
Specific fuel consumption 522/10.56
49.43 litres/T
Original specific fuel consumption 65.2 litres/T
65.2 – 49.43
15.77 litres/T

N-3 For a double extraction cum condensing turbine with data as given in the following
diagram, evaluate

a. Power generated if the efficiency of the turbine is 90 %


b. Cooling water flow rate circulation in the condenser if the range is 7oC

Ans.
a. Power generated if the efficiency of the turbine is 90 %

Input heat to turbine = 72000 x 3439.6


= 2.477 x 108 kJ/hr
= 2.477 x 108 / 3600
= 68792 kW

6
_________________________
Bureau of Energy Efficiency
Paper 4 – Set A, Energy Auditor Key

Output heat of different streams

1st extraction = 5000 x 2957.2


= 0.148 x 108 kJ/hr
= 4107 kW

2nd t extraction = 50,000 x 2768.8


= 1.38 x 108 kJ/hr
= 38456 kW

Condenser input heat load


= 17,000 x 2633.4
= 0.448 x 108 kJ/hr
= 12436 kW

Total heat leaving the turbine = 4107 + 38456 + 12436


= 54999 kW

Heat available for power generation = 68792 – 54999

= 13793 kW

Power generation at 0.9 turbine efficiency = 13793 x 0.9


= 12414 kW

b. Cooling water flow rate circulation in the condenser if the range is


7oC

Condenser heat load = 17,000 x (2633.4 – 191.8)


= 2442 kJ/hr
= 2442 x 4.18
= 10208000 kCal/hr

At a range of 7oC cooling water flow rate = 10208000 / 7

= 1458 m3/hr

N-4 A steam radiator is used for heating air with steam. Saturated steam enters the
radiator at a temperature of 133oC. Air enters the radiator at 30oC and leaves at
85oC. The heat transfer area is 794 m2. The heat duty of the radiator is 14,50,000
kCal/hr. If the correction factor is 0.95 calculate the overall heat transfer
coefficient in kW/m2 K.

Ans.

7
_________________________
Bureau of Energy Efficiency
Paper 4 – Set A, Energy Auditor Key

LMTD, counter flow [(133-30) – (133-85)]/ln [(133-30)/133-85)]


72oC

Corrected LMTD 72 x 0.95


68.4oC

Heat duty, Q 14,50,000 kCal/hr


1683 kW

Area 794 m2
Overall heat transfer Q/ A x corrected LMTD
coefficient
1683 / (794 x 68.4)
0.031 kW/m2 K

-------- End of Section - III ---------

8
_________________________
Bureau of Energy Efficiency
Paper 4 – Set B, Energy Auditor Key

Regn No: _________________


Name: ___________________
(To be written by the candidates)

7th NATIONAL CERTIFICATION EXAMINATION


FOR
ENERGY AUDITORS – Nov., 2008

PAPER – 4: Energy Performance Assessment for Equipment and Utility


Systems
Date: 23.11.2008 Timings: 1400-1600 HRS Duration: 2 HRS Max. Marks: 100

Section - I: SHORT DESCRIPTIVE QUESTIONS Marks: 10 x 1 = 10

(i) Answer all Ten questions


(ii) Each question carries One mark
(iii) Answer should not exceed 50 words

S-1 What will be the synchronous speed of a VFD driven 4-pole induction motor operating at
38 Hz ?

Ans. Ns = 120 x f/P


= 120 x 38/4
= 1140 RPM

S-2 If the power consumed by a refrigeration compressor is 2 kW per ton of refrigeration, what
is the energy efficiency ratio?

Ans.
12000 Btu
EER = 2000 W = 6

S-3 Explain why heat rate of back pressure turbine is greater than that of a condensing
turbine.

Ans. As it does not take into account of the heat content of the exhaust steam used in the
process.

S-4 Why line current method used for estimating loading of a motor is not applicable for motor
loading less than 75%.
Ans. At lower loadings, power factor of a motor degrades significantly and ampere-load curve
becomes nonlinear
S-5 Explain why actual air delivered is always converted to (FAD) while measuring delivered
air volume flow rates in an air compressor.

Ans. As air is compressible, its volume flow rate will vary with pressure on delivery side and
hence for comparison purposes the volume flow rates are always converted to their value
at standard atmospheric pressure.

S-6 What is the minimum wind speed which is acceptable for viable power generation from a

1
_________________________
Bureau of Energy Efficiency
Paper 4 – Set B, Energy Auditor Key

wind turbine?

Ans. 15 kmph

S-7 If the dry bulb temperature of air is 35oC and the wet bulb temperature is 35oC what will be
the relative humidity %.
Ans. 100 %

S-8 For which fuel the difference between GCV and NCV will be smaller, Coal or Natural Gas?
Ans. Coal

S-9 What is the conversion efficiency range of a biomass gasifier ?

Ans. 60 – 70 %

S-10 How many units of energy will be generated by a wind turbine of 250 kW operating at a
capacity factor 0.2 in 8760 hours ?

Ans. 250 x 0.2 x 8760 =

4,38,000kWh

-------- End of Section - I ---------

Section - II: LONG DESCRIPTIVE QUESTIONS Marks: 2 x 5 = 10

(i) Answer all Two questions


(ii) Each question carries Five marks

L-1 A trial for finding out the actual capacity of a reciprocating instrument
air compressor of nominal capacity of 900 Nm^3/Hr was done.

The following observations were made:


Atmospheric pressure : 1.033 kg/sq.cm
Ambient temperature : 30 deg. C
Receiver capacity 12 m^3
Additional hold-up volume : 10% of receiver volume
Initial pressure : 0.2 kg/sq.cm g
Final pressure : 7.0 kg/sq.cm g
Pump-up time : 5 min:30sec
Motor power(avg) : 115 kW ( as per power analyzer)
Discharge temperature : 45 deg.C
Calculate:
i) The actual compressor capacity
ii) The specific power consumption, in kW/m3/hr

Ans.
i) Actual capacity,FAD Q= ( P2 – P1) * V * (273+t1) Nm^3/mte

2
_________________________
Bureau of Energy Efficiency
Paper 4 – Set B, Energy Auditor Key

P0 T (273+t2)

where ,
P2 = final pressure at receiver after pump-up ,kg/sq.cm a
P1 = initial pressure at receiver after bleeding, kg/sq.cm a
P0 = atmospheric pressure, kg./sq.cm a
V = total storage volume , m^3
T = pump-up time ,mte

Q = (8.033-1.233) * 12*1.1/5.5 *303/318


1.033

= 15.05 Nm3/mte, say 15 Nm3/mte

ii) specific power consumption:= 115 kW/Nm3


15*60
= 0.127 kW/Nm3/hr

L-2 The following parameters were observed during the performance testing of pump.

Flow rate of fluid :900m3/hr.


Density of fluid :950kg/m3
Discharge pressure : 5.0kg/cm2(a)
Suction head :5 metre above the pump centerline.
Measured power :190kW
Motor efficiency :90%
Calculate the pump efficiency.

Ans. Hydraulic power = (900/3600) x 45 x 950 x 9.81/ 1000


= 104.7 kW

Pump shaft power= 190 x 0.9

= 171kW
Pump efficiency = 104.7/171
= 61.2 %

-------- End of Section - II ---------

3
_________________________
Bureau of Energy Efficiency
Paper 4 – Set B, Energy Auditor Key

Section - III: Numerical Questions Marks: 4 x 20 = 80

(i) Answer all Four questions


(ii) Each question carries Twenty marks

N-1 A furnace oil fired boiler is generating steam 20 t/hr @10 kg/cm2 ( enthalpy – 650
kcal/kg & feed water temp-80 0C) The evaporation ratio of the oil fired boiler is
14. The GCV of the fuel is 10,200 kCal/kg. Due to high furnace oil cost the
management wants to covert from oil firing to Agro residue briquettes firing with a
GCV of 3200 kcal/kg. The expected efficiency of the new Briquette fired boiler is
75%. The cost of furnace oil is Rs.28000/t and briquette cost is Rs.4000/t. The
annual operating hrs of the boiler is 8000 hrs. The emission factor for furnace .oil
is 3 t CO2/ton.

a. Find out the annual savings for the company by shifting to Briquettes.
b. In addition the management wants to claim carbon credits for fuel switch.
Calculate the estimated carbon credits for this measure.

Ans.

Parameter Unit F.Oil Briquettes


Steam
Generation T/hr 20 20
Steam Enthalpy kcal/kg 650 650
Feed water temp oC 80 80
Evap. Ratio t/t 14
Efficiency % 0.75
GCV kcal/kg 10200 3,200
20(650-80)
Fuel Consumption t/hr (20/14) /(3200*0.75)
1.43 4.75
fuel cost Rs/T 28000.00 4000
Cost of operation (1.43 x 28000) (4.75x 4000)
Rs./hr 40000 19000
Energy Cost
Savings (40000-19000) =
21000
annual operating
hrs hrs 8000 8000
annual cost (21000 x 8000)
Savings Rs. Lakh/hr /100000=
1680 lakhs
Carbon Credits

4
_________________________
Bureau of Energy Efficiency
Paper 4 – Set B, Energy Auditor Key

t
Emission Factor CO2/Ton 3
annual F.Oil
savings Ton/year (1.43x 8000)
11440
Expected Carbon
Credits CERs 11440x 3
34320

N-2 The following are the operating parameters of rerolling mill furnace

Weight of input material - 10 T/hr


Furnace oil consumption - 600 litres/hr
Specific gravity of oil - 0.92
Final material temperature - 1200oC
Initial material temperature - 40oC
Outlet flue gas temperature - 650oC
Specific heat of the material – 0.12 kCal/kg/oC
GCV of oil - 10,000 kCal/kg
Percentage yield - 92 %

a. Calculate furnace efficiency by direct method


b. Calculate Specific fuel consumption on finished product basis

The management installed a recuperator to preheat combustion air from 40oC to 300oC
resulted in following benefits:

Increase in material input by 10 %


Reduction in fuel consumption by 13 %
Yield improvement from 92% to 95%

c. Calculate the furnace efficiency after the modifications


d. Reduction in specific fuel consumption after installing the waste heat
recovery

Ans.
a) Furnace efficiency by direct method

Heat input 600 lit/hr x 0.92 x 10000


55,20,000 kCal/hr
Heat output 10,000 x 0.12 x (1200 – 40)
1,39,2000 kcal/hr
Efficiency 1,39,2000 /55,20,000
25.21 %

b) Specific fuel consumption on finished product basis

Weight of finished products 10 x 0.92


9.2 T/hr

5
_________________________
Bureau of Energy Efficiency
Paper 4 – Set B, Energy Auditor Key

Furnace oil consumption 600 litres/hr


Specific fuel consumption 600/9.2
65.2 litres/ton

c) Furnace efficiency with 10 % increase in material input


Fuel consumption after modification 600 x 0.87
522 litres/hr
Production after modification 10 + 10 x 0.1
11 T/hr
Heat input 522 lit/hr x 0.92 x 10000
48,02,400 kCal/hr
Heat output 11,000 x 0.12 x (1200 – 40)
15,31,200 kcal/hr
Efficiency 15,31,200/48,02,400
31.9 %

d) Reduction in Specific fuel


consumption
Yield of finished product 11 x 0.95
10.45 T/hr
Fuel consumption 522 litres
Specific fuel consumption 522/10.45
49.95 litres/T
Original specific fuel consumption 65.2 litres/T
65.2 – 49.95
15.25 litres/T

N-3 For a double extraction cum condensing turbine with data as given in the following
diagram, evaluate

a. Power generated if the efficiency of the turbine is 90 %


b. Cooling water flow rate circulation in the condenser if the range is 8oC

6
_________________________
Bureau of Energy Efficiency
Paper 4 – Set B, Energy Auditor Key

Ans.
a. Power generated if the efficiency of the turbine is 90 %

Input heat to turbine = 72000 x 3439.6


= 2.477 x 108 kJ/hr
= 2.477 x 108 / 3600
= 68792 kW

Output heat of different streams

1st extraction = 5000 x 2957.2


= 0.148 x 108 kJ/hr
= 4107 kW

2nd t extraction = 50,000 x 2768.8


= 1.38 x 108 kJ/hr
= 38456 kW

Condenser input heat load


= 17,000 x 2633.4
= 0.448 x 108 kJ/hr
= 12436 kW

Total heat leaving the turbine = 4107 + 38456 + 12436


= 54999 kW

Heat available for power generation = 68792 – 54999

= 13793 kW

Power generation at 0.9 turbine efficiency = 13793 x 0.9


= 12414 kW

b. Cooling water flow rate circulation in the condenser if the range is 8oC

Condenser heat load = 17,000 x (2633.4 – 191.8)


= 2442 kJ/hr
= 2442 x 4.18
= 10208000 kCal/hr

At a range of 8oC cooling water flow rate = 10208000 / 8

= 1276 m3/hr

N-4 A steam radiator is used for heating air with steam. Saturated steam enters the radiator at a
temperature of 133oC. Air enters the radiator at 30oC and leaves at 85oC. The heat transfer
area is 862 m2. The heat duty of the radiator is 14,50,000 kCal/hr. If the correction factor
is 0.95 calculate the overall heat transfer coefficient in kW/m2 K.

Ans.

7
_________________________
Bureau of Energy Efficiency
Paper 4 – Set B, Energy Auditor Key

LMTD, counter flow [(133-30) – (133-85)]/ln [(133-30)/133-85)]


72oC

Corrected LMTD 72 x 0.95


68.4oC

Heat duty, Q 14,50,000 kCal/hr


1683 kW

Area 862 m2
Overall heat transfer Q/ A x corrected LMTD
coefficient
1683 / (862 x 68.4)
0.0285 kW/m2 K

-------- End of Section - III ---------

8
_________________________
Bureau of Energy Efficiency
Paper 4 – Energy Auditor – Set A Solutions

Regn No: _________________


Name: ___________________
(To be written by the candidates)

NATIONAL CERTIFICATION EXAMINATION 2006


FOR
ENERGY AUDITORS

PAPER – 4: Energy Performance Assessment for Equipment and Utility


Systems

Date: 23.04.2006 Timings: 1400-1600 HRS Duration: 2 HRS Max. Marks: 100

General instructions:
o Please check that this question paper contains 4 printed pages
o Please check that this question paper contains 16 questions
o The question paper is divided into three sections
o All questions in all three sections are compulsory
o All parts of a question should be answered at one place
o Open book examination

Section - I: SHORT DESCRIPTIVE QUESTIONS Marks: 10 x 1 = 10

(i) Answer all Ten questions


(ii) Each question carries One mark
(iii) Answer should not exceed 50 words

S-1 State two causes for rise in exit flue gas temperature in a boiler.

1. Scale deposition on water side 2. Soot deposition on gas side

Any other relevant cause such as reduction in excess air levels, problems with
Air preheater, economizer etc. may be given mark

S-2 What are the disadvantages of heating the charge above the optimum
temperature in steel re-rolling furnaces?

1. Increased energy consumption 2. Increase in scale losses 3. High rejection


rates

S-3 State the impact of fouling factor on the overall heat transfer coefficient.

They are inversely proportional. Increase in fouling factor will result in decreased
overall heat transfer coefficient.

_________________________ 1
Bureau of Energy Efficiency
Paper 4 – Energy Auditor – Set A Solutions

S-4 List the basic parameters required for assessing refrigeration capacity.

Mass of circulating fluid, its specific heat and temperature difference

S-5 While using Pitot tube for airflow measurement in large ducts, series of traverse
measurements are recommended. Why?

Because the velocity is not uniform across the duct cross section

(m3/s) x pressure gain in Pascal


S-6 Static fan efficiency = . Right or wrong?
Power input to shaft in Watt

Justify your answer.

Right

The units are balanced in both SI and MKS system.

For example, if Volume=10 m3/s, Pressure gain is 500 Pa and power consumed
is 10,000 W, then the efficiency as per SI units given in the problem

(10 x 500 / 10,000) x 100 = 50%

As per MKS the fan efficiency is


Volume in m 3 / Sec x total pressure in mmwc
Static Fan Efficiency % =
102 x Power input to the shaft in (kW) X 100

1mmwc = 9.81 Pa
Volume = 10 m3/s, Pressure gain in mmWC = 500/9.81 = 51 mmwc,
Power consumed = 10 kW

(10 x 51) x100 /(102 x 10) = 50%

S-7 State two methods of non-intrusive water flow measurements in a pipe.

1. Ultrasonic / Doppler 2. Tracer

S-8 What is meant by compression ratio for air compressor?

Ratio of discharge to suction pressures

S-9 How many volt-amperes (VA) does a 60 Watt incandescent light require?

60 VA

S-10 A reasonable range of capacity factors for wind electric generators is.…

0.25 – 0.30
_________________________ 2
Bureau of Energy Efficiency
Paper 4 – Energy Auditor – Set A Solutions

-------- End of Section - I ---------

Section - II: LONG DESCRIPTIVE QUESTIONS Marks: 2 x 5 = 10

(i) Answer all Two questions


(ii) Each question carries Five marks

L-1 A centrifugal clear water pump rated for 800 m3/hr was found to be operating at
576 m3/hr with discharge valve throttled. The pumps speed is 1485 RPM. The
discharge pressure of the pump before the throttle valve is 2 kg/cm 2g. The pump
draws the water from a sump 4 metres below the centerline of the pump. The
input power drawn by the motor is 124 kW at a motor efficiency of 92%.
(i) Find out the efficiency of the pump.
(ii) If the normal required water flow rate is 500 m3/hr to 700 m3/hr, what in your
opinion should be the most energy efficient option to get the required flow rate
variation?
(iii) And what would be the pump shaft power for that most energy efficient option if
the pump is delivering the flow rate of 550 m3/hr.

Ans:

(i) Hydraulic power, Ph (kW) = Q x (hd - hs) x r x g / 1000

Q = Volume flow rate (m3/s), r = density of the fluid (kg/m3), g = acceleration due to gravity (m/s2),
(hd - hs) = Total head in metres

hd - hs = 20 – ( - 4 ) = 24 m

hydraulic power = (576 x 24 x 1000 x 9.81) / (3600 x 1000)


= 37.67 kw

Input power to pump = 124 kW x 0.92 = 114 kW

Efficiency of the pump = (37.67 /114) x 100 = 33 %

(ii) Since the pump discharge requirement varies from 500 m3/h to 700 m3/h,
the ideal option would be to operate with a VSD (variable frequency drive,
hydraulic coupling)

(iii) For a flow rate 550 m3/h, the reduced speed of pump would be:

_________________________ 3
Bureau of Energy Efficiency
Paper 4 – Energy Auditor – Set A Solutions

(550/800) = (N1/1485)
N1 = 1021
The pump shaft power would be:
3
1021
= x 114 = 37 kW
1485

L-2 A 30 kW four pole induction motor operating at 50 Hz and rated for 415 V and
1440 RPM, the actual measured speed is 1460 RPM. Find out the percentage
loading of the motor if the voltage applied is 425 V.

Ans:
% Loading = Slip x 100%
(Ss – Sr) x (Vr / V)2

Synchronous speed = 120 x 50 / 4 = 1500 rpm

Slip = Synchronous Speed – Measured speed in rpm.


= 1500 – 1460 = 40 rpm.

% Loading = 40 x 100% = 69.9%


(1500 - 1440) x (415/425)2

-------- End of Section - II ---------

Section - III: Numerical Questions Marks: 4 x 20 = 80

(i) Answer all Four questions


(ii) Each question carries Twenty marks

N -1 A process plant requires 28 tons of steam per hour and 2250 kW of electric
power. The plant operates for 8000 hours per annum. Steam is generated at 2
bar (g) in a coal fired boiler with an efficiency of 75%. The feed water
temperature is 80OC. The calorific value of coal is 4000 kcal/kg. The cost of coal
is Rs. 2000/ton. Power is drawn from the grid at Rs. 4/kWh. The contract
demand is 3000 kVA with the electricity supply company and the plant is
charged for 100% of the contract demand at Rs. 300/kVA/month. The plant has
never exceeded its contract demand in the past.
The plant is planning for a back pressure cogeneration system using the same
coal with the following parameters. The power and steam demand are to be fully
met by the cogeneration plant and a contract demand of 1000 kVA with the grid
is to be kept for emergency purposes.

_________________________ 4
Bureau of Energy Efficiency
Paper 4 – Energy Auditor – Set A Solutions

Find out the IRR over a project life cycle of 6 years for the proposed
cogeneration system
Cogeneration System data:

Boiler generation pressure - 18 bar (g), 310OC


Boiler efficiency - 81%
Investment required - Rs. 20 crores
Generated power = 2250 kW
Steam enthalpy data:

Total enthalpy at 2 bar (g) = 647.13 kcal/kg


Total enthalpy at 18 bar (g), 310oC = 730.28 kcal/kg

Ans:
Existing condition
Coal consumption 28,000 x [(647.13) – 80]
0.75 x 4000

5293 kg/hr

Fuel cost per annum 5.293 x 2000 x 8000

Rs. 8.5 crores (approx)

Annual electrical energy charges 2250 x 8000 x Rs.4

Rs. 7.2 crores

Maximum demand charges 3000 x 12 x 300

Rs. 1.1 crores (approx)

Total electricity bill per annum 7.2 + 1.1 = Rs. 8.3 crores

Total energy bill per annum 8.5 + 8.3 = Rs. 16.8 crores

With cogeneration plant

Coal consumption 28,000 x [(730.28) – 80]


0.81 x 4000

5620 kg/hr

Incremental coal consumption 5620 – 5293 = 327 kg/hr

Incremental fuel cost per annum 0.327 x 8000 x Rs. 2000

_________________________ 5
Bureau of Energy Efficiency
Paper 4 – Energy Auditor – Set A Solutions

Rs. 0.52 crores

Maximum demand charges per 1000 x 12 x Rs.300


annum Rs. 0.36 crores

Total cost per year 8.5 + 0.52 + 0.36 = 9.38 crores


Savings = 16.8 – 9.38 = Rs. 7.42 crores

Investment Rs. 20 crores

1 1 1 1 1 1
20 = 7.42 + + + + +
(1+i)1 (1+i)2 (1+i)3 (1+i)4 (1+i)5 (1+i)6

IRR = 29 to 30%

N -2 In a double pipe heat exchanger hot fluid is entering at 220°C and leaving at
115°C. Cold fluid enters at 10oC and leaves at 75°C. The following data is
provided for hot and cold fluids.
Mass flow rate of hot fluid = 100 kg/hr
Cp of hot fluid = 1.1 kcal/kg°C
Cp of cold fluid = 0.95 kcal/kg°C
(i) Calculate LMTD
a) For parallel flow
b) For counter current flow
(ii) Which flow arrangement is preferable and why?
(iii) Find the mass flow rate of cold fluid if the heat loss during the exchange is 5%.

Ans:
a) LMTD Parallel flow
t1 − t 2
LMTD =
t1
In
t 2
t1 = 210°C
t2 = 40°C
210− 40
LMTD = = 102.5°C
210
ln
40

b) LMTD Counter current flow


t1 = 145°C

_________________________ 6
Bureau of Energy Efficiency
Paper 4 – Energy Auditor – Set A Solutions

t2 = 105°C
t1 − t 2
LMTD =
t1
ln
t 2
145−105
= = 123.9°C
145
ln
105

2) Counter flow is preferred since the LMTD is more, the area of the heat
exchanger will be less

3) Mass flow rate of cold fluid


Data:
m. Mass flow rate of hot fluid = 100 kg/hr
cph. = 1.1 kcal/kg°C
cpC = 0.95 kcal/kg°C
Cold fluid inlet temperature = 10°C
Cold fluid outlet temperature = 75°C
Hot fluid inlet temperature = 220°C
Hot fluid outlet temperature = 115°C
Q = m.hxCPh X th x 0.95 = mc x CPC x tc

Mass flow rate of cold fluid mc =

100 x 1.1 x (220 − 115) x 0.95


=
0.95 x (75 − 10)
= 177.7 kg/hr

N-3 An efficiency trial was conducted in furnace oil fired boiler during the conduct of
energy audit study and the following data were collected.
Boiler Data:
Rated capacity = 10 TPH (F&A 100oC)
Rated efficiency = 84%
Actual steam generation pressure = 7 kg/cm2 (g) saturated
Feed water temperature = 45oC
Boiler was found to be operating at rated steam pressure and flow conditions
Furnace Oil Data:
Furnace oil consumption = 600 litre per hour
GCV of oil = 10200 kcal/kg
_________________________ 7
Bureau of Energy Efficiency
Paper 4 – Energy Auditor – Set A Solutions

Specific gravity of oil = 0.92


% Carbon = 84%
% Hydrogen = 12%
% Sulphur = 3%
% Oxygen = Nil
% Nitrogen = 1%
Cost = Rs. 20/kg

Flue Gas Data:


% O2 in flue gas = 5.5% by volume
CO in flue gas = Nil
Flue gas temperature = 240oC
Specific heat of flue gas = 0.24 kcal/kgoC
Moisture in ambient air = 0.03 kg/kg of air
Ambient air temperature = 40oC
Assume surface heat and unaccounted losses = 2%
Determine the following:
(i) Boiler efficiency by indirect method
(ii) Find out the annual savings in Rs per year if the boiler was operating at its rated
efficiency.
(iii) Also suggest possible measures to improve the efficiency of the boiler.

Ans:

Theoritical air Requirement for Furnace Oil


[(11.6 x C) + {34.8 x ( H 2 − O2 / 8)}+ (4.35 x S )] /100 kg/kg of oil

Theoritical air required

11.6 x 84 + ((34.8 x (12-0 )) + 4.35 x 3


100

= 11.6 x 84 + 34.8 x 12 + 4.35 x 3 = 14.05 kg/kg of fuel

100

Excess air Supplied (EA) = 5.5 x 100


21-5.5

Excess air for 5.5 % O2 in flue gas = 35.48%

Actual air supplied (AAS) = 1 + EA x Theoretical air


100

1 + 35.48 x 14.05=19.03 kg of air/kg of oil


100

_________________________ 8
Bureau of Energy Efficiency
Paper 4 – Energy Auditor – Set A Solutions

Mass of Dry Flue gas = .84 x 44 + 19.03x 77 + .01 + (19.03 - 14.05) x 23


12 100 100

+ 0.03 x 64/32
= 3.08 + 14.65 + 0.01 + 1.14 + 0.06
= 18.94 kg of air / kg of oil

Calculation of All Losses :


1. Dry flue gas loss = 18.94 x 0.24 (240-40) x 100
10200

= 8.91%

2. Loss due to Hydrogen in the Fuel = 9x 0.12 (584 + 0.45 (240-40) x 100
10200
= 7.14%

3. Loss due to Moisture vapour


present in combustion air = 19.03 x .03 x .45 (240-40) x 100
10200
= 0.50%

4. Boiler surface heat loss and unaccounted losses (given) = 2%

(i) Boiler Efficiency = 100 – (8.91 + 7.14 + 0.5 +2%)

= 100-18.55 = 81.45%

Efficiency Improvement from Existing to Rated = 2.55%


Fuel Input after improvement
in boiler efficiency = 600 x 0.8145 = 581.79 lit./hr
0.84
(ii) Oil Saving per hour = 600-581.79 = 18.21 litre/hr.

(18.21 x 0.92) x Rs. 20 x 8000 = Rs. 26.80 lakhs

(iii) 1. Reduce excess air


2. Reduce flue gas temperature (by cleaning the boiler tubes to get rid of
scales or soot )

N-4 A V-belt centrifugal fan is supplying air to a chemical process. The performance
test on the fan gave the following parameters.
Ambient temperature 40oC
Density of air at 0oC 1.293 kg/m3
Diameter of the discharge air duct 1m
_________________________ 9
Bureau of Energy Efficiency
Paper 4 – Energy Auditor – Set A Solutions

Velocity pressure measured by Pitot tube in 47 mmWC


discharge duct
Pitot tube coefficient 0.9
Static pressure at fan inlet - 22 mmWC
Static pressure at fan outlet 188 mmWC
Power drawn by the motor coupled with the fan 72 kW
Belt transmission efficiency 95%
Motor efficiency at the operating load 90 %

(i) Find out the efficiency of the fan.


(ii) Due to modification in the chemical process, only half of the operating flow will
be required in future. This is to be effected by damper control method. The fan
characteristic curve shows that the total static head developed by the fan will be
333 mmWC and static fan efficiency will be 61% by damper control method.
Find out what will be the annual savings at 8000 hours of operation per year and
an energy cost of Rs. 4.50 /kWh. Assume that the motor efficiency and belt
transmission efficiency remains same.
(iii) List down the various energy conservation options to achieve the modified flow
rate.

Ans:
i)
Ambient temperature 40oC
Diameter of the discharge air duct 1m
Velocity pressure measured by Pitot tube 47 mmWC
Static pressure at fan inlet - 22 mmWC
Static pressure at fan outlet 188 mmWC
Power drawn by the motor 72 kW
Transmission efficiency 95%
Motor efficiency 90 %
Area of the discharge duct 3.14 x 1 x 1/4
0.785 m2
Pitot tube coefficient 0.9
Corrected gas density (273 x 1.293) / (273 + 40) = 1.127

Air velocity Cp x  2 x 9.81 x  p x 


0.9 x x Sq rt.(2 x 9.81 x 47 x 1.127)


1.127
25.7 m/s

Volume 25.7 x 0.785


20.17 m3/s
Power input to the shaft 72 x 0.95 x 0.9

_________________________ 10
Bureau of Energy Efficiency
Paper 4 – Energy Auditor – Set A Solutions

61.6 kW

Volume in m3 / Sec x total static pressure in mmwc


Static Fan Efficiency % =
102 x Power input to the shaft in (kW)

Fan static Efficiency 20.17 x (188 – (-22)


102 x 61.6
67 %

ii)
New flow 20.17/2 = 10.1
Fan shaft Power drawn due to flow 10.1 x 333
reduction to 50 % by damper closing 102 x 0.61
54 kW
Power drawn by the motor 54/(0.95 x 0.9) =63
Energy savings (72 – 63) x 8000 x Rs.4.50
Rs.3,24,000/annum

iii) various energy conservation measures

i. Pulley change
ii. Impeller trimming
iii. New fan
iv. New motor

-------- End of Section - III ---------

_________________________ 11
Bureau of Energy Efficiency
Paper 4 – Energy Auditor – Set B Solutions

Regn No: _________________


Name: ___________________
(To be written by the candidates)

NATIONAL CERTIFICATION EXAMINATION 2006


FOR
ENERGY AUDITORS

PAPER – 4: Energy Performance Assessment for Equipment and Utility


Systems

Date: 23.04.2006 Timings: 1400-1600 HRS Duration: 2 HRS Max. Marks: 100

General instructions:
o Please check that this question paper contains 4 printed pages
o Please check that this question paper contains 16 questions
o The question paper is divided into three sections
o All questions in all three sections are compulsory
o All parts of a question should be answered at one place
o Open book examination

Section - I: SHORT DESCRIPTIVE QUESTIONS Marks: 10 x 1 = 10

(i) Answer all Ten questions


(ii) Each question carries One mark
(iii) Answer should not exceed 50 words

S-1 A reasonable range of capacity factors for wind electric generators is.…

0.25 – 0.30

S-2 How many volt-amperes (VA) does a 100 Watt incandescent light require?

100 VA

S-3 What is meant by compression ratio for air compressor?

Ratio of discharge to suction pressures

S-4 State two methods of non-intrusive water flow measurements in a pipe.

1. Ultrasonic / Doppler 2. Tracer

_________________________ 1
Bureau of Energy Efficiency
Paper 4 – Energy Auditor – Set B Solutions

(m3/s) x pressure gain in Pascal


S-5 Static fan efficiency = . Right or wrong?
Power input to shaft in Watt
Justify your answer.

Right

The units are balanced in both SI and MKS system.

For example, if Volume=10 m3/s, Pressure gain is 500 Pa and power consumed
is 10,000 W, then the efficiency as per SI units given in the problem

(10 x 500 / 10,000) x 100 = 50%

As per MKS the fan efficiency is


Volume in m 3 / Sec x total pressure in mmwc
Static Fan Efficiency % =
102 x Power input to the shaft in (kW) X 100

1mmwc = 9.81 Pa
Volume = 10 m3/s, Pressure gain in mmWC = 500/9.81 = 51 mmwc,
Power consumed = 10 kW

(10 x 51) x100 /(102 x 10) = 50%

S-6 While using Pitot tube for airflow measurement in large ducts, series of traverse
measurements are recommended. Why?

Because the velocity is not uniform across the duct cross section

S-7 List the basic parameters required for assessing refrigeration capacity.

Mass flow rate of circulating fluid, its specific heat and temperature difference

S-8 State the impact of fouling factor on the overall heat transfer coefficient.

They are inversely proportional. Increase in fouling factor will result in decreased
overall heat transfer coefficient.

S-9 What are the disadvantages of heating the charge above the optimum
temperature in steel re-rolling furnaces?

1. Increased energy consumption 2. Increase in scale losses 3. High rejection


rates

S-10 State two causes for rise in exit flue gas temperature in a boiler.

1. Scale deposition on water side 2. Soot deposition on gas side

Any other relevant cause such as reduction in excess air levels, problems with
Air preheater, economizer etc.
_________________________ 2
Bureau of Energy Efficiency
Paper 4 – Energy Auditor – Set B Solutions

-------- End of Section - I ---------

Section - II: LONG DESCRIPTIVE QUESTIONS Marks: 2 x 5 = 10

(i) Answer all Two questions


(ii) Each question carries Five marks

L-1 A 30 kW four pole induction motor operating at 50 Hz and rated for 415 V and
1460 RPM, the actual measured speed is 1480 RPM. Find out the percentage
loading of the motor if the voltage applied is 425 V.

Ans:
% Loading = Slip x 100%
(Ss – Sr) x (Vr / V)2

Synchronous speed = 120 x 50 / 4 = 1500 rpm

Slip = Synchronous Speed – Measured speed in rpm.


= 1500 – 1480 = 20 rpm.

% Loading = 20 x 100% = 52.4%


(1500 - 1460) x (415/425)2

L-2 A centrifugal clear water pump rated for 800 m3/hr was found to be operating at
576 m3/hr with discharge valve throttled. The pumps speed is 1460 RPM. The
discharge pressure of the pump before the throttle valve is 2 kg/cm2g. The pump
draws the water from a sump 4 metres below the centerline of the pump. The
input power drawn by the motor is 115 kW at a motor efficiency of 92%.
(i) Find out the efficiency of the pump.
(ii) If the normal required water flow rate is 500 m3/hr to 700 m3/hr, what in your
opinion should be the most energy efficient option to get the required flow rate
variation?
(iii) And what would be the pump shaft power for that most energy efficient option if
the pump is delivering the flow rate of 550 m3/hr.

Ans:

(i) Hydraulic power, Ph (kW) = Q x (hd - hs) x r x g / 1000

Q = Volume flow rate (m3/s), r = density of the fluid (kg/m3), g = acceleration due to gravity (m/s2),
(hd - hs) = Total head in metres

_________________________ 3
Bureau of Energy Efficiency
Paper 4 – Energy Auditor – Set B Solutions

hd - hs = 20 – ( - 4 ) = 24 m

hydraulic power = (576 x 24 x 1000 x 9.81) / (3600 x 1000)


= 37.67 kw

Input power to pump = 115 kW x 0.92 = 105.8 kW

Efficiency of the pump = (37.67 /105.8) x 100 = 35.6 %

(ii) Since the pump discharge requirement varies from 500 m3/h to 700 m3/h,
the ideal option would be to operate with a VSD.

(iii) For a flow rate 550 m3/h, the reduced speed of pump would be:

(550/800) = (N1/1460)
N1 = 1004
The pump shaft power would be:
3
1004
= x 105.8 = 34.4 kW
1460

-------- End of Section - II ---------

Section - III: Numerical Questions Marks: 4 x 20 = 80

(i) Answer all Four questions


(ii) Each question carries Twenty marks

N-1 A V-belt centrifugal fan is supplying air to a chemical process. The performance
test on the fan gave the following parameters.
Ambient temperature 40oC
Density of air at 0oC 1.293 kg/m3
Diameter of the discharge air duct 1m
Velocity pressure measured by Pitot tube in 47 mmWC
discharge duct
Pitot tube coefficient 0.9
Static pressure at fan inlet - 22 mmWC

_________________________ 4
Bureau of Energy Efficiency
Paper 4 – Energy Auditor – Set B Solutions

Static pressure at fan outlet 188 mmWC


Power drawn by the motor coupled with the fan 68 kW
Belt transmission efficiency 95%
Motor efficiency at the operating load 90 %

(i) Find out the efficiency of the fan.


(ii) Due to modification in the chemical process, only half of the operating flow will
be required in future. This is to be effected by damper control method. The fan
characteristic curve shows that the total static head developed by the fan will be
333 mmWC and static fan efficiency will be 59% by damper control method.
Find out what will be the annual savings at 8000 hours of operation per year and
an energy cost of Rs. 4.50 /kWh. Assume that the motor efficiency and belt
transmission efficiency remains same.
(iii) List down the various energy conservation options to achieve the modified flow
rate.

Ans:

Ambient temperature 40oC


Diameter of the discharge air duct 1m
Velocity pressure measured by Pitot tube 47 mmWC
Static pressure at fan inlet - 22 mmWC
Static pressure at fan outlet 188 mmWC
Power drawn by the motor 68 kW
Transmission efficiency 95%
Motor efficiency 90 %
Area of the discharge duct 3.14 x 1 x 1/4
0.785 m2
Pitot tube coefficient 0.9
Corrected gas density (273 x 1.293) / (273 + 40) = 1.127

Air velocity Cp x  2 x 9.81 x  p x 


0.9 x x Sq rt.(2 x 9.81 x 47 x 1.127)


1.127
25.7 m/s

Volume 25.7 x 0.785


20.17 m3/s
Power input to the shaft 68 x 0.95 x 0.9
58.14 kW

_________________________ 5
Bureau of Energy Efficiency
Paper 4 – Energy Auditor – Set B Solutions

Volume in m3 / Sec x total static pressure in mmwc


Static Fan Efficiency % =
102 x Power input to the shaft in (kW)

Fan static Efficiency 20.17 x (188 – (-22)


102 x 58.14
71.4 %

ii)
New flow 20.17/2 = 10.1
Fan shaft Power drawn due to flow 10.1 x 333
reduction to 50 % by damper closing 102 x 0.59
55.9 kW
Power drawn by the motor 55.9/(0.95 x 0.9) = 65.4
Energy savings (68 – 65.4) x 8000 x Rs.4.50
Rs.93,600/annum

iii) various energy conservation measures

i. Pulley change
ii. Impeller trimming
iii. New fan
iv. New motor

N-2 An efficiency trial was conducted in furnace oil fired boiler during the conduct of
energy audit study and the following data were collected.
Boiler Data:
Rated capacity = 10 TPH (F&A 100oC)
Rated efficiency = 84%
Actual steam generation pressure = 7 kg/cm2 (g) saturated
Feed water temperature = 45oC
Boiler was found to be operating at rated steam pressure and flow conditions
Furnace Oil Data:
Furnace oil consumption = 600 litre per hour
GCV of oil = 10200 kcal/kg
Specific gravity of oil = 0.92
% Carbon = 84%
% Hydrogen = 12%
% Sulphur = 3%
% Oxygen = Nil
% Nitrogen = 1%
Cost = Rs. 20/kg

_________________________ 6
Bureau of Energy Efficiency
Paper 4 – Energy Auditor – Set B Solutions

Flue Gas Data:


% O2 in flue gas = 7% by volume
CO in flue gas = Nil
Flue gas temperature = 240oC
Specific heat of flue gas = 0.24 kcal/kgoC
Moisture in ambient air = 0.03 kg/kg of air
Ambient air temperature = 40oC
Assume surface heat and unaccounted losses = 2%
Determine the following:
(i) Boiler efficiency by indirect method
(ii) Find out the annual savings in Rs per year if the boiler was operating at its rated
efficiency.
(iii) Also suggest possible measures to improve the efficiency of the boiler.

Ans:

Theoritical air Requirement for Furnace Oil

Theoritical air required based on ultimate analysis.

11.6 x 84 + ((34.8 x (12-0 )) + 4.35 x 3


100

= 11.6 x 84 + 34.8 x 12 + 4.35 x 3 = 14.05 kg/kg of fuel


100

Excess air Supplied (EA) = 7 x 100


21- 7

Excess air for 7 % O2 in flue gas = 50%

Actual air supplied (ASS) = 1 + EA x Theoretical air


100

1 + 50 x 14.05=21.1 kg of air/kg of oil


100

Mass of Dry Flue gas = .84 x 44 + 21.1 x 77 + .01 + (21.1 - 14.05) x 23


12 100 100

+ 0.03 x 64/32
= 3.08 + 16.25 + 0.01 + 1.62 + 0.06
= 21.02 kg / kg of oil

_________________________ 7
Bureau of Energy Efficiency
Paper 4 – Energy Auditor – Set B Solutions

Calculation of All Losses :


1. Dry flue gas loss = 21.02 x 0.24 (240-40) x 100
10200

= 9.89 %

2. Loss due to Hydrogen in the Fuel = 9x 0.12 (584 + 0.45 (240-40) x 100
10200
= 7.14%

3. Loss due to Moisture vapour


present in combustion air = 21.02 x .03 x .45 (240-40) x 100
10200
= 0.56%

4. Boiler surface heat loss and unaccounted losses (given) = 2%

(i) Boiler Efficiency = 100 – (9.89 + 7.14 + 0.56 + 2%)

= 100 - 19.59= 80.41%

Fuel Input after improvement


in boiler efficiency = 600 x 0.8041 = 574 lit./hr
0.84
(ii) Oil Saving per hour = 600- 574 = 26 litre/hr.

(26 x 0.92) x Rs. 20 x 8000 = Rs. 38.27 lakhs

(iii) Reduce excess air


Reduce flue gas temperature (by cleaning the boiler tubes to get rid of
scales or soot )

N -3 In a double pipe heat exchanger hot fluid is entering at 220°C and leaving at
115°C. Cold fluid enters at 20oC and leaves at 75°C. The following data is
provided for hot and cold fluids.
Mass flow rate of hot fluid = 100 kg/hr
Cp of hot fluid = 1.1 kcal/kg°C
Cp of cold fluid = 0.95 kcal/kg°C
_________________________ 8
Bureau of Energy Efficiency
Paper 4 – Energy Auditor – Set B Solutions

(i) Calculate LMTD


a) For parallel flow
b) For counter current flow
(ii) Which flow arrangement is preferable and why?
(iii) Find the mass flow rate of cold fluid if the heat loss during the exchange is 5%.

Ans:

i)
a) LMTD Parallel flow
t1 − t 2
LMTD =
t1
In
t 2
t1 = 200°C
t2 = 40°C
200− 40
LMTD = = 99.41 oC
200
ln
40

b) LMTD Counter current flow


t1 = 145°C
t2 = 95°C
t1 − t 2
LMTD =
t1
ln
t 2
145− 95
= = 118.2°C
145
ln
95

ii) Counter flow is preferred since the LMTD is more the area of the heat
exchanger will be less

iii) Mass flow rate of cold fluid


Data:
m. Mass flow rate of hot fluid = 100 kg/hr
cph. = 1.1 kcal/kg°C
_________________________ 9
Bureau of Energy Efficiency
Paper 4 – Energy Auditor – Set B Solutions

cpC = 0.95 kcal/kg°C


Cold fluid inlet temperature = 20°C
Cold fluid outlet temperature = 75°C
Hot fluid inlet temperature = 220°C
Hot fluid outlet temperature = 115°C
Q = m.hxCPh X th x 0.95 = mc x CPC x tc
mh cPh Th x 0.95
Mass flow rate of cold fluid mc =
cPh x t c
100 x 1.1 x (220 − 115) x 0.95
=
0.95 x (75 − 20)
= 210 kg/hr

N -4 A process plant requires 28 tons of steam per hour and 2250 kW of electric
power. The plant operates for 8000 hours per annum. Steam is generated at 2
bar (g) in a coal fired boiler with an efficiency of 75%. The feed water
temperature is 80OC. The calorific value of coal is 4000 kcal/kg. The cost of coal
is Rs. 2000/ton. Power is drawn from the grid at Rs. 4/kWh. The contract
demand is 3000 kVA with the electricity supply company and the plant is
charged for 100% of the contract demand at Rs. 300/kVA/month. The plant has
never exceeded its contract demand in the past.
The plant is planning for a back pressure cogeneration system using the same
coal with the following parameters. The power and steam demand are to be fully
met by the cogeneration plant and a contract demand of 1000 kVA with the grid
is to be kept for emergency purposes.
Find out the IRR over a project life cycle of 6 years for the proposed
cogeneration system
Cogeneration System data:

Boiler generation pressure - 18 bar (g), 310OC


Boiler efficiency - 79%
Investment required - Rs. 20 crores
Generated power = 2250 kW
Steam enthalpy data:

Total enthalpy at 2 bar (g) = 647.13 kcal/kg


Total enthalpy at 18 bar (g), 310oC = 730.28 kcal/kg

Ans:

Existing condition
Coal consumption 28,000 x [(647.13) – 80]
0.75 x 4000

5293 kg/hr

_________________________ 10
Bureau of Energy Efficiency
Paper 4 – Energy Auditor – Set B Solutions

Fuel cost per annum 5.293 x 2000 x 8000

Rs. 8.5 crores (approx)

Annual electrical energy charges 2250 x 8000 x Rs.4

Rs. 7.2 crores

Maximum demand charges 3000 x 12 x 300

Rs. 1.1 crores (approx)

Total electricity bill per annum 7.2 + 1.1 = Rs. 8.3 crores

Total energy bill per annum 8.5 + 8.3 = Rs. 16.8 crores

With cogeneration plant

Coal consumption 28,000 x [(730.28) – 80]


0.79 x 4000

5762 kg/hr

Incremental coal consumption 5762 – 5293 = 469 kg/hr

Incremental fuel cost per annum 0.469 x 8000 x Rs. 2000

Rs. 0.75 crores

Maximum demand charges per 1000 x 12 x Rs.300


annum Rs. 0.36 crores

Total cost per year 8.5 + 0.75 + 0.36 = 9.61 crores


Savings = 16.8 – 9.61 = Rs. 7.19 crores

Investment Rs. 20 crores

1 1 1 1 1 1
20 = 7.19 + + + + +
(1+i)1 (1+i)2 (1+i)3 (1+i)4 (1+i)5 (1+i)6

IRR = 27 to 28%

-------- End of Section - III ---------

_________________________ 11
Bureau of Energy Efficiency
Paper 4

Model question paper 20006


NATIONAL CERTIFICATION EXAMINATION
FOR
ENERGY AUDITORS

PAPER – 4: ENERGY PERFORMANCE ASSESSMENT FOR EQUIPMENT AND


UTILITY SYSTEMS

Duration: 2 HRS Max. Marks: 100

Section - I: SHORT DESCRIPTIVE QUESTIONS Marks: 10 x 1 = 10

(i) Answer all Ten questions


(ii) Each question carries One marks
(iii) Answer should not exceed 50 words

S-1 How boiler capacity rating is specified and why..

Boilers capacity is always specified in terms of equivalent evaporation


(kg of steam / hour at 100°C). The amount of steam generation - "from
and at" 100°C. It is easy to compare the ratings of different boilers from
different manufactures.

S-2 List any two losses that are difficult to measure while determining the
furnace efficiency evaluation.

The losses that are difficult to measure in the furnace efficiency


evaluation are heat storage loss, loss of furnace gases around
charging door and opening, loss of heat by conduction through hearth
and loss due to formation of scales.

S-3 What is the relation between effectiveness and length of the heat
exchanger for the same duty?

Ratio of the cold fluid temperature range to that of the inlet temperature
difference of the hot and cold fluid. Higher the ratio lesser will be
requirement of heat transfer surface and there by length of heat
exchanger

S-4 Which method is used to find the calorific value of unburned contents in the fly
ash and bottom ash

Gravimetric analysis is used to find calorific value of fly ash and bottom ash

NPC-AIP-2006 1
Paper 4

S-5 How to determine the Friction & wind age loss alone in electric motor
efficiency evolution.?

To separate core and F & W losses, no load test should be repeated at variable
voltages. By plotting no-load input kW versus Voltage, the intercept will indicate F &
W kW loss component.

S-6 Explain the characteristic of power factor in electric motor with VSD and
without VSD drives?

Variable frequency drives should also offer a true system power factor of 0.95
or better across the operational speed range, to save on demand charges, and to protect
the equipment (especially motors). In conventional motor, PF will redce at part load.

S-7 What is the main difference in the system characteristics curves between fans
and pumps distribution network.

There is no static head in FAN system characteristic curves whereas pump


has static head
S-8 How to ensure the accurate measurements in squire duct while measuring the
airflow with pit tot tubes.

• When in use, the pitot tube shall be connected by means of airtight tubes to
a pressure measuring instrument.

• For rectangular duct, let us calculate the traverse points. 16 points are to be
measured. In small ducts or where traverse operations are otherwise
impossible, an accuracy of ± 5% can frequently be achieved by placing
Pitot in center of duct.

S-9 Which of the following is better choice for maximum power generation and
why?
Site A: wind speed of 25 KM per hour & capacity factor 20%
Site B: wind speed of 29 KM per hour & capacity factor 19%

Site-A is better choice because power generation is high for any scapacity of
wind mill

S-10 Compare the advantages between Direct burning process and Biogas
generation foe effective utilization of Cow dung.
Bio gas route has many benefits like energy conversion efficiency is high ,
useful energy is high per kg of Dung plus free manure .

-------- End of Section - I ---------

NPC-AIP-2006 2
Paper 4

Section - II: LONG DESCRIPTIVE QUESTIONS Marks: 2 x 5 = 10

(i) Answer all Two questions


(ii) Each question carries Five marks

L-1 (i) What are the methods of recycling of waste minimization and Give two
example each.

i) On-site Recovery and Reuse - Reuse of wasted materials in the same


process or for another useful application within the industry.
ii) Production of Useful by-product - Modification of the waste generation
process in order to transform the wasted material into a material that can be reused
or recycled for another application within or outside the company.

L-2 In an industrial office building lighting system, the ILER ratio has been
improved from 0.6 to 0.8 for reducing lighting power consumption by modifying
fittings layout. The initial lighting circuit load was 5KW. Calculate the
percentage of waste reduction and annual energy savings if operating period is
10 hours and 360 days /year.

Before modification
Annual energy wastage = (1 - ILER) x K.Watts x no. of operating hours
= (1 - 0.6) x 5 x 10 hrs/day x 360 days
= 7200 kWh/year
After modification
= (1 - 0.8) x 5 x 10 hrs/day x 360 days
= 3600 kWh/year
i) % waste reduction 50%
ii) Annual energy savings= 3600 kwh/year

-------- End of Section - II ---------

Section - III: Numerical Questions Marks: 4 x 20 = 80

(i) Answer all Four questions

NPC-AIP-2006 3
Paper 4

(ii) Each question carries Twenty marks

N -1 A process industry has planned to install a Cogeneration plant with capital


investment of Rs. 18 Crores. The estimated annual energy savings is Rs 6
Crores for the 6 years life cycle. The annual operating cost would be 20 % of
the annual savings. The minimum expected internal rate of return is 18%
considering their capital resources.

I. Find out whether the investment will meet the company expectations.
II. Perform the sensitivity analyses for the following two scenario changes and
suggest the project feasibility.
a. If Annual savings is increased by 20% than expected
b. If annual saving is increased by 20% and operating cost also
increased from the existing 20 to 30 %

Solution :
Investment = 18 Crores
Annual Savings = 6 Crores
Annual Operating Cost = 6x0.2 = 1.2 Crores
Annual Net Cash Flow = 6-1.2 = 4.8 Crores
Expected IRR by the = 18.0%
Company
(i ) NPV = CF0 CF1 CF6
------ + ----- … ….+ ------
(1+r)0 (1+r)1 (1+r)6

1 1 1 1
= -18+ 4.8 { ------ + ------ + --- .. +...----}
(1.18)1 (1.18)2 (1.18)3 (1.18)6
= -18 + 4.8 (3.5573)
= -18 + 17.07
= (-) 0.925
As NPV is negative, it is not meeting the
Company requirement. Therefore project is
not feasible.

(ii) Sensitivity Analysis

Scenario-1

Annual Saving with 20 % raise = 6 Crores x 1.2 = 7.2 Crores


Annual Operating Cost = 20% of Annual Savings
= 7.2 x 1.2 = 1.44
Annual net cash flow = 7.2 – 1.44 = 5.76

NPV = -18 + 5.76 (3.5573)


= -18 – 20.490

NPC-AIP-2006 4
Paper 4

= (+) 2.49 (Positive)

Scenario-2

Annual Saving with 20% raise = 6.0 x 1.22 = 7.2 Crores


Operating Cost = 30% of Annual Savings
= 7.2 x 1.3 = 2.16 Crores
Annual net cash flow = 7.2 – 2.16 = 5.04 Crores

NPV = -18+5.04 (3.5573)


= (-) 0.712

In this case, NPV is Negative

As with sensitivity factor NPV becomes


negative. Hence this project is not feasible.

N -2 A In a five star hotel building air conditioning system, Cold air at 23oc is
supplied from air handling unit. The cold air flow rate is 20,000 M3/hr at
a density of 1.2 Kg/m3 .The inlet and outlet enthalpy of the air are 105
KJ/Kg and 80 KJ/Kg. The COP of the system is 3.75. Hotel
management wants to install Double effective VAR SYSTEM .The
saturated steam at 5kg/cm2 will be supplied either from 500 KVA DG
Sets exhaust gas boiler or from the existing LDO Fuel fired boiler. The
plant is operating for 8760 hr. The investment VAR system is 20 lacs.
The investment for waste heat boiler is 6 lacs. Power cost is Rs 4/KWH.

As an energy auditor which option can be recommended to the hotel


management?
Option1: Supply steam from the existing LDO fired boiler to VAR
system and avoid the investment of waste heat boiler
Option2 - Supply steam from the waste heat boiler, which needed
investment of waste heat boiler in addition to VAR system
The steam consumption per TR will be 4.7 Kg/TR at 5kg/cm2 pressure.
The cost of LDO is Rs.16,800 Ton

Solution :
Existing Base Case VCR System
TR Rating = (20,000m3/hr x 1.2kg/m3) (105-80)KJ/kg
------------------------------------------------------
3024 x 4.187
= 47.38 TR
COP = 3.75

NPC-AIP-2006 5
Paper 4

COP = Refrigeration effect Kcal/hr


-----------------------------------------
Power Input Kcal/hr
Power Input = 47.38 x 3024 Kcal
------------------------------
3.75 x 860
= 44.43 KW
Annual Energy Consumption = 44.43 x 8760 = 3.892 Lakhs KWH
Annual cost in VCR system (Base =3.892 x4= Rs. 15.56 Lakhs
Case)

Option – 1 : VAR System with Steam Supply from Existing Boiler


Steam Consumption /TR = 4.7 kg/TR
Steam Consumption per hr = 4.7 x 47.38 = 222.7 kg/hr

Evaporation Ratio IN THE = 14


EXISTING BOILER
1ton of steam cost = Rs.16800
-------------- = Rs.1200 /Ton of steam
14
Investment for VAR system = Rs.20.00 Lakhs
Electricity cost saving per hr = 44.43 x 4 =Rs. 178
Steam cost per hr = 222.7 x 1.2 = 277
Cost Savings /hr = 178-277 = negative
Hence this option of project is not feasible

Option-2: With VAR & steam supply from WHR steam boiler of DG set

Total Investment = Rs. 20.00 + 6.00 = 26.00 Lakhs


Savings/hr = Rs. 178 – Free steamt = Rs.178
Annual Savings = Rs.178x8760 = Rs.15.60 lakhs
Simple Pay back period = 26/15.6 = 1.67 years.
Solution : Option 2 should be selected

N-3 An FAD test was carried out on a a1000 cfm compressor using nozzle
method .The following are the measurements and results of the test.

Standard Nozzle Diameter selected for 1000cfm capcity : 0.08 m


Flow coefficient of the Nozzle : 0.95
Receiver Pressure maintained = 4.2 kg / cm2 (a)
Inlet Pressure of the - 1.03 kg / cm2(a)
Inlet air temperature 35oC
Pressure before nozzle maintained= 1.06 kg / cm2 a
Temperature before the nozzle = 48oC
Pressure drop across the nozzle = 0.052 kg / cm2
Gas constant : 287 Joules / kg K

NPC-AIP-2006 6
Paper 4

After the test the compressor is taken for maintenance and overhaul. The isothermal
efficiency increased thereafter by 5 percent and the shaft power of compressor was
measured to be 115 kW. The expenditure was Rs. 1.5 lacs. What is the payback period
for the investment if the operating hours are 8000/annum and the energy cost is
Rs.4/kWh

Compressor capacity ,1000 cfm 1700 m3/hr

Nozzle size 0.080 m

1/ 2
T   2( P3 − P4 )(P3 x Ra ) 
Free air delivered, Q f (m / sec)= k x x d x 1
s 2
x  
4 P1  T3 


1/ 2
308  2 x 0.052 x (1.06 x 287) 
= 0.95 x x (0.080) 2 x x 
4 1.03  321 

= 0.44 m3/s

= 1614 m3/hr

P1 x Q f x loge r
Isothermal power (kW ) =
36.7

Isothermal power 1.03 x 1614 x ln (4.2/1.03)


36.7

63.7 kW

New power drawn 115 kW

New isothermal efficiency = 63.7 / 115 = 55.4 %

Old isothermal efficiency = 55.4 / 1.05 = 52.76

Shaft power drawn earlier 63.7 / 0.5276

120.7 kW

Annual savings = (120.7 – 115) x 8000 x 4

Rs. 1,82,400

Payback period 1.5/1.824=10 months

NPC-AIP-2006 7
Paper 4

N-4 A 10 Ton per hour heat treatment furnace is operating for 20 days
continuously in month. An air preheater was installed to preheat the
combustion air temperature. The measurements were taken before and
after WHR implementation
Before Installation of Air preheater
Fuel oil Consumption = 620 litre/hr
Specific Gravity = 0.88
Furnace Operating Temperature = 1000oC
Oxygen % at outlet of furnace = 4.5%

Measurement after Installation of Air preheater


%O2after Waste Heat Recovery = 4.5%
Investment for WHR = Rs. 100.00 lakhs
Flue gas temperature after WHR = 600 oC
Ambient temperature of air = 30oC
Fuel oil cost = Rs.17,000/ KL
GCV of fuel oil = 10200 kCal/kg

Find out pay back period for the investment?

Solution :

Existing loss in flue gas :

02 in Flue Gas before WHR = 4.5%


% Excess Air = 4.5 x 100 = 27.3%
21-4.5

Fuel Consumption = 620 lit. x 0.88 = 545.6kg/hr


Dry flue gas loss % = [14x1.273)+ 1] x .24(1000-30) x100
10200
= 40.6%

After WHR installation

After WHR Installation % O2 in flue gas = 4.5%

= 4..5 x 100 = 27.3 %


21-4.5

Dry flue gas loss = [ (14x1.27.3) +1] x0.24 (600-30) x100


10200
= 25.24%

NPC-AIP-2006 8
Paper 4

% Reduction in the dry flue gas loss = 40.6-25.24 = 15.36%

Fuel oil saving/hr = 620 lit/hr x 15.36 = 95.22 lit/hr


100
Operating hours = 20 days x 24 hrsx12months =5760 hrs
Annual fuel oil saving = 5760 x 95.22 = 548467 lit =548.5KL /Year
Annual cost saving = 548.5 KLx 17000 = 93.245 Lakhs

Simple Pay back period = 100 / 93.245 = 1.07 YEARS

End

NPC-AIP-2006 9
MODEL QUESTION PAPER - 2006

Paper 4 –Energy performance Assessment for Equipment and Utility Systems

Duration : 2 Hrs. Max. Marks: 100

Section-1: Short Questions Marks: 10x1 = 10

(i) Answer all Ten questions


(ii) Each question carries One mark

S-1 What is the effect of wet steam on process heating?


The total heat available for heating is reduced and hence the heating time increases.

S-2 Which is the predominant parameter which decides blow down quantity in a boiler?
The total dissolved solids(TDS) of boiler water dictates the extent of blowdown
required.

S-3 Which are the losses that can be controlled in case of furnace operations?
The heat losses due to openings,skin,and the sensible heat loss in flue gases.

S-4 The pipeline carrying steam had a plaster finish(Emmisivity E=0.85).This was
replaced by aluminium cladding finish(Emissivity E=0.2).The heat loss from the surface;
i) Increases ii)Decreases iii)Remains same.
Decreases.

S-5 What is a Mollier diagram?


The mollier diagram is a chart with Enthalpy(H) and Entropy(S)and used in
estimation of heat extraction in turbines.

S-6 How do you identify the ‘fouling” in a heat exchanger?


Increase in pressure drop and reduction of heat flux.

S-7 A 4 pole motor is operating at a frequency of 50Hz.What is the motor RPM?


RPM=(50*120)/4 =1500.

S-8 What are the limitations of the simple pay back period?
The payback method does not consider savings that are accrued after the payback
period has finished.The metod also does not consider the time value of money.

S-9 What are the limitations of “Motor Rewinding”?


The efficiency of motor comes down after rewinding.The no-load current increases.

S-10 Under what situations are solar PV systems economical?


In hilly and far flung areas where the conventional grid power supply system are
difficult to reach solar PV systems are found to be economical.

Section –II Long Questions Marks: 2x5 =10.

i) Answer all questions.


ii) Each question carries Five marks.

L-1 What possible improvements would you look for in a general lighting system?
• Look for natural lighting opportunities through windows and other openings.
• Explore the scope of introducing translucent sheets.
• Assess scope for more energy efficient lamps and luminaries.
• Assess scope for rearrangement of lighting fixtures.

L-2 List the various sources of heat that need to be considered for a room to be air-
conditioned?
• Heat ingress from roof.
• Heat ingress from walls.
• Heat load due to lamps.
• Heat load due to people working.
• Heat load due to machinery.

Section –III: Numerical Questions Marks: 4x20 = 80

i) Answer all Four questions.


ii) Each question carries Twenty marks.

N-1. A process plant has a back pressure turbine for power generation.The electrical power
output from turbine is 0.5MW.The process data are as under:
• The plant heat rate is 35,000 K.Cal/Kwh.
• Enthalpy of steam at inlet of turbine is 700 K.Cal/Kg.
• Enthalpy of feed water is 50K.Cal/Kg.
• Enthalpy of steam at outlet of turbine is 650 K.Cal/Kwh.
• Evaporation ratio of Boiler is 3.7 Kgs of steam/Kg of coal.

Calculate the following:


a) Flow rate of steam to turbine.
b) Power generation efficiency of turbo-alternator.
c) Overall plant fuel rate including boiler.

a) Overall plant heat rate


= Steam flow rate x (Enthalpy of steam –Enthalpy of feed water)/Power Output
Substituting values 35,000=Flow rate of steam x(700-50)/1000.
Flow rate of steam = 35000*500/650 =26,923 Kgs/hr.
b) Power generation efficiency of turbo-alternator :
Heat energy input into turbine per Kg of inlet steam =700-650 =50 K.Cal/Kg.
Energy Input into turbine =26923*50 =1346150 K.Cal/hr.
Power generation from turbine =500KW.
Equivalent thermal energy output = 500*860 =430000 K.Cal/hr.
Power Generation efficiency = 430000/1346150 = 31.94%

c) Evaporation ratio of boiler =3.7 Kg of steam/Kg of coal.


Flow rate of steam from boiler = 26923 kgs/hr.
Hence fuel flow rate to boiler = 26923/3.7 =7276 Kgs of coal/hr.
Overall plant fuel rate including boiler = 7276/500 =14.55 Kgs of coal/Kw

N-2 The following parameters were observed during the testing of a cooling water pump:
• Discharge pressure of pump : 4.0 Kg/cm2
• Flow rate of water : 600 m3/hr.
• Suction head : 1 metre below the pump centerline.
• Height of cooling tower : 5 metre
• Motor efficiency : 88%.
• Density of water : 1000 Kg/m3
• Current drawn by pump motor : 140 amps.
• Voltage measured : 415volts.
• Power factor measured : 0.9
Calculate the following:
a) Hydraulic power.
b) Pump Efficiency.

a) Hydraulic power = (600/3600)*(40-1)*1000*9.81/1000


= 63.76 KW.

b) Pump Efficiency :
Input Power to motor =1.732*0.415*140*0.9 =90.6KW
Pump shaft power = 90.6*0.88 = 79.73 KW.
Pump efficiency = (63.76/79.73)*100 = 79.97%

N-3 Field tests were carried out on a chilling system to find out the energy performance
ratios.The measurements data are as under:
* Temperature of the refrigerant entering the evaporator : (-1) 0C.
* Temperature of the refrigerant leaving the evaporator : (-5) 0C.
* Flow rate of the refrigerant : 20000Kgs/hr.
* Specific heat of the refrigerant : 2.4Kcal/Kg0C.
* Current input into compressor : 125A
* Voltage input measured : 415 V
* Power factor : 0.9
Find out the following :
a) Net refrigeration capacity.
b) Kw/ton rating.
c) Coeffient of performance (COP).
d) Energy Efficiency ratio.(EER).

a) Net refrigeration capacity = 20000*2.4*(-1 –(-5))/3024


= 63.49 TR.
b) Power input to compressor = 1.732*0.415*125*0.9 =80.86KW.
Kw/Ton rating = 80.86/63.49 = 1.27

c) COP =3.516/1.27 =2.77.


d) EER = 12/1.27 = 9.45

N-4 (i) What is the total weight of flue gas generated when 20Kg of butane(C4H10) is burned .
The % O2 measured in the flue gas is 3.0.
i) The flue gas is cooled from 500oC to 200 oC for generating steam. The total heat
required per Kg of steam generation is 500 K.Cal/Kg. Find out the quantity of
steam generated. Additional information as under:
Atomic Weights C=12, H=1, O=16;
Specific heat of flue gas =0.24 K.cal/Kg/ oC
Assume air is 77% Nitrogen (N2) & 23% Oxygen (O2)

2C4H10 +13O2 =8CO2 +10H2O


2(48+10) +13(32)=8(12+32)+10(2+16)
116Kg butane requires 416 Kg of Oxygen
20 Kg of butane requires 71.72 Kg of O2.
Air requirement is (100/23)*71.72
: 311.8Kg.
% O2 measured in the flue gas is 3.0%.
% Excess air : 3/(21-3) =16.67%.
Hence excess air quantity : (16.67/100)*311.8
: 51.98Kg.
Total flue gas quantity :311.8+51.98+20
: 383.78Kg.
Quantity of steam generated : 383.78*0.24*(500-200)/500
: 55.3 Kg of steam
MODEL QUESTION PAPER - 2006

Paper 4
Duration : 2 Hrs Max. Marks: 100

Section-1: Short Questions Marks: 10x1 = 10

(i) Answer all Ten questions ii) Each question carries One mark

S-1 What are the losses not accounted for in the Indirect method of testing boiler efficiency?
• Blowdown loss
• Soot blower steam consumption.
• Energy usage by auxiliary equipments like burners.

S-2 The unit of the overall coefficient of heat transfer is:


a)k.cal/m2hr b)k.cal/ m2hr0C c)k.cal/kg0C

S-3 What is renewable energy?


The renewable energy comes directly or indirectly from sun and wind and can never
be exhausted. Hence they are called renewable.

S-4 What is the difference between net present value method and internal rate of return
method?
The net present value method is essentially a comparison tool which enables a number
of projects to be compared whereas internal rate of return method is designed to assess
whether or not a single project will achieve a target rate of return.

S-5 What is anaerobic digestion?

Anaerobic digestion involves using bacteria to decompose organic matter in the


absence of oxygen.

S-6 What parameter in a psychometric chart is used for calculating refrigeration load in
an air conditioning system?

Enthalpy (inlet and outlet air)

S-7 What is the Capacity factor of wind turbine?


The capacity factor is the ratio of the wind turbines actual energy output for the year
divided by the energy output if the machine operated at it’s rated power output for the
entire year.
S-8 The losses in a variable speed drive is:
a) 12% b) 8% c) 5%
c) 5%

S-9 The efficiency of a solar cell is a) 5% b) 15% c)30%.


b)15%.

S-10 What is meant by “Solar window”?


The period of 4-5 hours in late morning and early afternoon (9am-3pm) is commonly called
the solar window.

Section –II Long Questions Marks: 2x5 =10.

i) Answer all questions.


ii) Each question carries Five marks.

L-1 The sensible heat loss in flue gas is very high in furnace applications. Explain?
For effective heat transfer from the gas to the furnace the temperature of the flue gas
should be at least 500C higher than the operating temperature. Hence the flue gas
temperatures are high leading to increased sensible heat loss.

L-2 What are the advantages of the solar water heater?


• There is saving in heating costs.
• Saving in electricity/gas which are used for heating applications.
• Solar energy is most readily available.
• Solar energy is a renewable source of energy.

Section –III: Numerical Questions Marks: 4x20 = 80

i) Answer all Four questions.


ii) Each question carries Twenty marks.

N-1. A process plant has a back pressure turbine for power generation.The electrical power
output from turbine is 1MW.The process data are as under:
• The plant heat rate is 33,000 K.Cal/Kwh.
• Enthalpy of steam at inlet of turbine is 700 K.Cal/Kg.
• Enthalpy of feed water is 70K.Cal/Kg.
• Enthalpy of steam at outlet of turbine is 650 K.Cal/Kwh.
• Evaporation ratio of Boiler is 3.5 Kgs of steam/Kg of coal.

Calculate the following:


a) Flow rate of steam to turbine.
b) Power generation efficiency of turbo-alternator.
c) Overall plant fuel rate including boiler.

a) Overall plant heat rate


= Steam flow rate x (Enthalpy of steam –Enthalpy of feed water)/Power Output
Substituting values 33,000=Flow rate of steam x(700-70)/1000.
Flow rate of steam = 33000*1000/630 =52,381 Kgs/hr.

b) Power generation efficiency of turbo-alternator :


Heat energy input into turbine per Kg of inlet steam =700-650 =50 K.Cal/Kg.
Energy Input into turbine =52381*50 =26,19050 K.Cal/hr.
Power generation from turbine =1000KW.
Equivalent thermal energy output = 1000*860 =860000 K.Cal/hr.
Power Generation efficiency = 860000/26,19050 = 32.84%

c) Evaporation ratio of boiler =3.5 Kg of steam/Kg of coal.


Flow rate of steam from boiler = 52,381 kgs/hr.
Hence fuel flow rate to boiler = 52381/3.5 =14,966 Kgs of coal/hr.
Overall plant fuel rate including boiler = 14,966/1000 =14.97 Kgs of coal/Kw

N-2 The following parameters were observed during the testing of a cooling water
pump:
• Discharge pressure of pump : 5.0 Kg/cm2
• Flow rate of water : 900 m3/hr.
• Suction head : 2 metre below the pump centerline.
• Height of cooling tower : 5 metre
• Motor efficiency : 90%.
• Density of water : 1000 Kg/m3
• Current drawn by pump motor : 260 amps.
• Voltage measured : 415volts.
• Power factor measured : 0.9
Calculate the following:
a) Hydraulic power.
b) Pump Efficiency.

a) Hydraulic power = (900/3600)*(50-2)*1000*9.81/1000


= 117.72 KW.

b) Pump Efficiency :
Input Power to motor =1.732*0.415*260*0.9 =168.2KW
Pump shaft power = 168.2*0.9 = 151.38 KW.
Pump efficiency = (117.72/151.38)*100 = 77.76%

N-3 Field tests were carried out on a chilling system to find out the energy performance
ratios.The measurements data are as under:
* Temperature of the refrigerant entering the evaporator : (-2) 0C.
* Temperature of the refrigerant leaving the evaporator : (-5) 0C.
* Flow rate of the refrigerant : 15000Kgs/hr.
* Specific heat of the refrigerant : 2.3Kcal/Kg0C.
* Current input into compressor : 70A
* Voltage input measured : 415 V
* Power factor : 0.9
Find out the following :
a) Net refrigeration capacity.
b) Kw/ton rating.
c) Coeffient of performance (COP).
d) Energy Efficiency ratio.(EER).

a) Net refrigeration capacity = 15000*2.3*(-2 –(-5))/3024


= 34.23 TR.
b) Power input to compressor = 1.732*0.415*70*0.9 =45.28KW.
Kw/Ton rating = 45.28/34.23 = 1.32
c) COP =3.516/1.32 =2.66.
d) EER = 12/1.32 = 9.09

N-4 (i) What is the total weight of flue gas generated when 10Kg of butane(C4H10) is burned .
The % O2 measured in the flue gas is 2.0.
ii) The flue gas is cooled from 600oC to 200 oC for generating steam. The total heat
required per Kg of steam generation is 550 K.Cal/Kg. Find out the quantity of steam
generated.
Additional Information:
Atomic Weights C=12, H=1, O=16;
Specific heat of flue gas =0.24 K.cal/Kg/ oC
Assume air is 77% Nitrogen (N2) & 23% Oxygen (O2)

2C4H10 +13O2 =8CO2 +10H2O


2(48+10) +13(32)=8(12+32)+10(2+16)
116Kg butane requires 416 Kg of Oxygen
10 Kg of butane requires 35.86 Kg of O2.
Air requirement is (100/23)*35.86
: 155.9Kg.
% O2 measured in the flue gas is 2.0%.
% Excess air : 2/(21-2) =10.53%.
Hence excess air quantity : (10.53/100)*155.9
: 16.42Kg.
Total flue gas quantity :155.9+16.42+10
: 182.32Kg.
Quantity of steam generated : 182.32*0.24*(600-200)/550
: 31.8 Kg of steam
Paper 4 – Energy Auditor – Set A

NATIONAL CERTIFICATION EXAMINATION 2005


FOR
ENERGY AUDITORS
Question Papers & Model solutions to the Question Papers
PAPER – 4: ENERGY PERFORMANCE ASSESSMENT FOR EQUIPMENT AND
UTILITY SYSTEMS

Date: 29.05.2005 Timings: 1400-1600 HRS Duration: 2 HRS Max. Marks: 100

General instructions:
o Please check that this question paper contains 4 printed pages
o Please check that this question paper contains 16 questions
o The question paper is divided into three sections
o All questions in all three sections are compulsory
o All parts of a question should be answered at one place

Section - I: SHORT DESCRIPTIVE QUESTIONS Marks: 10 x 1 = 10

(i) Answer all Ten questions


(ii) Each question carries One marks
(iii) Answer should not exceed 50 words

S-1 If the power consumed by a refrigeration compressor is 2 kW per ton of


refrigeration, what is the energy efficiency ratio?

12000 Btu
EER = =6
2000 W

S-2 When using a chemical cell oxygen measuring device for stack gas analysis,
state the equation to find out the excess air in %?

O2%
EA = x 100 %
21 - O2 %

S-3 Which has more energy content, 1 kg of Hydrogen or 1 kg of gasoline?

1 kg of Hydrogen

S-4 Write the overall heat transfer coefficient U, as a function of sensible heat
(qs) and latent heat (qL).

U = (qs + qL ) / (A x LMTD)

_________________________ 1
Bureau of Energy Efficiency
Paper 4 – Energy Auditor – Set A

S-5 Which loss is considered the most unreliable or complicated to measure in


electric motor efficiency testing?

The stray load loss, because this loss is only estimated and not
measured, the method to measure is very complicated and rarely used
on the shop floor.

S-6 The inclined manometer connected to a pitot tube is sensing which pressure in
a gas stream?

The difference between total and static pressure (also called velocity
pressure)

S-7 When using an ultrasonic flow meter for flow measurements in a water pipe
which major additional parameter must be guessed or known to calculate the
flow in cubic meter per second.

The actual inner diameter to calculate the free cross-sectional area of the
pipe.
S-8 What is the correction factor for actual free air discharge in a compressor
capacity test if compressed air discharge temperature is 150 C higher than
ambient air? Assume ambient air = 400 C.

Factor is (273 + 40) / (273 + 55) = 0.954

S-9 Which expression to state the energy efficiency of a chiller does not follow the
trend “a higher number means a more efficient system”?

The expression “power per ton” (in kW/ton) does not follow the trend.
The higher the number the more inefficient the chiller.

S-10 What have all boiler efficiency testing standards in common?

They do not include blow down as a loss in the efficiency determination


process.

-------- End of Section - I ---------

Section - II: LONG DESCRIPTIVE QUESTIONS Marks: 2 x 5 = 10

(i) Answer all Two questions


(ii) Each question carries Five marks

L-1 (i) List any four common losses of boilers and furnaces.
(ii) Which loss is unique to boilers and does not occur in furnaces?

(i) a) radiation losses


b) dry flue gas losses
c) losses due to moisture in the fuel

_________________________ 2
Bureau of Energy Efficiency
Paper 4 – Energy Auditor – Set A

d) losses due to Hydrogen in the fuel that forms water with Oxygen
in the combustion air
e) losses due to partial combustion of Carbon to CO
f) losses due to remaining carbon in the residue (ash)
g) losses due to humidity in the air
(ii) Blow down losses occur only in boilers

L-2 The suction head of a pump is 5 m below the pump centerline. The discharge
pressure is 3 kg/cm2. The flow rate of water is 100 m3 /hr. Find out the pump
efficiency if the actual power input at the shaft is 12 kW.

Discharge Head : 3 kg/cm2 equals 30 metre head.


Suction Head : - 5 metre.
Total Head : 30 – (-5) = 35 metre.
Hydraulic Power : (100/3600) x 1000 x 9.81 x 35/1000 = 9.54 kW
Pump Efficiency : 100 x 9.54/12 = 79.5%

-------- End of Section - II ---------

Section - III: Numerical Questions Marks: 4 x 20 = 80

(i) Answer all Four questions


(ii) Each question carries Twenty marks

N -1 You as an energy auditor have the task to quickly assess within 20 minutes the
technical/ financial performance of a paddy husk fired power plant to be
installed.

The plant owner provided you the following information.

• Nominal capacity : 7 MW
• Assumed plant load factor : 0.75
• Number of hours of operation : 8760/ year
• Analysis of paddy husk

Fuel property Weight %


Moisture 10.79
Mineral Matter 16.73
Carbon 33.95
Hydrogen 5.01
Nitrogen 1.00
Oxygen 32.52
GCV (kCal/kg) 3,568

_________________________ 3
Bureau of Energy Efficiency
Paper 4 – Energy Auditor – Set A

Provide solutions to the following to the plant owner.

(i) Tonnes of paddy husk to be fired per year if the power plant has an overall
efficiency of 25%.
(ii) The area required in square meters to store an inventory of paddy husk 30 cm
high for 4 days of operation. Assume paddy husk bulk density of 100 kg/m3.
(iii) Power plant capital cost is Rs. 28 crores and rice husk cost as delivered is Rs.
1500/ tonnne. Annual repair, maintenance and operation costs are 10% of
capital costs. What is the simple pay back period if electricity is sold at
Rs.3/kWh.

(i) Paddy husk energy needed per hour = 860 kCal x 7000 x 0.75
0.25
=18,060,000 kCal / hour

Tons of husk per hour = 18,060,000 = 5.06 tons/hour


1000 x 3568

Tons per year 5,06 x 8760 = 44,340 tons/year

(ii) Tons per day: 44,340/365 = 121.48 tons per day or


4 x 121.48 = 486 tons for four days, or
486 tons = 4859 m3 = a2 x 0.30 m
0.1 ton/ m3
Area = 16,197 m2

(iii) Annual revenue cash flow 7,000 x 8,760 x 0.75 x 3 Rs./kWh = Rs. 137,970,000
Annual R&M cost, 0.1 x Rs.28 crores = Rs. - 28,000,000

Fuel costs 44,340 x 1500 = Rs. - 66,510,000


Annual return = Rs. 43,460,000
Pay back period = 280,000,000 = 6.44 years
43,460,000

N -2 A performance evaluation of a large air fan resulted in the following data.

Pitot tube measurement average


velocity pressure : 75 mm water column
Suction pressure : - 20 mm water column
Outlet pressure : 480 mm water column
Area of duct : 8 square meter
Pitot tube constant : 0.85
Corrected gas density : 1.15 kg/ m3

(i) Calculate flow in m3/sec.

_________________________ 4
Bureau of Energy Efficiency
Paper 4 – Energy Auditor – Set A

(ii) Calculate the static fan efficiency based on the following 3 phase motor data.
Line current : 100 Amps
Line voltage : 11,000 volts
Power factor of electric motor : 0.9
Efficiency of motor at the operating load is 95%.

SQRT (2 x 9.81 x P x )
(i) Flow (v) = Cp x A

Cp = Pitot tube constant
A = Area of duct
P = Pitot tube measurement average velocity pressure
 = Corrected gas density
SQRT (2 x 9.81 x 75 x 1.15)
Flow (v) = 0.85 x 8
1.15
SQRT (1692)
Flow (v) = 0.85 x 8 = 243.2 m3/sec
1.15

(ii) Power input to fan = 1.732 x V x I x Power factor/ 1000


= 1.732 x 11000 x 100 x 0.95/ 1000 = 1628.946 kW
Static fan efficiency = (243.2 x 500) / (102 x 1628.946) = 73.2%

N-3 A multi-storied shopping mall has installed 5 x 110 TR reciprocating


compressors of which four compressors are in use and fully loaded for 14
hours per day. The specific power consumption of reciprocating compressor
is 0.8 kW/TR. Due to higher energy cost the shopping mall chief engineer has
decided to replace reciprocating compressors with screw compressors having
specific power consumption of 0.65 kW/TR. The chief engineer needs
following input from energy consultant:

(i) Comparison of power and electricity consumption of both reciprocating


and screw compressors?
(ii) Annual energy bill savings (for 320 days operation). Present unit cost is
Rs 6.00 per kWh
(iii) What should be the size of cooling tower required for proposed screw
compressors?
(i) Operating reciprocating compressors capacity : 440 TR
Sp. Power consumption of compressor (reciprocating) : 0.8 kW/TR
Power consumption per hour : 0.8 x 440 = 352 kW
Required screw compressor capacity : 440 TR
Sp. Power consumption of compressor (screw) : 0.65 kW/TR
Power consumption per hour : 286 kW

_________________________ 5
Bureau of Energy Efficiency
Paper 4 – Energy Auditor – Set A

By replacement of reciprocating compressors with screw compressors


reduction in power is 66 kW and in consumption 924 kWh/day.

(ii) Reduction in power consumption : 66 kW


Operating hours : 14 per day
Operating days : 320 days per year
Annual energy savings : 66 x 14 x 320
: 2.957 lakh kWh
Annual cost savings : Rs 2.957 x6 = Rs. 17.742 lakh

(iii) Operating refrigeration load : 440 TR


Vapour compression type refrigeration systems condenser load (TR) will
be around 1.2 time of evaporator load (TR)
Cooling tower capacity : 1.2 x 440 TR
: 528 TR

N-4 (i) What is the total weight of flue gas generated when 20 kg of Methane
(CH4 ) is burned with 10% excess air?
(ii) How much heat will be recovered from the flue gas by providing an
additional water heater if the flue gas is cooled from 3000 C to 1400 C?

Additional Information:

Atomic weights C=12, H = 1, O = 16;


specific heat of flue gas = 0.24 kCal/kgoC).
Assume combustion air is 77% Nitrogen (N2 ) and 23% Oxygen (O2 ) by weight.

CH4 + 2O2 = 2 H2 O + CO2


(a) (12+4) + 2(32) = 2(2+16) + (12+32)

16 kg of Methane require 64 kg of O2

20 kg of Methane require 64 = 80 kg of O2
16
Therefore, Air (theoretical) required = 100 x 80
23
= 347.8 kg

Excess Air @ 10% = 34.78 kg

Therefore, 347.8 + 34.78 + 20 = 402.6 kg of stack gas

Or long calculation

CO2 produced = 44 x20 = 55 kg


16

_________________________ 6
Bureau of Energy Efficiency
Paper 4 – Energy Auditor – Set A

H2 O produced = 36 x 20 = 45 kg
16

Nitrogen (in theoretical air) = 347.8 x77 = 267.81 kg.


100

Total Flue Gas = CO2 + H2 O + N2 + Excess Air

= 55+ 45 + 267.81 + 34.78 = 402.6 kg of stack gas


(b) Heat recovered = m cp t = 402.6 x 0.24 x (300-140)
= 15,459.8 kCal

-------- End of Section - III ---------

_________________________ 7
Bureau of Energy Efficiency
Paper 4 – Energy Auditor – Set B

NATIONAL CERTIFICATION EXAMINATION 2005


FOR
ENERGY AUDITORS
Question Papers & Model solutions to the Question Papers
PAPER – 4: ENERGY PERFORMANCE ASSESSMENT FOR EQUIPMENT AND
UTILITY SYSTEMS

Date: 29.05.2005 Timings: 1400-1600 HRS Duration: 2 HRS Max. Marks: 100

General instructions:
o Please check that this question paper contains 4 printed pages
o Please check that this question paper contains 16 questions
o The question paper is divided into three sections
o All questions in all three sections are compulsory
o All parts of a question should be answered at one place

Section - I: SHORT DESCRIPTIVE QUESTIONS Marks: 10 x 1 = 10

(i) Answer all Ten questions


(ii) Each question carries One marks

S-1 What have all boiler efficiency testing standards in common?

They do not include blowdown as a loss in the efficiency determination


process.

S-2 For which fuel the difference between GCV and LCV will be smaller, Coal or
Natural Gas?

Coal

S-3 The overall gas turbine efficiency is defined as  = ?


(Express the equation in units of kW and kWh)

 = power output , kW_________________________________


fuel input to gas turbine in kg/h x GCV of fuel in kWh/kg

In case GCV is expressed in kCal/kg – give ½ mark.

S-4 The more fouling fluid should be on which side of a shell & tube heat
exchanger?

Tube side (because it is easier to clean)

S-5 Which loss is considered the most unreliable or complicated to measure in


electric motor efficiency testing?

_________________________ 1
Bureau of Energy Efficiency
Paper 4 – Energy Auditor – Set B

The stray load loss, because this loss is either estimated or not
measured, because the method to measure stray load loss is very
complicated and rarely used on the shop floor.

S-6 Which pressure is sensed in a gas stream by an inclined manometer


connected to a pitot tube?

The difference between total and static pressure also called velocity
pressure.

S-7 What would you call, one lumen per square metre?

1 Lux.

S-8 What is the correction factor for actual free air discharge in a compressor
capacity test if compressed air temperature is 100 C higher than ambient air?
(Assume ambient air = 400 C)

Factor is (273 + 40) / (273 + 50) = 0.969

S-9 In the performance assessment of a refrigeration system, which performance


ratio (energy efficiency) does not follow the trend “a higher ratio means a more
efficient refrigeration system”?

The expression “power per ton” (in kW/ton) does not follow the trend.
The higher the number the more inefficient the chiller.

S-10 Why does a wind generator produces less power in summer than in winter at
the same wind speed?

The wind generator produces less power in summer because the air
density in summer is lower due to warmer temperatures. (Or because the
air density in winter is higher due to lower temperature)

-------- End of Section - I ---------

Section - II: LONG DESCRIPTIVE QUESTIONS Marks: 2 x 5 = 10

(i) Answer all Two questions


(ii) Each question carries Five marks

L-1 The suction head of a pump is 5 m below the pump centerline. The discharge
pressure is 3 kg/cm2. The flow rate of water is 100 m3 /hr. Find out the pump
efficiency if the actual power input at the shaft is 15 kW.

Discharge Head : 3 kg/cm2 equals 30 metre head.


Suction Head : - 5 metre.
Total Head : 30 – (-5) = 35 metre.
Hydraulic Power : (100/3600) x 1000 x 9.81 x 35/1000 = 9.54 kW

_________________________ 2
Bureau of Energy Efficiency
Paper 4 – Energy Auditor – Set B

Pump Efficiency : 100 x 9.54/15 = 63.6%

L-2 (i) List any four common losses of boilers and furnaces.
(ii) Which loss is unique to boilers and does not occur in furnaces

(i) a) radiation losses from the boiler hull


b) dry flue gas losses in the stack gas
c) losses due to moisture in the fuel
d) losses due to Hydrogen in the fuel that forms water by reacting
with Oxygen of in the combustion air
e) losses due to incomplete combustion of Carbon to CO
f) losses due to remaining carbon in the residue (ash)
g) losses due to humidity in the air
(ii) Blow down losses occur only in boilers

-------- End of Section - II ---------

Section - III: Numerical Questions Marks: 4 x 20 = 80

(i) Answer all Four questions


(ii) Each question carries Twenty marks

N -1 An energy auditor or energy manager has the task to quickly assess within 20
minutes the technical/ financial performance of a newly installed paddy husk
power plant.

The owner of the proposed plant provided the following information.

• Nominal capacity of power plant: 5 MW


• Plant load factor: 0.70
• Analysis of paddy husk, as given below

Fuel property Weight %


Moisture 10.79
Ash 16.73
Carbon 33.95
Hydrogen 5.01
Nitrogen 1.00
Oxygen 32.52
GCV (kCal/kg) 3,568

What solutions will be provided by the energy auditor to the plant owners on the
following.

_________________________ 3
Bureau of Energy Efficiency
Paper 4 – Energy Auditor – Set B

(i) Tonnes of paddy husk fired per year if the power plant has an efficiency of
25% measured by the direct method.
(ii) The storage area required in square meters to store an inventory of paddy
husks 30 cm high for 4 days of operation. Assume paddy husks bulk density of
100 kg/m3.
(iii) Power plant capital cost is Rs. 20 crore and paddy husks cost as delivered is
Rs. 1200/ tonne. Annual repair, maintenance and operation costs are 10% of
capital cost. What is the simple pay back period if power is sold at Rs.3/kWh.

(i) Paddy husk energy needed per hour = 860 kCal x 5000 x 0.70
0.25
=12,040,000 kCal / hour

Tons of husk per hour = 12,040,000 = 3.3744 tons/hour


1000 x 3568

Tons per year 3.3744 x 8760 = 29,560 tons/year

(ii) Tons per day: 29,560/365 = 81 tons per day or

4 x 81 = 324 tons for four days, or

324 tons = 3240 m3 = a2 x 0.30 m


0.1 ton/ m3

Area = 10,800 m2 or about one hectare land

(iii) Annual revenue cash flow 5,000 x 8,760 x 0.7x 3 Rs./kWh = 91,980,000

Annual R&M cost, 0.1 x Rs.20 crores = - 20,000,000

Fuel costs 29,560 x 1200 = Rs. - 35,472,000

Annual return = Rs. 36,508,000

Pay back period = 200,000,000 = 5.478 years


36,508,000

N-2 A 15 kW rated motor burns out. The financial manager of the firm wants to
rewind the motor for Rs.3000 to save money. The Energy Manager wants to
buy a new premium motor for Rs.20,000/- after selling motor for Rs. 5,000. He
claims that he can save much more money in the next five years than the cost
difference of the above two options. Other data is as under:

Operating hours/year = 8000


Rewound motor efficiency = 89%
New premium motor efficiency = 93%
Motor loading = 75%
Power cost = Rs.4/kWh

_________________________ 4
Bureau of Energy Efficiency
Paper 4 – Energy Auditor – Set B

(i) How much money does the energy manager actually save over 5 years
and what is the simple pay back period ?
(ii) The financial manager claims the financial risk is still too high because
operating hours may go down drastically in the next years. How many
operating hours/year are required to recover the cost difference within 5
years.

(i) Electricity cost savings over 5 years:

15 x 0.75 (100 _ 100) x 8,000 x 4 x 5 = Rs.86,988.


( 89 93)

Pay back period = (20,000 – 5,000)


(86988/ 5)
= 0.86 years (or) 10.3 months

(ii) 15 x 0.75 (100 _ 100) x “Hours” x 4 x 5 = Rs.15,000


( 89 93)

“Hours” = 1379 hours per year.

N-3 (i) What is the total weight of flue gas generated when 10 kg of Methane
(CH4 ) is burned with 10% excess air?
(ii) How much heat will be recovered from the flue gas by providing an
additional water heater if the flue gas is cooled from 3500 C to 2100 C?

Additional Information:

Atomic weights C=12, H = 1, O = 16;


specific heat of flue gas = 0.24 kcal/kgoC).
Assume combustion air is 77% Nitrogen (N2 ) and 23% Oxygen (O2 ) by weight.

CH4 + 2O2 = 2 H2 O + CO2


(a) (12+4) + 2(32) = 2(2+16) + (12+32)

16 kg of Methane require 64 kg of O2

10 kg of Methane require 64 = 40 kg of O2
16
Therefore, Air (theoretical) required = 100 x 40
23
= 173.91 kg.

Excess Air @ 10% = 17.39 kg

Therefore, 173.91 + 17.39 + 10 = 201.3 kg of stack gas

Or long calculation

CO2 produced = 44 x10 = 27.5 kg


16

_________________________ 5
Bureau of Energy Efficiency
Paper 4 – Energy Auditor – Set B

H2 O produced = 36 x 10 = 22.5 kg
16

Nitrogen (in theoretical air) = 173.91 x77 = 133.91 kg.


100

Excess Air = 17.39 kg.

Total Flue Gas = CO2 + H2 O + N2 + Excess Air

= 27.5 + 22.5 + 133.91 + 17.39 = 201.30 kg of stack gas

(b) Heat recovered = m cp t = 201.30 x 0.24 x (350-210)


= 6763.7 kCal

N-4 A multi-storied shopping mall has installed 5 x 110 TR reciprocating


compressors of which four compressors are in use and fully loaded for 16
hours per day. Specific power consumption of reciprocating compressor is 0.8
kW/TR. Due to higher energy cost shopping mall chief engineer has decided
to replace reciprocating compressors with screw compressors having specific
power consumption on 0.65 kW/Tk. Chief engineer need following input from
energy consultant.

(i) Comparison of power and electricity consumption of both reciprocating


and screw compressors?
(ii) Annual cost savings (for 350 days operation). Present unit cost is Rs 6.50
per kWh, investment for a 220 TR screw compressor is Rs 30 lakh.
(iii) What should be the size of cooling tower required for proposed screw
compressors?
(i) Operating reciprocating compressors capacity : 440 TR
(If candidates have used 5 x 110 = 550 TR, then 70% of marks
can be given)
Sp. Power consumption of compressor (reciprocating) : 0.8 kW/TR
Power consumption per hour : 0.8 x 440 = 352 kW
Required screw compressor capacity : 440 TR
Sp. Power consumption of compressor (screw) : 0.65 kW/TR
Power consumption per hour : 286 kW
By replacement of reciprocating compressors with screw compressors
reduction in power is 66 kW per hour and in consumption 1056 kWh/day.

(ii) Annual cost savings:


Reduction in power consumption : 66 kW
Operating hours : 16 per day
Operating days : 350 days per year
Annual energy savings : 66 x 16 x 350

_________________________ 6
Bureau of Energy Efficiency
Paper 4 – Energy Auditor – Set B

: 3.696 lakh kWh


Annual cost savings : Rs 24.024 lakh (@ Rs 6.50 per kWh)

(iii) Cooling tower capacity:


Operating refrigeration load : 440 TR
Vapour compression type refrigeration systems condenser load (TR) will
be around 1.2 time of evaporator load (TR)
Cooling tower capacity : 1.2 x 440 TR
: 528 TR
-------- End of Section - III ---------

_________________________ 7
Bureau of Energy Efficiency
Paper EA4 - Energy Auditor - Set A

NATIONAL CERTIFICATION EXAMINATION 2004


for

ENERGY AUDITORS

PAPER – EA4: Energy Performance Assessment for Equipment and Utility Systems

Date: 23.05.2004 Timings: 1400 – 1600 HRS Duration: 2 HRS. Max. Marks: 100

General instructions:
o Please check that this question paper contains 3 printed pages
o Please check that this question paper contains 16 questions
o The question paper is divided into three sections
o All questions in all three sections are compulsory
o All parts of a question should be answered at one place
o Open book examination

Section – I: Short Questions Marks: 10 x 1 = 10

(i) Answer all Ten questions


(ii) Each question carries One mark

S-1 Define “sub - stoichiometric ratio” in combustion technology. State one


technology applying “sub – stoichiometric” combustion?

S-2 Why does radiation heat loss increase considerably with increase in the
temperature of a furnace wall?

S-3 Draw the schematic diagram of topping cycle cogeneration system.

S-4 Name the two most common types of extended surface heat exchangers.

S-5 Explain why efficiency testing of electric motors yields different results, even if
same measuring equipment is used.

S-6 What is meant by the term “specific power requirement” with respect to air
compressors? State the units.

S-7 Define “lamp circuit efficacy”. State the units.

S-8 Explain why a project with a high IRR is not necessarily more attractive than a
project with a lower IRR.

S-9 How do you classify solar thermal devices?

S-10 Explain how performance of an internal combustion engine changes when the
fuel is switched from petrol to producer gas.

-------- End of Section - I ---------

________________________ 1
Bureau of Energy Efficiency
Paper EA4 - Energy Auditor - Set A

Section - II: Long Questions Marks: 2 x 5 = 10

(i) Answer all Two questions


(ii) Each question carries Five marks

L-1 Assume that the boiler efficiency is calculated by the direct method using
either NCV or GCV.
(i) What is the difference in the efficiency calculation using NCV or
GCV?
(ii) State an example where boiler efficiency is more than 100%, if not,
explain why this is not possible.

L-2 Which is one of the first essential steps in determining the suitability of a
variable speed drive in a pump system? Explain why?

-------- End of Section - II ---------

Section - III: Numerical Questions Marks: 4 x 20 = 80

(i) Answer all Four questions


(ii) Each question carries Twenty marks

N-1 Determine the simple payback period of the incremental investment for two
transformers with the following details:

Option A Option B
Capacity 500 kVA 500 kVA
Efficiency at rated capacity 98% 98.5%
Capital cost Rs. 3.15 lakhs Rs. 4.05 lakhs

Assume the following for both the transformers


Operating PF at rated capacity = 0.9
No load losses = same
Energy charge = Rs. 4.50/kWh

For the analysis consider two cases for the length of time during which the
transformers are used at rated capacity
(a) 10 hours/day and 250 days/year of operation
(b) 16 hours/day and 300 days/year of operation

________________________ 2
Bureau of Energy Efficiency
Paper EA4 - Energy Auditor - Set A

N-2 A reciprocating single stage compressor coupled with an electric motor has a
mechanical shaft power requirement of 50 kW at a discharge pressure of 700
kPa. Determine the energy cost savings if the discharge pressure is reduced to
600 kPa for both isothermal and adiabatic compression processes. Assume
the following for the existing and modified pressure conditions:
Intake air pressure = 1 atmosphere, motor operating efficiency = 90%
Average load factor = 75%, operating hours = 8000 hours/year
Average energy charge = Rs. 4.5/kWh No change in remaining parameters

N-3 It is proposed to install at the beginning of the year a heat recovery equipment
in a food processing industry. The capital cost of the equipment is Rs 20,000/.
The savings accrued by the unit are constant and Rs 5,000/- annually. The
discount rate is 8%.
(i) Calculate the Net Present Value (NPV) for 5 years.
(ii) Is the investment recovered after 5 years? Explain!
(iii) Is the investment recovered after 7 years? Explain!
(iv) Estimate the IRR for this investment after 7 years if the salvage value
of the equipment is Rs 2,000 at the end of 7th year.

N -4 The following are the data collected for a boiler using furnace oil as the fuel.
Determine the boiler efficiency based on GCV by indirect method ignoring
radiation and convection losses.
Ultimate chemical analysis (% weight): Carbon : 84, Hydrogen : 12,
Nitrogen : 0.5, Oxygen : 1.5, Sulphur: 1.5, Moisture : 0.5, NCV of fuel 9,763
kCal/kg and humidity 0.025 kg moisture /kg of dry air.
Flue gas analysis: CO2 : 9.8% volume, flue gas exit temperature : 190oC and
ambient temperature : 30oC.

-------- End of Section - III ---------

________________________ 3
Bureau of Energy Efficiency
Paper EA4 - Energy Auditor - Set B

NATIONAL CERTIFICATION EXAMINATION 2004


for

ENERGY AUDITORS

PAPER – EA4: Energy Performance Assessment for Equipment and Utility Systems

Date: 23.05.2004 Timings: 1400 – 1600 HRS Duration: 2 HRS. Max. Marks: 100

General instructions:
o Please check that this question paper contains 3 printed pages
o Please check that this question paper contains 16 questions
o The question paper is divided into three sections
o All questions in all three sections are compulsory
o All parts of a question should be answered at one place
o Open book examination

Section – I: Short Questions Marks: 10 x 1 = 10

(i) Answer all Ten questions


(ii) Each question carries One mark

S-1 Why does an energy auditor prefer to establish the efficiency of a steam boiler
based on the indirect method?

S-2 Explain why a project with a high IRR is not necessarily more attractive then a
project with a lower IRR.

S-3 What are the two most important rules to improve measurement accuracy
when measuring airflow with a pitot tube in a large duct?

S-4 Name the two most common types of extended surface heat exchangers.

S-5 Define volumetric efficiency for a compressor. State the units.

S-6 Is there a conceptual difference between COP and EER? Explain how they
relate!

S-7 Define color rendering index.

S-8 Define the term “overall plant fuel rate”. State the units.

S-9 Which three variables need to be measured to calculate the efficiency of a


pump. State which units apply in an SI system?

S-10 Under what circumstances does cogeneration not make any sense?

-------- End of Section - I ---------

_________________________ 1
Bureau of Energy Efficiency
Paper EA4 - Energy Auditor - Set B

Section - II: Long Questions Marks: 2 x 5 = 10

(i) Answer all Two questions


(ii) Each question carries Five marks

L-1 Assume that the boiler efficiency is calculated by the direct method using
either NCV or GCV.
(i) What is the difference in the efficiency calculation using NCV or
GCV?
(ii) State an example where boiler efficiency is more than 100%, if not,
explain why this is not possible.

L-2 Which is one of the first essential steps in determining the suitability of a
variable speed drive in a pump system? Explain why!

-------- End of Section - II ---------

Section - III: Numerical Questions Marks: 4 x 20 = 80

(i) Answer all Four questions


(ii) Each question carries Twenty marks

N-1 Determine the simple payback period of the incremental investment for two
transformers with the following details

Option A Option B
Capacity 315 kVA 315 kVA
Efficiency at rated capacity 98% 99%
Capital cost Rs. 2.2 lakhs Rs. 3.0 lakhs

Assume the following for both the transformers


Operating PF at rated capacity = 0.9
No load losses = same
Energy charge = Rs. 4.50/kWh

For the analysis consider two cases for the length of time during which the
transformers are used at rated capacity
(a) 10 hours/day and 250 days/year of operation
(b) 16 hours/day and 300 days/year of operation

_________________________ 2
Bureau of Energy Efficiency
Paper EA4 - Energy Auditor - Set B

N-2 A reciprocating single stage compressor coupled with an electric motor has a
mechanical shaft power requirement of 75 kW at a discharge pressure of 600
kPa. Determine the energy cost savings if the discharge pressure is reduced to
500 kPa of both isothermal and adiabatic compression processes. Assume
the following for the existing and modified pressure conditions:
Intake air pressure = 1 atmosphere, motor operating efficiency = 95%
Average load factor = 75%, operating hours = 7000 hours/year
Average energy charge = Rs. 4.50/kWh No change in remaining parameters

N-3 It is proposed to install at the beginning of the year a heat recovery equipment
in a food processing industry. The capital cost of the equipment is Rs 20,000/-.
The savings accrued by the unit are constant and Rs 5,000/- annually. The
discount rate is 25%.
(i) Calculate the Net Present Value (NPV) for 5 years.
(ii) Is the investment recovered after 5 years? Explain!
(iii) Is the investment recovered after 7 years? Explain!
(iv) Estimate the IRR for this investment after 7 years if the salvage value
of the equipment is Rs 2,000 at the end of 7th year.

N -4 The following are the data collected for a boiler using furnace oil as the fuel.
Determine the boiler efficiency based on GCV by indirect method ignoring
radiation and convection losses.
Ultimate chemical analysis (% weight) : Carbon : 84, Hydrogen : 12,
Nitrogen : 0.5, Oxygen : 1.5, Sulphur: 1.5, Moisture : 0.5, NCV of fuel 9,763
kCal/kg and humidity 0.015 kg moisture/kg of dry air.
Flue gas analysis: CO2 : 10.5% volume, flue gas temperature : 180oC and
ambient temperature : 20oC

-------- End of Section - III ---------

_________________________ 3
Bureau of Energy Efficiency
2nd NATIONAL CERTIFICATION EXAMINATION 2005
FOR
ENERGY AUDITORS

MODEL TEST SERIES –1

Paper – 4: Energy Performance Assessment for Equipment and Utility Systems

Duration: 2 HRS Max. Marks: 100

General instructions:
o Please check that this question paper contains 16 questions
o The question paper is divided into three sections
o All questions in all three sections are compulsory
o All parts of a question should be answered at one place
o Open Book Examination

Section – I: Short Questions Marks: 10 x 1 = 10


(i) Answer all Ten questions
(ii) Each question carries One mark

1 How boiler rated capacity is specified?

2 What are the components of surface heat loss in the furnaces and its dependent
factors affecting loss?

3 How Efficiency and Power Factor varies in motors with VSD Drives.

4 What are the three reasons for poor fan performance in the field?

5 How do you determine system resistance for a pump?

6 What are the measuring instruments required for a compressed air delivery test by
nozzle method ?

7 A refrigeration plant operating at 600 T is consuming 564 kW of compressor power.


Find out the kW/ton, COP and EER

8 What is Lamp Circuit Efficacy?

9 What is the capacity factor of wind turbine.

10 Name four type of wastage due to poorhouse keeping?

Paper 4/ Series 1 Dr. Ambedkar Institute of Productivity, NPC-Chennai. Page 1 of 3


Section - II: Long Questions Marks: 2 x 5 = 10
(i) Answer all Two questions
(ii) Each question carries Five marks

1 Compare the advantage of biogas generation over the direct burning of 25 kg


biomass?

2 What are the Factors Affecting Furnace Performance ?

Section - III: Numerical Questions Marks: 4 x 20 = 80


(i) Answer all Four questions
(ii) Each question carries Twenty marks

1 A Boiler Efficiency trial was conducted in an Furnace Oil fired process boiler during
Energy Audit Study and the following data were collected.

Boiler Data :

Boiler Rated Capacity = 10 TPH (F&A 100oC)


Rated Boiler Efficiency = 84%
Actual steam generation pressure = 7 kg/cm2 (g) Saturated
Feed water Temperature = 45oC

Fuel Oil Data :

Furnace Oil Consumption = 600 litre per hour


GCV of Oil = 10200 Kcal/kg
Specific gravity of oil = 0.92
% Carbon = 84%
% Hydrogen = 12%
% Sulphur = 3%
% Oxygen = Nil
% Nitrogen = 1%

Flue Gas Data :

% O2 in Flue Gas = 5.5% by volume


CO = Nil
Flue Gas Temperature = 240oC
Specific Heat of Flue Gas = 0.24
Moisture at Ambient air = 0.03 kg/kg of air
Ambient air temperature = 40oC

Determine the boiler efficiency by indirect method by assuming 2% boiler surface


heat loss. Also find out the fuel oil saving in litre per hour, if efficiency of boiler was

Paper 4/ Series 1 Dr. Ambedkar Institute of Productivity, NPC-Chennai. Page 2 of 3


improved to Rated efficiency.

2 A process plant requires 28 tonnes of steam per hour. The power requirement is 2250
kW. The plant operates for 8000 hours per annum.

Steam is generated at 2 bar in a coal fired boiler with an efficiency of 75% The feed
water temperature is 80OC. The calorific value of coal is 4000 kcal/kg. The cost of
coal is Rs.2000/tonne.

Power is drawn from the grid at Rs. 4/kWh. The contract demand is 3000 kVA and
the company is charged for 100 % of the contract demand at Rs. 300/kVA.

The company is planning for a back pressure cogeneration plant using the same coal
with the following parameters. The power and steam demand were fully met by the
cogeneration plant.

Boiler generation pressure - 18 bar, 310OC


Boiler efficiency - 81 %
A contract demand of 1000 kVA with the grid was kept for emergency purpose.
Investment required - Rs. 20 crores
Generated power = 2250 kW
Find out the IRR over a project life cycle of 6 years.

3 A centrifugal pump at base of cooling tower pumps 120 m3/hr at pressure 2.1 kg/cm2.
The cooling tower range was measured to be 4oC. What is the power input at the
motor? (Efficiency of pump is 65% and motor efficiency is 82%).

Pump is throttled such that cooling tower range was 6oC. What is the new flow rate
under throttled conditions? (Pressure drop across the throttle valve is 0.5 kg/cm2).

If instead of throttling, the existing impeller is replaced with a new impeller at a cost
of Rs.25000/- find the operating point and differential savings vis-à-vis throttling case
considering 8760 operating hours and unit cost of Rs.4/- (assume pump efficiency of
63% and motor efficiency of 82%)

4 In an air conditioning system, the air flow rate is 80,000 m3/hr at a density of 1.2
kg/m3.. The inlet and outlet enthalpies of air at Air Handling Unit is 105 kJ/kg and 79
kJ/kg. The COP of the system is 3.82. It is proposed to replace it with a vapour
absorption system at a cost of Rs. 50 lacs with an annual maintenance cost of Rs.4
lacs.. The steam consumption will be 4.5 kg/hr/TR. The steam is to be generated by a
coal fired boiler with an evaporation ratio of 4.5. The energy electrical energy cost is
Rs.4 per kWh and the system operates for 8000 hrs per annum. The cost of coal is
Rs.2000/tonne. Find out the payback period for the investment.

Paper 4/ Series 1 Dr. Ambedkar Institute of Productivity, NPC-Chennai. Page 3 of 3

You might also like